Set One: Cardio

Ace your homework & exams now with Quizwiz!

The nurse is caring for a patient who is 24 hours postpacemaker insertion. Which nursing intervention is most appropriate at this time? A. Reinforcing the pressure dressing as needed B. Encouraging range-of-motion exercises of the involved arm C. Assessing the incision for any redness, swelling, or discharge D. Applying wet-to-dry dressings every 4 hours to the insertion site

C. Assessing the incision for any redness, swelling, or discharge After pacemaker insertion, it is important for the nurse to observe signs of infection by assessing for any redness, swelling, or discharge from the incision site. The nonpressure dressing is kept dry until removed, usually 24 hours postoperatively. It is important for the patient to limit activity of the involved arm to minimize pacemaker lead displacement.

A patient has had synchronized cardioversion for unstable VT. Which interventions does the nurse include in the patient's care after the procedure? (select all that apply) a. administer therapeutic hypothermia b. Assess vital signs and the level of consciousness. c. Administer antidysrhythmic drug therapy d. Monitor for dysrhthmias e. Monitor for loss of capture f. Assess for chest burns from electrodes

Bcdf

A patient has had a permanent pacemaker surgically implanted. What are the nursing responsibilities for the care of this patient related to the surgery? (select all that apply) a. Administer short-acting sedatives b. Assess the implantation site for bleeding c. Teach about and monitor for the initial activity restrictions d. Observe for overstimulation of the chest wall, which could lead to pneumothorax e. Monitor the ECG rhythm to check that the pacemaker is working correctly

Bce

Excessive vagal simulation can result form which activities? (select all that apply) a. Jogging b. Carotid sinus massage c. Suctioning d. Voiding e. Valsalva maneuver

Bce

The patient has sustained SVT and the health care provider orders IV adnosine. Which important actions must the nurse perform when this drug is given? ( select all that apply) a. Inject the drug slowly over one minute. b. Have emergency equipment at the bedside. c. Follow the drug injection with a normal saline bolus. d. Have injectable beta blocker drugs at the bedside. e. Monitor the patient for bradycardia, nuasea, and vomiting

Bce

The student nurse is looking at a patient's ECG rhythm strip and suspects a normal sinus rhythm (NSR) Which ECG criteria are included for NSR? (SATA) a. Rate: Atrial and ventricular rates of 40 to 120 beats/min b. Rhythm: Atrial and ventricular rhythms regular c. P waves: Present, consistent configuration, one P wave before each QRS complex d. P-R interval 0.24 second e. QRS duration: 0.04 to 0.10 second and constant

Bce

An asymptomatic patient with acute decompensated heart failure (ADHF) suddenly becomes dyspneic. Before dangling the patient on the bedside, what should the nurse assess first? Urine output Heart rhythm Breath sounds Blood pressure

Blood pressure The nurse should evaluate the blood pressure before dangling the patient on the bedside because the blood pressure can decrease as blood pools in the periphery and preload decreases. If the patient's blood pressure is low or marginal, the nurse should put the patient in the semi-Fowler's position and use other measures to improve gas exchange.

An asymptomatic patient with acute decompensated heart failure (ADHF) suddenly becomes dyspneic. Before dangling the patient on the bedside, what should the nurse assess first? Urine output Heart rhythm Breath sounds Blood pressure

Blood pressure The nurse should evaluate the blood pressure before dangling the patient on the bedside because the blood pressure can decrease as blood pools in the periphery and preload decreases. If the patient's blood pressure is low or marginal, the nurse should put the patient in the semi-Fowler's position and use other measures to improve gas exchange.

A 54-year-old male patient who had bladder surgery 2 days ago develops acute decompensated heart failure (ADHF) with severe dyspnea. Which action by the nurse would be indicated first? A. Perform a bladder scan to assess for urinary retention. B. Restrict the patient's oral fluid intake to 500 mL per day. C. Assist the patient to a sitting position with arms on the overbed table. D. Instruct the patient to use pursed-lip breathing until the dyspnea subsides.

C. Assist the patient to a sitting position with arms on the overbed table. The nurse should place the patient with ADHF in a high Fowler's position with the feet horizontal in the bed or dangling at the bedside. This position helps decrease venous return because of the pooling of blood in the extremities. This position also increases the thoracic capacity, allowing for improved ventilation. Pursed-lip breathing helps with obstructive air trapping but not with acute pulmonary edema. Restricting fluids takes considerable time to have an effect.

The nurse is caring for a patient who has been receiving warfarin (Coumadin) and digoxin (Lanoxin) as treatment for atrial fibrillation. Because the warfarin has been discontinued before surgery, the nurse should diligently assess the patient for which complication early in the postoperative period until the medication is resumed? A. Decreased cardiac output B. Increased blood pressure C. Cerebral or pulmonary emboli D. Excessive bleeding from incision or IV sites

C. Cerebral or pulmonary emboli Warfarin is an anticoagulant that is used to prevent thrombi from forming on the walls of the atria during atrial fibrillation. Once the medication is terminated, thrombi could again form. If one or more thrombi detach from the atrial wall, they could travel as cerebral emboli from the left atrium or pulmonary emboli from the right atrium.

A 72-year-old man with a history of aortic stenosis is admitted to the emergency department. He reports severe left-sided chest pressure radiating to the jaw. Which medication, if ordered by the health care provider, should the nurse question? A. Aspirin B. Oxygen C. Nitroglycerin D. Morphine sulfate

C. Nitroglycerin Aspirin, oxygen, nitroglycerin, and morphine sulfate are all commonly used to treat acute chest pain suspected to be caused by myocardial ischemia. However, nitroglycerin should be used cautiously or avoided in patients with aortic stenosis as a significant reduction in blood pressure may occur. Chest pain can worsen because of a drop in blood pressure.

The nurse would determine that a postoperative patient is not receiving the beneficial effects of enoxaparin (Lovenox) after noting what during a routine shift assessment? A. Generalized weakness and fatigue B. Crackles bilaterally in the lung bases C. Pain and swelling in lower extremity D. Abdominal pain with decreased bowel sounds

C. Pain and swelling in lower extremity Enoxaparin is a low-molecular-weight heparin used to prevent the development of deep vein thromboses (DVTs) in the postoperative period. Pain and swelling in the lower extremity can indicate development of DVT and therefore may signal ineffective medication therapy.

2. A patient with chronic heart failure who has been following a low-sodium diet tells the nurse at the clinic about a 5-pound weight gain in the last 3 days. The nurse's first action will be to a. ask the patient to recall the dietary intake for the last 3 days because there may be hidden sources of sodium in the patient's diet. b. instruct the patient in a low-calorie, low-fat diet because the weight gain has likely been caused by excessive intake of inappropriate foods. c. assess the patient for clinical manifestations of acute heart failure because an exacerbation of the chronic heart failure may be occurring. d. educate the patient about the use of diuretic therapy because it is likely that the patient will need medications to reduce the hypervolemia.

C Rationale: The 5-pound weight gain over 3 days indicates that the patient's chronic heart failure may be worsening; it is important that the patient be immediately assessed for other clinical manifestations of decompensation, such as lung crackles. A dietary recall to detect hidden sodium in the diet and teaching about diuretic therapy are appropriate interventions but are not the first nursing actions indicated. There is no evidence that the patient's weight gain is caused by excessive dietary intake of fat or calories, so the answer beginning "instruct the patient in a low-calorie, low-fat diet" describes an inappropriate action. Cognitive Level: Application Text Reference: p. 826 Nursing Process: Assessment NCLEX: Physiological Integrity

Integrated Process: Nursing Process (Planning) 24. A client with third-degree heart block is admitted to the telemetry unit. The nurse observes wide QRS complexes on the monitor with a heart rate of 35 beats/min. What priority assessment does the nurse perform? a. Pulmonary auscultation b. Pulse strength and amplitude c. Level of consciousness d. Mobility and gait stability

C A heart rate of 40 beats/min or less, with widened QRS complexes, could have hemodynamic consequences, and the client is at risk for inadequate cerebral perfusion. The nurse should assess for level of consciousness, lightheadedness, confusion, syncope, and seizure activity. DIF: Cognitive Level: Application/Applying or higher REF: N/A TOP: Client Needs Category: Physiological Integrity (Reduction of Risk Potential—Potential for Alterations in Body Systems)

Integrated Process: Nursing Process (Assessment) 25. The nurse is caring for a client with a temporary pacemaker. The client's bedside monitor shows a spike followed by a QRS complex. What is the nurse's best action? a. Remove the pacemaker; it is not needed. b. Decrease the threshold of the pacemaker. c. Document the finding in the client's chart. d. Set the pacemaker to the synchronous mode.

C A spike followed by a QRS complex indicates "capture," meaning that the pacemaker has successfully depolarized or captured the ventricle. No action other than documentation of this finding is necessary. DIF: Cognitive Level: Application/Applying or higher REF: N/A TOP: Client Needs Category: Physiological Integrity (Physiological Adaptation—Illness Management)

Physiological Integrity 17. During discharge teaching with a 68-year-old patient who had a mitral valve replacement with a mechanical valve, the nurse instructs the patient on the a. use of daily aspirin for anticoagulation. b. correct method for taking the radial pulse. c. need for frequent laboratory blood testing. d. need to avoid any physical activity for 1 month.

C Anticoagulation with warfarin (Coumadin) is needed for a patient with mechanical valves to prevent clotting on the valve. This will require frequent international normalized ratio (INR) testing. Daily aspirin use will not be effective in reducing the risk for clots on the valve. Monitoring of the radial pulse is not necessary after valve replacement. Patients should resume activities of daily living as tolerated. DIF: Cognitive Level: Apply (application) REF: 826 TOP: Nursing Process: Implementation MSC:

Physiological Integrity 29. Which action by the nurse will determine if the therapies ordered for a patient with chronic constrictive pericarditis are effective? a. Assess for the presence of a paradoxical pulse. b. Monitor for changes in the patient's sedimentation rate. c. Assess for the presence of jugular venous distention (JVD). d. Check the electrocardiogram (ECG) for ST segment changes.

C Because the most common finding on physical examination for a patient with chronic constrictive pericarditis is jugular venous distention, a decrease in JVD indicates improvement. Paradoxical pulse, ST-segment ECG changes, and changes in sedimentation rates occur with acute pericarditis but are not expected in chronic constrictive pericarditis. DIF: Cognitive Level: Apply (application) REF: 817 TOP: Nursing Process: Evaluation MSC:

Integrated Process: Nursing Process (Analysis) 18. The nurse has administered adenosine (Adenocard). What is the expected therapeutic response? a. Increased intraocular pressure b. A brief tonic-clonic seizure c. A short period of asystole d. Hypertensive crisis

C Clients usually respond to this medication with a short period of asystole, bradycardia, hypotension, dyspnea, and chest pain. DIF: Cognitive Level: Comprehension/Understanding REF: p. 733 TOP: Client Needs Category: Physiological Integrity (Pharmacological and Parenteral Therapies—Expected Actions/Outcomes)

Integrated Process: Nursing Process (Implementation) 22. The physician is about to perform carotid sinus massage on a client with supraventricular tachycardia. What equipment is most important for the nurse to have ready? a. Emesis basin b. Magnesium sulfate c. Resuscitation cart d. Padded tongue blade

C Complications of this procedure include bradydysrhythmias, asystole, ventricular fibrillation, and cerebral damage. The resuscitation cart, complete with defibrillator, should be available whenever this procedure is initiated. The other equipment is not needed. DIF: Cognitive Level: Application/Applying or higher REF: N/A TOP: Client Needs Category: Physiological Integrity (Reduction of Risk Potential—Potential for Complications from Diagnostic Tests/Treatments/Procedures)

Physiological Integrity 22. The nurse will plan discharge teaching about the need for prophylactic antibiotics when having dental procedures for which patient? a. Patient admitted with a large acute myocardial infarction. b. Patient being discharged after an exacerbation of heart failure. c. Patient who had a mitral valve replacement with a mechanical valve. d. Patient being treated for rheumatic fever after a streptococcal infection.

C Current American Heart Association guidelines recommend the use of prophylactic antibiotics before dental procedures for patients with prosthetic valves to prevent infective endocarditis (IE). The other patients are not at risk for IE. DIF: Cognitive Level: Apply (application) REF: 813 TOP: Nursing Process: Planning MSC:

Nitrodisc

-nitrate -place on hairless part of body

Physiological Integrity 19. When caring for a patient with infective endocarditis of the tricuspid valve, the nurse should monitor the patient for the development of a. flank pain. b. splenomegaly. c. shortness of breath. d. mental status changes.

C Embolization from the tricuspid valve would cause symptoms of pulmonary embolus. Flank pain, changes in mental status, and splenomegaly would be associated with embolization from the left-sided valves. DIF: Cognitive Level: Apply (application) REF: 812 TOP: Nursing Process: Planning MSC:

A nurse is providing teaching to a client who has a new prescription for digoxin (Lanoxin) Which of the following may indicate dig toxicity & should be reported to the provider? 1. Fatigue 2. constipation 3. Anorexia 4. Rash 5. Diplopia

1. Fatigue Not constipation but -- nausea, vomiting & diarrhea 3. Anorexia b/c GI disturbances 5. Diplopia -- visual changes , halo, yellow-tinged vision.

22. While providing discharge instructions to the patient who has had an implantable cardioverter-defibrillator (ICD) inserted, the nurse teaches the patient that if the ICD fires, \ he or she should do what? a. Lie down. b. Call the cardiologist. c. Push the reset button on the pulse generator. d. Immediately take his or her antidysrhythmic medication.

22. b. If the cardioverter-defibrillator delivers a shock, the patient has experienced a lethal dysrhythmia and needs to notify the cardiologist. The patient will want to lie down to allow recovery from the dysrhythmia. In the event that the patient loses consciousness or there is repetitive firing, a call should be placed to the emergency medical services (EMS) system by anyone who finds the patient.

25. The use of catheter ablation therapy to "burn" areas of the cardiac conduction system is indicated for the treatment of a. sinus arrest. b. heart blocks. c. tachydysrhythmias. d. premature ventricular tachycardia.

25. c. Catheter ablation therapy uses radiofrequency energy to ablate or "burn" accessory pathways or ectopic sites in the atria, AV node, or ventricles that cause tachydysrhythmias.

28. The patient is brought to the emergency department with acute coronary syndrome (ACS). What changes should the nurse expect to see on the ECG if only myocardial injury has occurred? a. Absent P wave b. A wide Q wave c. Inverted T wave d. ST segment elevation

28. d. ST segment elevation is seen in myocardial injury. An absent or buried P wave can occur with PVCs, ventricular tachycardia, or ventricular fibrillation. A wide pathologic Q wave may be seen with infarction. T wave inversion may be seen with cardiac ischemia or within hours following an MI.

What nursing action should the nurse prioritize during the care of a patient who has recently recovered from rheumatic fever? A. Teach the patient how to manage his or her physical activity. B. Teach the patient about the need for ongoing anticoagulation. C. Teach the patient about the need for continuous antibiotic prophylaxis. D. Teach the patient about the need to maintain standard infection control procedures.

C. Teach the patient about the need for continuous antibiotic prophylaxis. Patients with a history of rheumatic fever frequently require ongoing antibiotic prophylaxis, an intervention that necessitates education. This consideration is more important than activity management in preventing recurrence. Anticoagulation is not indicated in this patient population. Standard precautions are indicated for all patients.

A nurse in a provider's office is monitoring serum electrolytes for 4 older adult clients who take digoxin(Lanoxin) & furosemide(Lasix). Which of the following electrolyte values puts a client at risk for dig toxicity? 1. Calcium 9.2 mg/dL 2. Calcium 10.3 mg/dL 3. Potassium 3.4 mEq/L 4. Potassium 4.8 mEq/L

3. Potassium 3.4 mEq/L

A nurse is measuring the blood pressure (BP) of a 68-year-old patient. What intervention should the nurse perform for this patient? 1. Measure BP one hour after eating. 2. Inflate the cuff until the pulse disappears. 3. Recommend a BP goal of 120/80 mm Hg. 4. Check for an auscultatory gap

4. Check for an auscultatory gap The nurse measuring the BP of a 68-year-old patient should check for an auscultatory gap. Some elderly patients have a wide gap between the first Korotkoff sound and subsequent beats. Elderly patients experience a postprandial drop in BP; the greatest drop occurs approximately one hour after eating. The BP returns to preprandial levels three to four hours after eating. When measuring BP, the nurse should inflate the cuff 20 to 30 mm Hg after the radial pulse disappears. The recommended BP goal for this patient would be less than 140/90 mm Hg. Text Reference - p. 725

Integrated Process: Nursing Process (Analysis) 33. The nurse is assessing a client's ECG. What is the nurse's interpretation of the following ECG strip? a. Sinus rhythm with premature ventricular contractions (PVCs) b. Ventricular tachycardia c. Ventricular fibrillation d. Sinus rhythm with premature atrial contractions (PACs)

A Sinus rhythm with PVCs has an underlying regular sinus rhythm with ventricular depolarization that sometime precedes atrial depolarization. DIF: Cognitive Level: Application/Applying or higher REF: N/A TOP: Client Needs Category: Physiological Integrity (Physiological Adaptation-Pathophysiology)

All of the following might be considered helpful in the treatment of heart failure EXCEPT for: ADH/vasopressin a Beta 1 adrenergic receptor antagonist AT II receptor antagonists an aldosterone blocking diuretic an ACE inhibitor

ADH/vasopressin

An elderly patient presents to the emergency department after a fall. She states she does not remember the incident. What is most important to assess first? A. Heart rate and rhythm B. Hemoglobin C. Home environment D. Alcohol consumption

ANS: A Although all options will be assessed eventually, determining a cardiac cause for this brief lapse of consciousness is most important. Reference: 839

Which assessment finding in a patient admitted with acute decompensated heart failure (ADHF) requires the most immediate action by the nurse? a. O2 saturation of 88% b. Weight gain of 1 kg (2.2 lb) c. Heart rate of 106 beats/min d. Urine output of 50 mL over 2 hours

ANS: A A decrease in O2 saturation to less than 92% indicates hypoxemia, and the nurse should start supplemental O2 immediately. An increase in apical pulse rate, 1-kg weight gain, and decreases in urine output also indicate worsening heart failure and require nursing actions, but the low O2 saturation rate requires the most immediate nursing action.

A nurse assesses a client with tachycardia. Which clinical manifestation requires immediate intervention by the nurse? a. Mid-sternal chest pain b. Increased urine output c. Mild orthostatic hypotension d. P wave touching the T wave

ANS: A Chest pain, possibly angina, indicates that tachycardia may be increasing the client's myocardial workload and oxygen demand to such an extent that normal oxygen delivery cannot keep pace. This results in myocardial hypoxia and pain. Increased urinary output and mild orthostatic hypotension are not life-threatening conditions and therefore do not require immediate intervention. The P wave touching the T wave indicates significant tachycardia and should be assessed to determine the underlying rhythm and cause; this is an important assessment but is not as critical as chest pain, which indicates cardiac cell death.

A nurse teaches a client who experiences occasional premature atrial contractions (PACs) accompanied by palpitations that resolve spontaneously without treatment. Which statement should the nurse include in this client's teaching? a. "Minimize or abstain from caffeine." b. "Lie on your side until the attack subsides." c. "Use your oxygen when you experience PACs." d. "Take amiodarone (Cordarone) daily to prevent PACs."

ANS: A PACs usually have no hemodynamic consequences. For a client experiencing infrequent PACs, the nurse should explore possible lifestyle causes, such as excessive caffeine intake and stress. Lying on the side will not prevent or resolve PACs. Oxygen is not necessary. Although medications may be needed to control symptomatic dysrhythmias, for infrequent PACs, the client first should try lifestyle changes to control them.

Which patient should the nurse assign as the roommate for a patient who has aplastic anemia? a. A patient with chronic heart failure b. A patient who has viral pneumonia c. A patient who has right leg cellulitis d. A patient with multiple abdominal drains

ANS: A Patients with aplastic anemia are at risk for infection because of the low white blood cell production associated with this type of anemia, so the nurse should avoid assigning a roommate with any possible infectious process

The charge nurse observes a new registered nurse (RN) doing discharge teaching for a patient with hypertension who has a new prescription for enalapril (Vasotec). The charge nurse will need to intervene if the new RN tells the patient to a. increase the dietary intake of high-potassium foods. b. make an appointment with the dietitian for teaching. c. check the blood pressure (BP) at home at least once a day. d. move slowly when moving from lying to sitting to standing.

ANS: A The ACE inhibitors cause retention of potassium by the kidney, so hyperkalemia is a possible adverse effect. The other teaching by the new RN is appropriate for a patient with newly diagnosed hypertension who has just started therapy with enalapril.

The nurse is reviewing the laboratory test results for a patient who has recently been diagnosed with hypertension. Which result is most important to communicate to the health care provider? a. Serum creatinine of 2.8 mg/dL b. Serum potassium of 4.5 mEq/L c. Serum hemoglobin of 14.7 g/dL d. Blood glucose level of 96 mg/dL

ANS: A The elevated creatinine indicates renal damage caused by the hypertension. The other laboratory results are normal

A nurse cares for a client with congestive heart failure who has a regular cardiac rhythm of 128 beats/min. For which physiologic alterations should the nurse assess? (Select all that apply.) a. Decrease in cardiac output b. Increase in cardiac output c. Decrease in blood pressure d. Increase in blood pressure e. Decrease in urine output f. Increase in urine output

ANS: A, D, E Elevated heart rates in a healthy client initially cause blood pressure and cardiac output to increase. However, in a client who has congestive heart failure or a client with long-term tachycardia, ventricular filling time, cardiac output, and blood pressure eventually decrease. As cardiac output and blood pressure decrease, urine output will fall

The emergency department patient is in paroxysmal supraventricular tachycardia (PSVT) at a rate of 170 beats/minute. Which treatment do you anticipate first? A. Sotalol (Betapace) by slow IVP B. Adenosine (Adenocard) by fast IVP C. Defibrillation D. Digoxin (Lanoxin)

ANS: B PSVT is a dysrhythmia originating in an ectopic focus anywhere above the bifurcation of the bundle of His. Treatment includes vagal stimulation (e.g., Valsalva maneuver, coughing) and intravenous (IV) adenosine as the first drug of choice. The drug has a short half-life and is given rapid IVP. Other drugs are β-adrenergic blockers, calcium channel blockers, and amiodarone. Defibrillation is used if the vagal stimulation and drug therapy are ineffective and the patient becomes hemodynamically unstable. Digoxin is not used for this dysrhythmia but typically is used for atrial fibrillation. Reference: 826

Which patient requires the most rapid assessment and care by the emergency department nurse? a. The patient with hemochromatosis who reports abdominal pain b. The patient with neutropenia who has a temperature of 101.8° F c. The patient with sickle cell anemia who has had nausea and diarrhea for 24 hours d. The patient with thrombocytopenia who has oozing after having a tooth extracted

ANS: B A neutropenic patient with a fever is assumed to have an infection and is at risk for rapidly developing sepsis. Rapid assessment, cultures, and initiation of antibiotic therapy are needed. The other patients also require rapid assessment and care but not as urgently as the neutropenic patient

The nurse has just finished teaching a hypertensive patient about the newly prescribed drug, ramipril (Altace). Which patient statement indicates that more teaching is needed? a. "The medication may not work well if I take aspirin." b. "I can expect some swelling around my lips and face." c. "The doctor may order a blood potassium level occasionally." d. "I will call the doctor if I notice that I have a frequent cough."

ANS: B Angioedema occurring with angiotensin-converting enzyme (ACE) inhibitor therapy is an indication that the ACE inhibitor should be discontinued. The patient should be taught that if any swelling of the face or oral mucosa occurs, the health care provider should be immediately notified because this could be life threatening. The other patient statements indicate that the patient has an accurate understanding of ACE inhibitor therapy.

A routine complete blood count indicates that an active 80-year-old man may have myelodysplastic syndrome. The nurse will plan to teach the patient about a. blood transfusion b. bone marrow biopsy. c. filgrastim (Neupogen) administration. d. erythropoietin (Epogen) administration.

ANS: B Bone marrow biopsy is needed to make the diagnosis and determine the specific type of myelodysplastic syndrome. The other treatments may be necessary if there is progression of the myelodysplastic syndrome, but the initial action for this asymptomatic patient will be a bone marrow biopsy.

Which intervention will be included in the nursing care plan for a patient with immune thrombocytopenic purpura (ITP)? a. Assign the patient to a private room. b. Avoid intramuscular (IM) injections. c. Use rinses rather than a soft toothbrush for oral care. d. Restrict activity to passive and active range of motion.

ANS: B IM or subcutaneous injections should be avoided because of the risk for bleeding. A soft toothbrush can be used for oral care. There is no need to restrict activity or place the patient in a private room

A 53-year-old patient with Stage D heart failure and type 2 diabetes asks the nurse whether heart transplant is a possible therapy. Which response by the nurse is most appropriate? a. "Because you have diabetes, you would not be a candidate for a heart transplant." b. "The choice of a patient for a heart transplant depends on many different factors." c. "Your heart failure has not reached the stage in which heart transplants are needed." d. "People who have heart transplants are at risk for multiple complications after surgery."

ANS: B Indications for a heart transplant include end-stage heart failure (Stage D), but other factors such as coping skills, family support, and patient motivation to follow the rigorous posttransplant regimen are also considered. Diabetic patients who have well-controlled blood glucose levels may be candidates for heart transplant. Although heart transplants can be associated with many complications, this response does not address the patient's question.

The nurse is assessing a patient with myocarditis before administering the scheduled dose of digoxin (Lanoxin). Which finding is most important for the nurse to communicate to the health care provider? a. Leukocytosis b. Irregular pulse c. Generalized myalgia d. Complaint of fatigue

ANS: B Myocarditis predisposes the heart to digoxin-associated dysrhythmias and toxicity. The other findings are common symptoms of myocarditis and there is no urgent need to report these.

A patient with a history of a transfusion-related acute lung injury (TRALI) is to receive a transfusion of packed red blood cells (PRBCs). Which action by the nurse will decrease the risk for TRALI for this patient? a. Infuse the PRBCs slowly over 4 hours. b. Transfuse only leukocyte-reduced PRBCs. c. Administer the scheduled diuretic before the transfusion. d. Give the PRN dose of antihistamine before the transfusion.

ANS: B TRALI is caused by a reaction between the donor and the patient leukocytes that causes pulmonary inflammation and capillary leaking. The other actions may help prevent respiratory problems caused by circulatory overload or by allergic reactions, but they will not prevent TRALI

After receiving change-of-shift report on four patients admitted to a heart failure unit, which patient should the nurse assess first? a. A patient who reported dizziness after receiving the first dose of captopril b. A patient who is cool and clammy, with new-onset confusion and restlessness c. A patient who has crackles bilaterally in the lung bases and is receiving oxygen. d. A patient who is receiving IV nesiritide (Natrecor) and has a blood pressure of 100/62

ANS: B The patient who has "wet-cold" clinical manifestations of heart failure is perfusing inadequately and needs rapid assessment and changes in management. The other patients also should be assessed as quickly as possible but do not have indications of severe decreases in tissue perfusion.

The nurse in the emergency department received change-of-shift report on four patients with hypertension. Which patient should the nurse assess first? a. 52-year-old with a BP of 212/90 who has intermittent claudication b. 43-year-old with a BP of 190/102 who is complaining of chest pain c. 50-year-old with a BP of 210/110 who has a creatinine of 1.5 mg/dL d. 48-year-old with a BP of 200/98 whose urine shows microalbuminuria

ANS: B The patient with chest pain may be experiencing acute myocardial infarction and rapid assessment and intervention is needed. The symptoms of the other patients also show target organ damage, but are not indicative of acute processes.

When teaching the patient with newly diagnosed heart failure about a 2000-mg sodium diet, the nurse explains that foods to be restricted include a. canned and frozen fruits. b. yogurt and milk products. c. fresh or frozen vegetables. d. eggs and other high-protein foods.

ANS: B Yogurt and milk products (e.g., cheese) naturally contain a significant amount of sodium, and the intake of these should be limited for patients on a diet that limits sodium to 2000 mg daily. The other foods listed have minimal levels of sodium and can be eaten without restriction.

Which teaching points should you include when providing discharge instructions to a patient with a new permanent pacemaker and to the caregiver (select all that apply)? A. Avoid or limit air travel. B. Take and record a pulse rate daily. C. Obtain and wear a Medic Alert ID or bracelet at all times. D. Avoid lifting the arm on the side of the pacemaker above the shoulder. E. Avoid microwave ovens because they interfere with pacemaker function.

ANS: B,C,D Air travel is not restricted. Inform airport security about the pacemaker because it may set off the metal detector. If a hand-held screening wand is used, it should not be placed directly over the pacemaker. Manufacturer information may vary regarding the effect of metal detectors on the function of the pacemaker. The patient should monitor the pulse and inform the primary care provider if it drops below predetermined rate. A Medic Alert ID or bracelet should be worn at all times. The patient should avoid lifting the arm on the pacemaker side above the shoulder until it is approved by the primary care provider. Microwave ovens are safe to use and do not interfere with pacemaker function. Table 36-13 provides additional discharge teaching guidelines for a patient with a pacemaker. Reference: 837

Which action by the nurse will determine if the therapies ordered for a patient with chronic constrictive pericarditis are effective? a. Assess for the presence of a paradoxical pulse. b. Monitor for changes in the patient's sedimentation rate. c. Assess for the presence of jugular venous distention (JVD). d. Check the electrocardiogram (ECG) for ST segment changes.

ANS: C Because the most common finding on physical examination for a patient with chronic constrictive pericarditis is jugular venous distention, a decrease in JVD indicates improvement. Paradoxical pulse, ST-segment ECG changes, and changes in sedimentation rates occur with acute pericarditis but are not expected in chronic constrictive pericarditis.

The nurse asks a client who has experienced ventricular dysrhythmias about substance abuse. The client asks, "Why do you want to know if I use cocaine?" How should the nurse respond? a. "Substance abuse puts clients at risk for many health issues." b. "The hospital requires that I ask you about cocaine use." c. "Clients who use cocaine are at risk for fatal dysrhythmias." d. "We can provide services for cessation of substance abuse."

ANS: C Clients who use cocaine or illicit inhalants are particularly at risk for potentially fatal dysrhythmias. The other responses do not adequately address the client's question.

The nurse will plan discharge teaching about the need for prophylactic antibiotics when having dental procedures for which patient? a. Patient admitted with a large acute myocardial infarction. b. Patient being discharged after an exacerbation of heart failure. c. Patient who had a mitral valve replacement with a mechanical valve. d. Patient being treated for rheumatic fever after a streptococcal infection.

ANS: C Current American Heart Association guidelines recommend the use of prophylactic antibiotics before dental procedures for patients with prosthetic valves to prevent infective endocarditis (IE). The other patients are not at risk for IE.

Which information should the nurse include when teaching a patient with newly diagnosed hypertension? a. Dietary sodium restriction will control BP for most patients. b. Most patients are able to control BP through lifestyle changes. c. Hypertension is usually asymptomatic until significant organ damage occurs. d. Annual BP checks are needed to monitor treatment effectiveness.

ANS: C Hypertension is usually asymptomatic until target organ damage has occurred. Lifestyle changes and sodium restriction are used to help manage blood pressure, but drugs are needed for most patients. BP should be checked by the health care provider every 3 to 6 months.

The RN is caring for a patient with a hypertensive crisis who is receiving sodium nitroprusside (Nipride). Which of the following nursing actions can the nurse delegate to an experienced LPN/LVN? a. Titrate nitroprusside to maintain BP at 160/100 mm Hg. b. Evaluate effectiveness of nitroprusside therapy on BP. c. Set up the automatic blood pressure machine to take BP every 15 minutes. d. Assess the patient's environment for adverse stimuli that might increase BP.

ANS: C LPN/LVN education and scope of practice include correct use of common equipment such as automatic blood pressure machines. The other actions require more nursing judgment and education and should be done by RNs.

A patient with a history of hypertension treated with a diuretic and an angiotensin-converting enzyme (ACE) inhibitor arrives in the emergency department complaining of a severe headache and has a BP of 240/118 mm Hg. Which question should the nurse ask first? a. Did you take any acetaminophen (Tylenol) today? b. Do you have any recent stressful events in your life? c. Have you been consistently taking your medications? d. Have you recently taken any antihistamine medications?

ANS: C Sudden withdrawal of antihypertensive medications can cause rebound hypertension and hypertensive crisis. Although many over-the-counter medications can cause hypertension, antihistamines and acetaminophen do not increase BP. Stressful events will increase BP but not usually to the level seen in this patient.

A patient with chronic heart failure who is taking a diuretic and an angiotensin-converting enzyme (ACE) inhibitor and who is on a low-sodium diet tells the home health nurse about a 5-pound weight gain in the last 3 days. The nurse's priority action will be to a. have the patient recall the dietary intake for the last 3 days. b. ask the patient about the use of the prescribed medications. c. assess the patient for clinical manifestations of acute heart failure. d. teach the patient about the importance of restricting dietary sodium.

ANS: C The 5-pound weight gain over 3 days indicates that the patient's chronic heart failure may be worsening. It is important that the patient be assessed immediately for other clinical manifestations of decompensation, such as lung crackles. A dietary recall to detect hidden sodium in the diet, reinforcement of sodium restrictions, and assessment of medication compliance may be appropriate interventions but are not the first nursing actions indicated.

A patient has just been diagnosed with hypertension and has been started on captopril . Which information is *most* important to include when teaching the patient about this drug? a. Include high-potassium foods such as bananas in the diet. b. Increase fluid intake if dryness of the mouth is a problem. c. Change position slowly to help prevent dizziness and falls. d. Check blood pressure (BP) in both arms before taking the drug.

ANS: C The angiotensin-converting enzyme (ACE) inhibitors frequently cause orthostatic hypotension, and patients should be taught to change position slowly to allow the vascular system time to compensate for the position change. Increasing fluid intake may counteract the effect of the drug, and the patient is taught to use gum or hard candy to relieve dry mouth. The BP should be taken in the nondominant arm by newly diagnosed patients in the morning, before taking the drug, and in the evening. Because ACE inhibitors cause potassium retention, increased intake of high-potassium foods is inappropriate.

A patient recovering from heart surgery develops pericarditis and complains of level 6 (0 to 10 scale) chest pain with deep breathing. Which ordered PRN medication will be the most appropriate for the nurse to give? a. Fentanyl 1 mg IV b. IV morphine sulfate 4 mg c. Oral ibuprofen (Motrin) 600 mg d. Oral acetaminophen (Tylenol) 650 mg

ANS: C The pain associated with pericarditis is caused by inflammation, so nonsteroidal antiinflammatory drugs (NSAIDs) (e.g., ibuprofen) are most effective. Opioid analgesics are usually not used for the pain associated with pericarditis.

The nurse is caring for a 70-yr-old patient who uses hydrochlorothiazide and enalapril (Norvasc) but whose self-monitored blood pressure (BP) continues to be elevated. Which patient information may indicate a need for a change? a. Patient takes a daily multivitamin tablet. b. Patient checks BP daily just after getting up. c. Patient drinks wine three to four times a week. d. Patient uses ibuprofen (Motrin) treat osteoarthritis.

ANS: D Because use of nonsteroidal antiinflammatory drugs (NSAIDs) can prevent adequate BP control, the patient may need to avoid the use of ibuprofen. A multivitamin tablet will help supply vitamin D, which may help lower BP. BP decreases while sleeping, so self-monitoring early in the morning will result in obtaining pressures that are at their lowest. The patient's alcohol intake is not excessive.

After the nurse teaches the patient with stage 1 hypertension about diet modifications that should be implemented, which diet choice indicates that the teaching has been effective? a. The patient avoids eating nuts or nut butters. b. The patient restricts intake of chicken and fish. c. The patient has two cups of coffee in the morning. d. The patient has a glass of low-fat milk with each meal.

ANS: D For the prevention of hypertension, the Dietary Approaches to Stop Hypertension (DASH) recommendations include increasing the intake of calcium-rich foods. Caffeine restriction and decreased protein intake are not included in the recommendations. Nuts are high in beneficial nutrients and 4 to 5 servings weekly are recommended in the DASH diet.

Which action should the nurse take when giving the initial dose of oral labetalol to a patient with hypertension? a. Encourage the use of hard candy to prevent dry mouth. b. Teach the patient that headaches often occur with this drug. c. Instruct the patient to call for help if heart palpitations occur. d. Ask the patient to request assistance before getting out of bed.

ANS: D Labetalol decreases sympathetic nervous system activity by blocking both a and b adrenergic receptors, leading to vasodilation and a decrease in heart rate, which can cause severe orthostatic hypotension. Heart palpitations, dry mouth, dehydration, and headaches are possible side effects of other antihypertensives.

A critical action by the nurse caring for a patient with an acute exacerbation of polycythemia vera is to a. place the patient on bed rest. b. administer iron supplements. c. avoid use of aspirin products. d. monitor fluid intake and output.

ANS: D Monitoring hydration status is important during an acute exacerbation because the patient is at risk for fluid overload or underhydration. Aspirin therapy is used to decrease risk for thrombosis. The patient should be encouraged to ambulate to prevent deep vein thrombosis (DVT). Iron is contraindicated in patients with polycythemia vera

The nurse establishes the nursing diagnosis of ineffective health maintenance related to lack of knowledge regarding long-term management of rheumatic fever when a 30-year-old recovering from rheumatic fever without carditis says which of the following? a. "I will need prophylactic antibiotic therapy for 5 years." b. "I will need to take aspirin or ibuprofen (Motrin) to relieve my joint pain." c. "I will call the doctor if I develop excessive fatigue or difficulty breathing." d. "I will be immune to further episodes of rheumatic fever after this infection."

ANS: D Patients with a history of rheumatic fever are more susceptible to a second episode. Patients with rheumatic fever without carditis require prophylaxis until age 20 and for a minimum of 5 years. The other patient statements are correct and would not support the nursing diagnosis of ineffective health maintenance.

Which laboratory result will the nurse expect to show a decreased value if a patient develops heparin-induced thrombocytopenia (HIT)? a. Prothrombin time b. Erythrocyte count c. Fibrinogen degradation products d. Activated partial thromboplastin time

ANS: D Platelet aggregation in HIT causes neutralization of heparin, so that the activated partial thromboplastin time will be shorter and more heparin will be needed to maintain therapeutic levels. The other data will not be affected by HIT

When a patient with hypertension who has a new prescription for atenolol (Tenormin) returns to the health clinic after 2 weeks for a follow-up visit, the BP is unchanged from the previous visit. Which action should the nurse take first? a. Provide information about the use of multiple drugs to treat hypertension. b. Teach the patient about the reasons for a possible change in drug therapy. c. Remind the patient that lifestyle changes also are important in BP control. d. Question the patient about whether the medication is actually being taken.

ANS: D Since noncompliance with antihypertensive therapy is common, the nurse's initial action should be to determine whether the patient is taking the atenolol as prescribed. The other actions also may be implemented, but these would be done after assessing patient compliance with the prescribed therapy.

A nurse assesses a client's electrocardiogram (ECG) and observes the reading shown below: How should the nurse document this client's ECG strip? a. Ventricular tachycardia b. Ventricular fibrillation c. Sinus rhythm with premature atrial contractions (PACs) d. Sinus rhythm with premature ventricular contractions (PVCs)

ANS: D Sinus rhythm with PVCs has an underlying regular sinus rhythm with ventricular depolarization that sometimes precede atrial depolarization. Ventricular tachycardia and ventricular fibrillation rhythms would not have sinus beats present. Premature atrial contractions are atrial contractions initiated from another region of the atria before the sinus node initiates atrial depolarization.

The nurse is caring for a patient with mitral regurgitation. Referring to the figure below, where should the nurse listen to best hear any murmur that the patient has? a. 1 b. 2 c. 3 d. 4

ANS: D Sounds from the mitral valve are best heard at the apex of the heart, fifth intercostal space, midclavicular line.

Which actions are the responsibilities of the monitor tech? (SATA) a. Watch the bank of monitors on a unit b. Notify the health care provider of any changes c. Print routine ECG strips d. Apply battery-operated transmitter leads to patients. e. Interpret the rhythems

Ace

Physiological Integrity 14. A 21-year-old woman is scheduled for percutaneous transluminal balloon valvuloplasty to treat mitral stenosis. Which information should the nurse include when explaining the advantages of valvuloplasty over valve replacement to the patient? a. Biologic valves will require immunosuppressive drugs after surgery. b. Mechanical mitral valves need to be replaced sooner than biologic valves. c. Lifelong anticoagulant therapy will be needed after mechanical valve replacement. d. Ongoing cardiac care by a health care provider is not necessary after valvuloplasty.

C Long-term anticoagulation therapy is needed after mechanical valve replacement, and this would restrict decisions about career and childbearing in this patient. Mechanical valves are durable and last longer than biologic valves. All valve repair procedures are palliative, not curative, and require lifelong health care. Biologic valves do not activate the immune system, and immunosuppressive therapy is not needed. DIF: Cognitive Level: Apply (application) REF: 824-825 TOP: Nursing Process: Implementation MSC:

Physiological Integrity 30. Which statement by a patient with restrictive cardiomyopathy indicates that the nurse's discharge teaching about self-management has been most effective? a. "I will avoid taking aspirin or other antiinflammatory drugs." b. "I will need to limit my intake of salt and fluids even in hot weather." c. "I will take antibiotics when my teeth are cleaned at the dental office." d. "I should begin an exercise program that includes things like biking or swimming."

C Patients with restrictive cardiomyopathy are at risk for infective endocarditis and should use prophylactic antibiotics for any procedure that may cause bacteremia. The other statements indicate a need for more teaching by the nurse. Dehydration and vigorous exercise impair ventricular filling in patients with restrictive cardiomyopathy. There is no need to avoid salt (unless ordered), aspirin, or NSAIDs. DIF: Cognitive Level: Apply (application) REF: 830 | 813 TOP: Nursing Process: Evaluation MSC:

The nurse is preparing to administer a nitroglycerin patch to a patient. When providing instructions regarding the use of the patch, what should the nurse include in the teaching? Avoid high-potassium foods Avoid drugs to treat erectile dysfunction Avoid over-the-counter H2-receptor blockers Avoid nonsteroidal antiinflammatory drugs (NSAIDS)

Avoid drugs to treat erectile dysfunction The use of erectile drugs concurrent with nitrates creates a risk of severe hypotension and possibly death. High-potassium foods, NSAIDs, and H2-receptor blockers do not pose a risk in combination with nitrates.

The nurse is preparing to administer a nitroglycerin patch to a patient. When providing instructions regarding the use of the patch, what should the nurse include in the teaching? Avoid high-potassium foods Avoid drugs to treat erectile dysfunction Avoid over-the-counter H2-receptor blockers Avoid nonsteroidal antiinflammatory drugs (NSAIDS)

Avoid drugs to treat erectile dysfunction The use of erectile drugs concurrent with nitrates creates a risk of severe hypotension and possibly death. High-potassium foods, NSAIDs, and H2-receptor blockers do not pose a risk in combination with nitrates.

Integrated Process: Nursing Process (Analysis) 35. A nurse assesses the following ECG strip from a client's telemetry monitor. What does the nurse do next? a. Measure hourly urine output. b. Assess the client's vital signs. c. Administer 0.5 mg atropine IV. d. Prepare for external pacing.

B Assessing the client's vital signs allows the nurse to determine if he or she is stable or unstable and symptomatic with the bradycardia. The client's stability with the bradycardia will determine the need for specific interventions. DIF: Cognitive Level: Application/Applying or higher REF: N/A TOP: Client Needs Category: Physiological Integrity (Physiological Adaptation-Pathophysiology)

Integrated Process: Nursing Process (Assessment) 14. The nurse is caring for a client with atrial fibrillation. What manifestation most alerts the nurse to the possibility of a serious complication from this condition? a. Sinus tachycardia b. Speech alterations c. Fatigue d. Dyspnea with activity

B Clients with atrial fibrillation are at risk for embolic stroke. Evidence of embolic events includes changes in mentation, speech, sensory function, and motor function. Clients with atrial fibrillation often have a rapid ventricular response as a result. Fatigue is a nonspecific complaint. Clients with atrial fibrillation often have dyspnea as a result of the decreased cardiac output caused by the rhythm disturbance. DIF: Cognitive Level: Application/Applying or higher REF: N/A TOP: Client Needs Category: Physiological Integrity (Reduction of Risk Potential—Potential for Complications from Surgical Procedures and Health Alterations)

Physiological Integrity 32. After receiving report on the following patients, which patient should the nurse assess first? a. Patient with rheumatic fever who has sharp chest pain with a deep breath b. Patient with acute aortic regurgitation whose blood pressure is 86/54 mm Hg c. Patient with infective endocarditis who has a murmur and splinter hemorrhages d. Patient with dilated cardiomyopathy who has bilateral crackles at the lung bases

B Hypotension in patients with acute aortic regurgitation may indicate cardiogenic shock. The nurse should immediately assess this patient for other findings such as dyspnea or chest pain. The findings in the other patients are typical of their diagnoses and do not indicate a need for urgent assessment and intervention. DIF: Cognitive Level: Analyze (analysis) REF: 823 OBJ: Special Questions: Multiple Patients; Prioritization TOP: Nursing Process: Assessment MSC:

Physiological Integrity 31. The nurse is assessing a patient with myocarditis before administering the scheduled dose of digoxin (Lanoxin). Which finding is most important for the nurse to communicate to the health care provider? a. Leukocytosis b. Irregular pulse c. Generalized myalgia d. Complaint of fatigue

B Myocarditis predisposes the heart to digoxin-associated dysrhythmias and toxicity. The other findings are common symptoms of myocarditis and there is no urgent need to report these. DIF: Cognitive Level: Apply (application) REF: 818 OBJ: Special Questions: Prioritization TOP: Nursing Process: Assessment MSC:

Physiological Integrity 2. During the assessment of a 25-year-old patient with infective endocarditis (IE), the nurse would expect to find a. substernal chest pressure. b. a new regurgitant murmur. c. a pruritic rash on the chest. d. involuntary muscle movement.

B New regurgitant murmurs occur in IE because vegetations on the valves prevent valve closure. Substernal chest discomfort, rashes, and involuntary muscle movement are clinical manifestations of other cardiac disorders such as angina and rheumatic fever. DIF: Cognitive Level: Understand (comprehension) REF: 811 | 821 TOP: Nursing Process: Assessment MSC:

20. An elderly patient with a 40-pack-year history of smoking and a recent myocardial infarction is admitted to the medical unit with acute shortness of breath; the nurse need to rule out pneumonia versus heart failure. The diagnostic test that the nurse will monitor to help in determining whether the patient has heart failure is a. 12-lead electrocardiogram (ECG). b. arterial blood gases (ABGs). c. B-type natriuretic peptide (BNP). d. serum creatine kinase (CK).

C Rationale: BNP is secreted when ventricular pressures increase, as with heart failure, and elevated BNP indicates a probable or very probable diagnosis of heart failure. 12-lead ECGs, ABGs, and CK may also be used in determining the causes or effects of heart failure but are not as clearly diagnostic of heart failure as BNP. Cognitive Level: Application Text Reference: p. 827 Nursing Process: Assessment NCLEX: Physiological Integrity

The laboratory results of a patient diagnosed with heart failure shows a serum digoxin (Lanoxin) level of 2.1 ng/mL. Which medication is appropriate to administer at this time? A Furosemide (Lasix) B Digoxin immune fab (DigiFab) C An increased dose of digoxin (Lanoxin) D Potassium chloride (K-tab)

B Digoxin immune fab (DigiFab)

The nurse provides discharge instructions for a 40-year-old woman who is newly diagnosed with cardiomyopathy. Which statement, if made by the patient, indicates that further teaching is necessary? A."I will avoid lifting heavy objects." B. "I can drink alcohol in moderation." C."My family will need to take a CPR course." D. "I will reduce stress by learning guided imagery."

B. "I can drink alcohol in moderation." Patients with cardiomyopathy should avoid alcohol consumption, especially in patients with alcohol-related dilated cardiomyopathy. Avoiding heavy lifting and stress, as well as family members learning CPR, are recommended teaching points.

What are the risk factors for AF? (select all that apply) a. Chronic obstructive pulmonary disease (COPD) b. Hypertension c. Diabetes mellitus d. Valvular disease

Bde

What is the priority assessment by the nurse caring for a patient receiving IV nesiritide to treat heart failure? Urine output Lung sounds Blood pressure Respiratory rate

Blood pressure. Although all identified assessments are appropriate for a patient receiving IV nesiritide, the priority assessment would be monitoring for hypotension, the main adverse effect of nesiritide.

16. A home health care patient has recently started taking oral digoxin (Lanoxin) and furosemide (Lasix) for control of heart failure. The patient data that will require the most immediate action by the nurse is if the patient's a. weight increases from 120 pounds to 122 pounds over 3 days. b. liver is palpable 2 cm below the ribs on the right side. c. serum potassium level is 3.0 mEq/L after 1 week of therapy. d. has 1 to 2+ edema in the feet and ankles in the morning.

C Rationale: Hypokalemia potentiates the actions of digoxin and increases the risk for digoxin toxicity, which can cause life-threatening dysrhythmias. The other data indicate that the patient's heart failure requires more effective therapies, but they do not require nursing action as rapidly as the low serum potassium level. Cognitive Level: Application Text Reference: pp. 832-833 Nursing Process: Assessment NCLEX: Physiological Integrity

3. During assessment of a 72-year-old with ankle swelling, the nurse notes jugular venous distention (JVD) with the head of the patient's bed elevated 45 degrees. The nurse knows this finding indicates a. decreased fluid volume. b. incompetent jugular vein valves. c. elevated right atrial pressure. d. jugular vein atherosclerosis.

C Rationale: The jugular veins empty into the superior vena cava and then into the right atrium, so JVD with the patient sitting at a 45-degree angle reflects elevated right atrial pressure. JVD is an indicator of excessive fluid volume (increased preload), not decreased fluid volume; it is not caused by incompetent jugular vein valves or atherosclerosis. Cognitive Level: Comprehension Text Reference: p. 825 Nursing Process: Assessment NCLEX: Physiological Integrity

Integrated Process: Teaching/Learning 10. The nurse identifies a client's rhythm to be a sustained supraventricular tachycardia. What medication does the nurse administer? a. Atropine (Atropine) b. Epinephrine (Adrenalin) c. Lidocaine (Xylocaine) d. Diltiazem (Cardizem)

D Diltiazem, a calcium channel blocker, slows depolarization through the conduction system and is commonly used as an agent to terminate a sustained episode of supraventricular tachycardia. DIF: Cognitive Level: Application/Applying or higher REF: N/A TOP: Client Needs Category: Physiological Integrity (Pharmacological and Parenteral Therapies—Expected Actions/Outcomes)

Integrated Process: Nursing Process (Analysis) 4. When analyzing a client's electrocardiograph (ECG) tracing, the nurse observes that not all QRS complexes are preceded by a P wave. What is the nurse's interpretation of this observation? a. The client has hyperkalemia causing irregular QRS complexes. b. Ventricular tachycardia is overriding the normal atrial rhythm. c. The client's chest leads are not making sufficient contact with the skin. d. Ventricular and atrial depolarizations are initiated from different sites.

D Normal rhythm shows one P wave preceding each QRS, indicating that all depolarization is initiated at the sinoatrial node. QRS complexes without a P wave indicate a different source of initiation of depolarization. DIF: Cognitive Level: Comprehension/Understanding REF: p. 718 TOP: Client Needs Category: Physiological Integrity (Physiological Adaptation—Pathophysiology)

Physiological Integrity 11. The nurse establishes the nursing diagnosis of ineffective health maintenance related to lack of knowledge regarding long-term management of rheumatic fever when a 30-year-old recovering from rheumatic fever without carditis says which of the following? a. "I will need prophylactic antibiotic therapy for 5 years." b. "I will need to take aspirin or ibuprofen (Motrin) to relieve my joint pain." c. "I will call the doctor if I develop excessive fatigue or difficulty breathing." d. "I will be immune to further episodes of rheumatic fever after this infection."

D Patients with a history of rheumatic fever are more susceptible to a second episode. Patients with rheumatic fever without carditis require prophylaxis until age 20 and for a minimum of 5 years. The other patient statements are correct and would not support the nursing diagnosis of ineffective health maintenance. DIF: Cognitive Level: Apply (application) REF: 820 TOP: Nursing Process: Diagnosis MSC:

9. During a visit to an elderly patient with chronic heart failure, the home care nurse finds that the patient has severe dependent edema and that the legs appear to be weeping serous fluid. Based on these data, the best nursing diagnosis for the patient is a. activity intolerance related to venous congestion. b. disturbed body image related to massive leg swelling. c. impaired skin integrity related to peripheral edema. d. impaired gas exchange related to chronic heart failure.

C Rationale: The patient's findings of severe dependent edema and weeping serous fluid from the legs support the nursing diagnosis of impaired skin integrity. There is less evidence for the nursing diagnoses of activity intolerance, disturbed body image, and impaired gas exchange, although the nurse will further assess the patient to determine whether there are other clinical manifestations of heart failure to indicate that these diagnoses are appropriate. Cognitive Level: Application Text Reference: p. 836 Nursing Process: Diagnosis NCLEX: Physiological Integrity

13. The nurse plans discharge teaching for a patient with chronic heart failure who has prescriptions for digoxin (Lanoxin), hydrochlorothiazide (HydroDIURIL), and a potassium supplement. Appropriate instructions for the patient include a. avoid dietary sources of potassium because too much can cause digitalis toxicity. b. take the pulse rate daily and never take digoxin if the pulse is below 60 beats/min. c. take the hydrochlorothiazide before bedtime to maximize activity level during the day. d. notify the health care provider immediately if nausea or difficulty breathing occurs.

D Rationale: Difficulty breathing is an indication of acute decompensated heart failure and suggests that the medications are not achieving the desired effect. Nausea is an indication of digoxin toxicity and should be reported so that the provider can assess the patient for toxicity and adjust the digoxin dose, if necessary. Digoxin toxicity is potentiated by hypokalemia, rather than hyperkalemia. Patients should be taught to check their pulse daily before taking the digoxin and, if the pulse is less than 60, to call their provider before taking the digoxin. Diuretics should be taken early in the day to avoid sleep disruption. Cognitive Level: Application Text Reference: p. 835 Nursing Process: Implementation NCLEX: Health Promotion and Maintenance

18. A 55-year-old patient with inoperable coronary artery disease and end-stage heart failure asks the nurse whether heart transplant is a possible therapy. The nurse's response to the patient will be based on the knowledge that a. heart transplants are experimental surgeries that are not covered by most insurance. b. the patient is too old to be placed on the transplant list. c. the diagnoses and symptoms indicate that the patient is not an appropriate candidate. d. candidacy for heart transplant depends on many factors.

D Rationale: Indications for a heart transplant include inoperable coronary artery disease and refractory end-stage heart failure, but other factors such as coping skills, family support, and patient motivation to follow the rigorous post-transplant regimen are also considered. Heart transplants are not considered experimental; rather, transplantation has become the treatment of choice for patients who meet the criteria. The patient is not too old for a transplant. The patient's diagnoses and symptoms indicate that the patient may be an appropriate candidate for a heart transplant. Cognitive Level: Comprehension Text Reference: p. 837 Nursing Process: Planning NCLEX: Health Promotion and Maintenance

Safe and Effective Care Environment 34. Which action could the nurse delegate to unlicensed assistive personnel (UAP) trained as electrocardiogram (ECG) technicians working on the cardiac unit? a. Select the best lead for monitoring a patient with an admission diagnosis of Dressler syndrome. b. Obtain a list of herbal medications used at home while admitting a new patient with pericarditis. c. Teach about the need to monitor the weight daily for a patient who has hypertrophic cardiomyopathy. d. Check the heart monitor for changes in rhythm while a patient who had a valve replacement ambulates.

D Under the supervision of registered nurses (RNs), UAP check the patient's cardiac monitor and obtain information about changes in heart rate and rhythm with exercise. Teaching and obtaining information about home medications (prescribed or complementary) and selecting the best leads for monitoring patients require more critical thinking and should be done by the RN. DIF: Cognitive Level: Apply (application) REF: 15 OBJ: Special Questions: Delegation TOP: Nursing Process: Planning MSC:

What should the nurse recognize as an indication for the use of dopamine in the care of a patient with heart failure? Acute anxiety Hypotension and tachycardia Peripheral edema and weight gain Paroxysmal nocturnal dyspnea (PND)

Hypotension and tachycardia Dopamine is a β-adrenergic agonist whose inotropic action is used for treatment of severe heart failure accompanied by hemodynamic instability. Such a state may be indicated by tachycardia accompanied by hypotension. PND, anxiety, edema, and weight gain are common signs and symptoms of heart failure, but these do not necessarily warrant the use of dopamine.

A patient has developed left heart failure. Which of the following symptoms might have precipitated this condition?

Myocardial infarction

A 20-year-old college student being treated for a kidney infection developed a temperature of 104ºF in spite of treatment with antibiotics. Her pulse was high, her blood pressure was low, and her skin was hot, dry, and flushed. The nurse knows that this patient most likely is experiencing which of the following types of shock?

Septic

Heart failure patients are at risk for atrial fibrillation.

TRUE

Severe stenosis of the pulmonary semilunar valve can cause right-sided HF.

TRUE

The pulmonary capillary wedge pressure represents the left ventricular pressures.

TRUE

You might see hyperglycemia and DIC in septic shock.

TRUE

What describes the action of the natrieuretic peptides and nitric oxide in thier counterregulatory processes in response to heart failure?

Vasodilation and decreased blood pressure

A patient with newly discovered high BP has an average reading of 158/98 mm Hg after 3 months of exercise and diet modifications. Which management strategy will be a priority for this patient? a. Medication will be required because the BP is still not at goal. b. BP monitoring should continue for another 3 months to confirm a diagnosis of hypertension. c. Lifestyle changes are less important, since they were not effective, and medications will be started. d. More vigorous changes in the patient's lifestyle are needed for a longer time before starting medications.

a

A patient's ECG rhythm strip is irregular. Which method does the nurse use for an accurate assessment? a. 6-second strip method b. Memory emthod c. Big block method d. Commerical ECG rate ruler

a

The health care provider prescribes sprinolactone for the patient with chronic heart failure, what diet modifications related to the use of this drug should the nurse include in the patient teaching

a Decrease in both sodium and potassium

A patient with PAD has a nursing diagnosis of ineffective peripheral tissue perfusion. What should be included in the teaching plan for this patient (select all that apply)? a. Keep legs and feet warm. b. Apply cold compresses when the legs become swollen. c. Walk at least 30 minutes per day to the point of discomfort. d. Use nicotine replacement therapy as a substitute for smoking. e. Inspect lower extremities for pulses, temperature, and any injury.

a, c, e

When caring for a patient with infective endocarditis, the nurse will assess the patient for which of the following vascular manifestations (select all that apply)? a. Osler's nodes b. Janeway's lesions c. Splinter hemorrhages d. Subcutaneous nodules e. Erythema marginatum lesions

a. Osler's nodes b. Janeway's lesions c. Splinter hemorrhages Rationale: Osler's nodes, Janeway's lesions, and splinter hemorrhages are all vascular manifestations of infective endocarditis. Subcutaneous nodules and erythema marginatum lesions occur with rheumatic fever.

Priority Decision: A patient has atrial fibrillation and develops an acute arterial occlusion at the iliac artery bifurcation. What are the six Ps of acute arterial occlusion the nurse may assess in this patient that require immediate notification of the physician? a. b. c. d. e. f

a. Pain b. Pallor c. Pulselessness d. Paresthesia e. Paralysis f. Poikilothermia

A patient with a recent diagnosis of heart failure has been prescribed furosemide (Lasix) in an effort to physiologically do what for the patient? a. Reduce preload. b. Decrease afterload. c. Increase contractility. d. Promote vasodilation.

a. Reduce preload. Diuretics such as furosemide are used in the treatment of HF to mobilize edematous fluid, reduce pulmonary venous pressure, and reduce preload. They do not directly influence afterload, contractility, or vessel tone.

Which is a priority nursing intervention for a patient during the acute phase of rheumatic fever? a. administration of antibiotics as ordered b. management of pain with opioid analgesics c. encouragement of fluid intake for hydration d. performance of frequent, active range-of motion exercises

a. administration of antibiotics as ordered Rationale: The primary goal of managing a patient with acute rheumatic fever is to control and eradicate the infecting organism. The nurse should administer antibiotics as ordered to treat the streptococcal infection and teach the patient that completion of the full course of antibiotic therapy is essential for successful treatment.

A patient has an admitting diagnosis of acute left-sided infective endocarditis. The nurse explains to the patient that this diagnosis is best confirmed with a. blood cultures b. a complete blood count c. a cardiac catheterization d. a transesophageal echocardiogram

a. blood cultures Rationale: Although a complete blood cell count (CBC) will reveal a mild leukocytosis and erythrocyte sedimentation rates (ESRs) will be elevated in patients with infective endocarditis, these are nonspecific findings, and blood cultures are the primary diagnostic tool for infective endocarditis. Transesophageal echocardiograms can identify vegetations on valves but are used when blood cultures are negative, and cardiac catheterizations are used when surgical intervention is being considered.

The patient is in third-degree heart block (complete heart block) and is symptomatic. The treatment for this patient is which of the following? (Select all that apply.) a. transcutaneous pacemaker. b. atropine IV. c. temporary transvenous pacemaker. d. permanent pacemaker. e. amiodarone IV.

a. transcutaneous pacemaker. c. temporary transvenous pacemaker. d. permanent pacemaker.

According to the Vaughn-Williams classification of antidysrhythmics, which class II drug controls dysrhythmias associated with excessive beta-adrenergic stimulation? b. Propranolol hydrochloride ( Inderal)

b

How does the nurse interpret the measurement of the PR interval when the interval to be measured is six small boxes on the ECG strip? a. Atrium is taking longer to repolarize b. Longer-than-normal impulse time from the SA node to the ventricles is shown. c. There is a problem with the length of time the ventricles are depolarizing d. This is the normal length fo time for the P-R interval.

b

The bedside cardiac monitor of a postoperative patient who becomes confused shows sinus rhythm, but there is no palpable pulse. How does the nurse interpret these findings? a. ECG monitor artifact or dysfunction b. Pulseless electrical activity with inadequate perfusion c. A paced rhythm with hypotension d. Idioventricular rhythm as seen in the dying heart

b

The heart monitor of a patient shows a rhythm that appears as a wandering or fuzzy baseline. What is the priority action for the nurse? a. Immediately obtain a 12-lead ECG to assess the actual rhythm b. Assess the patient to differentiate artifact from actual lethal rhythms c. Check to see if the patient has a DNR order d. Ask the patient care technician to take vital signs on the patient

b

The nurse discovers a patient is unconscious and without palpable pulses and immediately initiates CPR. For what reason is CPR started on this patient? a. To identify the underlying heart rhythm b. For the rapid return of a pulse, blood pressure, and consciousness c. To prevent rib fractures or lacerations of the liver and spleen d. To mimic cardiac function until the defibrillator arrives

b

The nurse has four patients on telemetry monitors and is analyzing the ECG rhythm strips a. Analyze the P waves b. Determine the heart rate c. Measure the QRS duration d. Measure teh PR interval

b

The nurse hears in report that a patient has sinus arrhythmia. In order to validate that this is associated with the changes in intrathoracic pressure, what does the nurse do next? a. Count the respiratory and pulse rate at rest and then count both rates after moderate exertion b. Observe that the heart rate increases slightly during inspiration and decreases slightly during exhalation. c. Ask the patient to hold the breath and take an apical pulse; then have the patient resume normal breathing. d. Have the patient take a deep breath and count the patient apical pulse rate while the patient slowly exhales

b

The nurse is assessing a patient's ECG rhythm strip and checking the regularity of the atrial rhythm. What is the correct technique? a. Place one caliper point on a QRS complex; place the other point on the precise spot on the next QRS complex b. Place one caliper point on a P wave; place the other point on the precise spot on the next P Wave c. Place one caliper point at the beginning of the P wave; place the other point at the end of the P-R segment d. Place one caliper point at the beginning of the QRS complex; place and the other point where the S-T segment begins

b

The nurse is assisting the provider to perform temporary pacing for a patient who has atropine-refractory symptomatic bradycardia. What is the desired outcome for this patient as evidenced by the cardiac monitor? a. No spike, but a complete QRS complex indicating atrial depolarization b A spike followed by a QRS complex indicating ventricular depolarization c. Two spikes, followed by a QRS complex indiccating ventricular depolarization d. A spike before and after a QRS complex indicating atrial depolarization

b

The nurse is reviewing a patient's ECG and interprets a wide distorted QRS complex of 0.14 second followed by a P wave. What does this finding indicate? a. Wide but normal complex, and no cause for concern b. Premature ventricular contraction c. Problem with the speed set on the ECG machine d. Delayed time for the electrical impulse through the ventricles

b

The nurse is teaching a community group how to use an AED. What is the first step for using the AED that the nurse teaches? a. Rescuer presses the "analyze" button on the machine b. Place the patient on a firm, dry surface c. Rescuer stops CPR and directs anyone present to move away d. Place two large adhesive-patch electrodes on the patient's chest

b

The patient at highest risk for venous thromboembolism (VTE) is a. a 62-year-old man with spider veins who is having arthroscopic knee surgery. b. a 32-year-old woman who smokes, takes oral contraceptives, and is planning a trip to Europe. c. a 26-year-old woman who is 3 days postpartum and received maintenance IV fluids for 12 hours during her labor. d. an active 72-year-old man at home recovering from transurethral resection of the prostate for benign prostatic hyperplasia.

b

The patient is diagnosed with a superficial vein thrombosis (SVT). Which characteristic should the nurse know about SVT? a. Embolization to lungs may result in death. b. Clot may extend to deeper veins if untreated. c. Vein is tender to pressure and there is edema. d. Typically found in the iliac, inferior, or superior vena cava.

b

What does stimulation of the sympathetic nervous system produce? a. Delayed electrical impulse that causes hypotension and bradypnea b. Contractility and dilation of coronary vessels and increased heart rate. c. Virtually no effect on the ventricles of the heart or vital signs d. A slowed AV conduction time that results in a slow heart rate

b

Which drug for symptomatic bradycardia does the nurse prepare to administer to a patient with bradydysrhythmia? a. Epinephrine b. Atropine c. Calcium d. Lidocaine

b

A patient was just diagnosed with acute arterial ischemia in the left leg secondary to atrial fibrillation. Which early clinical manifestation must be reported to the physician immediately to save the patient's limb? a. Paralysis Incorrect b. Paresthesia c. Cramping d. Referred pain

b The physician must be notified immediately if any of the six Ps of acute arterial ischemia occur to prevent ischemia from quickly progressing to tissue necrosis and gangrene. The six Ps are paresthesia, pain, pallor, pulselessness, and poikilothermia, with paralysis being a very late sign indicating the death of nerves to the extremity.

Which assessment findings of the left lower extremity will the nurse identify as consistent with arterial occlusion (select all that apply.)? a. Edematous b. Cold and mottled c. Complaints of paresthesia d. Pulse not palpable with Doppler e. Capillary refill less than three seconds f. Erythema and warmer than right lower extremity

b, c, d

Which are clinical findings in a person with an acute lower extremity VTE (select all that apply)? a. Pallor and coolness of foot and calf b. Mild to moderate calf pain and tenderness c. Grossly diminished or absent pedal pulses d. Unilateral edema and induration of the thigh e. Palpable cord along a superficial varicose vein

b, d

A patient with chronic heart failure has atrial fibrillation and a left ventricular ejection fraction (LVEF) of 18%. To decrease the risk of complications from these conditions, what drug does the nurse anticipate giving? a. Diuretic b. Anticoagulant c. β-Adrenergic blocker d. Potassium supplement

b. Thrombus formation occurs in the heart when the chambers do not contract normally and empty completely. Both atrial fibrillation and very low left ventricular output (LVEF <20%) lead to thrombus formation, which is treated with anticoagulants to prevent the release of emboli into the circulation as well as antidysrhythmics or cardioversion to control atrial fibrillation.

The unit is very busy and short staffed. What could be delegated to the unlicensed assistive personnel (UAP)? a. Administer antihypertensive medications to stable patients. b. Obtain orthostatic blood pressure (BP) readings for older patients. c. Check BP readings for the patient receiving IV enalapril (Vasotec). d. Teach about home BP monitoring and use of automatic BP monitoring equipment

b. Unlicensed assistive personnel (UAP) may check postural changes in BP as directed. The licensed practical nurse (LPN) may administer antihypertensive medications to stable patients. The RN must monitor the patient receiving IV enalapril (Vasotec), as he or she is in a hypertensive crisis. The RN must also do the teaching related to home BP monitoring.

The normal rate for the SA node when the patient is at rest is: a. 40 to 60 beats per minute. b. 60 to 100 beats per minute. c. 20 to 40 beats per minute. d. more than100 beats per minute.

b. 60 to 100 beats per minute.

25. When caring for a patient on the first postoperative day after an abdominal aortic aneurysm repair, which assessment finding is most important for the nurse to communicate to the health care provider? a. Presence of flatus b. Loose, bloody stools c. Hypoactive bowel sounds d. Abdominal pain with palpation

b. Loose, bloody stools

The nurse conducts a complete physical assessment on a patient admitted with infective endocarditis. Which of the following findings are significant? a. Respiratory rate of 18 and heart rate of 90 b. Regurgitant murmur at the mitral valve area c. Heart rate of 94 and capillary refill time of 2 seconds d. Point of maximal impulse palpable in fourth intercostal space

b. Regurgitant murmur at the mitral valve area Rationale: A regurgitant murmur would indicate valvular disease, which can be a complication of endocarditis. All the other findings are within normal limits.

After advanced cardiac life support (ACLS) is performed, a patient who experienced VF has a return of spontaneous circulation. To protect the patient's nervous system, which intervention does the nurse anticipate with be performed? a. Neurologic checks every 4 hours b. Administration of IV mannitol c. Application of a cooling blanket d. Continuous ECG monitoring

c

What is the normal measurement of the PR interval in an ECG? a. Less than 0.11 second b. 0.06 to 0.10 second c. 0.12 to 0.20 second d. 0.16 to 0.26 second

c

The patient who is being admitted has had a history of uncontrolled hypertension. High SVR is most likely to cause damage to which organ? a. Brain b. Retina c. Heart d. Kidney

c. The increased systemic vascular resistance (SVR) of hypertension directly increases the workload of the heart and heart failure occurs when the heart can no longer pump effectively against the increased resistance.

A mechanical prosthetic valve is most likely to be preferred over a biologic valve for valve replacement in a a. 41-year-old man with peptic ulcer disease b. a 22-year-old woman who desires to have children c. a 35-year-old man with a history of seasonal asthma d. 62-year-old woman with early Alzheimer's disease

c. a 35-year-old man with a history of seasonal asthma Rationale: Mechanical prosthetic valves require long-term anticoagulation, and this is a factor in making a decision about the type of valve to use for replacement. Patients who cannot take anticoagulant therapy, such as women of childbearing age, patients at risk for hemorrhage, or patient who may not be compliant with anticoagulation therapy, may be candidates for the less durable biologic valves.

The nurse notices that the patient has a first-degree AV block. Everything else about the rhythm is normal. The nurse should: a. prepare to place the patient on a transcutaneous pacemaker. b. give the patient atropine to shorten the PR interval. c. monitor the rhythm and patient's condition. d. give the patient an antiarrhythmic medication.

c. monitor the rhythm and patient's condition.

When assessing the 12-lead electrocardiogram (ECG) or a rhythm strip, it is helpful to understand that the electrical activity is viewed in relation to the positive electrode of that particular lead. When an electrical signal is aimed directly at the positive electrode, the inflection will be: a. negative. b. upside down. c. upright. d. equally positive and negative.

c. upright.

What does the T wave on an ECG represent? A. Ventricular depolarization b. Atrial repolarization c. Atrial depolarization d. Ventricular repolarization

d

The acronym FACES is used to help teach patients to identify early symptoms of heart failure. What does this acronym mean? a. Frequent activity leads to cough in the elderly and swelling b. Factors of risk: activity, cough, emotional upsets, salt intake c. Follow activity plan, continue exercise, and know signs of problems d. Fatigue, limitation of activities, chest congestion/cough, edema, shortness of breath

d. FACES is used to teach patients to identify early HF symptoms. F = Fatigue; A = Activity limitations; C =Chest congestion/cough; E = Edema; S = Shortness of breath. The other options are not correct.

The nurse is working on the night shift when she notices sinus bradycardia on the patient's cardiac monitor. The nurse should: a. give atropine to increase heart rate. b. begin transcutaneous pacing of the patient. c. start a dopamine infusion to stimulate heart function. d. assess for hemodynamic instability.

d. assess for hemodynamic instability.

6. A patient at the clinic says, I have always taken a walk after dinner, but lately my leg cramps and hurts after just a few minutes of starting. The pain goes away after I stop walking, though. The nurse should a. check for the presence of tortuous veins bilaterally on the legs. b. ask about any skin color changes that occur in response to cold. c. assess for unilateral swelling, redness, and tenderness of either leg. d. assess for the presence of the dorsalis pedis and posterior tibial pulses.

d. assess for the presence of the dorsalis pedis and posterior tibial pulses.

An older adult patient with chronic heart failure (HF) and atrial fibrillation asks the nurse why warfarin has been prescribed to continue at home. What is the best response by the nurse? "The medication prevents blood clots from forming in your heart." "The medication dissolves clots that develop in your coronary arteries." "The medication reduces clotting by decreasing serum potassium levels." "The medication increases your heart rate so that clots do not form in your heart."

"The medication prevents blood clots from forming in your heart." Chronic HF causes enlargement of the chambers of the heart and an altered electrical pathway, especially in the atria. When numerous sites in the atria fire spontaneously and rapidly, atrial fibrillation occurs. Atrial fibrillation promotes thrombus formation within the atria with an increased risk of stroke and requires treatment with cardioversion, antidysrhythmics, and/or anticoagulants. Warfarin is an anticoagulant that interferes with hepatic synthesis of vitamin K-dependent clotting factors.

An older adult patient with chronic heart failure (HF) and atrial fibrillation asks the nurse why warfarin has been prescribed to continue at home. What is the best response by the nurse? "The medication prevents blood clots from forming in your heart." "The medication dissolves clots that develop in your coronary arteries." "The medication reduces clotting by decreasing serum potassium levels." "The medication increases your heart rate so that clots do not form in your heart."

"The medication prevents blood clots from forming in your heart." Chronic HF causes enlargement of the chambers of the heart and an altered electrical pathway, especially in the atria. When numerous sites in the atria fire spontaneously and rapidly, atrial fibrillation occurs. Atrial fibrillation promotes thrombus formation within the atria with an increased risk of stroke and requires treatment with cardioversion, antidysrhythmics, and/or anticoagulants. Warfarin is an anticoagulant that interferes with hepatic synthesis of vitamin K-dependent clotting factors.

The client is prescribed digoxin (Lanoxin) for treatment of HR. Which of the following statements by the client indicates the need for further teaching by the nurse? "I should not get short of breath anymore." "This drug will help my heart muscle pump less." "I may notice my heart rate decrease." "I may feel tired during early treatment."

"This drug will help my heart muscle pump less." Rationale: The ability to increase the strength of contractions is a characteristic of cardiac glycosides. It may result in a decrease in pulse. Initially the client may experience some fatigue. Symptoms of CHF, such as dyspnea, should improve.

captopril (Capoten)

-ACE inhibitor -monitor the BP for several hours after 1st dose -take on an empty stomach -avoid antacids, NSAIDs, caution with K+sparing diuretics -monitor creatinine, K+ levels, CBC, UA -report cough -hypotension may occur as a first dose effect, esp in volume depleted people

-sartan endings

-ARBs -monitor BP for several hours after 1st dose -take on an empty stomach -avoid antacids, NSAIDs, caution with K+sparing diuretics -monitor creatinine, K+ levels, CBC, UA -decreased problem with cough

Regarding heart failure, the nurse knows that which of the following statements are correct? Select all that apply.

-Aortic stenosis can cause left-sided failure. -Cardiomyopathy is a common cause of heart failure. -Compensated heart failure may be clinically asymptomatic.

The diagnosis is left-sided heart failure. The nurse knows that the most common causes of left-sided failure include which of the following? Select all that apply.

-Hypertension -Acute myocardial infarction

digitalis/digoxin (Lanoxin)

-cardiac glycosides -positive inotropic agent (increase force of heart contraction) -take apical pulse for full minute--note pattern, regularity before giving dose -monitor level: 0.5-2.0 -observe for signs of toxicity: confusion, visual changes, dysrhythmias, anorexia, N/V, fatigue, muscle weakness -notify HCP if <60

beta blockers

-carvedilol (Coreg), metoprolol (Toprol XL) -monitor BP and pulse initially and with each dose increase -teach patient: twice weekly- BP checks, pulse, activity tolerance, orthopnea -teach pt to take daily weights, report evidence of volume overload (increased weight, dyspnea, edema) -teach pt never to stop medication abruptly or without HCP advice -diabetics monitor glucose carefully -arise carefully -stopping therapy quickly may cause angina, dysrhythmias, or MI

thiazide diuretics

-hydrochlorothiazide, chlorothiazide -inexpensive -measure I&O, daily weight -assess for edema -monitor electrolytes, glucose, BUN, Cr, Uric acid, bili, Mg, Chol -give in the morning with food -monitor for digitalis toxicity

Nitro-Dur

-nitrate -assess for HA, dizziness,hypotension, tachycardia -HA diminishes with tolerance

isosorbide

-nitrate -observe for postural hypotension -supervise ambulation -dizziness -avoid erectile dysfunction meds

morphine

-reduces preload and after load -decreases respirations and anxiety, which improves gas exchange and decreases dyspnea -decreases oxygen demand

potassium-sparing diuretics

-spironolactone (Aldactone) -assess potassium level -monitor digitalis level if given also -give in the morning -may be given with a loop diuretic

The nurse is obtaining a health history from a patient with hypertension. Nonmodifiable risk factors for the development of hypertension include which of these? Select all that apply. 1. Age 65 years 2. Excessive dietary sodium 3. African American ethnicity 4. Excessive alcohol consumption 5. A family history of hypertension

1. Age 65 years 3. African American ethnicity 5. A family history of hypertension Nonmodifiable risk factors for hypertension include increasing age, African American ethnicity, and a family history of hypertension. Consumption of excessive dietary sodium and excessive alcohol consumption are considered modifiable risk factors. Text Reference - p. 713

The nurse is taking the blood pressure of an older patient and compares the result to the previous reading. The nurse observes that the systolic blood pressure of the previous reading was 30 mm Hg higher than the reading the nurse just obtained. Which of these is a possible explanation? 1. An aucultatory gap may be present. 2. The patient's hypertension is improving. 3. The equipment the nurse used is not working properly. 4. The patient was more relaxed with the most recent blood pressure measurement.

1. An aucultatory gap may be present. Careful technique is important in assessing blood pressure in older adults. Some older people have a wide gap between the first Korotkoff sound and subsequent beats. This is called the auscultatory gap. Failure to inflate the cuff high enough may result in underestimating systolic blood pressure. Text Reference - p. 725

The nurse is creating a plan of care for a patient with a new diagnosis of hypertension. Which is a potential nursing diagnosis for the patient taking antihypertensive medications? Select all that apply. 1. Anxiety 2. Constipation 3. Impaired memory 4. Sexual dysfunction 5. Urge urinary incontinence

1. Anxiety 4. Sexual dysfunction Nursing diagnoses associated with patients taking medications for hypertension include anxiety (related to complexity of management regimen) and sexual dysfunction (related to side effects of antihypertensive drugs). Constipation, impaired memory, and urge urinary incontinence are not side effects of antihypertensive drugs. Text Reference - p. 720

A patient whose blood pressure is 200/120 mm Hg is diagnosed with hypertensive crisis. The patient receives a prescription for clonidine. The nurse instructs the patient to avoid engaging in hazardous activities like operating machinery. What is the reason for this instruction? 1. Clonidine causes drowsiness. 2. Clonidine causes orthostatic hypotension. 3. Clonidine causes a rapid decrease in BP. 4. Clonidine causes rebound hypertension

1. Clonidine causes drowsiness. Clonidine is an adrenergic inhibitor that reduces sympathetic outflow from the central nervous system. It lowers BP by reducing peripheral sympathetic tone, dilating the blood vessels, and decreasing the systemic vascular resistance. Drowsiness is a side effect of the drug, and the patient is advised to avoid engaging in hazardous activities, because this can increase the risk of injury. The drug also causes orthostatic hypotension, so the patient is advised to change position slowly. The drug does not cause a rapid decrease in BP. The drug can cause rebound hypertension if discontinued abruptly. Text Reference - p. 717

1. Assessment of an IV cocaine user with infective endocarditis should focus on which signs and symptoms (select all that apply)? a. Retinal hemorrhages b. Splinter hemorrhages c. Presence of Osler's nodes d. Painless nodules over bony prominences e. Painless erythematous macules on the palms and soles

1. Correct answers: a, b, c, e Rationale: Clinical manifestations of infective endocarditis may include hemorrhagic retinal lesions (Roth's spots), splinter hemorrhages (black, longitudinal streaks) that may occur in the nail beds, Osler's nodes (painful, tender, red or purple, pea-size lesions) on the fingertips or toes, and Janeway's lesions (flat, painless, small, red spots) seen on the fingertips, palms, soles of feet, and toes.

A patient is prescribed lisinopril for the treatment of hypertension. The patient asks about side effects of this medication. Which side effects should the nurse include? Select all that apply. 1. Cough 2. Edema 3. Dizziness 4. Impotence 5. Hypotension 6. Muscle stiffness

1. Cough 3. Dizziness 5. Hypotension Cough, dizziness, and hypotension are side effects of angiotensin-converting enzyme (ACE) inhibitors. Peripheral edema is a side effect of calcium channel blockers. Impotence is a side effect of thiazide diuretics, aldosterone receptor blockers, central-acting alpha-adrenergic antagonists, peripheral-acting alpha-adrenergic antagonists, beta-adrenergic blockers, and mixed alpha 1 and beta 1 blockers. Muscle stiffness is not associated with an ACE inhibitor. Test-Taking Tip: Avoid spending excessive time on any one question. Most questions can be answered in one to two minutes. Text Reference - p. 724

A patient is being discharged from the hospital. The primary health care provider prescribes propranolol for hypertension. Which instruction should the nurse include in the patient's discharge teaching plan? 1. Do not stop taking abruptly. 2. Take initial doses at bedtime. 3. Monitor for peripheral edema. 4. Take with orange juice.

1. Do not stop taking abruptly. Patients should not stop taking this medication abruptly, because this may cause rebound hypertension. The initial dose of alpha-1 adrenergic blockers should be taken at bedtime because of the possible profound orthostatic hypotension with syncope within 90 minutes after the initial dose. Calcium channel blockers may cause peripheral edema. Beta blockers are not potassium wasting, so it is not necessary to take them with orange juice. Text Reference - p. 725

The nurse teaches a patient with hypertension that symptoms of uncontrolled hypertension may include which of the following? Select all that apply. 1. Fatigue 2. Dizziness 3. Palpitations 4. Shortness of breath 5. Cluster headaches

1. Fatigue 2. Dizziness 3. Palpitations Uncontrolled hypertension may result in fatigue, dizziness, and palpitations. Cluster headaches and shortness of breath do not occur with uncontrolled hypertension. Test-Taking Tip: Be alert for details about what you are being asked to do. In this question type, you are asked to select all options that apply to a given situation or client. All options likely relate to the situation, but only some of the options may relate directly to the situation. Text Reference - p. 713

The nurse teaches a patient with hypertension that uncontrolled hypertension may damage organs in the body primarily by which mechanism? 1. Hypertension promotes atherosclerosis and damage to the walls of the arteries. 2. Hypertension causes direct pressure on organs, resulting in necrosis and replacement of cells with scar tissue. 3. Hypertension causes thickening of the capillary membranes, leading to hypoxia of organ systems. 4.Hypertension increases blood viscosity, which contributes to intravascular coagulation and tissue necrosis distal to occlusions.

1. Hypertension promotes atherosclerosis and damage to the walls of the arteries. Hypertension is a major risk factor for the development of atherosclerosis by mechanisms not yet fully known. However, once atherosclerosis develops, it damages the walls of arteries and reduces circulation to target organs and tissues.

A nurse is preparing to measure the blood pressure of a patient who is lying down on a bed. What technique should the nurse use to ensure that the BP reading is accurate? 1. Measure the BP twice, waiting one minute between measurements. 2. Rest the patient's arms on the bed during the BP measurement. 3. Measure the BP in both arms and record the lowest reading. 4. Place the BP cuff on the forearm when measuring the BP.

1. Measure the BP twice, waiting one minute between measurements. When recording BP, two measurements should be taken one minute apart. This allows the blood to drain from the arm and prevents inaccurate readings. If the patient is in a lying down position, the arm should be placed on a pillow so that it is at the heart level. Atherosclerosis in the subclavian artery may result in a falsely low BP in the affected side; therefore, the arm which has the highest recording of the BP should be used for further measurements. The upper arm, not the forearm, is the preferred site of BP cuff placement due to its accuracy of recordings. Text Reference - p. 723

A nurse works in a medical unit. The nurse has assessed the patients and planned care for them. Which activities can be delegated to unlicensed assistive personnel (UAP)? Select all that apply. 1. Report high or low BP readings to the registered nurse. 2. Make appropriate referrals to other health care professionals. 3. Teach patients about lifestyle management and medication use. 4. Check for postural changes in BP. 5. Assess patients for hypertension risk factors and develop risk modification plans.

1. Report high or low BP readings to the registered nurse. 4. Check for postural changes in BP. Reporting high or low BP readings and checking for postural changes in BP are repetitive activities and do not require nursing judgment. Therefore, these activities can be delegated to unlicensed assistive personnel. Making appropriate referrals requires understanding of the collaborative care and judgment regarding the requirement of the referrals; this activity cannot be delegated and is the role of a registered nurse. Patient education about lifestyle management and medication use requires sound knowledge; therefore, this activity should be performed by the nurse. Assessment and development of risk modification plans requires assessment and planning skills; this activity should not be delegated and should be performed by the nurse. Text Reference - p. 724

A nurse is preparing discharge teaching for a patient with orthostatic hypotension. Which instructions should be a part of the discharge plan? Select all that apply. 1. Rise slowly from a supine to sitting position. 2. Avoid sleeping with the head elevated. 3. Lie down or sit if dizziness occurs. 4. Do not stand still for prolonged periods. 5. Perform leg exercises to increase venous return.

1. Rise slowly from a supine to sitting position. 3. Lie down or sit if dizziness occurs. 4. Do not stand still for prolonged periods. 5. Perform leg exercises to increase venous return. Orthostatic hypotension is a condition in which there is a decrease in blood pressure upon rising to a standing position from a lying down or sitting position. The patient should be instructed to rise slowly from the sitting and lying down positions and to move only when no dizziness occurs. The patient should sit or lie down if there is dizziness. This prevents the risk of falling. Standing still for prolonged periods may cause venous stasis and worsen hypotension. Doing leg exercises helps to increase venous return to the heart and lowers blood pressure. Sleeping with the head elevated helps to keep the blood flow to the brain uniform and prevents orthostatic hypotension. Text Reference - p. 723

The nurse is teaching the male patient about the most common side effects of antihypertensive medications. Which information would the nurse discuss with the patient? Select all that apply. 1. Sexual dysfunction 2. Resistant hypertension 3. Orthostatic hypotension 4. Frequent voiding and dry mouth 5. Rebound hypotension if the drug is stopped abruptly

1. Sexual dysfunction 3. Orthostatic hypotension 4. Frequent voiding and dry mouth Reduced libido or erectile dysfunction are examples of sexual dysfunction as a side effect of antihypertensive medications. Alteration of the autonomic nervous system mechanism by antihypertensive medications leads to orthostatic hypotension. Diuretics are one class of medications for treatment of hypertension that cause frequent urination and dry mouth. "Resistant hypertension" is a term used to describe failure to reach desired blood pressure (BP) in the patient who takes multiple antihypertensive medications. Rebound hypertension results from abrupt stopping of antihypertensive medication use. Text Reference - p. 721

The nurse is preparing a presentation on complications of hypertension. Which information would the nurse include? Select all that apply. 1. Stroke as a result of carotid artery atherosclerosis 2. Heart failure as a result of increased heart contractility 3. Blurred vision or loss of vision secondary to retinal damage 4. Right ventricular hypertrophy as a result of increased workload 5. Coronary artery disease caused by an increase in the elasticity of arterial walls

1. Stroke as a result of carotid artery atherosclerosis 3. Blurred vision or loss of vision secondary to retinal damage Embolic stroke may be a result of cerebral blood flow obstruction by a portion of atherosclerotic plaque or a blood clot formed in the carotid arteries. Hypertension leads to retinal damage that is manifested by blurred vision or loss of vision and retinal hemorrhage. Heart failure is a result of decreased heart contractility along with decreased stroke volume and cardiac output. Hypertension leads to increased cardiac workload that causes left ventricular hypertrophy. Coronary artery disease is caused by decreased elasticity of arterial walls and narrowing of the lumen. Text Reference - p. 713

Which test result would indicate the presence of target organ damage resulting from uncontrolled hypertension? 1. Urine protein 3+ 2. Blood urea nitrogen (BUN) 18 mg/dL 3. Uric acid 8.2 mg/dL 4. Triglycerides 144 mg/dL

1. Urine protein 3+ Urine protein should not be present. This increased level indicates target organ damage to the kidneys. The BUN is normal, the elevated uric acid level indicates gout, and the triglyceride level is normal. Test-Taking Tip: Avoid taking a wild guess at an answer. However, should you feel insecure about a question, eliminate the alternatives that you believe are definitely incorrect, and reread the information given to make sure you understand the intent of the question. This approach increases your chances of randomly selecting the correct answer or getting a clearer understanding of what is being asked. Although there is no penalty for guessing on the NCLEX examination, the subsequent question will be based, to an extent, on the response you give to the question at hand; that is, if you answer a question incorrectly, the computer will adapt the next question accordingly based on your knowledge and skill performance on the examination up to that point. Text Reference - p. 714

1. What accurately describes electrocardiographic (ECG) monitoring? a. Depolarization of the cells in the ventricles produces the T wave on the ECG. b. An abnormal cardiac impulse that arises in the atria, ventricles, or atrioventricular (AV) junction can create a premature beat that is known as an artifact. c. Lead placement for V1 includes one lead each for right arm, right leg, left arm, and left leg with the fifth lead on the fourth intercostal space to the right of the sternal border. d. If the sinoatrial (SA) node fails to discharge an impulse or discharges very slowly, a secondary pacemaker in the AV node is able to discharge at a rate of 30 to 40 times per minute.

1. c. The V1 leads are placed toward each limb and centrally at the fourth intercostal space to the right of the sternal border. Depolarization of the ventricular cells produces the QRS interval on the ECG. The T wave is produced by repolarization of the ventricular cells. Abnormal cardiac impulses from the atria, ventricles, or atrioventricular (AV) junction create ectopic beats. Artifacts are seen with leads or electrodes that are not secure, with muscle activity or electrical interference. The rate produced by the AV node pacing in a junctional escape rhythm is 40 to 60 bpm. If the His-Purkinje system is blocked, the heart rate is 20 to 40 bpm.

a. Although obtaining the ECG, chest radiograph, and blood work are all important, the nurse's priority action should be to relieve the crushing chest pain. Therefore, administering morphine sulfate is the priority action.

10] A 60-year-old male client comes into the emergency department with a complaint of crushing substernal chest pain that radiates to his shoulder and left arm. The admitting diagnosis is acute myocardial infarction (MI). Immediate admission orders include oxygen by nasal cannula at 4L/minute, blood work, a chest radiograph, a 12-lead electrocardiogram (ECG), and 2mg of morphine sulfate given IV. The nurse should first: a. Administer the morphine b. Obtain a 12-lead ECG c. Obtain the blood work d. Order the chest radiograph

11. The nurse is evaluating the telemetry ECG rhythm strip. How should the nurse document the distorted P wave causing an irregular rhythm? a. Atrial flutter b. Sinus bradycardia c. Premature atrial contraction (PAC) d. Paroxysmal supraventricular tachycardia (PSVT)

11. c. The premature atrial contraction (PAC) has a distorted P wave that may feel like a skipped beat to the patient. Atrial flutter is an atrial tachydysrhythmia with recurring, regular, saw-toothed flutter waves from the same focus in the right or possibly left atrium. Sinus bradycardia has a regular heart rate less than 100 bpm. Paroxysmal supraventricular tachycardia (PSVT) starts in an ectopic focus above the bundle of His and may be triggered by PAC. If seen, the P wave may have an abnormal shape and has a spontaneous start and termination with a rate of 150 to 220 bpm.

15. A patient with no history of heart disease has a rhythm strip that shows an occasional distorted P wave followed by normal AV and ventricular conduction. What should the nurse question the patient about? a. The use of caffeine b. The use of sedatives c. Any aerobic training d. Holding of breath during exertion

15. a. A distorted P wave with normal conduction of the impulse through the ventricles is characteristic of a premature atrial contraction. In a normal heart, this dysrhythmia is frequently associated with emotional stress or the use of caffeine, tobacco, or alcohol. Sedatives rarely slow the heart rate (HR). Aerobic conditioning and holding of breath during exertion (Valsalva maneuver) often cause bradycardia.

16. Priority Decision: A patient's rhythm strip indicates a normal HR and rhythm with normal P waves and QRS complexes, but the PR interval is 0.26 second. What is the most appropriate action by the nurse? a. Continue to assess the patient. b. Administer atropine per protocol. c. Prepare the patient for synchronized cardioversion. d. Prepare the patient for placement of a temporary pacemaker.

16. a. A rhythm pattern that is normal except for a prolonged P-R interval is characteristic of a first-degree heart block. First-degree heart blocks are not treated but are observed for progression to higher degrees of heart block. Atropine is administered for bradycardia. Synchronized cardioversion is used for atrial fibrillation with a rapid ventricular response or supraventricular tachycardia (SVT). Pacemakers are used for higher-degree heart blocks.

19. Which rhythm pattern finding is indicative of PVCs? a. A QRS complex ≥0.12 second followed by a P wave b. Continuous wide QRS complexes with a ventricular rate of 160 bpm c. P waves hidden in QRS complexes with a regular rhythm of 120 bpm d. Saw-toothed P waves with no measurable PR interval and an irregular rhythm

19. a. PVC is an ectopic beat that causes a wide, distorted QRS complex ≥0.12 second because the impulse is not conducted normally through the ventricles. Because it is premature, it precedes the P wave and the P wave may be hidden in the QRS complex, or the ventricular impulse may be conducted retrograde and the P wave may be seen following the PVC but the rhythm is not regular. Continuous wide QRS complexes with a ventricular rate between 150 and 250 bpm are seen in ventricular tachycardia, whereas saw-toothed P waves are characteristic of atrial flutter.

A nurse is providing care for a patient who continues to experience hypertension despite taking a calcium channel blocker daily. A diuretic has been prescribed. How does a diuretic help control blood pressure? Select all that apply. 1. It causes vasodilation. 2. It reduces plasma volume. 3. It promotes sodium and water excretion. 4. It reduces the vascular response to catecholamines. 5. It prevents extracellular calcium from moving into the cells.

2. It reduces plasma volume. 3. It promotes sodium and water excretion. 4. It reduces the vascular response to catecholamines. Diuretics are an important component of BP treatment. Diuretics tend to reduce the plasma volume by promoting excretion of sodium and water. The net result is a reduction in the circulating volume, which causes a decrease in the BP. Diuretics also reduce the vascular response to catecholamines. The blood vessels do not constrict in response to catecholamines; as a result, the BP is reduced. Diuretics do not cause vasodilation or prevent the movement of extracellular calcium into the cells; these effects are brought about by calcium channel blockers. Text Reference - p. 720

The nurse understands that which medication may increase the risk of hypoglycemia unawareness in a patient with diabetes mellitus? 1. Cardizem 2. Metoprolol 3. Prednisone 4. Hydromorphone

2. Metoprolol Metoprolol, a beta blocker, slows the heart rate. Tachycardia is a classic symptom of hypoglycemia; therefore patients must be made aware of failure of the heart rate to respond to decreasing blood sugars and should be instructed to look for other signs of hypoglycemia. Cardizem and hydromorphone will not affect blood sugars or signs/symptoms of hypoglycemia. Prednisone will increase, not decrease, blood sugar levels. Text Reference - p. 718

The nurse is obtaining data from a patient who has been on medication for hypertension and diabetes for four years. The patient has been experiencing blurred vision due to retinal damage caused by hypertension. What are the other manifestations of target organ disease? Select all that apply. 1. Pneumonia 2. Nocturia 3. Aneurysm 4. Transient ischemic attack 5. Anemia

2. Nocturia 3. Aneurysm 4. Transient ischemic attack Hypertension affects the kidneys; the earliest manifestation of renal disease is nocturia. Hypertension speeds up the process of atherosclerosis in the peripheral blood vessels, leading to aneurysms. Adequate control of blood pressure (BP) reduces the risk of transient ischemic attack. Pneumonia and anemia are not manifestations of target organ disease. Text Reference - p. 714

When teaching a patient about dietary management of stage 1 hypertension, which instruction is most appropriate? 1. Restrict all caffeine 2. Restrict sodium intake 3. Increase protein intake 4. Use calcium supplements

2. Restrict sodium intake The patient should decrease intake of sodium. This will help to control hypertension, which can be aggravated by excessive salt intake, which in turn leads to fluid retention. Caffeine and protein intake do not affect hypertension. Calcium supplements are not recommended to lower blood pressure. Text Reference - p. 712

When providing dietary instruction to a patient with hypertension, the nurse would advise the patient to restrict intake of which meat? 1. Broiled fish 2. Roasted duck 3. Roasted turkey 4. Baked chicken breast

2. Roasted duck Roasted duck is high in fat, which should be avoided by the patient with hypertension. Weight loss may slow the progress of atherosclerosis and overall cardiovascular disease risk. Broiled fish, roasted turkey, and baked chicken breast are lower in fat and are therefore acceptable in the diet. Test-Taking Tip: Once you have decided on an answer, look at the stem again. Does your choice answer the question that was asked? If the question stem asks "why," be sure the response you have chosen is a reason. If the question stem is singular, then be sure the option is singular, and the same for plural stems and plural responses. Many times, checking to make sure that the choice makes sense in relation to the stem will reveal the correct answer. Text Reference - p. 715

A patient has been asked to monitor blood pressure (BP) levels at home twice a day. What should the nurse teach the patient about measuring BP in the supine position? 1. Take at least two consecutive readings one after another. 2. Support the arm with a pillow during measurement. 3. Take the reading immediately after lying down. 4. Use the arm with the lowest BP for all future measurements.

2. Support the arm with a pillow during measurement. When measuring BP in a supine position, the patient should support the arm with a small pillow to raise the position of the hand to the level of the heart. Record the average pressure by taking two consecutive readings at least one minute apart; this allows the blood to drain from the arm and prevents inaccurate readings. The first reading should be taken after two to three minutes of rest in a supine position. If bilateral BP measurements are not equal, the patient should use the arm with the highest BP for all future measurements. Text Reference - p. 723

2. What accurately describes the PR interval (select all that apply)? a. 0.16 seconds b. <0.12 seconds c. 0.06 to 0.12 seconds d. 0.12 to 0.20 seconds e. Time of depolarization and repolarization of ventricles f. Measured from beginning of P wave to beginning of QRS complex

2. d, f. The expected PR interval is 0.12 to 0.20 seconds and is measured from the beginning of the P wave to the beginning of the QRS complex. The T wave is 0.16 seconds, the QRS interval is <0.12 seconds, the P wave is 0.06 to 0.12 seconds, and the QT interval is the time of depolarization and repolarization of the ventricles

20. In the patient experiencing ventricular fibrillation (VF), what is the rationale for using cardiac defibrillation? a. Enhance repolarization and relaxation of ventricular myocardial cells b. Provide an electrical impulse that stimulates normal myocardial contractions c. Depolarize the cells of the myocardium to allow the SA node to resume pacemaker function d. Deliver an electrical impulse to the heart at the time of ventricular contraction to convert the heart to a sinus rhythm

20. c. The intent of defibrillation is to apply an electrical current to the heart that will depolarize the cells of the myocardium so that subsequent repolarization of the cells will allow the SA node to resume the role of pacemaker. An artificial pacemaker provides an electrical impulse that stimulates normal myocardial contractions. Synchronized cardioversion involves delivery of a shock that is programmed to occur during the QRS complex of the ECG but this cannot be done during ventricular fibrillation because there is no normal ventricular contraction or QRS complex.

21. What action is included in the nurse's responsibilities in preparing to administer defibrillation? a. Applying gel pads to the patient's chest b. Setting the defibrillator to deliver 50 joules c. Setting the defibrillator to a synchronized mode d. Sedating the patient with midazolam (Versed) before defibrillation

21. a. In preparation for defibrillation the nurse should apply conductive materials (e.g., saline pads, electrode gel, defibrillator gel pads) to the patient's chest to decrease electrical impedance and prevent burns. For defibrillation, the initial shock is 200 joules with biphasic defibrillators and the synchronizer switch used for cardioversion must be turned off. Sedatives may be used before cardioversion if the patient is conscious but the patient in ventricular fibrillation is unconscious.

23. A patient with a sinus node dysfunction has a permanent pacemaker inserted. Before discharge, what should the nurse include when teaching the patient? a. Avoid cooking with microwave ovens. b. Avoid standing near antitheft devices in doorways. c. Use mild analgesics to control the chest spasms caused by the pacing current. d. Start lifting the arm above the shoulder right away to prevent a "frozen shoulder."

23. b. The patient should avoid standing near antitheft devices in doorways of department stores and libraries but walking through them at normal pace is fine. High-output electrical generators or large magnets, such as those used in magnetic resonance imaging (MRI), can reprogram pacemakers and should be avoided. Microwave ovens pose no problems to pacemaker function but the arm should not be raised above the shoulder for 1 week after placement of the pacemaker. The pacing current of an implanted pacemaker is not felt by the patient but an external pacemaker may cause uncomfortable chest muscle contractions.

d. The woman who is 65-years-old, over weight and has an elevated LDL is at greatest risk. Total cholesterol >200, LDL >100, HDL <40 in men, HDL <50 in women, men 45-years and older, women 55-years and older, smoking and obesity increase the risk of CAD. Atorvastatin is a medication to reduce LDL and decrease risk of CAD. The combination of postmenopausal, obesity and high LDL cholesterol places this client at greatest risk.

23] The nurse is assessing clients at a health fair. Which client is at greatest risk for coronary artery disease? a. a 32-year-old female with mitral valve prolapse who quit smoking 10 years ago. b. a 43-year-old male with a family history of CAD and cholesterol level of 158 c. A 56-year-old male with an HDL of 60 who takes atorvastatin (Lipitor) d. A 65-year-old female who is obese with an LDL of 188

24. Priority Decision: A patient on the cardiac telemetry unit goes into ventricular fibrillation and is unresponsive. Following initiation of the emergency call system (Code Blue), what is the next priority for the nurse in caring for this patient? a. Begin CPR. b. Get the crash cart. c. Administer amiodarone IV. d. Defibrillate with 360 joules.

24. a. Until the defibrillator is available, the patient needs CPR. Defibrillation is needed as soon as possible, so someone should bring the crash cart to the room. Defibrillation would be with 360 joules for monophasic defibrillators and 120 to 200 joules for biphasic defibrillators. Amiodarone is an antidysrhythmic that is part of the advanced cardiac life support (ACLS) protocol for ventricular fibrillation.

c. CRP is a marker of inflammation and is elevated in the presence of cardiovascular disease. The high sensitivity CRP (hs-CRP) is the blood test for greater accuracy in measuring the CRP to evaluate cardiovascular risk. The family history, post-menopausal age, LDL above optimum levels and elevated CRP place the client at risk of CAD. Statin medications can decrease LDL, whereas statins and aspirin can reduce CRP and decrease the risk of MI and stroke.

24] A 58-year-old female with a family history of CAD is being seen for her annual physical exam. Fasting lab test results include: Total cholesterol 198; LDL cholesterol 120; HDL cholesterol 58; Triglycerides 148; Blood sugar 102; and C-reactive protein (CRP) 4.2. The health care provider informs the client that she will be started on a statin medication and aspirin. The client asks the nurse why she needs to take these medications. Which is the best response by the nurse? a. "The labs indicate severe hyperlipidemia and the medications will lower your LDL, along with a low-fat diet." b. "The triglycerides are elevated and will not return to normal without these medications." c. "The CRP is elevated indicating inflammation seen in cardiovascular disease, which can be lowered by the medications ordered." d. "The medications are not indicated since your lab values are all normal."

27. A 54-year-old patient who has no structural heart disease has an episode of syncope. An upright tilt table test is performed to rule out neurocardiogenic syncope. The nurse explains to the patient that if neurocardiogenic syncope is the problem, the patient will experience what? a. No change in HR or BP b. Palpitations and dizziness c. Tachydysrhythmias and chest pain d. Marked bradycardia and hypotension

27. d. One of the most common causes of syncope is neurocardiogenic syncope, or "vasovagal" syncope. In this type of syncope there is accentuated adrenergic activity in the upright position, with intense activation of cardiopulmonary receptors resulting in marked bradycardia and hypotension. Normally testing with the upright tilt table causes activation of the renin-angiotensin-aldosterone system and compensation to increase CO and maintain BP when blood pools in the extremities. However, patients with neurocardiogenic syncope experience a marked decrease in BP and HR.

A patient has a new prescription for doxazosin. When providing education about this drug, the nurse will include which instructions? 1. "Weigh yourself daily, and report any weight loss to your prescriber." 2. "Increase your potassium intake by eating more bananas and apricots." 3. "Take this drug at bedtime because of the risk of orthostatic hypotension." 4. "The impaired taste associated with this medication usually goes away in two to three weeks."

3. "Take this drug at bedtime because of the risk of orthostatic hypotension." A patient who is starting doxazosin should take the first dose while lying down because there is a first-dose hypotensive effect with this medication. Taking the drug at bedtime reduces risks associated with orthostatic hypotension. The patient does not need to increase potassium intake. Doxazosin does not cause impaired taste. It does not cause weight loss, because it is not a diuretic. Test-Taking Tip: Relax during the last hour before an exam. Your brain needs some recovery time to function effectively. Text Reference - p. 723

For what change in vital signs would the nurse assess a patient experiencing postural hypotension? 1. Increased systolic blood pressure, decreased pulse rate 2. Increased diastolic blood pressure, increased pulse rate 3. Decreased systolic blood pressure, decreased diastolic blood pressure, increased pulse rate 4. Decreased systolic blood pressure, increased diastolic blood pressure, no change in pulse rate

3. Decreased systolic blood pressure, decreased diastolic blood pressure, increased pulse rate A decrease in both systolic and diastolic blood pressure and an increase in pulse would be seen in a patient with postural hypotension. Blood pressure drops as the volume of circulating blood decreases when a patient abruptly stands from a lying or sitting position. The pulse rate increases as the heart attempts to compensate by increasing the amount of circulating blood by increasing cardiac output. Increased systolic blood pressure and decreased pulse rate; increased diastolic blood pressure and increased pulse rate; and decreased systolic blood pressure, increased diastolic blood pressure, and no change in pulse rate are all incorrect. Text Reference - p. 723

The patient has chronic hypertension. Today the patient has gone to the emergency department and the patient's blood pressure has risen to 200/140. What is the priority assessment for the nurse to make? 1. Is the patient pregnant? 2.Does the patient need to urinate? 3. Does the patient have a headache or confusion? 4. Is the patient taking antiseizure medications as prescribed?

3. Does the patient have a headache or confusion? The nurse's priority assessments include neurologic deficits, retinal damage, heart failure, pulmonary edema, and renal failure. The headache or confusion could be seen with hypertensive encephalopathy, from increased cerebral capillary permeability leading to cerebral edema. Pregnancy can lead to secondary hypertension. Needing to urinate and taking antiseizure medication do not indicate a hypertensive emergency. Text Reference - p. 726

The nurse understands that catapres has which side effects? 1. Cough and confusion 2. Sweating and shaking 3. Dry mouth and sedation 4. Gynecomastia and dizziness

3. Dry mouth and sedation Dry mouth and sedation are side effects of central-acting alpha-adrenergic antagonists. Cough is a possible side effect of angiotensin-converting enzyme inhibitors. Confusion, sweating, and shaking are not common side effects for hypertension drug therapy. Gynecomastia is a side effect of aldosterone receptor blockers. Dizziness is a side effect for hypertension drug therapy; gynecomastia is not. Text Reference - p. 717

A patient is scheduled a dose of metoprolol. The nurse should withhold the dose and consult the health care provider after noting which assessment finding? 1. Migraine headache 2. Pulse 112 beats/minute 3. Expiratory wheezing 4. Blood sugar 217 mg/dL

3. Expiratory wheezing Metoprolol is a β-adrenergic-blocking agent that reduces blood pressure and could affect the β2 receptors in the lungs with larger doses or with drug accumulation. It should be used cautiously in patients with wheezing or respiratory disorders because it could cause bronchospasm, a potentially life-threatening adverse effect. Metoprolol will not worsen migraine, will decrease the elevated pulse rate, and will not lower or further elevate the blood sugar. Text Reference - p. 718

The nurse is assessing a patient and auscultates a "swooshing" sound heard over the chest wall when the stethoscope is lifted just off of the chest. The nurse would document this finding as a(n): 1. Severe bruit 2. Atrial gallop 3. Grade VI murmur 4. Pericardial friction rub

3. Grade VI murmur A murmur is classified as turbulent blood flow, which produces the classic swooshing sound as it passes through the valve and is graded on a scale of I toVI, with VI being the loudest, heard when the stethoscope is not touching the chest wall. A bruit is auscultated over arteries. An atrial gallop is an extra heart sound and is not associated with turbulent blood flow. A pericardial friction rub is a scratching sound caused when inflamed surfaces of the pericardium move against each other, indicating cardiac inflammation. Text Reference - p. 726

When teaching how lisinopril will help lower the patient's blood pressure, which mechanism of action should the nurse use to explain it? 1. Blocks β-adrenergic effects 2. Relaxes arterial and venous smooth muscle 3. Inhibits conversion of angiotensin I to angiotensin II 4. Reduces sympathetic outflow from the central nervous system (CNS)

3. Inhibits conversion of angiotensin I to angiotensin II Lisinopril is an angiotensin-converting enzyme (ACE) inhibitor that inhibits the conversion of angiotensin I to angiotensin II, which reduces angiotensin II-mediated vasoconstriction and sodium and water retention. Beta blockers result in vasodilation and decreased heart rate. Direct vasodilators relax arterial and venous smooth muscle. Central acting α-adrenergic antagonists reduce sympathetic outflow from the CNS to produce vasodilation and decreased systemic vascular resistance (SVR) and blood pressure (BP). Test-Taking Tip: Do not read too much into the question or worry that it is a "trick." If you have nursing experience, ask yourself how a classmate who is inexperienced would answer this question from only the information provided in the textbooks or given in the lectures. Text Reference - p. 719

The nurse is teaching a patient, recently diagnosed with hypertension (HTN), about diagnostic studies prescribed by a primary health care provider. Which information would the nurse include? Select all that apply. 1. Echocardiography to evaluate cardiac status 2. ECG to evaluate degree of left ventricular hypertrophy 3. Lipid profile to provide information about the risk factor for HTN 4. Uric acid level because it frequently decreases with diuretic therapy 5. Blood urea nitrogen (BUN) and serum creatinine levels to provide information on renal function

3. Lipid profile to provide information about the risk factor for HTN 5. Blood urea nitrogen (BUN) and serum creatinine levels to provide information on renal function An elevated lipid profile is an additional risk factor for hypertension because having elevated blood lipids leads to development of atherosclerosis. BUN, creatinine, and urinalysis provide information about baseline renal function and help to identify renal damage. Echocardiography evaluates the degree of ventricular hypertrophy, whereas ECG is used to assess baseline cardiac function. Diuretic therapy frequently leads to an increase in uric acid. Text Reference - p. 715

In caring for a patient admitted with poorly controlled hypertension, the nurse should understand that which laboratory test result would indicate the presence of target organ damage? 1. Blood urea nitrogen (BUN) of 15 mg/dL 2. Serum uric acid of 3.8 mg/dL 3. Serum creatinine of 2.6 mg/dL 4. Serum potassium of 3.5 mEq/L

3. Serum creatinine of 2.6 mg/dL The normal serum creatinine level is 0.6 to 1.3 mg/dL. This elevated level indicates target organ damage to the kidneys. BUN of 15 mg/dL, serum uric acid of 3.8 mg/dL, and serum potassium of 3.5 mEq/L are within normal limits. Text Reference - p. 715

A patient reports chest pain and is admitted to the emergency department. The patient is obese, smokes cigarettes, and drinks alcohol in moderate amounts. The patient had taken labetalol for high blood pressure (BP) for one week and then stopped taking the medication the morning of admission. The nurse recognizes that the probable reason for the patient's angina is what? 1. Leading a sedentary lifestyle after a lifetime of obesity 2. Smoking cigarettes 3. Stopping labetalol abruptly after a week of treatment 4. Alcohol consumption

3. Stopping labetalol abruptly after a week of treatment Labetalol is an alpha- and beta-adrenergic blocker and reduces BP by causing vasodilatation and a decrease in heart rate. The patient should not stop the drug abruptly, because it may precipitate angina and heart failure. Obesity, a sedentary lifestyle, smoking, and alcohol consumption are risk factors for cardiovascular disease but are unlikely to cause angina. Text Reference - p. 718

The nurse just received the shift report. Which patient should the nurse assess first? 1. The patient who is complaining about dizziness and whose blood pressure (BP) is 150/92. 2. The patient with a hip fracture who is complaining about pain 2 out of 10 3. The patient who is complaining about severe headache and has a nose bleed 4. The patient complaining of fatigue and who just received an angiotensin-converting enzyme (ACE) inhibitor.

3. The patient who is complaining about severe headache and has a nose bleed Severe headache and nose bleed are signs of hypertensive crisis that is an emergency situation, and therefore the nurse has to see this patient first. Dizziness is one of the symptoms of hypertension and the patient has an elevated blood pressure, but it is not an emergency situation. Pain 2 out of 10 is mild pain and therefore this patient is not a priority. Fatigue is one of the symptoms of hypertension, but the patient just received antihypertensive medication. Text Reference - p. 714

In reviewing medication instructions with a patient being discharged on antihypertensive medications, which statement would be most appropriate for the nurse to make when discussing guanethidine? 1. "A fast heart rate is a side effect to watch for while taking guanethidine." 2. "Stop the drug and notify your health care provider if you experience any nausea or vomiting." 3. "Because this drug may affect the lungs in large doses, it also may help your breathing." 4. "Make position changes slowly, especially when rising from lying down to a standing position."

4. "Make position changes slowly, especially when rising from lying down to a standing position." Guanethidine is a peripheral-acting α-adrenergic antagonist and can cause marked orthostatic hypotension. For this reason, the patient should be instructed to rise slowly, especially when moving from a recumbent to a standing position. Support stockings also may be helpful. Tachycardia or lung effects are not evident with guanethidine, nor are nausea and vomiting. Test-Taking Tip: Start by reading each of the answer options carefully. Usually at least one of them will be clearly wrong. Eliminate this one from consideration. Now you have reduced the number of response choices by one and improved the odds. Continue to analyze the options. If you can eliminate one more choice in a four-option question, you have reduced the odds to 50/50. While you are eliminating the wrong choices, recall often occurs. One of the options may serve as a trigger that causes you to remember what a few seconds ago had seemed completely forgotten. Text Reference - p. 718

A 65-year-old patient without any past medical problems has his or her blood pressure checked at a primary health care provider's office during an annual physical examination. The blood pressure (BP) reading is 158/92. The patient is asking the nurse who was checking the blood pressure: "Does this mean that I have hypertension?" What is the most appropriate answer from the nurse? 1. "Do not worry, everything is fine." 2. "It is a normal blood pressure reading for a person of your age." 3. "Yes, you have hypertension, because your blood pressure is over 140/90." 4. "You need to have a follow-up appointment to recheck your blood pressure."

4. "You need to have a follow-up appointment to recheck your blood pressure." The diagnosis of hypertension is made based on two or more elevated blood pressure readings. Considering the fact that the patient does not have any medical problems and that this reading is the first elevated blood pressure reading, a follow-up office visit is required. Providing false reassurance to the patient is leading to misinformation. For any person of age 18 and older, BP higher than 140/90 is considered elevated. Diagnosing the patient with a medical diagnosis is not within the nursing scope of practice and cannot be done based on one elevated BP reading. Test-Taking Tip: Avoid looking for an answer pattern or code. There may be times when four or five consecutive questions have the same letter or number for the correct answer. Text Reference - p. 715

4. Priority nursing management for a patient with myocarditis includes interventions related to a. meticulous skin care. b. antibiotic prophylaxis. c. tight glycemic control. d. oxygenation and ventilation.

4. Correct answer: d Rationale: General supportive measures for management of myocarditis include interventions to improve ventilation and oxygenation (oxygen therapy, bed rest, and restricted activity).

A nurse provides care to a patient who is admitted to an emergency department with hypertensive crisis. The patient had been taking sodium nitroprusside for the past three days. What is the reason that blood tests to assess thiocyanate levels are prescribed for this patient? 1. The patient may have very low BP due to the sodium nitroprusside. 2. The patient may have adverse effects on target organs. 3. The patient may have reduced excretion of sodium nitroprusside. 4. The patient may have toxic levels of sodium nitroprusside.

4. The patient may have toxic levels of sodium nitroprusside. Sodium nitroprusside causes arterial vasodilation and reduces systemic vascular resistance. This in turn decreases the blood pressure. Sodium nitroprusside is metabolized to cyanide and then to thiocyanate, which can reach lethal levels. Therefore, thiocyanate levels should be monitored in patients receiving the drug for more than three days or at doses greater than 4mcg/kg/min. In hypertensive crisis, the patient usually has very high BP despite the BP-lowering effect of sodium nitroprusside. Serum thiocyanate levels do not indicate adverse effects of hypertension on target organs or reduced excretion of sodium nitroprusside. Text Reference - p. 719

b. Late onset of puberty is not generally considered to be a risk factor for the development of atherosclerosis. Risk factors for atherosclerosis include family history of atherosclerosis, cigarette smoking, hypertension, high blood cholesterol level, male gender, diabetes mellitus, obesity, and physical inactivity

40] Which of the following is not a risk factor for the development of atherosclerosis? a. Family history of early heart attack b. Late onset of puberty c. Total blood cholesterol level greater than 220 mg/dL d. Elevated fasting blood glucose concentration

c. Nitroglycerin produces peripheral vasodilation, which reduces myocardial oxygen consumption and demand. Vasodilation in coronary arteries and collateral vessels may also increase blood flow to the ischemic areas of the heart. Nitroglycerin does not have an effect on pericardial spasticity or conductivity in the myocardium.

41] As an initial step in treating a client with angina, the physician prescribes nitroglycerin tablets, 0.3 mg given sublingually. This drug's principal effects are produced by: a. Antispasmodic effects on the pericardium b. Causing an increased myocardial oxygen demand c. Vasodilation of peripheral vasculature d. Improved conductivity in the myocardium

c. The correct protocol for nitroglycerin use involves immediate administration, with subsequent doses taken at 5-minute intervals as needed, for a total dose of three tablets. Sublingual nitroglycerin appears in the bloodstream within 2 to 3 minutes and is metabolized within about 10 minutes.

43] Sublingual nitroglycerine tablets begin to work within 1 to 2 minutes. How should the nurse instruct the client to use the drug when chest pain occurs? a. Take one tablet every 2 to 5 minutes until the pains stops b. Take one tablet and rest for 10 minutes. Call the physician if pain persists after 10 minutes c. Take one tablet, then an additional tablet every 5 minutes for a total of three tablets. Call the physician if pain persists after three tablets d. Take one tablet. If pain persists after 5 minutes, take two tablets. If pain still persists 5 minutes later, call the physician

5. When teaching a patient about the long-term consequences of rheumatic fever, the nurse should discuss the possibility of a. valvular heart disease. b. pulmonary hypertension. c. superior vena cava syndrome. d. hypertrophy of the right ventricle.

5. Correct answer: a Rationale: Rheumatic heart disease is a chronic condition resulting from rheumatic fever that is characterized by scarring and deformity of the heart valves.

5. What describes the SA node's ability to discharge an electrical impulse spontaneously? a. Excitability b. Contractility c. Conductivity d. Automaticity

5. d. Automaticity describes the ability to discharge an electrical impulse spontaneously. Excitability is a property of myocardial tissue that enables it to be depolarized by an impulse. Contractility is the ability of the chambers to respond mechanically to an impulse. Conductivity is the ability to transmit an impulse along a membrane.

6. Which is a priority nursing intervention for a patient during the acute phase of rheumatic fever? a. Administration of antibiotics as ordered b. Management of pain with opioid analgesics c. Encouragement of fluid intake for hydration d. Performance of frequent active range-of-motion exercises

6. Correct answer: a Rationale: The primary goal of managing a patient with acute rheumatic fever is to control and eradicate the infecting organism. The nurse should administer antibiotics as ordered to treat the streptococcal infection and teach the patient that completion of the full course of antibiotic therapy is essential for successful treatment.

6. What describes refractoriness? a. Abnormal electrical impulses b. Period in which heart tissue cannot be stimulated c. Areas of the heart do not repolarize at the same rate because of depressed conduction d. Sodium migrates rapidly into the cell so it is positive compared to the outside of the cell

6. b. Refractoriness is the period in which heart tissue cannot be stimulated. Ectopic foci are abnormal electrical impulses. Reentrant excitation causing premature beats may occur when areas of the heart do not repolarize simultaneously with depressed conduction. Depolarization of cardiac cells occurs when sodium migrates rapidly into the cell.

7. Which clinical finding would most likely indicate decreased cardiac output in a patient with aortic valve regurgitation? a. Reduction in peripheral edema and weight b. Carotid venous distention and new-onset atrial fibrillation c. Significant pulsus paradoxus and diminished peripheral pulses d. Shortness of breath on minimal exertion and a diastolic murmur

7. Correct answer: d Rationale: Clinical manifestations of aortic regurgitation (AR) that indicate decreased cardiac output include severe dyspnea, chest pain, and hypotension. Other manifestations of chronic AR include water-hammer pulse (i.e., a strong, quick beat that collapses immediately), soft or absent S1, presence of S3 or S4, and soft, high-pitched diastolic murmur. A low-pitched diastolic murmur may be heard in severe AR. Early manifestations may include exertional dyspnea, orthopnea, and paroxysmal nocturnal dyspnea.

7. The patient's PR interval comprises six small boxes on the ECG graph. What does the nurse determine that this indicates? a. A normal finding b. A problem with ventricular depolarization c. A disturbance in the repolarization of the atria d. A problem with conduction from the SA node to the ventricular cells

7. d. The normal PR interval is 0.12 to 0.20 seconds and reflects the time taken for the impulse to spread through the atria, AV node and bundle of His, the bundle branches, and Purkinje fibers. A PR interval of six small boxes is 0.24 second and indicates that the conduction of the impulse from the atria to the Purkinje fibers is delayed.

b. A low urine output and confusion are signs of decreased tissue perfusion. Orthopnea is a sign of left-sided heart failure. Crackles, edema and weight gain should be monitored closely, but the levels are not as high a priority. With atrial fibrillation there is a loss of atrial kick, but the blood pressure and heart rate are stable

7] The nurse has completed an assessment on a client with a decreased cardiac output. Which findings should receive highest priority? a. BP 110/62, atrial fibrillation with HR 82, bibasilar crackles b. Confusion, urine output 15mL over the last 2 hours, orthopnea. c. SpO2 92 on 2 liters nasal cannula, respirations 20, 1+ edema of lower extremities. d. Weight gain of 1kg in 3 days, BP 130/80, mild dyspnea with exercise.

8. The nurse plans close monitoring for the patient during electrophysiologic testing because this test a. requires the use of dyes that irritate the myocardium. b. causes myocardial ischemia, resulting in dysrhythmias. c. involves the use of anticoagulants to prevent thrombus and embolism. d. induces dysrhythmias that may require cardioversion or defibrillation to correct.

8. d. Electrophysiologic testing involves electrical stimulation to various areas of the atrium and ventricle to determine the inducibility of dysrhythmias and frequently induces ventricular tachycardia or ventricular fibrillation. The patient may have "near-death" experiences and requires emotional support if this occurs. Dye and anticoagulants are used for coronary angiograms.

9. What should the nurse reading the monitor strip call a rhythm with a regular PR interval but a blocked QRS complex? a. Asystole b. Atrial fibrillation c. First-degree AV block d. Type II second-degree AV block

9. d. In type II second-degree AV block, a P wave is nonconducted without progressive P-R interval lengthening. It is usually from a block in a bundle branch, occurs in a ratio of 2 P waves-to-1 QRS complex, 3:1, and so on. Atrial fibrillation has a chaotic P wave. Asystole is absence of ventricular activity. First-degree AV block is a prolonged AV conduction time, so the P-R interval is prolonged.

Integrated Process: Nursing Process (Analysis) 7. A client with tachycardia is experiencing clinical manifestations. Which manifestation requires immediate intervention by the nurse? a. Mid-sternal chest pain b. Increased urine output c. Mild orthostatic hypotension d. P wave touching the T wave

A Chest pain, possibly angina, indicates that tachycardia may be increasing the client's myocardial workload and oxygen demand to such an extent that normal oxygen delivery cannot keep pace. This results in myocardial hypoxia and pain. DIF: Cognitive Level: Application/Applying or higher REF: N/A TOP: Client Needs Category: Physiological Integrity (Reduction of Risk Potential—Potential for Complications from Surgical Procedures and Health Alterations)

Physiological Integrity 26. The nurse is caring for a 78-year-old patient with aortic stenosis. Which assessment data obtained by the nurse would be most important to report to the health care provider? a. The patient complains of chest pressure when ambulating. b. A loud systolic murmur is heard along the right sternal border. c. A thrill is palpated at the second intercostal space, right sternal border. d. The point of maximum impulse (PMI) is at the left midclavicular line.

A Chest pressure (or pain) occurring with aortic stenosis is caused by cardiac ischemia, and reporting this information would be a priority. A systolic murmur and thrill are expected in a patient with aortic stenosis. A PMI at the left midclavicular line is normal. DIF: Cognitive Level: Apply (application) REF: 823 OBJ: Special Questions: Prioritization TOP: Nursing Process: Assessment MSC:

Physiological Integrity 28. The nurse is caring for a 64-year-old patient admitted with mitral valve regurgitation. Which information obtained by the nurse when assessing the patient should be communicated to the health care provider immediately? a. The patient has bilateral crackles. b. The patient has bilateral, 4+ peripheral edema. c. The patient has a loud systolic murmur across the precordium. d. The patient has a palpable thrill felt over the left anterior chest.

A Crackles that are audible throughout the lungs indicate that the patient is experiencing severe left ventricular failure with pulmonary congestion and needs immediate interventions such as diuretics. A systolic murmur and palpable thrill would be expected in a patient with mitral regurgitation. Although 4+ peripheral edema indicates a need for a change in therapy, it does not need to be addressed urgently. DIF: Cognitive Level: Apply (application) REF: 821-822 OBJ: Special Questions: Prioritization TOP: Nursing Process: Assessment MSC:

Physiological Integrity 5. A patient is admitted to the hospital with possible acute pericarditis. The nurse should plan to teach the patient about the purpose of a. echocardiography. b. daily blood cultures. c. cardiac catheterization. d. 24-hour Holter monitor.

A Echocardiograms are useful in detecting the presence of the pericardial effusions associated with pericarditis. Blood cultures are not indicated unless the patient has evidence of sepsis. Cardiac catheterization and 24-hour Holter monitor is not a diagnostic procedure for pericarditis. DIF: Cognitive Level: Apply (application) REF: 816 TOP: Nursing Process: Planning MSC:

Chapter 36: Care of Patients with Dysrhythmias 1. A client's cardiac status is being observed by telemetry monitoring. The nurse observes a P wave that changes shape in lead II. What conclusion does the nurse make about the P wave? a. It originates from an ectopic focus. b. The P wave was replaced by U waves. c. It is from the sinoatrial (SA) node. d. Multiple P waves are present.

A If the P wave is firing consistently from the SA node, the P wave will have a consistent shape in a given lead. If the impulse is from an ectopic focus, then the P wave will vary in shape in that lead. DIF: Cognitive Level: Comprehension/Understanding REF: p. 715 TOP: Client Needs Category: Physiological Integrity (Physiological Adaptation—Pathophysiology)

Integrated Process: Nursing Process (Implementation) 9. A client experiences occasional premature atrial contractions (PACs) accompanied by palpitations that resolve spontaneously without treatment. What instruction does the nurse include in the client's teaching plan? a. "Minimize or abstain from caffeine." b. "Lie on your side until the attack subsides." c. "Use your oxygen when you experience PACs." d. "Take quinidine (Cardioquin) daily to prevent PACs."

A PACs usually have no hemodynamic consequences. For a client experiencing infrequent bouts of PACs, the nurse should explore possible lifestyle causes, such as excessive caffeine intake and stress. Lying on the side will not prevent or resolve PACs. Oxygen is not necessary. Although medications may be needed to control symptomatic dysrhythmias, for infrequent PACs, the client first should try lifestyle changes to control them. DIF: Cognitive Level: Application/Applying or higher REF: N/A TOP: Client Needs Category: Health Promotion and Maintenance (Self-Care)

Physiological Integrity 7. The nurse suspects cardiac tamponade in a patient who has acute pericarditis. To assess for the presence of pulsus paradoxus, the nurse should a. note when Korotkoff sounds are auscultated during both inspiration and expiration. b. subtract the diastolic blood pressure (DBP) from the systolic blood pressure (SBP). c. check the electrocardiogram (ECG) for variations in rate during the respiratory cycle. d. listen for a pericardial friction rub that persists when the patient is instructed to stop breathing.

A Pulsus paradoxus exists when there is a gap of greater than 10 mm Hg between when Korotkoff sounds can be heard during only expiration and when they can be heard throughout the respiratory cycle. The other methods described would not be useful in determining the presence of pulsus paradoxus. DIF: Cognitive Level: Understand (comprehension) REF: 816 TOP: Nursing Process: Assessment MSC:

6. A patient in the intensive care unit with ADHF complains of severe dyspnea and is anxious, tachypneic, and tachycardic. All these medications have been ordered for the patient. The first action by the nurse will be to a. administer IV morphine sulfate 2 mg. b. give IV diazepam (Valium) 2.5 mg. c. increase dopamine (Intropin) infusion by 2 mcg/kg/min. d. increase nitroglycerin (Tridil) infusion by 5 mcg/min.

A Rationale: Morphine improves alveolar gas exchange, improves cardiac output by reducing ventricular preload and afterload, decreases anxiety, and assists in reducing the subjective feeling of dyspnea. Diazepam may decrease patient anxiety, but it will not improve the cardiac output or gas exchange. Increasing the dopamine may improve cardiac output but will also increase the heart rate and myocardial oxygen consumption. Nitroglycerin will improve cardiac output and may be appropriate for this patient, but it will not directly reduce anxiety and will not act as quickly as morphine to decrease dyspnea. Cognitive Level: Analysis Text Reference: pp. 828-829 Nursing Process: Implementation NCLEX: Physiological Integrity

21. A patient with ADHF who is receiving nesiritide (Natrecor) asks the nurse how the medication will work to help improve the symptoms of dyspnea and orthopnea. The nurse's reply will be based on the information that nesiritide will a. dilate arterial and venous blood vessels, decreasing ventricular preload and afterload. b. improve the ability of the ventricular myocardium to contract, strengthening contractility. c. enhance the speed of impulse conduction through the heart, increasing the heart rate. d. increase calcium sensitivity in vascular smooth muscle, boosting systemic vascular resistance.

A Rationale: Nesiritide, a recombinant form of BNP, causes both arterial and venous vasodilation, leading to reductions in preload and afterload. Inotropic medications, such as dopamine and dobutamine, may be used in ADHF to improve ventricular contractility. Nesiritide does not increase impulse conduction or calcium sensitivity in the heart. Cognitive Level: Application Text Reference: p. 829 Nursing Process: Implementation NCLEX: Physiological Integrity

19. An outpatient who has developed heart failure after having an acute myocardial infarction has a new prescription for carvedilol (Coreg). After 2 weeks, the patient returns to the clinic. The assessment finding that will be of most concern to the nurse is that the patient a. has BP of 88/42. b. has an apical pulse rate of 56. c. complains of feeling tired. d. has 2+ pedal edema.

A Rationale: The patient's BP indicates that the dose of carvedilol may need to be decreased because the mean arterial pressure is only 57. Bradycardia is a frequent adverse effect of -Adrenergic blockade, but the rate of 56 is not as great a concern as the hypotension. -adrenergic blockade will initially worsen symptoms of heart failure in many patients, and patients should be taught that some increase in symptoms, such as fatigue and edema, is expected during the initiation of therapy with this class of drugs. Cognitive Level: Application Text Reference: p. 832 Nursing Process: Assessment NCLEX: Analysis

1. A patient with a history of chronic heart failure is admitted to the emergency department with severe dyspnea and a dry, hacking cough. The patient has pitting edema in both ankles, blood pressure (BP) of 170/100, an apical pulse rate of 92, and respirations 28. The most important assessment for the nurse to accomplish next is to a. auscultate the lung sounds. b. assess the orientation. c. check the capillary refill. d. palpate the abdomen.

A Rationale: When caring for a patient with severe dyspnea, the nurse should use the ABCs to guide initial care. This patient's severe dyspnea and cough indicate that acute decompensated heart failure (ADHF) is occurring. ADHF usually manifests as pulmonary edema, which should be detected and treated immediately to prevent ongoing hypoxemia and cardiac/respiratory arrest. The other assessments will provide useful data about the patient's volume status and should also be accomplished rapidly, but detection (and treatment) of fluid-filled alveoli is the priority. Cognitive Level: Application Text Reference: pp. 824-825 Nursing Process: Assessment NCLEX: Physiological Integrity

Physiological Integrity 16. While caring for a patient with aortic stenosis, the nurse identifies a nursing diagnosis of acute pain related to decreased coronary blood flow. A priority nursing intervention for this patient would be to a. promote rest to decrease myocardial oxygen demand. b. teach the patient about the need for anticoagulant therapy. c. teach the patient to use sublingual nitroglycerin for chest pain. d. raise the head of the bed 60 degrees to decrease venous return.

A Rest is recommended to balance myocardial oxygen supply and demand and to decrease chest pain. The patient with aortic stenosis requires higher preload to maintain cardiac output, so nitroglycerin and measures to decrease venous return are contraindicated. Anticoagulation is not recommended unless the patient has atrial fibrillation. DIF: Cognitive Level: Apply (application) REF: 823 | 826 TOP: Nursing Process: Implementation MSC:

Integrated Process: Nursing Process (Analysis) 19. A client's electrocardiograph (ECG) tracing shows a run of sustained ventricular tachycardia. What is the nurse's first action? a. Assess airway, breathing, and level of consciousness. b. Administer an amiodarone bolus followed by a drip. c. Cardiovert the client with a biphasic defibrillator. d. Begin cardiopulmonary resuscitation (CPR).

A The first action that the nurse should take when ventricular tachycardia is observed is to assess the client's airway, breathing, and level of consciousness. If the client is unconscious or has experienced respiratory arrest, defibrillation and CPR are begun. DIF: Cognitive Level: Application/Applying or higher REF: N/A TOP: Client Needs Category: Physiological Integrity (Physiological Adaptation—Medical Emergencies)

Physiological Integrity 27. Two days after an acute myocardial infarction (MI), a patient complains of stabbing chest pain that increases with a deep breath. Which action will the nurse take first? a. Auscultate the heart sounds. b. Check the patient's temperature. c. Notify the patient's health care provider. d. Give the PRN acetaminophen (Tylenol).

A The patient's clinical manifestations and history are consistent with pericarditis, and the first action by the nurse should be to listen for a pericardial friction rub. Checking the temperature and notifying the health care provider are also appropriate actions but would not be done before listening for a rub. It is not stated for what symptom (e.g., headache) or finding (e.g., increased temperature) the PRN acetaminophen (Tylenol) is ordered. DIF: Cognitive Level: Apply (application) REF: 815 OBJ: Special Questions: Prioritization TOP: Nursing Process: Implementation MSC:

Integrated Process: Nursing Process (Implementation) 21. A client with ischemic heart disease has an electrocardiograph (ECG) tracing that shows a PR interval of 0.24 second. What is the nurse's best action? a. Document the finding in the chart. b. Measure blood pressure. c. Notify the health care provider. d. Administer oxygen.

A This prolonged PR interval indicates a first-degree heart block. First-degree heart block in a stable client requires no intervention. DIF: Cognitive Level: Application/Applying or higher REF: N/A TOP: Client Needs Category: Physiological Integrity (Physiological Adaptation—Pathophysiology)

Integrated Process: Nursing Process (Analysis) 34. The nurse notes the following rhythm on a client's telemetry monitor. How does the nurse interpret these findings? a. Ventricular tachycardia b. Second-degree heart block c. Supraventricular tachycardia d. Premature ventricular contractions

A Ventricular tachycardia occurs with repetitive firing of an irritable ventricular ectopic focus, usually at a rate of 140 to 180 beats/min or more. DIF: Cognitive Level: Application/Applying or higher REF: N/A TOP: Client Needs Category: Physiological Integrity (Physiological Adaptation-Pathophysiology)

At a clinic visit, the nurse provides dietary teaching for a 56-year-old woman who was recently hospitalized with an exacerbation of chronic heart failure. The nurse determines that teaching is successful if the patient makes which statement? A "I will limit the amount of milk and cheese in my diet." B "I can add salt when cooking foods but not at the table." C "I will take an extra diuretic pill when I eat a lot of salt." D "I can have unlimited amounts of foods labeled as reduced sodium ."

A "I will limit the amount of milk and cheese in my diet." Milk products should be limited to 2 cups per day for a 2500-mg sodium-restricted diet. Salt should not be added during food preparation or at the table. Diuretics should be taken as prescribed (usually daily) and not based on sodium intake. Foods labeled as reduced sodium contain at least 25% less sodium than regular.

A70-year-old woman with chronic heart failure and atrial fibrillation asks the nurse why warfarin (Coumadin) has been prescribed for her to continue at home. Which response by the nurse is accurate? A "The medication prevents blood clots from forming in your heart." B "The medication dissolves clots that develop in your coronary arteries." C "The medication reduces clotting by decreasing serum potassium levels." D "The medication increases your heart rate so that clots do not form in your heart."

A "The medication prevents blood clots from forming in your heart." Chronic heart failure causes enlargement of the chambers of the heart and an altered electrical pathway, especially in the atria. When numerous sites in the atria fire spontaneously and rapidly, atrial fibrillation occurs. Atrial fibrillation promotes thrombus formation within the atria with an increased risk of stroke and requires treatment with cardioversion, antidysrhythmics, and/or anticoagulants. Warfarin is an anticoagulant that interferes with hepatic synthesis of vitamin K-dependent clotting factors.

Answer: 3 Patients with atrial fibrillation are at incredibly high risk for clots, even with anticoagulation therapy. Shortness of breath could indicate a PE, and this should be immediately investigated by the nurse. The patient should be NPO for at least 4 hr. prior to the procedure related to anesthesia use, but this is not as urgent of a concern. The patient should also withhold Digoxin therapy for 48 hours to ensure that, once cardioverted, NSR returns.

A 26-year-old client with atrial fibrillation that has not responded to medication therapy has arrived at the hospital for an elective cardioversion. Which of the following patient statements most concerns the nurse? 1) "I can't wait to stop taking this Coumadin. I've been on this crap for weeks now." 2) "I'm starving. I haven't eaten anything in 3 hours." 3) "I feel really short of breath, can I lie down?" 4) "I haven't taken my Digoxin since 9 o'clock last night. Is that okay?"

The evaluation team for cardiac transplantation is evaluating patients. Which patient is most likely to receive the most benefit from a new heart?

A 52 year old woman with end-stage coronary artery disease who has limited financial resources but is emotionally stable and has strong social support

A patient admitted with heart failure appears very anxious and complains of shortness of breath. Which nursing actions would be appropriate to alleviate this patient's anxiety (select all that apply)? A Administer ordered morphine sulfate. B Position patient in a semi-Fowler's position. C Position patient on left side with head of bed flat. D Instruct patient on the use of relaxation techniques. E Use a calm, reassuring approach while talking to patient.

A Administer ordered morphine sulfate. B Position patient in a semi-Fowler's position. D Instruct patient on the use of relaxation techniques. E Use a calm, reassuring approach while talking to patient. Morphine sulfate reduces anxiety and may assist in reducing dyspnea. The patient should be positioned in semi-Fowler's position to improve ventilation that will reduce anxiety. Relaxation techniques and a calm reassuring approach will also serve to reduce anxiety.

The patient has heart failure (HF) with an ejection fraction of less than 40%. What core measures should the nurse expect to include in the plan of care for this patient (select all that apply)? A Left ventricular function is documented. B Controlling dysrhythmias will eliminate HF. C Prescription for digoxin (Lanoxin) at discharge D Prescription for angiotensin-converting enzyme (ACE) inhibitor at discharge E Education materials about activity, medications, weight monitoring, and what to do if symptoms worsen

A Left ventricular function is documented. D Prescription for angiotensin-converting enzyme (ACE) inhibitor at discharge E Education materials about activity, medications, weight monitoring, and what to do if symptoms worsen The Joint Commission has identified these three core measures for heart failure patients. Although controlling dysrhythmias will improve CO and workload, it will not eliminate HF. Prescribing digoxin for all HF patients is no longer done because there are newer effective drugs and digoxin toxicity occurs easily related to electrolyte levels and the therapeutic range must be maintained.

A patient with a recent diagnosis of heart failure has been prescribed furosemide (Lasix) in an effort to physiologically do what for the patient? A Reduce preload. B Decrease afterload. C Increase contractility. D Promote vasodilation.

A Reduce preload. Diuretics such as furosemide are used in the treatment of HF to mobilize edematous fluid, reduce pulmonary venous pressure, and reduce preload. They do not directly influence afterload, contractility, or vessel tone.

The patient with chronic heart failure is being discharged from the hospital. What information should the nurse emphasize in the patient's discharge teaching to prevent progression of the disease to ADHF? A Take medications as prescribed. B Use oxygen when feeling short of breath. C Only ask the physician's office questions. D Encourage most activity in the morning when rested.

A Take medications as prescribed. The goal for the patient with chronic HF is to avoid exacerbations and hospitalization. Taking the medications as prescribed along with nondrug therapies such as alternating activity with rest will help the patient meet this goal. If the patient needs to use oxygen at home, it will probably be used all the time or with activity to prevent respiratory acidosis. Many HF patients are monitored by a care manager or in a transitional program to assess the patient for medication effectiveness and monitor for patient deterioration and encourage the patient. This nurse manager can be asked questions or can contact the health care provider if there is evidence of worsening HF.

A Prinzmetal's or variant angina is atypical angina that occurs unpredictably (unrelated to activity) and often at night. It is caused by coronary artery spasm with or without an atherosclerotic lesion. This client is at risk for silent ischemia and is associated with a higher relative risk of serious or fatal cardiac events. Stable angina occurs with exertion and is relieved with rest and nitroglycerin tablets. Unstable angina occurs with increasing frequency, severity, and duration. Pain is unpredictable, occurs with decreasing levels of activity or stress, and may occur at rest. It is a precursor to myocardial infarction. The client has classic symptoms of variant angina and so is not experiencing nonanginal pain. Variant angina is prolonged, severe, and occurs at the same time every day, usually in the morning.

A client tells the nurse that the anginal pain is unpredictable but usually occurs at night. The nurse questions the client and family further about precipitating factors at home; however, there does not appear to be any environmental or emotional cause for the pain. The nurse concludes that the client should contact the physician to facilitate admission to the hospital, because the client has which type of angina? A) Stable angina B) Variant angina C) Unstable angina D) Nonanginal pain

Answer(s): 2, 3 The ATRIAL rate is 220-300 bpm. Ventricular is about 75-150. The rhythm is regular, with the P wave appearing as little flutter or a "saw tooth pattern". The PR interval is not measurable r/t this saw-tooth P wave. The QRS is normal.

A group of nursing students are discussing atrial flutter. These students recognize that which of the following are seen with atrial flutter? Select all that apply: 1) Ventricular rate of 220-300 bpm. 2) Regular rhythm 3) Saw-tooth pattern 4) Measurable PR interval 5) Long QRS interval

Answer(s): 2, 3, 4, 5 Diarrhea will not stimulate a vagal response, but vomiting can. Chronic constipation will cause a consistent vagal response. Digoxin, beta blockers, and calcium channel blockers can all contribute to first degree blocks. Relate this to bradycardia.

A group of nursing students are studying AV blocks and ask their instructor, "what causes a first-degree block?" The nursing instructor responds that which of the following can cause a first-degree block: Select all that apply 1) Diarrhea 2) Chronic constipation 3) Diltiazem 4) Digoxin 5) Metoprolol

Which action will the nurse include in the plan of care for a 72-year-old woman admitted with multiple myeloma? a. Monitor fluid intake and output. b. Administer calcium supplements. c. Assess lymph nodes for enlargement. d. Limit weight bearing and ambulation.

A high fluid intake and urine output helps prevent the complications of kidney stones caused by hypercalcemia and renal failure caused by deposition of Bence-Jones protein in the renal tubules. Weight bearing and ambulation are encouraged to help bone retain calcium. Lymph nodes are not enlarged with multiple myeloma. Calcium supplements will further increase the patient's calcium level and are not used

A client with HF has an order for lisnopril (Prinivil, Zestril) Which of the following conditions in the client's history would lead a nurse to confirm the order with the provider? 1. A history of HT previously treated with diuretics. 2. A history of seasonal allergies currently treated with antihistamines. 3. A history of angioedema after taking enalapril (Vasotec) 4. A history of alcoholism, currently abstaining.

A history of angioedema after taking enalapril (Vasotec)

Answer: 1) Although electrolytes are likely the culprit, the nurse first needs to first assess the patient. Then, the nurse should look in the patient's chart and evaluate or request an order for electrolyte levels. This may eventually need to be documented, but the nurse can be held liable for neglect if he/she does not assess the patient first. The physician may or may not need to be contacted.

A nurse on a CVT unit views the monitor and sees the patient in room 452 has just begun having occasional PVCs. Which action should the nurse take first? 1) Check on the patient 2) Check last magnesium and potassium levels 3) Document the occurrence and watch for further PVCs 4) Contact the physician

Answer: 2) Patients with a-fib are at risk for pulmonary and systemic emboli, and new onset of confusion may indicate a stroke in this patient. Patients with atrial flutter may feel more tired some days than others.

A nurse working on a CVT unit receives report from day shift. After receiving report, which patient should the nurse see first? 1) A 23-year-old professional tennis player with a HR of 47 bpm. 2) A 69-year-old male with atrial fibrillation who has new onset confusion. 3) A 72-year-old female with atrial flutter who reports feeling unusually tired today and yesterday. 4) A 33-year-old female with sinus tachycardia who is asking for her at-home Metoprolol.

Answer: 2) Cardioversion is used if the patient is unstable. Anticoagulants are used if the arrhythmia has stuck around for 48 hr +. Adenosine may be used with a narrow QRS and regular RR interval. I made up Altemose.

A nursing student is aware that which of the following is the treatment for unstable atrial flutter? 1) Adenosine (Adenocard) 6 mg rapid IVP. 2) Cardioversion with adjacent Heparin therapy 3) Defibrillation STAT followed by CPR. 4) Altemose 3 mg IVP over 1-2 seconds.

ANS: C The rhythm is a second-degree AV block, type I (Mobitz I or Wenckebach heart block). The rhythm is identified by a gradual lengthening of the PR interval. Type I AV block is usually a result of myocardial ischemia or infarction and typically is transient and well tolerated. You should assess for bradycardia, hypotension, and angina. If the patient becomes symptomatic, atropine or a temporary pacemaker may be needed. Reference: 830

A patient admitted with acute coronary syndrome (ACS) has continuous ECG monitoring. An examination of the rhythm strip reveals the following characteristics: atrial rate of 74 beats/minute and regular; ventricular rate of 62 beats/minute and irregular; P wave with a normal shape; PR interval that lengthens progressively until a P wave is not conducted; and QRS complex with a normal shape. Your priority nursing intervention involves A. performing synchronized cardioversion. B. administering 1 mg of epinephrine by IVP. C. observing for symptoms of hypotension or angina. D. preparing the patient for a transcutaneous pacemaker.

C

A patient has been admitted to the cardiac unit with a diagnosis of right ventricular failure. Which of the following assessment findings would the healthcare provider expect to observe? is most likely to be observed by the healthcare provider? A) Fatigue and hemoptysis B) Bradycardia and circumoral cyanosis C) Peripheral edema and jugular vein distension D) Dyspnea and pulmonary crackles

ANS: A A head-up tilt test is a common component of the diagnostic workup after episodes of syncope. Intravenous β-blockers are not indicated, and addressing pacemakers is premature and inappropriate at this stage of diagnosis. Patient education about antiplatelet aggregators is not directly relevant to the patient's syncope. Reference: 839

A patient has sought care after an episode of syncope of unknown origin. Which nursing action should you prioritize in the patient's subsequent diagnostic workup? A. Preparing to assist with a head-up tilt test B. Assessing the patient's knowledge of pacemakers C. Preparing an intravenous dose of a β-adrenergic blocker D. Teaching the patient about the role of antiplatelet aggregators

ANS: A Normally, the patient in asystole cannot be successfully resuscitated. However, administration of atropine and epinephrine may prompt the return of depolarization and ventricular contraction. Reference: 832

A patient in asystole is likely to receive which drug treatments? A. Atropine and epinephrine B. Lidocaine and amiodarone C. Digoxin and procainamide D. β-Adrenergic blockers and dopamine

D) Edema in the feet is common for Right ventricular failure

A patient who has a history of pulmonary valve stenosis tells the healthcare provider, "I don't have a lot of energy anymore, and both of my feet get swollen in the late afternoon." Which of these problems does the healthcare provider conclude is the likely cause of these clinical findings? Please choose from one of the following options. A) Acute pericarditis B) Deep vein thrombosis (DVT) C) Peripheral artery disease D) Right ventricular failure

Answer: B. Antiarrhythmic medications are prescribed with the use of an ICD in order to prevent the tachycardic (or other deadly arrhythmia) from occurring in the first place. This makes sure that the ICD is used only when absolutely necessary.

A patient with cardiomyopathy has been given an ICD. Which of the following medications would the nurse expect to see in the MAR for this patient? A) Warfarin B) Cardizem C) Nitroglycerin D) Digoxin

A patient admitted with heart failure appears very anxious and complains of shortness of breath. Which nursing actions would be appropriate to alleviate this patient's anxiety (select all that apply)? A. Administer ordered morphine sulfate. B. Position patient in a semi-Fowler's position. C. Position patient on left side with head of bed flat. D. Instruct patient on the use of relaxation techniques. E. Use a calm, reassuring approach while talking to patient.

A, B, D, E. Morphine sulfate reduces anxiety and may assist in reducing dyspnea. The patient should be positioned in semi-Fowler's position to improve ventilation that will reduce anxiety. Relaxation techniques and a calm reassuring approach will also serve to reduce anxiety.

Integrated Process: Nursing Process (Intervention) MULTIPLE RESPONSE 1. A client has a consistently regular heart rate of 128 beats/min. Which related physiologic alterations does the nurse assess for? (Select all that apply.) a. Decrease in cardiac output b. Increase in cardiac output c. Increase in blood pressure d. Decrease in blood pressure e. Increase in urine output

A, D Consistently elevated heart rates initially cause blood pressure and cardiac output to increase. However, ventricular filling time, cardiac output, and blood pressure eventually decrease. As cardiac output and blood pressure decrease, urine output will fall. DIF: Cognitive Level: Application/Applying or higher REF: N/A TOP: Client Needs Category: Physiological Integrity (Physiological Adaptation—Pathophysiology)

The patient has heart failure (HF) with an ejection fraction of less than 40%. What core measures should the nurse expect to include in the plan of care for this patient (select all that apply)? A. Left ventricular function is documented. B. Controlling dysrhythmias will eliminate HF. C. Prescription for digoxin (Lanoxin) at discharge D. Prescription for angiotensin-converting enzyme (ACE) inhibitor at discharge E. Education materials about activity, medications, weight monitoring, and what to do if symptoms worsen

A, D, E. The Joint Commission has identified these three core measures for heart failure patients. Although controlling dysrhythmias will improve CO and workload, it will not eliminate HF. Prescribing digoxin for all HF patients is no longer done because there are newer effective drugs and digoxin toxicity occurs easily related to electrolyte levels and the therapeutic range must be maintained.

A70-year-old woman with chronic heart failure and atrial fibrillation asks the nurse why warfarin (Coumadin) has been prescribed for her to continue at home. Which response by the nurse is accurate? A. "The medication prevents blood clots from forming in your heart." B. "The medication dissolves clots that develop in your coronary arteries." C. "The medication reduces clotting by decreasing serum potassium levels." D. "The medication increases your heart rate so that clots do not form in your heart."

A. "The medication prevents blood clots from forming in your heart." Chronic heart failure causes enlargement of the chambers of the heart and an altered electrical pathway, especially in the atria. When numerous sites in the atria fire spontaneously and rapidly, atrial fibrillation occurs. Atrial fibrillation promotes thrombus formation within the atria with an increased risk of stroke and requires treatment with cardioversion, antidysrhythmics, and/or anticoagulants. Warfarin is an anticoagulant that interferes with hepatic synthesis of vitamin K-dependent clotting factors.

The nurse is reviewing the laboratory test results for a 68-year-old patient whose warfarin (Coumadin) therapy was terminated during the preoperative period. The nurse concludes that the patient is in the most stable condition for surgery after noting which INR (international normalized ratio) result? A. 1.0 B. 1.8 C. 2.7 D. 3.4

A. 1.0 The therapeutic range for INR is 2.0 to 3.0 for many clinical diagnoses. The larger the INR number, the greater the amount of anticoagulation. For this reason, the safest value before surgery is 1.0, meaning that the anticoagulation has been reversed.

Which person should the nurse identify as having the highest risk for abdominal aortic aneurysm? A. A 70-year-old male, with high cholesterol and hypertension B. A 40-year-old female with obesity and metabolic syndrome C. A 60-year-old male with renal insufficiency who is physically inactive D. A 65-year-old female with hyperhomocysteinemia and substance abuse

A. A 70-year-old male, with high cholesterol and hypertension The most common etiology of descending abdominal aortic aneurysm (AAA) is atherosclerosis. Male gender, age 65 years or older, and tobacco use are the major risk factors for AAAs of atherosclerotic origin. Other risk factors include the presence of coronary or peripheral artery disease, high blood pressure, and high cholesterol.

A 40-year-old man tells the nurse he has a diagnosis for the color and temperature changes of his limbs but can't remember the name of it. He says he must stop smoking and avoid trauma and exposure of his limbs to cold temperatures to get better. This description should allow the nurse to ask the patient if he has which diagnosis? A. Buerger's disease B. Venous thrombosis C. Acute arterial ischemia D. Raynaud's phenomenon

A. Buerger's disease Buerger's disease is a nonatherosclerotic, segmental, recurrent inflammatory disorder of small and medium-sized veins and arteries of upper and lower extremities leading to color and temperature changes of the limbs, intermittent claudication, rest pain, and ischemic ulcerations. It primarily occurs in men younger than 45 years old with a long history of tobacco and/or marijuana use. Buerger's disease treatment includes smoking cessation, trauma and cold temperature avoidance, and a walking program. Venous thrombosis is the formation of a thrombus in association with inflammation of the vein. Acute arterial ischemia is a sudden interruption in arterial blood flow to a tissue caused by embolism, thrombosis, or trauma. Raynaud's phenomenon is characterized by vasospasm-induced color changes of the fingers, toes, ears, and nose.

The patient had myocarditis and is now experiencing fatigue, weakness, palpitations, and dyspnea at rest. The nurse assesses pulmonary crackles, edema, and weak peripheral pulses. Sinoatrial tachycardia is evident on the cardiac monitor. The Doppler echocardiography shows dilated cardiomyopathy. What collaborative and nursing care of this patient should be done to improve cardiac output and the quality of life? (Select all that apply.) A. Decrease preload and afterload. B. Relieve left ventricular outflow obstruction. C. Control heart failure by enhancing myocardial contractility. D. Improve diastolic filling and the underlying disease process. E. Improve ventricular filling by reducing ventricular contractility.

A. Decrease preload and afterload. C. Control heart failure by enhancing myocardial contractility. The patient is experiencing dilated cardiomyopathy. To improve cardiac output and quality of life, drug, nutrition, and cardiac rehabilitation will be focused on controlling heart failure by decreasing preload and afterload and improving cardiac output, which will improve the quality of life. Relief of left ventricular outflow obstruction and improving ventricular filling by reducing ventricular contractility is done for hypertrophic cardiomyopathy. There are no specific treatments for restrictive cardiomyopathy, but interventions are aimed at improving diastolic filling and the underlying disease process.

The patient had myocarditis and is now experiencing fatigue, weakness, palpitations, and dyspnea at rest. The nurse assesses pulmonary crackles, edema, and weak peripheral pulses. Sinoatrial tachycardia is evident on the cardiac monitor. The Doppler echocardiography shows dilated cardiomyopathy. What collaborative and nursing care of this patient should be done to improve cardiac output and the quality of life? (Select all that apply.) A. Decrease preload and afterload. B. Relieve left ventricular outflow obstruction. C. Control heart failure by enhancing myocardial contractility. D. Improve diastolic filling and the underlying disease process. E. Improve ventricular filling by reducing ventricular contractility

A. Decrease preload and afterload. C. Control heart failure by enhancing myocardial contractility. The patient is experiencing dilated cardiomyopathy. To improve cardiac output and quality of life, drug, nutrition, and cardiac rehabilitation will be focused on controlling heart failure by decreasing preload and afterload and improving cardiac output, which will improve the quality of life. Relief of left ventricular outflow obstruction and improving ventricular filling by reducing ventricular contractility is done for hypertrophic cardiomyopathy. There are no specific treatments for restrictive cardiomyopathy, but interventions are aimed at improving diastolic filling and the underlying disease process

While doing an admission assessment, the nurse notes clubbing of the patient's fingers. Based on this finding, the nurse will question the patient about which disease process? A. Endocarditis B. Acute kidney injury C. Myocardial infarction D. Chronic thrombophlebitis

A. Endocarditis Clubbing of the fingers is a loss of the normal angle between the base of the nail and the skin. This finding can be found in endocarditis, congenital defects, and/or prolonged oxygen deficiency. Clinical manifestations of acute kidney injury, myocardial infarction, and chronic thrombophlebitis will not include clubbing of the fingers.

While doing an admission assessment, the nurse notes clubbing of the patient's fingers. Based on this finding, the nurse will question the patient about which disease process? A. Endocarditis B. Acute kidney injury C. Myocardial infarction D. Chronic thrombophlebitis

A. Endocarditis Clubbing of the fingers is a loss of the normal angle between the base of the nail and the skin. This finding can be found in endocarditis, congenital defects, and/or prolonged oxygen deficiency. Clinical manifestations of acute kidney injury, myocardial infarction, and chronic thrombophlebitis will not include clubbing of the fingers.

When caring for a patient with infective endocarditis, the nurse will assess the patient for which vascular manifestations (select all that apply)? A. Osler's nodes B. Janeway's lesions C. Splinter hemorrhages D. Subcutaneous nodules E. Erythema marginatum lesions

A. Osler's nodes B. Janeway's lesions C. Splinter hemorrhages Osler's nodes, Janeway's lesions, and splinter hemorrhages are all vascular manifestations of infective endocarditis. Subcutaneous nodules and erythema marginatum lesions occur with rheumatic fever.

When caring for a patient with infective endocarditis, the nurse will assess the patient for which vascular manifestations (select all that apply)? A. Osler's nodes B. Janeway's lesions C. Splinter hemorrhages D. Subcutaneous nodules E. Erythema marginatum lesions

A. Osler's nodes B. Janeway's lesions C. Splinter hemorrhages Osler's nodes, Janeway's lesions, and splinter hemorrhages are all vascular manifestations of infective endocarditis. Subcutaneous nodules and erythema marginatum lesions occur with rheumatic fever.

The nurse is teaching a community group about preventing rheumatic fever. What information should the nurse include? A. Prompt recognition and treatment of streptococcal pharyngitis B. Completion of 4 to 6 days of antibiotic therapy for infective endocarditis of respiratory infections in children born with heart defects C. Avoidance of respiratory infections in children who have rheumatoid arthritis D. Requesting antibiotics before dental surgery for individuals with rheumatoid arthritis

A. Prompt recognition and treatment of streptococcal pharyngitis The nurse should emphasize the need for prompt and adequate treatment of streptococcal pharyngitis infection, which can lead to the complication of rheumatic fever.

While admitting a patient with pericarditis, the nurse will assess for what manifestations of this disorder? A. Pulsus paradoxus B. Prolonged PR intervals C. Widened pulse pressure D. Clubbing of the fingers

A. Pulsus paradoxus Pericarditis can lead to cardiac tamponade, an emergency situation. Pulsus paradoxus greater than 10 mm Hg is a sign of cardiac tamponade that should be assessed at least every 4 hours in a patient with pericarditis. Prolonged PR intervals occur with first-degree AV block. Widened pulse pressure occurs with valvular heart disease. Clubbing of fingers may occur in subacute forms of infective endocarditis and valvular heart disease.

While admitting a patient with pericarditis, the nurse will assess for what manifestations of this disorder? A. Pulsus paradoxus B. Prolonged PR intervals C. Widened pulse pressure D. Clubbing of the fingers

A. Pulsus paradoxus Pericarditis can lead to cardiac tamponade, an emergency situation. Pulsus paradoxus greater than 10 mm Hg is a sign of cardiac tamponade that should be assessed at least every 4 hours in a patient with pericarditis. Prolonged PR intervals occur with first-degree AV block. Widened pulse pressure occurs with valvular heart disease. Clubbing of fingers may occur in subacute forms of infective endocarditis and valvular heart disease.

What medications should the nurse expect to include in the teaching plan to decrease the risk of cardiovascular events and death for PAD patients (select all that apply)? A. Ramipril (Altace) B. Cilostazol (Pletal) C. Simvastatin (Zocor) D. Clopidogrel (Plavix) E. Warfarin (Coumadin) F. Aspirin (acetylsalicylic acid)

A. Ramipril (Altace) C. Simvastatin (Zocor) F. Aspirin (acetylsalicylic acid) Angiotensin-converting enzyme inhibitors (e.g., ramipril [Altace]) are used to control hypertension. Statins (e.g., simvastatin [Zocor]) are used for lipid management. Aspirin is used as an antiplatelet agent. Cilostazol (Pletal) is used for intermittent claudication, but it does not reduce CVD morbidity and mortality risks. Clopidogrel may be used if the patient cannot tolerate aspirin. Anticoagulants (e.g., warfarin [Coumadin]) are not recommended to prevent CVD events in PAD patients.

A patient with a recent diagnosis of heart failure has been prescribed furosemide (Lasix) in an effort to physiologically do what for the patient? A. Reduce preload. B. Decrease afterload. C. Increase contractility. D. Promote vasodilation.

A. Reduce preload. Diuretics such as furosemide are used in the treatment of HF to mobilize edematous fluid, reduce pulmonary venous pressure, and reduce preload. They do not directly influence afterload, contractility, or vessel tone.

Assessment of a patient's peripheral IV site reveals that phlebitis has developed over the past several hours. Which intervention should the nurse implement first? A. Remove the patient's IV catheter. B. Apply an ice pack to the affected area. C. Decrease the IV rate to 20 to 30 mL/hr. D. Administer prophylactic anticoagulants.

A. Remove the patient's IV catheter.

The patient with chronic heart failure is being discharged from the hospital. What information should the nurse emphasize in the patient's discharge teaching to prevent progression of the disease to ADHF? A. Take medications as prescribed. B. Use oxygen when feeling short of breath. C. Only ask the physician's office questions. D. Encourage most activity in the morning when rested.

A. Take medications as prescribed. The goal for the patient with chronic HF is to avoid exacerbations and hospitalization. Taking the medications as prescribed along with nondrug therapies such as alternating activity with rest will help the patient meet this goal. If the patient needs to use oxygen at home, it will probably be used all the time or with activity to prevent respiratory acidosis. Many HF patients are monitored by a care manager or in a transitional program to assess the patient for medication effectiveness and monitor for patient deterioration and encourage the patient. This nurse manager can be asked questions or can contact the health care provider if there is evidence of worsening HF.

The nurse is admitting a 68-year-old preoperative patient with a suspected abdominal aortic aneurysm (AAA). The medication history reveals that the patient has been taking warfarin (Coumadin) on a daily basis. Based on this history and the patient's admission diagnosis, the nurse should prepare to administer which medication? A. Vitamin K B. Cobalamin C. Heparin sodium D. Protamine sulfate

A. Vitamin K Coumadin is a Vitamin K antagonist anticoagulant that could cause excessive bleeding during surgery if clotting times are not corrected before surgery. For this reason, vitamin K is given as the antidote for warfarin (Coumadin).

A nurse cares for a client who has a heart rate averaging 56 beats/min with no adverse symptoms. Which activity modification should the nurse suggest to avoid further slowing of the heart rate? a. "Make certain that your bath water is warm." b. "Avoid straining while having a bowel movement." c. "Limit your intake of caffeinated drinks to one a day." d. "Avoid strenuous exercise such as running."

ANS: "Avoid straining while having a bowel movement." Bearing down strenuously during a bowel movement is one type of Valsalva maneuver, which stimulates the vagus nerve and results in slowing of the heart rate. Such a response is not desirable in a person who has bradycardia. The other instructions are not appropriate for this condition.

The nurse asks a client who has experienced ventricular dysrhythmias about substance abuse. The client asks, "Why do you want to know if I use cocaine?" How should the nurse respond? a. "Substance abuse puts clients at risk for many health issues." b. "The hospital requires that I ask you about cocaine use." c. "Clients who use cocaine are at risk for fatal dysrhythmias." d. "We can provide services for cessation of substance abuse."

ANS: "Clients who use cocaine are at risk for fatal dysrhythmias." Clients who use cocaine or illicit inhalants are particularly at risk for potentially fatal dysrhythmias. The other responses do not adequately address the client's question.

After teaching a client who has an implantable cardioverter-defibrillator (ICD), a nurse assesses the client's understanding. Which statement by the client indicates a correct understanding of the teaching? a. "I should wear a snug-fitting shirt over the ICD." b. "I will avoid sources of strong electromagnetic fields." c. "I should participate in a strenuous exercise program." d. "Now I can discontinue my antidysrhythmic medication."

ANS: "I will avoid sources of strong electromagnetic fields." The client being discharged with an ICD is instructed to avoid strong sources of electromagnetic fields. Clients should avoid tight clothing, which could cause irritation over the ICD generator. The client should be encouraged to exercise but should not engage in strenuous activities that cause the heart rate to meet or exceed the ICD cutoff point because the ICD can discharge inappropriately. The client should continue all prescribed medications.

A nurse teaches a client who experiences occasional premature atrial contractions (PACs) accompanied by palpitations that resolve spontaneously without treatment. Which statement should the nurse include in this client's teaching? a. "Minimize or abstain from caffeine." b. "Lie on your side until the attack subsides." c. "Use your oxygen when you experience PACs." d. "Take amiodarone (Cordarone) daily to prevent PACs."

ANS: "Minimize or abstain from caffeine." PACs usually have no hemodynamic consequences. For a client experiencing infrequent PACs, the nurse should explore possible lifestyle causes, such as excessive caffeine intake and stress. Lying on the side will not prevent or resolve PACs. Oxygen is not necessary. Although medications may be needed to control symptomatic dysrhythmias, for infrequent PACs, the client first should try lifestyle changes to control them.

A nurse teaches a client with a new permanent pacemaker. Which instructions should the nurse include in this client's teaching? (Select all that apply.) a. "Until your incision is healed, do not submerge your pacemaker. Only take showers." b. "Report any pulse rates lower than your pacemaker settings." c. "If you feel weak, apply pressure over your generator." d. "Have your pacemaker turned off before having magnetic resonance imaging (MRI)." e. "Do not lift your left arm above the level of your shoulder for 8 weeks."

ANS: "Until your incision is healed, do not submerge your pacemaker. Only take showers." "Report any pulse rates lower than your pacemaker settings." "Do not lift your left arm above the level of your shoulder for 8 weeks." The client should not submerge in water until the site has healed; after the incision is healed, the client may take showers or baths without concern for the pacemaker. The client should be instructed to report changes in heart rate or rhythm, such as rates lower than the pacemaker setting or greater than 100 beats/min. The client should be advised of restrictions on physical activity for 8 weeks to allow the pacemaker to settle in place. The client should never apply pressure over the generator and should avoid tight clothing. The client should never have MRI because, whether turned on or off, the pacemaker contains metal. The client should be advised to inform all health care providers that he or she has a pacemaker.

A patient has sought care after an episode of syncope of unknown origin. Which nursing action should you prioritize in the patient's subsequent diagnostic workup? A. Preparing to assist with a head-up tilt test B. Assessing the patient's knowledge of pacemakers C. Preparing an intravenous dose of a β-adrenergic blocker D. Teaching the patient about the role of antiplatelet aggregators

ANS: A A head-up tilt test is a common component of the diagnostic workup after episodes of syncope. Intravenous β-blockers are not indicated, and addressing pacemakers is premature and inappropriate at this stage of diagnosis. Patient education about antiplatelet aggregators is not directly relevant to the patient's syncope. Reference: 839

The charge nurse is explaining the concept of pacemaker failure to capture to the new graduate. What information should the charge nurse give? A. It occurs when the electrical charge is insufficient. B. It occurs when the pacemaker does not recognize spontaneous heart activity. C. A complication is ventricular tachycardia. D. First-line treatment when this occurs is to turn down the electrical charge.

ANS: A Failure to capture occurs when the electrical charge to the myocardium is insufficient to produce atrial or ventricular contraction. It can result in serious bradycardia, and treatment includes increasing the electrical charge. Failure to sense occurs when the pacemaker fails to recognize spontaneous atrial or ventricular activity and fires inappropriately. This can result in ventricular tachycardia. Reference: 836

The patient has a heart rate of 40 beats/minute. The P waves are regular, and the Q waves are regular, but there is no relationship between the P wave and QRS complex. What treatment do you anticipate? A. Pacemaker B. Continue to monitor C. Carotid massage D. Defibrillation

ANS: A In third-degree atrioventricular (AV) block, there is no correlation between the impulse from the atrium to the ventricles and the ventricular rhythm seen. A pacemaker eventually is required. Action must be taken because this usually results in reduced cardiac output with subsequent ischemia if untreated. Carotid massage is vagal stimulation, and it can cause bradycardia. There is a problem in conduction, not abnormal contraction, and defibrillation is not used. Reference: 830

A patient in asystole is likely to receive which drug treatments? A. Atropine and epinephrine B. Lidocaine and amiodarone C. Digoxin and procainamide D. β-Adrenergic blockers and dopamine

ANS: A Normally, the patient in asystole cannot be successfully resuscitated. However, administration of atropine and epinephrine may prompt the return of depolarization and ventricular contraction. Reference: 832

Which assessment finding in a patient admitted with acute decompensated heart failure (ADHF) requires the most immediate action by the nurse? a. Oxygen saturation of 88% b. Weight gain of 1 kg (2.2 lb) c. Heart rate of 106 beats/minute d. Urine output of 50 mL over 2 hours

ANS: A A decrease in oxygen saturation to less than 92% indicates hypoxemia. The nurse should administer supplemental oxygen immediately to the patient. An increase in apical pulse rate, 1-kg weight gain, and decreases in urine output also indicate worsening heart failure and require nursing actions, but the low oxygen saturation rate requires the most immediate nursing action.

The nurse has just finished teaching a hypertensive patient about the newly prescribed ramipril (Altace). Which patient statement indicates that more teaching is needed? a. "A little swelling around my lips and face is okay." b. "The medication may not work as well if I take any aspirin." c. "The doctor may order a blood potassium level occasionally." d. "I will call the doctor if I notice that I have a frequent cough."

ANS: A Angioedema occurring with angiotensin-converting enzyme (ACE) inhibitor therapy is an indication that the ACE inhibitor should be discontinued. The patient should be taught that if any swelling of the face or oral mucosa occurs, the health care provider should be immediately notified because this could be life threatening. The other patient statements indicate that the patient has an accurate understanding of ACE inhibitor therapy

An appropriate nursing intervention for a patient with non-Hodgkin's lymphoma whose platelet count drops to 18,000/µL during chemotherapy is to a. check all stools for occult blood. b. encourage fluids to 3000 mL/day. c. provide oral hygiene every 2 hours. d. check the temperature every 4 hours.

ANS: A Because the patient is at risk for spontaneous bleeding, the nurse should check stools for occult blood. A low platelet count does not require an increased fluid intake. Oral hygiene is important, but it is not necessary to provide oral care every 2 hours. The low platelet count does not increase risk for infection, so frequent temperature monitoring is not indicated

The nurse is caring for a 78-year-old patient with aortic stenosis. Which assessment data obtained by the nurse would be most important to report to the health care provider? a. The patient complains of chest pressure when ambulating. b. A loud systolic murmur is heard along the right sternal border. c. A thrill is palpated at the second intercostal space, right sternal border. d. The point of maximum impulse (PMI) is at the left midclavicular line.

ANS: A Chest pressure (or pain) occurring with aortic stenosis is caused by cardiac ischemia, and reporting this information would be a priority. A systolic murmur and thrill are expected in a patient with aortic stenosis. A PMI at the left midclavicular line is normal.

The nurse is caring for a 64-year-old patient admitted with mitral valve regurgitation. Which information obtained by the nurse when assessing the patient should be communicated to the health care provider immediately? a. The patient has bilateral crackles. b. The patient has bilateral, 4+ peripheral edema. c. The patient has a loud systolic murmur across the precordium. d. The patient has a palpable thrill felt over the left anterior chest.

ANS: A Crackles that are audible throughout the lungs indicate that the patient is experiencing severe left ventricular failure with pulmonary congestion and needs immediate interventions such as diuretics. A systolic murmur and palpable thrill would be expected in a patient with mitral regurgitation. Although 4+ peripheral edema indicates a need for a change in therapy, it does not need to be addressed urgently.

A patient is admitted to the hospital with possible acute pericarditis. The nurse should plan to teach the patient about the purpose of a. echocardiography. b. daily blood cultures. c. cardiac catheterization. d. 24-hour Holter monitor.

ANS: A Echocardiograms are useful in detecting the presence of the pericardial effusions associated with pericarditis. Blood cultures are not indicated unless the patient has evidence of sepsis. Cardiac catheterization and 24-hour Holter monitor is not a diagnostic procedure for pericarditis.

Which menu choice indicates that the patient understands the nurse's teaching about best dietary choices for iron-deficiency anemia? a. Omelet and whole wheat toast b. Cantaloupe and cottage cheese c. Strawberry and banana fruit plate d. Cornmeal muffin and orange juice

ANS: A Eggs and whole grain breads are high in iron. The other choices are appropriate for other nutritional deficiencies but are not the best choice for a patient with iron-deficiency anemia

Which information obtained by the nurse assessing a patient admitted with multiple myeloma is most important to report to the health care provider? a. Serum calcium level is 15 mg/dL. b. Patient reports no stool for 5 days. c. Urine sample has Bence-Jones protein. d. Patient is complaining of severe back pain.

ANS: A Hypercalcemia may lead to complications such as dysrhythmias or seizures, and should be addressed quickly. The other patient findings will also be discussed with the health care provider, but are not life threatening

Which patient statement to the nurse indicates a need for additional instruction about taking oral ferrous sulfate? a. "I will call my health care provider if my stools turn black." b. "I will take a stool softener if I feel constipated occasionally." c. "I should take the iron with orange juice about an hour before eating." d. "I should increase my fluid and fiber intake while I am taking iron tablets."

ANS: A It is normal for the stools to appear black when a patient is taking iron, and the patient should not call the doctor about this. The other patient statements are correct

A patient in the intensive care unit with acute decompensated heart failure (ADHF) complains of severe dyspnea and is anxious, tachypneic, and tachycardic. All of the following medications have been ordered for the patient. The nurse's priority action will be to a. give IV morphine sulfate 4 mg. b. give IV diazepam (Valium) 2.5 mg. c. increase nitroglycerin (Tridil) infusion by 5 mcg/min. d. increase dopamine (Intropin) infusion by 2 mcg/kg/min.

ANS: A Morphine improves alveolar gas exchange, improves cardiac output by reducing ventricular preload and afterload, decreases anxiety, and assists in reducing the subjective feeling of dyspnea. Diazepam may decrease patient anxiety, but it will not improve the cardiac output or gas exchange. Increasing the dopamine may improve cardiac output, but it will also increase the heart rate and myocardial oxygen consumption. Nitroglycerin will improve cardiac output and may be appropriate for this patient, but it will not directly reduce anxiety and will not act as quickly as morphine to decrease dyspnea.

A patient in the intensive care unit with acute decompensated heart failure (ADHF) complains of severe dyspnea and is anxious, tachypneic, and tachycardic. Several drugs have been ordered for the patient. The nurse's priority action will be to a. give PRN IV morphine sulfate 4 mg. b. give PRN IV diazepam (Valium) 2.5 mg. c. increase nitroglycerin infusion by 5 mcg/min. d. increase dopamine infusion by 2 mcg/kg/min.

ANS: A Morphine improves alveolar gas exchange, improves cardiac output by reducing ventricular preload and afterload, decreases anxiety, and assists in reducing the subjective feeling of dyspnea. Diazepam may decrease patient anxiety, but it will not improve the cardiac output or gas exchange. Increasing the dopamine may improve cardiac output, but it will also increase the heart rate and myocardial oxygen consumption. Nitroglycerin will improve cardiac output and may be appropriate for this patient, but it will not directly reduce anxiety and will not act as quickly as morphine to decrease dyspnea.

Propranolol (Inderal) is prescribed for a patient diagnosed with hypertension. The nurse should consult with the health care provider before giving this medication when the patient reveals a history of a. asthma. b. daily alcohol use. c. peptic ulcer disease. d. myocardial infarction (MI).

ANS: A Nonselective b-blockers block b1- and b2-adrenergic receptors and can cause bronchospasm, especially in patients with a history of asthma. b-Blockers will have no effect on the patient's peptic ulcer disease or alcohol use. b-Blocker therapy is recommended after MI

A patient is diagnosed with hypertension and nadolol (Corgard) is prescribed. The nurse should consult with the health care provider before giving this medication upon finding a history of a. asthma. b. peptic ulcer disease. c. alcohol dependency. d. myocardial infarction (MI).

ANS: A Nonselective β-blockers block β1- and β2-adrenergic receptors and can cause bronchospasm, especially in patients with a history of asthma. β-blockers will have no effect on the patient's peptic ulcer disease or alcohol dependency. β-blocker therapy is recommended after MI.

A 19-year-old woman with immune thrombocytopenic purpura (ITP) has an order for a platelet transfusion. Which information indicates that the nurse should consult with the health care provider before obtaining and administering platelets? a. The platelet count is 42,000/mL. b. Petechiae are present on the chest. c. Blood pressure (BP) is 94/56 mm Hg. d. Blood is oozing from the venipuncture site.

ANS: A Platelet transfusions are not usually indicated until the platelet count is below 10,000 to 20,000/mL unless the patient is actively bleeding. Therefore the nurse should clarify the order with the health care provider before giving the transfusion. The other data all indicate that bleeding caused by ITP may be occurring and that the platelet transfusion is appropriate

The nurse suspects cardiac tamponade in a patient who has acute pericarditis. To assess for the presence of pulsus paradoxus, the nurse should a. note when Korotkoff sounds are auscultated during both inspiration and expiration. b. subtract the diastolic blood pressure (DBP) from the systolic blood pressure (SBP). c. check the electrocardiogram (ECG) for variations in rate during the respiratory cycle. d. listen for a pericardial friction rub that persists when the patient is instructed to stop breathing.

ANS: A Pulsus paradoxus exists when there is a gap of greater than 10 mm Hg between when Korotkoff sounds can be heard during only expiration and when they can be heard throughout the respiratory cycle. The other methods described would not be useful in determining the presence of pulsus paradoxus.

While caring for a patient with aortic stenosis, the nurse identifies a nursing diagnosis of acute pain related to decreased coronary blood flow. A priority nursing intervention for this patient would be to a. promote rest to decrease myocardial oxygen demand. b. teach the patient about the need for anticoagulant therapy. c. teach the patient to use sublingual nitroglycerin for chest pain. d. raise the head of the bed 60 degrees to decrease venous return.

ANS: A Rest is recommended to balance myocardial oxygen supply and demand and to decrease chest pain. The patient with aortic stenosis requires higher preload to maintain cardiac output, so nitroglycerin and measures to decrease venous return are contraindicated. Anticoagulation is not recommended unless the patient has atrial fibrillation.

The charge nurse observes a new registered nurse (RN) doing discharge teaching for a patient with hypertension who has a new prescription for enalapril (Vasotec). The charge nurse will need to intervene if the new RN tells the patient to a. increase the dietary intake of high-potassium foods. b. make an appointment with the dietitian for teaching. c. check the blood pressure (BP) with a home BP monitor at least once a day. d. move slowly when moving from lying to sitting to standing.

ANS: A The ACE inhibitors cause retention of potassium by the kidney, so hyperkalemia is a possible adverse effect. The other teaching by the new RN is appropriate for a patient with newly diagnosed hypertension who has just started therapy with enalapril

The nurse is reviewing the laboratory tests for a patient who has recently been diagnosed with hypertension. Which result is most important to communicate to the health care provider? a. Serum creatinine of 2.6 mg/dL b. Serum potassium of 3.8 mEq/L c. Serum hemoglobin of 14.7 g/dL d. Blood glucose level of 98 mg/dL

ANS: A The elevated creatinine indicates renal damage caused by the hypertension. The other laboratory results are normal.

The nurse is reviewing the laboratory test results for a patient who has recently been diagnosed with hypertension. Which result is *most* important to communicate to the health care provider? a. Serum creatinine of 2.8 mg/dL c. Serum hemoglobin of 14.7 g/dL b. Serum potassium of 4.5 mEq/L d. Blood glucose level of 96 mg/dL

ANS: A The elevated serum creatinine indicates renal damage caused by the hypertension. The other laboratory results are normal.

A nurse prepares to discharge a client with cardiac dysrhythmia who is prescribed home health care services. Which priority information should be communicated to the home health nurse upon discharge? a. Medication reconciliation b. Immunization history c. Religious beliefs d. Nutrition preferences

ANS: A The home health nurse needs to know current medications the client is taking to ensure assessment, evaluation, and further education related to these medications. The other information will not assist the nurse to develop a plan of care for the client.

The nurse caring for a patient with type A hemophilia being admitted to the hospital with severe pain and swelling in the right knee will a. immobilize the joint. b. apply heat to the knee. c. assist the patient with light weight bearing. d. perform passive range of motion to the knee.

ANS: A The initial action should be total rest of the knee to minimize bleeding. Ice packs are used to decrease bleeding. Range of motion (ROM) and weight-bearing exercise are contraindicated initially, but after the bleeding stops, ROM and physical therapy are started

Which nursing action should the nurse take first in order to assist a patient with newly diagnosed stage 1 hypertension in making needed dietary changes? a. Have the patient record dietary intake for 3 days. b. Give the patient a detailed list of low-sodium foods. c. Teach the patient about foods that are high in sodium. d. Help the patient make an appointment with a dietitian.

ANS: A The initial nursing action should be assessment of the patient's baseline dietary intake through a 3-day food diary. The other actions may be appropriate, but assessment of the patient's baseline should occur first.

Which nursing action should the nurse take *first* to assist a patient with newly diagnosed stage 1 hypertension in making needed dietary changes? a. Collect a detailed diet history. b. Provide a list of low-sodium foods. c. Help the patient make an appointment with a dietitian. d. Teach the patient about foods that are high in potassium.

ANS: A The initial nursing action should be assessment of the patient's baseline dietary intake through a thorough diet history. The other actions may be appropriate, but assessment of the patient's baseline should occur first.

After receiving change-of-shift report on a heart failure unit, which patient should the nurse assess first? a. A patient who is cool and clammy, with new-onset confusion and restlessness b. A patient who has crackles bilaterally in the lung bases and is receiving oxygen. c. A patient who had dizziness after receiving the first dose of captopril (Capoten) d. A patient who is receiving IV nesiritide (Natrecor) and has a blood pressure of 100/62

ANS: A The patient who has "wet-cold" clinical manifestations of heart failure is perfusing inadequately and needs rapid assessment and changes in management. The other patients also should be assessed as quickly as possible but do not have indications of severe decreases in tissue perfusion.

The nurse on the intermediate care unit received change-of-shift report on four patients with hypertension. Which patient should the nurse assess *first*? a. 48-yr-old with a blood pressure of 160/92 mm Hg who reports chest pain b. 52-yr-old with a blood pressure of 198/90 mm Hg who has intermittent claudication c. 50-yr-old with a blood pressure of 190/104 mm Hg who has a creatinine of 1.7 mg/dL d. 43-yr-old with a blood pressure of 172/98 mm Hg whose urine shows microalbuminuria

ANS: A The patient with chest pain may be experiencing acute myocardial infarction and rapid assessment and intervention are needed. The symptoms of the other patients also show target organ damage but are not indicative of acute processes.

The nurse on the intermediate care unit received change-of-shift report on four patients with hypertension. Which patient should the nurse assess first? a. 43-year-old with a (blood pressure (BP) of 160/92 who is complaining of chest pain b. 52-year-old with a BP of 212/90 who has intermittent claudication c. 50-year-old with a BP of 190/104 who has a creatinine of 1.7 mg/dL d. 48-year-old with a BP of 172/98 whose urine shows microalbuminuria

ANS: A The patient with chest pain may be experiencing acute myocardial infarction, and rapid assessment and intervention are needed. The symptoms of the other patients also show target organ damage but are not indicative of acute processes

Two days after an acute myocardial infarction (MI), a patient complains of stabbing chest pain that increases with a deep breath. Which action will the nurse take first? a. Auscultate the heart sounds. b. Check the patient's temperature. c. Notify the patient's health care provider. d. Give the PRN acetaminophen (Tylenol).

ANS: A The patient's clinical manifestations and history are consistent with pericarditis, and the first action by the nurse should be to listen for a pericardial friction rub. Checking the temperature and notifying the health care provider are also appropriate actions but would not be done before listening for a rub. It is not stated for what symptom (e.g., headache) or finding (e.g., increased temperature) the PRN acetaminophen (Tylenol) is ordered.

During a visit to a 78-yr-old patient with chronic heart failure, the home care nurse finds that the patient has ankle edema, a 2-kg weight gain over the past 2 days, and complains of "feeling too tired to get out of bed." Based on these data, a correct nursing diagnosis for the patient is a. activity intolerance related to fatigue. b. impaired skin integrity related to edema. c. disturbed body image related to weight gain. d. impaired gas exchange related to dyspnea on exertion.

ANS: A The patient's statement supports the diagnosis of activity intolerance. There are no data to support the other diagnoses, although the nurse will need to assess for additional patient problems.

During a visit to a 78-year-old with chronic heart failure, the home care nurse finds that the patient has ankle edema, a 2-kg weight gain over the past 2 days, and complains of "feeling too tired to get out of bed." Based on these data, the best nursing diagnosis for the patient is a. activity intolerance related to fatigue. b. disturbed body image related to weight gain. c. impaired skin integrity related to ankle edema. d. impaired gas exchange related to dyspnea on exertion.

ANS: A The patient's statement supports the diagnosis of activity intolerance. There are no data to support the other diagnoses, although the nurse will need to assess for other patient problems.

A nurse supervises an unlicensed assistive personnel (UAP) applying electrocardiographic monitoring. Which statement should the nurse provide to the UAP related to this procedure? a. "Clean the skin and clip hairs if needed." b. "Add gel to the electrodes prior to applying them." c. "Place the electrodes on the posterior chest." d. "Turn off oxygen prior to monitoring the client."

ANS: A To ensure the best signal transmission, the skin should be clean and hairs clipped. Electrodes should be placed on the anterior chest, and no additional gel is needed. Oxygen has no impact on electrocardiographic monitoring.

A nurse cares for a client who is on a cardiac monitor. The monitor displayed the rhythm shown below: Which action should the nurse take first? a. Assess airway, breathing, and level of consciousness. b. Administer an amiodarone bolus followed by a drip. c. Cardiovert the client with a biphasic defibrillator. d. Begin cardiopulmonary resuscitation (CPR).

ANS: A Ventricular tachycardia occurs with repetitive firing of an irritable ventricular ectopic focus, usually at a rate of 140 to 180 beats/min or more. Ventricular tachycardia is a lethal dysrhythmia. The nurse should first assess if the client is alert and breathing. Then the nurse should call a Code Blue and begin CPR. If this client is pulseless, the treatment of choice is defibrillation. Amiodarone is the antidysrhythmic of choice, but it is not the first action.

A nurse is assessing clients on a medical-surgical unit. Which client should the nurse identify as being at greatest risk for atrial fibrillation? a. A 45-year-old who takes an aspirin daily b. A 50-year-old who is post coronary artery bypass graft surgery c. A 78-year-old who had a carotid endarterectomy d. An 80-year-old with chronic obstructive pulmonary disease

ANS: A 50-year-old who is post coronary artery bypass graft surgery Atrial fibrillation occurs commonly in clients with cardiac disease and is a common occurrence after coronary artery bypass graft surgery. The other conditions do not place these clients at higher risk for atrial fibrillation.

A nurse is teaching a client with premature ectopic beats. Which education should the nurse include in this client's teaching? (Select all that apply.) a. Smoking cessation b. Stress reduction and management c. Avoiding vagal stimulation d. Adverse effects of medications e. Foods high in potassium

ANS: A, B, D A client who has premature beats or ectopic rhythms should be taught to stop smoking, manage stress, take medications as prescribed, and report adverse effects of medications. Clients with premature beats are not at risk for vasovagal attacks or potassium imbalances.

A nurse teaches a client with a new permanent pacemaker. Which instructions should the nurse include in this client's teaching? (Select all that apply.) a. "Until your incision is healed, do not submerge your pacemaker. Only take showers." b. "Report any pulse rates lower than your pacemaker settings." c. "If you feel weak, apply pressure over your generator." d. "Have your pacemaker turned off before having magnetic resonance imaging (MRI)." e. "Do not lift your left arm above the level of your shoulder for 8 weeks."

ANS: A, B, E The client should not submerge in water until the site has healed; after the incision is healed, the client may take showers or baths without concern for the pacemaker. The client should be instructed to report changes in heart rate or rhythm, such as rates lower than the pacemaker setting or greater than 100 beats/min. The client should be advised of restrictions on physical activity for 8 weeks to allow the pacemaker to settle in place. The client should never apply pressure over the generator and should avoid tight clothing. The client should never have MRI because, whether turned on or off, the pacemaker contains metal. The client should be advised to inform all health care providers that he or she has a pacemaker.

1. Based on the Joint Commission Core Measures for patients with heart failure, which topics should the nurse include in the discharge teaching plan for a patient who has been hospitalized with chronic heart failure (select all that apply)? a. How to take and record daily weight b. Importance of limiting aerobic exercise c. Date and time of follow-up appointment d. Symptoms indicating worsening heart failure e. Actions and side effects of prescribed medications

ANS: A, C, D, E The Joint Commission Core Measures state that patients should be taught about prescribed medications, follow-up appointments, weight monitoring, and actions to take for worsening symptoms. Patients with heart failure are encouraged to begin or continue aerobic exercises such as walking, while self-monitoring to avoid excessive fatigue. DIF: Cognitive Level: Apply (application) REF: 779 TOP: Nursing Process: Planning MSC: NCLEX: Physiological Integrity

Based on the Joint Commission Core Measures for patients with heart failure, which topics should the nurse include in the discharge teaching plan for a patient who has been hospitalized with chronic heart failure (select all that apply)? a. How to take and record daily weight b. Importance of limiting aerobic exercise c. Date and time of follow-up appointment d. Symptoms indicating worsening heart failure e. Actions and side effects of prescribed medications

ANS: A, C, D, E The Joint Commission Core Measures state that patients should be taught about prescribed medications, follow-up appointments, weight monitoring, and actions to take for worsening symptoms. Patients with heart failure are encouraged to begin or continue aerobic exercises such as walking, while self-monitoring to avoid excessive fatigue.

A nurse performs an admission assessment on a 75-year-old client with multiple chronic diseases. The client's blood pressure is 135/75 mm Hg and oxygen saturation is 94% on 2 liters per nasal cannula. The nurse assesses the client's rhythm on the cardiac monitor and observes the reading shown below: Which action should the nurse take first? a. Begin external temporary pacing. b. Assess peripheral pulse strength. c. Ask the client what medications he or she takes. d. Administer 1 mg of atropine.

ANS: Ask the client what medications he or she takes. This client is stable and therefore does not require any intervention except to determine the cause of the bradycardia. Bradycardia is often caused by medications. Clients who have multiple chronic diseases are often on multiple medications that can interact with each other. The nurse should assess the client's current medications first.

A nurse cares for a client who is on a cardiac monitor. The monitor displayed the rhythm shown below: Which action should the nurse take first? a. Assess airway, breathing, and level of consciousness. b. Administer an amiodarone bolus followed by a drip. c. Cardiovert the client with a biphasic defibrillator. d. Begin cardiopulmonary resuscitation (CPR).

ANS: Assess airway, breathing, and level of consciousness. Ventricular tachycardia occurs with repetitive firing of an irritable ventricular ectopic focus, usually at a rate of 140 to 180 beats/min or more. Ventricular tachycardia is a lethal dysrhythmia. The nurse should first assess if the client is alert and breathing. Then the nurse should call a Code Blue and begin CPR. If this client is pulseless, the treatment of choice is defibrillation. Amiodarone is the antidysrhythmic of choice, but it is not the first action.

A nurse cares for a client with an intravenous temporary pacemaker for bradycardia. The nurse observes the presence of a pacing spike but no QRS complex on the client's electrocardiogram. Which action should the nurse take next? a. Administer intravenous diltiazem (Cardizem). b. Assess vital signs and level of consciousness. c. Administer sublingual nitroglycerin. d. Assess capillary refill and temperature.

ANS: Assess vital signs and level of consciousness. In temporary pacing, the wires are threaded onto the epicardial surface of the heart and exit through the chest wall. The pacemaker spike should be followed immediately by a QRS complex. Pacing spikes seen without subsequent QRS complexes imply loss of capture. If there is no capture, then there is no ventricular depolarization and contraction. The nurse should assess for cardiac output via vital signs and level of consciousness. The other interventions would not determine if the client is tolerating the loss of capture.

You are watching the cardiac monitor, and a patient's rhythm suddenly changes. There are no P waves. Instead, there are fine, wavy lines between the QRS complexes. The QRS complexes each measure 0.08 second (narrow), but they occur irregularly with a rate of 120 beats/minute. You correctly interpret that this rhythm is A. sinus tachycardia. B. atrial fibrillation. C. ventricular fibrillation. D. ventricular tachycardia.

ANS: B Atrial fibrillation is represented on the cardiac monitor by irregular R-R intervals and small fibrillatory (F) waves. There are no normal P waves because the atria are not contracting, just fibrillating. Reference: 827

The patient has a pacemaker set for 70 beats/minute. When taking the patient's pulse, you obtain a heart rate of 60 beats/minute. What is the best interpretation of this finding? A. The patient's heart has become more effective. B. The pacemaker is not working properly. C. The patient is tolerating a lower heart rate now. D. The pacemaker is sensing a ventricular rhythm.

ANS: B If the pacemaker is set for 70 beats/minute, a slower rate means that the pacemaker is not working properly and the patient's spontaneous rate has taken over. This situation needs to be evaluated. Reference: 836-837

The patient has a permanent cardiac pacemaker. On the electrocardiographic tracing, you notice a spike before each P wave. What action should you take? A. Assess the patient for syncope. B. Document the findings. C. Notify the physician. D. Take blood pressure in both arms.

ANS: B These pacer spikes show that the pacemaker is firing and the atrium is responding to the impulse. It is a normal, expected finding in this situation. Reference: 834-835

You obtain a 6-second rhythm strip, and document the following analysis: atrial rate of 70 beats/minute, regular; ventricular rate of 40 beats/minute, regular; QRS of 0.04 second; no relationship between P waves and QRS complexes; and atria and ventricles beating independently of each other. What is the correct interpretation of this rhythm strip? A. Sinus dysrhythmias B. Third-degree heart block C. Wenckebach phenomenon D. Premature ventricular contractions

ANS: B Third-degree heart block represents a loss of communication between the atrium and ventricles. This is depicted on the rhythm strip because there is no relationship between the P waves, representing atrial contraction, and QRS complexes, representing ventricular contraction. The atria are beating totally on their own at 70 beats/minute, whereas the ventricles are pacing themselves at 40 beats/minute. Reference: 830

After receiving change-of-shift report for several patients with neutropenia, which patient should the nurse assess first? a. 56-year-old with frequent explosive diarrhea b. 33-year-old with a fever of 100.8° F (38.2° C) c. 66-year-old who has white pharyngeal lesions d. 23-year old who is complaining of severe fatigue

ANS: B Any fever in a neutropenic patient indicates infection and can quickly lead to sepsis and septic shock. Rapid assessment and (if prescribed) initiation of antibiotic therapy within 1 hour are needed. The other patients also need to be assessed but do not exhibit symptoms of potentially life-threatening problems

A nurse is assessing clients on a medical-surgical unit. Which client should the nurse identify as being at greatest risk for atrial fibrillation? a. A 45-year-old who takes an aspirin daily b. A 50-year-old who is post coronary artery bypass graft surgery c. A 78-year-old who had a carotid endarterectomy d. An 80-year-old with chronic obstructive pulmonary disease

ANS: B Atrial fibrillation occurs commonly in clients with cardiac disease and is a common occurrence after coronary artery bypass graft surgery. The other conditions do not place these clients at higher risk for atrial fibrillation.

A nurse evaluates prescriptions for a client with chronic atrial fibrillation. Which medication should the nurse expect to find on this client's medication administration record to prevent a common complication of this condition? a. Sotalol (Betapace) b. Warfarin (Coumadin) c. Atropine (Sal-Tropine) d. Lidocaine (Xylocaine)

ANS: B Atrial fibrillation puts clients at risk for developing emboli. Clients at risk for emboli are treated with anticoagulants, such as heparin, enoxaparin, or warfarin. Sotalol, atropine, and lidocaine are not appropriate for this complication.

A nurse cares for a client who has a heart rate averaging 56 beats/min with no adverse symptoms. Which activity modification should the nurse suggest to avoid further slowing of the heart rate? a. "Make certain that your bath water is warm." b. "Avoid straining while having a bowel movement." c. "Limit your intake of caffeinated drinks to one a day." d. "Avoid strenuous exercise such as running."

ANS: B Bearing down strenuously during a bowel movement is one type of Valsalva maneuver, which stimulates the vagus nerve and results in slowing of the heart rate. Such a response is not desirable in a person who has bradycardia. The other instructions are not appropriate for this condition.

A patient with hypertension who has just started taking atenolol (Tenormin) returns to the health clinic after 2 weeks for a follow-up visit. The blood pressure (BP) is unchanged from the previous visit. Which action should the nurse take *first*? a. Tell the patient why a change in drug dosage is needed. b. Ask the patient if the medication is being taken as prescribed. c. Inform the patient that multiple drugs are often needed to treat hypertension. d. Question the patient regarding any lifestyle changes made to help control BP.

ANS: B Because nonadherence with antihypertensive therapy is common, the nurse's initial action should be to determine whether the patient is taking the atenolol as prescribed. The other actions also may be implemented, but these would be done after assessing patient adherence with the prescribed therapy.

A patient with hypertension who has just started taking atenolol (Tenormin) returns to the health clinic after 2 weeks for a follow-up visit. The blood pressure (BP) is unchanged from the previous visit. Which action should the nurse take first? a. Inform the patient about the reasons for a possible change in drug dosage. b. Question the patient about whether the medication is actually being taken. c. Inform the patient that multiple drugs are often needed to treat hypertension. d. Question the patient regarding any lifestyle changes made to help control BP.

ANS: B Because noncompliance with antihypertensive therapy is common, the nurse's initial action should be to determine whether the patient is taking the atenolol as prescribed. The other actions also may be implemented, but these would be done after assessing patient compliance with the prescribed therapy

Which collaborative problem will the nurse include in a care plan for a patient admitted to the hospital with idiopathic aplastic anemia? a. Potential complication: seizures b. Potential complication: infection c. Potential complication: neurogenic shock d. Potential complication: pulmonary edema

ANS: B Because the patient with aplastic anemia has pancytopenia, the patient is at risk for infection and bleeding. There is no increased risk for seizures, neurogenic shock, or pulmonary edema

A nurse assesses a client with atrial fibrillation. Which manifestation should alert the nurse to the possibility of a serious complication from this condition? a. Sinus tachycardia b. Speech alterations c. Fatigue d. Dyspnea with activity

ANS: B Clients with atrial fibrillation are at risk for embolic stroke. Evidence of embolic events includes changes in mentation, speech, sensory function, and motor function. Clients with atrial fibrillation often have a rapid ventricular response as a result. Fatigue is a nonspecific complaint. Clients with atrial fibrillation often have dyspnea as a result of the decreased cardiac output caused by the rhythm disturbance.

The nurse identifies the nursing diagnosis of decreased cardiac output related to valvular insufficiency for the patient with infective endocarditis (IE) based on which assessment finding(s)? a. Fever, chills, and diaphoresis b. Urine output less than 30 mL/hr c. Petechiae on the inside of the mouth and conjunctiva d. Increase in heart rate of 15 beats/minute with walking

ANS: B Decreased renal perfusion caused by inadequate cardiac output will lead to decreased urine output. Petechiae, fever, chills, and diaphoresis are symptoms of IE, but are not caused by decreased cardiac output. An increase in pulse rate of 15 beats/minute is normal with exercise.

Which information obtained by the nurse caring for a patient with thrombocytopenia should be immediately communicated to the health care provider? a. The platelet count is 52,000/µL. b. The patient is difficult to arouse. c. There are purpura on the oral mucosa. d. There are large bruises on the patient's back.

ANS: B Difficulty in arousing the patient may indicate a cerebral hemorrhage, which is life threatening and requires immediate action. The other information should be documented and reported but would not be unusual in a patient with thrombocytopenia

A nurse assists with the cardioversion of a client experiencing acute atrial fibrillation. Which action should the nurse take prior to the initiation of cardioversion? a. Administer intravenous adenosine. b. Turn off oxygen therapy. c. Ensure a tongue blade is available. d. Position the client on the left side.

ANS: B For safety during cardioversion, the nurse should turn off any oxygen therapy to prevent fire. The other interventions are not appropriate for a cardioversion. The client should be placed in a supine position.

Which assessment finding for a patient who is receiving IV furosemide (Lasix) to treat stage 2 hypertension is most important to report to the health care provider? a. Blood glucose level of 175 mg/dL b. Blood potassium level of 3.0 mEq/L c. Most recent blood pressure (BP) reading of 168/94 mm Hg d. Orthostatic systolic BP decrease of 12 mm Hg

ANS: B Hypokalemia is a frequent adverse effect of the loop diuretics and can cause life-threatening dysrhythmias. The health care provider should be notified of the potassium level immediately and administration of potassium supplements initiated. The elevated blood glucose and BP also indicate a need for collaborative interventions but will not require action as urgently as the hypokalemia. An orthostatic drop of 12 mm Hg is common and will require intervention only if the patient is symptomatic

Which assessment finding for a patient who is receiving furosemide (Lasix) to treat stage 2 hypertension is most important to report to the health care provider? a. Blood glucose level of 180 mg/dL b. Blood potassium level of 3.0 mEq/L c. Early morning BP reading of 164/96 mm Hg d. Orthostatic systolic BP decrease of 12 mm Hg

ANS: B Hypokalemia is a frequent adverse effect of the loop diuretics and can cause life-threatening dysrhythmias. The health care provider should be notified of the potassium level immediately and administration of potassium supplements initiated. The elevated blood glucose and BP also indicate a need for collaborative interventions but will not require action as urgently as the hypokalemia. An orthostatic drop of 12 mm Hg is common and will require intervention only if the patient is symptomatic.

Which assessment finding for a patient who is receiving IV furosemide (Lasix) to treat stage 2 hypertension is *most* important to report to the health care provider? a. Blood glucose level of 175 mg/dL b. Serum potassium level of 3.0 mEq/L c. Orthostatic systolic BP decrease of 12 mm Hg d. Most recent blood pressure (BP) reading of 168/94 mm Hg

ANS: B Hypokalemia is a frequent adverse effect of the loop diuretics and can cause life-threatening dysrhythmias. The health care provider should be notified of the potassium level immediately and administration of potassium supplements initiated. The elevated blood glucose and BP also indicate a need for collaborative interventions but will not require action as urgently as the hypokalemia. An orthostatic drop of 12 mm Hg will require intervention only if the patient is symptomatic.

After receiving report on the following patients, which patient should the nurse assess first? a. Patient with rheumatic fever who has sharp chest pain with a deep breath b. Patient with acute aortic regurgitation whose blood pressure is 86/54 mm Hg c. Patient with infective endocarditis who has a murmur and splinter hemorrhages d. Patient with dilated cardiomyopathy who has bilateral crackles at the lung bases

ANS: B Hypotension in patients with acute aortic regurgitation may indicate cardiogenic shock. The nurse should immediately assess this patient for other findings such as dyspnea or chest pain. The findings in the other patients are typical of their diagnoses and do not indicate a need for urgent assessment and intervention.

After assessing a client who is receiving an amiodarone intravenous infusion for unstable ventricular tachycardia, the nurse documents the findings and compares these with the previous assessment findings: Vital Signs Nursing Assessment Time: 0800 Temperature: 98° F Heart rate: 68 beats/min Blood pressure: 135/60 mm Hg Respiratory rate: 14 breaths/min Oxygen saturation: 96% Oxygen therapy: 2 L nasal cannula Time: 1000 Temperature: 98.2° F Heart rate: 50 beats/min Blood pressure: 132/57 mm Hg Respiratory rate: 16 breaths/min Oxygen saturation: 95% Oxygen therapy: 2 L nasal cannula Time: 0800 Client alert and oriented. Cardiac rhythm: normal sinus rhythm. Skin: warm, dry, and appropriate for race. Respirations equal and unlabored. Client denies shortness of breath and chest pain. Time: 1000 Client alert and oriented. Cardiac rhythm: sinus bradycardia. Skin: warm, dry, and appropriate for race. Respirations equal and unlabored. Client denies shortness of breath and chest pain. Client voids 420 mL of clear yellow urine. Based on the assessments, which action should the nurse take? a. Stop the infusion and flush the IV. b. Slow the amiodarone infusion rate. c. Administer IV normal saline. d. Ask the client to cough and deep breathe.

ANS: B IV administration of amiodarone may cause bradycardia and atrioventricular (AV) block. The correct action for the nurse to take at this time is to slow the infusion, because the client is asymptomatic and no evidence reveals AV block that might require pacing. Abruptly ceasing the medication could allow fatal dysrhythmias to occur. The administration of IV fluids and encouragement of coughing and deep breathing exercises are not indicated, and will not increase the client's heart rate.

A nurse cares for a client with an intravenous temporary pacemaker for bradycardia. The nurse observes the presence of a pacing spike but no QRS complex on the client's electrocardiogram. Which action should the nurse take next? a. Administer intravenous diltiazem (Cardizem). b. Assess vital signs and level of consciousness. c. Administer sublingual nitroglycerin. d. Assess capillary refill and temperature.

ANS: B In temporary pacing, the wires are threaded onto the epicardial surface of the heart and exit through the chest wall. The pacemaker spike should be followed immediately by a QRS complex. Pacing spikes seen without subsequent QRS complexes imply loss of capture. If there is no capture, then there is no ventricular depolarization and contraction. The nurse should assess for cardiac output via vital signs and level of consciousness. The other interventions would not determine if the client is tolerating the loss of capture.

An older patient has been diagnosed with possible white coat hypertension. Which planned action by the nurse *best* addresses the suspected cause of the hypertension? a. Instruct the patient about the need to decrease stress levels. b. Teach the patient how to self-monitor and record BPs at home. c. Schedule the patient for regular blood pressure (BP) checks in the clinic. d. Inform the patient and caregiver that major dietary changes will be needed.

ANS: B In the phenomenon of "white coat" hypertension, patients have elevated BP readings in a clinical setting and normal readings when BP is measured elsewhere. Having the patient self-monitor BPs at home will provide a reliable indication about whether the patient has hypertension. Regular BP checks in the clinic are likely to be high in a patient with white coat hypertension. There is no evidence that this patient has elevated stress levels or a poor diet, and those factors do not cause white coat hypertension.

The nurse notes scleral jaundice in a patient being admitted with hemolytic anemia. The nurse will plan to check the laboratory results for the a. Schilling test. b. bilirubin level. c. stool occult blood test. d. gastric analysis testing.

ANS: B Jaundice is caused by the elevation of bilirubin level associated with red blood cell (RBC) hemolysis. The other tests would not be helpful in monitoring or treating a hemolytic anemia

Which problem reported by a patient with hemophilia is most important for the nurse to communicate to the physician? a. Leg bruises b. Tarry stools c. Skin abrasions d. Bleeding gums

ANS: B Melena is a sign of gastrointestinal bleeding and requires collaborative actions such as checking hemoglobin and hematocrit and administration of coagulation factors. The other problems indicate a need for patient teaching about how to avoid injury, but are not indicators of possible serious blood loss

A patient who is receiving methotrexate for severe rheumatoid arthritis develops a megaloblastic anemia. The nurse will anticipate teaching the patient about increasing oral intake of a. iron. b. folic acid. c. cobalamin (vitamin B12). d. ascorbic acid (vitamin C).

ANS: B Methotrexate use can lead to folic acid deficiency. Supplementation with oral folic acid supplements is the usual treatment. The other nutrients would not correct folic acid deficiency, although they would be used to treat other types of anemia

During the assessment of a 25-year-old patient with infective endocarditis (IE), the nurse would expect to find a. substernal chest pressure. b. a new regurgitant murmur. c. a pruritic rash on the chest. d. involuntary muscle movement.

ANS: B New regurgitant murmurs occur in IE because vegetations on the valves prevent valve closure. Substernal chest discomfort, rashes, and involuntary muscle movement are clinical manifestations of other cardiac disorders such as angina and rheumatic fever.

An appropriate nursing intervention for a hospitalized patient with severe hemolytic anemia is to a. provide a diet high in vitamin K. b. alternate periods of rest and activity. c. teach the patient how to avoid injury. d. place the patient on protective isolation.

ANS: B Nursing care for patients with anemia should alternate periods of rest and activity to encourage activity without causing undue fatigue. There is no indication that the patient has a bleeding disorder, so a diet high in vitamin K or teaching about how to avoid injury is not needed. Protective isolation might be used for a patient with aplastic anemia, but it is not indicated for hemolytic anemia

It is important for the nurse providing care for a patient with sickle cell crisis to a. limit the patient's intake of oral and IV fluids. b. evaluate the effectiveness of opioid analgesics. c. encourage the patient to ambulate as much as tolerated. d. teach the patient about high-protein, high-calorie foods.

ANS: B Pain is the most common clinical manifestation of a crisis and usually requires large doses of continuous opioids for control. Fluid intake should be increased to reduce blood viscosity and improve perfusion. Rest is usually ordered to decrease metabolic requirements. Patients are instructed about the need for dietary folic acid, but high-protein, high-calorie diets are not emphasized

To assess the patient with pericarditis for evidence of a pericardial friction rub, the nurse should a. listen for a rumbling, low-pitched, systolic murmur over the left anterior chest. b. auscultate by placing the diaphragm of the stethoscope on the lower left sternal border. c. ask the patient to cough during auscultation to distinguish the sound from a pleural friction rub. d. feel the precordial area with the palm of the hand to detect vibrations with cardiac contraction.

ANS: B Pericardial friction rubs are heard best with the diaphragm at the lower left sternal border. The nurse should ask the patient to hold his or her breath during auscultation to distinguish the sounds from a pleural friction rub. Friction rubs are not typically low pitched or rumbling and are not confined to systole. Rubs are not assessed by palpation.

The nurse is admitting a patient with possible rheumatic fever. Which question on the admission health history will be most pertinent to ask? a. "Do you use any illegal IV drugs?" b. "Have you had a recent sore throat?" c. "Have you injured your chest in the last few weeks?" d. "Do you have a family history of congenital heart disease?"

ANS: B Rheumatic fever occurs as a result of an abnormal immune response to a streptococcal infection. Although illicit IV drug use should be discussed with the patient before discharge, it is not a risk factor for rheumatic fever, and would not be as pertinent when admitting the patient. Family history is not a risk factor for rheumatic fever. Chest injury would cause musculoskeletal chest pain rather than rheumatic fever.

A patient with a history of hypertension treated with a diuretic and an angiotensin-converting enzyme (ACE) inhibitor arrives in the emergency department complaining of a severe headache and nausea and has a blood pressure (BP) of 238/118 mm Hg. Which question should the nurse ask first? a. "Did you take any acetaminophen (Tylenol) today?" b. "Have you been consistently taking your medications?" c. "Have there been any recent stressful events in your life?" d. "Have you recently taken any antihistamine medications?"

ANS: B Sudden withdrawal of antihypertensive medications can cause rebound hypertension and hypertensive crisis. Although many over-the-counter medications can cause hypertension, antihistamines and acetaminophen do not increase BP. Stressful events will increase BP but not usually to the level seen in this patient

A patient with a history of hypertension treated with a diuretic and an angiotensin-converting enzyme (ACE) inhibitor arrives in the emergency department complaining of a severe headache and nausea and has a blood pressure (BP) of 238/118 mm Hg. Which question should the nurse ask to follow up on these findings? a. "Have you recently taken any antihistamines?" b. "Have you consistently taken your medications?" c. "Did you take any acetaminophen (Tylenol) today?" d. "Have there been recent stressful events in your life?"

ANS: B Sudden withdrawal of antihypertensive medications can cause rebound hypertension and hypertensive crisis. Although many over-the-counter medications can cause hypertension, antihistamines and acetaminophen do not increase BP. Stressful events will increase BP but not usually to the level seen in this patient.

Which blood pressure (BP) finding by the nurse indicates that no changes in therapy are needed for a 48-yr-old patient with newly diagnosed hypertension? a. 98/56 mm Hg c. 128/92 mm Hg b. 128/76 mm Hg d. 142/78 mm Hg

ANS: B The 8th Joint National Committee's recommended goal for antihypertensive therapy for a 30- to 59-yr-old patient with hypertension is a BP below 140/90 mm Hg. The BP of 98/56 mm Hg may indicate overtreatment of the hypertension and an increased risk for adverse drug effects. The other two blood pressures indicate a need for modifications in the patient's treatment.

A 28-year-old man with von Willebrand disease is admitted to the hospital for minor knee surgery. The nurse will review the coagulation survey to check the a. platelet count. b. bleeding time. c. thrombin time. d. prothrombin time.

ANS: B The bleeding time is affected by von Willebrand disease. Platelet count, prothrombin time, and thrombin time are normal in von Willebrand disease

After teaching a client who has an implantable cardioverter-defibrillator (ICD), a nurse assesses the client's understanding. Which statement by the client indicates a correct understanding of the teaching? a. "I should wear a snug-fitting shirt over the ICD." b. "I will avoid sources of strong electromagnetic fields." c. "I should participate in a strenuous exercise program." d. "Now I can discontinue my antidysrhythmic medication."

ANS: B The client being discharged with an ICD is instructed to avoid strong sources of electromagnetic fields. Clients should avoid tight clothing, which could cause irritation over the ICD generator. The client should be encouraged to exercise but should not engage in strenuous activities that cause the heart rate to meet or exceed the ICD cutoff point because the ICD can discharge inappropriately. The client should continue all prescribed medications.

The nurse is caring for a client on the medical-surgical unit who suddenly becomes unresponsive and has no pulse. The cardiac monitor shows the rhythm below: After calling for assistance and a defibrillator, which action should the nurse take next? a. Perform a pericardial thump. b. Initiate cardiopulmonary resuscitation (CPR). c. Start an 18-gauge intravenous line. d. Ask the client's family about code status.

ANS: B The client's rhythm is ventricular fibrillation. This is a lethal rhythm that is best treated with immediate defibrillation. While the nurse is waiting for the defibrillator to arrive, the nurse should start CPR. A pericardial thump is not a treatment for ventricular fibrillation. If the client does not already have an IV, other members of the team can insert one after defibrillation. The client's code status should already be known by the nurse prior to this event.

Which action will the admitting nurse include in the care plan for a 30-year old woman who is neutropenic? a. Avoid any injections. b. Check temperature every 4 hours. c. Omit fruits or vegetables from the diet. d. Place a "No Visitors" sign on the door.

ANS: B The earliest sign of infection in a neutropenic patient is an elevation in temperature. Although unpeeled fresh fruits and vegetables should be avoided, fruits and vegetables that are peeled or cooked are acceptable. Injections may be required for administration of medications such as filgrastim (Neupogen). The number of visitors may be limited and visitors with communicable diseases should be avoided, but a "no visitors" policy is not needed

Which blood pressure (BP) finding by the nurse indicates that no changes in therapy are needed for a patient with stage 1 hypertension who has a history of diabetes mellitus? a. 102/60 mm Hg b. 128/76 mm Hg c. 139/90 mm Hg d. 136/82 mm Hg

ANS: B The goal for antihypertensive therapy for a patient with hypertension and diabetes mellitus is a BP <130/80 mm Hg. The BP of 102/60 may indicate overtreatment of the hypertension and an increased risk for adverse drug effects. The other two blood pressures indicate a need for modifications in the patient's treatment

Which BP finding by the nurse indicates that no changes in therapy are needed for a patient with stage 1 hypertension who has a history of heart failure? a. 108/64 mm Hg b. 128/76 mm Hg c. 140/90 mm Hg d. 136/ 82 mm Hg

ANS: B The goal for antihypertensive therapy for a patient with hypertension and heart failure is a BP of <130/80 mm Hg. The BP of 108/64 may indicate overtreatment of the hypertension and an increased risk for adverse drug effects. The other two blood pressures indicate a need for modifications in the patient's treatment.

When developing a community health program to decrease the incidence of rheumatic fever, which action would be most important for the community health nurse to include? a. Vaccinate high-risk groups in the community with streptococcal vaccine. b. Teach community members to seek treatment for streptococcal pharyngitis. c. Teach about the importance of monitoring temperature when sore throats occur. d. Teach about prophylactic antibiotics to those with a family history of rheumatic fever.

ANS: B The incidence of rheumatic fever is decreased by treatment of streptococcal infections with antibiotics. Family history is not a risk factor for rheumatic fever. There is no immunization that is effective in decreasing the incidence of rheumatic fever. Teaching about monitoring temperature will not decrease the incidence of rheumatic fever.

A 30-year-old man with acute myelogenous leukemia develops an absolute neutrophil count of 850/µL while receiving outpatient chemotherapy. Which action by the outpatient clinic nurse is most appropriate? a. Discuss the need for hospital admission to treat the neutropenia. b. Teach the patient to administer filgrastim (Neupogen) injections. c. Plan to discontinue the chemotherapy until the neutropenia resolves. d. Order a high-efficiency particulate air (HEPA) filter for the patient's home.

ANS: B The patient may be taught to self-administer filgrastim injections. Although chemotherapy may be stopped with severe neutropenia (neutrophil count less than 500/µL), administration of filgrastim usually allows the chemotherapy to continue. Patients with neutropenia are at higher risk for infection when exposed to other patients in the hospital. HEPA filters are expensive and are used in the hospital, where the number of pathogens is much higher than in the patient's home environment

Which action will the nurse in the hypertension clinic take in order to obtain an accurate baseline blood pressure (BP) for a new patient? a. Deflate the BP cuff at a rate of 5 to 10 mm Hg per second. b. Have the patient sit in a chair with the feet flat on the floor. c. Assist the patient to the supine position for BP measurements. d. Obtain two BP readings in the dominant arm and average the results.

ANS: B The patient should be seated with the feet flat on the floor. The BP is obtained in both arms, and the results of the two arms are not averaged. The patient does not need to be in the supine position. The cuff should be deflated at 2 to 3 mm Hg per second

Which action will the nurse in the hypertension clinic take to obtain an accurate baseline blood pressure (BP) for a new patient? a. Deflate the BP cuff at a rate of 5 to 10 mm Hg per second. b. Have the patient sit in a chair with the feet flat on the floor. c. Assist the patient to the supine position for BP measurements. d. Obtain two BP readings in the dominant arm and average the results.

ANS: B The patient should be seated with the feet flat on the floor. The BP is obtained in both arms, and the results of the two arms are not averaged. The patient does not need to be in the supine position. The cuff should be deflated at 2 to 3 mm Hg per second.

After receiving the following information about four patients during change-of-shift report, which patient should the nurse assess first? a. Patient with acute pericarditis who has a pericardial friction rub b. Patient who has just returned to the unit after balloon valvuloplasty c. Patient who has hypertrophic cardiomyopathy and a heart rate of 116 d. Patient with a mitral valve replacement who has an anticoagulant scheduled

ANS: B The patient who has just arrived after balloon valvuloplasty will need assessment for complications such as bleeding and hypotension. The information about the other patients is consistent with their diagnoses and does not indicate any complications or need for urgent assessment or intervention.

Several patients call the outpatient clinic and ask to make an appointment as soon as possible. Which patient should the nurse schedule to be seen first? a. 44-year-old with sickle cell anemia who says "my eyes always look sort of yellow" b. 23-year-old with no previous health problems who has a nontender lump in the axilla c. 50-year-old with early-stage chronic lymphocytic leukemia who reports chronic fatigue d. 19-year-old with hemophilia who wants to learn to self-administer factor VII replacement

ANS: B The patient's age and presence of a nontender axillary lump suggest possible lymphoma, which needs rapid diagnosis and treatment. The other patients have questions about treatment or symptoms that are consistent with their diagnosis but do not need to be seen urgently

The nurse is assessing a patient who has been admitted to the intensive care unit (ICU) with a hypertensive emergency. Which finding is most important to report to the health care provider? a. Urine output over 8 hours is 200 mL less than the fluid intake. b. The patient is unable to move the left arm and leg when asked to do so. c. Tremors are noted in the fingers when the patient extends the arms. d. The patient complains of a severe headache with pain at level 8/10 (0 to 10 scale).

ANS: B The patient's inability to move the left arm and leg indicates that a hemorrhagic stroke may be occurring and will require immediate action to prevent further neurologic damage. The other clinical manifestations also likely are caused by the hypertension and will require rapid nursing actions, but they do not require action as urgently as the neurologic changes.

The nurse is assessing a patient who has been admitted to the intensive care unit (ICU) with a hypertensive emergency. Which finding is most important to report to the health care provider? a. Urine output over 8 hours is 250 mL less than the fluid intake. b. The patient cannot move the left arm and leg when asked to do so. c. Tremors are noted in the fingers when the patient extends the arms. d. The patient complains of a headache with pain at level 8/10 (0 to 10 scale).

ANS: B The patient's inability to move the left arm and leg indicates that a hemorrhagic stroke may be occurring and will require immediate action to prevent further neurologic damage. The other clinical manifestations are also likely caused by the hypertension and will require rapid nursing actions, but they do not require action as urgently as the neurologic changes

The nurse is assessing a patient who has been admitted to the intensive care unit (ICU) with a hypertensive emergency. Which finding is *most* important to report to the health care provider? a. Urine output over 8 hours is 250 mL less than the fluid intake. b. The patient cannot move the left arm and leg when asked to do so. c. Tremors are noted in the fingers when the patient extends the arms. d. The patient complains of a headache with pain at level 7 of 10 (0 to 10 scale).

ANS: B The patient's inability to move the left arm and leg indicates that a stroke may be occurring and will require immediate action to prevent further neurologic damage. The other clinical manifestations are also likely caused by the hypertension and will require rapid nursing actions, but they do not require action as urgently as the neurologic changes.

A patient with rheumatic fever has subcutaneous nodules, erythema marginatum, and polyarthritis. Based on these findings, which nursing diagnosis would be most appropriate? a. Pain related to permanent joint fixation b. Activity intolerance related to arthralgia c. Risk for infection related to open skin lesions d. Risk for impaired skin integrity related to pruritus

ANS: B The patient's joint pain will lead to difficulty with activity. The skin lesions seen in rheumatic fever are not open or pruritic. Although acute joint pain will be a problem for this patient, joint inflammation is a temporary clinical manifestation of rheumatic fever and is not associated with permanent joint changes.

After receiving change-of-shift report on a heart failure unit, which patient should the nurse assess first? a. Patient who is taking carvedilol (Coreg) and has a heart rate of 58 b. Patient who is taking digoxin and has a potassium level of 3.1 mEq/L c. Patient who is taking isosorbide dinitrate/hydralazine (BiDil) and has a headache d. Patient who is taking captopril (Capoten) and has a frequent nonproductive cough

ANS: B The patient's low potassium level increases the risk for digoxin toxicity and potentially fatal dysrhythmias. The nurse should assess the patient for other signs of digoxin toxicity and then notify the health care provider about the potassium level. The other patients also have side effects of their medications, but their symptoms do not indicate potentially life-threatening complications.

After receiving change-of-shift report on a heart failure unit, which patient should the nurse assess first? a. Patient who is taking carvedilol (Coreg) and has a heart rate of 58 b. Patient who is taking digoxin and has a potassium level of 3.1 mEq/L c. Patient who is taking captopril and has a frequent nonproductive cough d. Patient who is taking isosorbide dinitrate/hydralazine (BiDil) and has a headache

ANS: B The patient's low potassium level increases the risk for digoxin toxicity and potentially life-threatening dysrhythmias. The nurse should assess the patient for other signs of digoxin toxicity and then notify the health care provider about the potassium level. The other patients also have side effects of their drugs, but their symptoms do not indicate potentially life-threatening complications.

A patient with septicemia develops prolonged bleeding from venipuncture sites and blood in the stools. Which action is most important for the nurse to take? a. Avoid venipunctures. b. Notify the patient's physician. c. Apply sterile dressings to the sites. d. Give prescribed proton-pump inhibitors.

ANS: B The patient's new onset of bleeding and diagnosis of sepsis suggest that disseminated intravascular coagulation (DIC) may have developed, which will require collaborative actions such as diagnostic testing, blood product administration, and heparin administration. The other actions also are appropriate, but the most important action should be to notify the physician so that DIC treatment can be initiated rapidly

The nurse obtains the following information from a patient newly diagnosed with prehypertension. Which finding is most important to address with the patient? a. Low dietary fiber intake b. No regular aerobic exercise c. Weight 5 pounds above ideal weight d. Drinks a beer with dinner on most nights

ANS: B The recommendations for preventing hypertension include exercising aerobically for 30 minutes most days of the week. A weight that is 5 pounds over the ideal body weight is not a risk factor for hypertension. The Dietary Approaches to Stop Hypertension (DASH) diet is high in fiber, but increasing fiber alone will not prevent hypertension from developing. The patient's alcohol intake is within guidelines and will not increase the hypertension risk

The nurse obtains the following information from a patient newly diagnosed with prehypertension. Which finding is *most* important to address with the patient? a. Low dietary fiber intake b. No regular physical exercise c. Drinks a beer with dinner every night d. Weight is 5 pounds above ideal weight

ANS: B The recommendations for preventing hypertension include exercising aerobically for 30 minutes most days of the week. A weight that is 5 pounds over the ideal body weight is not a risk factor for hypertension. The Dietary Approaches to Stop Hypertension (DASH) diet is high in fiber, but increasing fiber alone will not prevent hypertension from developing. The patient's alcohol intake is within guidelines and will not increase the hypertension risk.

A 68-year-old woman with acute myelogenous leukemia (AML) asks the nurse whether the planned chemotherapy will be worth undergoing. Which response by the nurse is appropriate? a. "If you do not want to have chemotherapy, other treatment options include stem cell transplantation." b. "The side effects of chemotherapy are difficult, but AML frequently goes into remission with chemotherapy." c. "The decision about treatment is one that you and the doctor need to make rather than asking what I would do." d. "You don't need to make a decision about treatment right now because leukemias in adults tend to progress quite slowly."

ANS: B This response uses therapeutic communication by addressing the patient's question and giving accurate information. The other responses either give inaccurate information or fail to address the patient's question, which will discourage the patient from asking the nurse for information

When planning care for a patient hospitalized with a streptococcal infective endocarditis (IE), which intervention is a priority for the nurse to include? a. Monitor labs for streptococcal antibodies. b. Arrange for placement of a long-term IV catheter. c. Teach the importance of completing all oral antibiotics. d. Encourage the patient to begin regular aerobic exercise.

ANS: B Treatment for IE involves 4 to 6 weeks of IV antibiotic therapy in order to eradicate the bacteria, which will require a long-term IV catheter such as a peripherally inserted central catheter (PICC) line. Rest periods and limiting physical activity to a moderate level are recommended during the treatment for IE. Oral antibiotics are not effective in eradicating the infective bacteria that cause IE. Blood cultures, rather than antibody levels, are used to monitor the effectiveness of antibiotic therapy.

Which admission order written by the health care provider for a patient admitted with infective endocarditis (IE) and a fever would be a priority for the nurse to implement? a. Administer ceftriaxone (Rocephin) 1 g IV. b. Order blood cultures drawn from two sites. c. Give acetaminophen (Tylenol) PRN for fever. d. Arrange for a transesophageal echocardiogram.

ANS: B Treatment of the IE with antibiotics should be started as quickly as possible, but it is essential to obtain blood cultures before initiating antibiotic therapy to obtain accurate sensitivity results. The echocardiogram and acetaminophen administration also should be implemented rapidly, but the blood cultures (and then administration of the antibiotic) have the highest priority.

The nurse is planning to administer a transfusion of packed red blood cells (PRBCs) to a patient with blood loss from gastrointestinal hemorrhage. Which action can the nurse delegate to unlicensed assistive personnel (UAP)? a. Verify the patient identification (ID) according to hospital policy. b. Obtain the temperature, blood pressure, and pulse before the transfusion. c. Double-check the product numbers on the PRBCs with the patient ID band. d. Monitor the patient for shortness of breath or chest pain during the transfusion.

ANS: B UAP education includes measurement of vital signs. UAP would report the vital signs to the registered nurse (RN). The other actions require more education and a larger scope of practice and should be done by licensed nursing staff members

You are caring for a patient 24 hours after pacemaker insertion. Which nursing intervention is most appropriate at this time? A. Reinforcing the pressure dressing as needed B. Encouraging range-of-motion exercises of the involved arm C. Assessing the incision for any redness, swelling, or discharge D. Applying wet-to-dry dressings every 4 hours to the insertion site

ANS: C After pacemaker insertion, it is important for the patient to limit activity of the involved arm to minimize pacemaker lead displacement. The nonpressure dressing is kept dry until removed, usually 24 hours postoperatively. It is important for you to observe signs of infection by assessing for redness, swelling, or discharge from the incision site. Reference: 836-837

Important teaching for the patient scheduled for a radiofrequency catheter ablation procedure includes explaining that A. ventricular bradycardia may be induced and treated during the procedure. B. catheters will be placed in both femoral arteries to allow double-catheter use. C. the procedure will destroy areas of the conduction system that are causing rapid heart rhythms. D. a general anesthetic will be given to prevent the awareness of any "sudden cardiac death" experiences.

ANS: C Radiofrequency catheter ablation therapy uses electrical energy to "burn" or ablate areas of the conduction system as definitive treatment for tachydysrhythmias. Reference: 837

What is the purpose of the head-up tilt test? A. To determine whether a fluid volume deficit exists B. To assess for dysrhythmias when under stress C. To determine whether there is positional decreased venous return to the heart D. To evaluate for peripheral vascular disease

ANS: C The head-up tilt test is used to see whether there is cardioneurogenic syncope with increased venous pooling that occurs in the upright position. This reduces the venous return to the heart. Reference: 839

A patient admitted with acute coronary syndrome (ACS) has continuous ECG monitoring. An examination of the rhythm strip reveals the following characteristics: atrial rate of 74 beats/minute and regular; ventricular rate of 62 beats/minute and irregular; P wave with a normal shape; PR interval that lengthens progressively until a P wave is not conducted; and QRS complex with a normal shape. Your priority nursing intervention involves A. performing synchronized cardioversion. B. administering 1 mg of epinephrine by IVP. C. observing for symptoms of hypotension or angina. D. preparing the patient for a transcutaneous pacemaker.

ANS: C The rhythm is a second-degree AV block, type I (Mobitz I or Wenckebach heart block). The rhythm is identified by a gradual lengthening of the PR interval. Type I AV block is usually a result of myocardial ischemia or infarction and typically is transient and well tolerated. You should assess for bradycardia, hypotension, and angina. If the patient becomes symptomatic, atropine or a temporary pacemaker may be needed. Reference: 830

You are monitoring the electrocardiogram of a patient admitted with ACS. Which ECG characteristics most suggest ischemia? A. Sinus rhythm with a pathologic Q wave B. Sinus rhythm with an elevated ST segment C. Sinus rhythm with a depressed ST segment D. Sinus rhythm with premature atrial contractions

ANS: C The typical ECG changes seen in myocardial ischemia include ST-segment depression or T-wave inversion, or both. Reference: 837

Which ECG characteristics are consistent with a diagnosis of ventricular tachycardia (VT)? A. Unmeasurable rate and rhythm B. Rate of 150 beats/minute; inverted P wave C. Rate of 200 beats/minute; P wave not visible D. Rate of 125 beats/minute; normal QRS complex

ANS: C VT is associated with a rate of 150 to 250 beats/minute, and the P wave is not normally visible. P-wave inversion and a normal QRS complex are not associated with VT. Rate and rhythm are not measurable in ventricular fibrillation. Reference: 831

A telemetry nurse assesses a client with third-degree heart block who has wide QRS complexes and a heart rate of 35 beats/min on the cardiac monitor. Which assessment should the nurse complete next? a. Pulmonary auscultation b. Pulse strength and amplitude c. Level of consciousness d. Mobility and gait stability

ANS: C A heart rate of 40 beats/min or less with widened QRS complexes could have hemodynamic consequences. The client is at risk for inadequate cerebral perfusion. The nurse should assess for level of consciousness, light-headedness, confusion, syncope, and seizure activity. Although the other assessments should be completed, the client's level of consciousness is the priority.

A 56-yr-old patient who has no previous history of hypertension or other health problems suddenly develops a blood pressure (BP) of 198/110 mm Hg. After reconfirming the BP, it is appropriate for the nurse to tell the patient that a. a BP recheck should be scheduled in a few weeks. b. dietary sodium and fat content should be decreased. c. diagnosis, treatment, and ongoing monitoring will be needed. d. there is an immediate danger of a stroke, requiring hospitalization.

ANS: C A sudden increase in BP in a patient older than age 50 years with no previous hypertension history or risk factors indicates that the hypertension may be secondary to some other problem. The BP will need treatment and ongoing monitoring. If the patient has no other risk factors, a stroke in the immediate future is unlikely. There is no indication that dietary salt or fat intake have contributed to this sudden increase in BP, and reducing intake of salt and fat alone will not be adequate to reduce this BP to an acceptable level.

A patient who is receiving dobutamine for the treatment of acute decompensated heart failure (ADHF) has the following nursing interventions included in the plan of care. Which action will be most appropriate for the registered nurse (RN) to delegate to an experienced licensed practical/vocational nurse (LPN/LVN)? a. Teach the patient the reasons for remaining on bed rest. b. Change the peripheral IV site according to agency policy. c. Monitor the patient's blood pressure and heart rate every hour. d. Titrate the rate to keep the systolic blood pressure >90 mm Hg.

ANS: C An experienced LPN/LVN would be able to monitor BP and heart rate and would know to report significant changes to the RN. Teaching patients, making adjustments to the drip rate for vasoactive drugs, and inserting a new peripheral IV catheter require RN level education and scope of practice.

A patient who is receiving dobutamine (Dobutrex) for the treatment of acute decompensated heart failure (ADHF) has the following nursing interventions included in the plan of care. Which action will be most appropriate for the registered nurse (RN) to delegate to an experienced licensed practical/vocational nurse (LPN/LVN)? a. Assess the IV insertion site for signs of extravasation. b. Teach the patient the reasons for remaining on bed rest. c. Monitor the patient's blood pressure and heart rate every hour. d. Titrate the rate to keep the systolic blood pressure >90 mm Hg.

ANS: C An experienced LPN/LVN would be able to monitor BP and heart rate and would know to report significant changes to the RN. Teaching patients, making adjustments to the drip rate for vasoactive medications, and monitoring for serious complications such as extravasation require RN level education and scope of practice.

When a patient with splenomegaly is scheduled for splenectomy, which action will the nurse include in the preoperative plan of care? a. Discourage deep breathing to reduce risk for splenic rupture. b. Teach the patient to use ibuprofen (Advil) for left upper quadrant pain. c. Schedule immunization with the pneumococcal vaccine (Pneumovax). d. Avoid the use of acetaminophen (Tylenol) for 2 weeks prior to surgery.

ANS: C Asplenic patients are at high risk for infection with Pneumococcus and immunization reduces this risk. There is no need to avoid acetaminophen use before surgery, but nonsteroidal antiinflammatory drugs (NSAIDs) may increase bleeding risk and should be avoided. The enlarged spleen may decrease respiratory depth and the patient should be encouraged to take deep breaths

Which diagnostic test will be most useful to the nurse in determining whether a patient admitted with acute shortness of breath has heart failure? a. Serum troponin b. Arterial blood gases c. B-type natriuretic peptide d. 12-lead electrocardiogram

ANS: C B-type natriuretic peptide (BNP) is secreted when ventricular pressures increase, as they do with heart failure. Elevated BNP indicates a probable or very probable diagnosis of heart failure. A 12-lead electrocardiogram, arterial blood gases, and troponin may also be used in determining the causes or effects of heart failure but are not as clearly diagnostic of heart failure as BNP.

A 52-year-old patient has a new diagnosis of pernicious anemia. The nurse determines that the patient understands the teaching about the disorder when the patient states, "I a. need to start eating more red meat and liver." b. will stop having a glass of wine with dinner." c. could choose nasal spray rather than injections of vitamin B12." d. will need to take a proton pump inhibitor like omeprazole (Prilosec)."

ANS: C Because pernicious anemia prevents the absorption of vitamin B12, this patient requires injections or intranasal administration of cobalamin. Alcohol use does not cause cobalamin deficiency. Proton pump inhibitors decrease the absorption of vitamin B12. Eating more foods rich in vitamin B12 is not helpful because the lack of intrinsic factor prevents absorption of the vitamin

A patient with heart failure has a new order for captopril (Capoten) 12.5 mg PO. After administering the first dose and teaching the patient about the drug, which statement by the patient indicates that teaching has been effective? a. "I will be sure to take the medication with food." b. "I will need to eat more potassium-rich foods in my diet." c. "I will call for help when I need to get up to use the bathroom." d. "I will expect to feel more short of breath for the next few days."

ANS: C Captopril can cause hypotension, especially after the initial dose, so it is important that the patient not get up out of bed without assistance until the nurse has had a chance to evaluate the effect of the first dose. The angiotensin-converting enzyme (ACE) inhibitors are potassium sparing, and the nurse should not teach the patient to purposely increase sources of dietary potassium. Increased shortness of breath is expected with the initiation of -adrenergic blocker therapy for heart failure, not for ACE inhibitor therapy. ACE inhibitors are best absorbed when taken an hour before eating.

A patient with heart failure has a new order for captopril 12.5 mg PO. After giving the first dose and teaching the patient about the drug, which statement by the patient indicates that teaching has been effective? a. "I will be sure to take the medication with food." b. "I will need to eat more potassium-rich foods in my diet." c. "I will call for help when I need to get up to use the bathroom." d. "I will expect to feel more short of breath for the next few days."

ANS: C Captopril can cause hypotension, especially after the initial dose, so it is important that the patient not get up out of bed without assistance until the nurse has had a chance to evaluate the effect of the first dose. The angiotensin-converting enzyme (ACE) inhibitors are potassium sparing, and the nurse should not teach the patient to purposely increase sources of dietary potassium. Increased shortness of breath is expected with the initiation of -adrenergic blocker therapy for heart failure, not for ACE inhibitor therapy. ACE inhibitors are best absorbed when taken an hour before eating.

A nurse cares for a client with atrial fibrillation who reports fatigue when completing activities of daily living. What interventions should the nurse implement to address this client's concerns? a. Administer oxygen therapy at 2 liters per nasal cannula. b. Provide the client with a sleeping pill to stimulate rest. c. Schedule periods of exercise and rest during the day. d. Ask unlicensed assistive personnel to help bathe the client.

ANS: C Clients who have atrial fibrillation are at risk for decreased cardiac output and fatigue when completing activities of daily living. The nurse should schedule periods of exercise and rest during the day to decrease fatigue. The other interventions will not assist the client with self-care activities.

When caring for a patient with infective endocarditis of the tricuspid valve, the nurse should monitor the patient for the development of a. flank pain. b. splenomegaly. c. shortness of breath. d. mental status changes.

ANS: C Embolization from the tricuspid valve would cause symptoms of pulmonary embolus. Flank pain, changes in mental status, and splenomegaly would be associated with embolization from the left-sided valves.

Which action will be included in the plan of care when the nurse is caring for a patient who is receiving sodium nitroprusside (Nipride) to treat a hypertensive emergency? a. Organize nursing activities so that the patient has undisturbed sleep for 6 to 8 hours at night. b. Assist the patient up in the chair for meals to avoid complications associated with immobility. c. Use an automated noninvasive blood pressure machine to obtain frequent BP measurements. d. Place the patient on NPO status to prevent aspiration caused by nausea and the associated vomiting.

ANS: C Frequent monitoring of BP is needed when the patient is receiving rapid-acting IV antihypertensive medications. This can be most easily accomplished with an automated BP machine or arterial line. The patient will require frequent assessments, so allowing 6 to 8 hours of undisturbed sleep is not appropriate. When patients are receiving IV vasodilators, bed rest is maintained to prevent decreased cerebral perfusion and fainting. There is no indication that this patient is nauseated or at risk for aspiration, so an NPO status is unnecessary.

A patient has been diagnosed with possible white coat hypertension. Which action will the nurse plan to take next? a. Schedule the patient for frequent BP checks in the clinic. b. Instruct the patient about the need to decrease stress levels. c. Tell the patient how to self-monitor and record BPs at home. d. Teach the patient about ambulatory blood pressure monitoring.

ANS: C Having the patient self-monitor BPs at home will provide a reliable indication about whether the patient has hypertension. Frequent BP checks in the clinic are likely to be high in a patient with white coat hypertension. Ambulatory blood pressure monitoring may be used if the data from self-monitoring is unclear. Although elevated stress levels may contribute to hypertension, instructing the patient about this is unlikely to reduce BP.

An older patient has been diagnosed with possible white coat hypertension. Which action will the nurse plan to take next? a. Schedule the patient for regular blood pressure (BP) checks in the clinic. b. Instruct the patient about the need to decrease stress levels. c. Tell the patient how to self-monitor and record BPs at home. d. Inform the patient that ambulatory blood pressure monitoring will be needed.

ANS: C Having the patient self-monitor BPs at home will provide a reliable indication about whether the patient has hypertension. Regular BP checks in the clinic are likely to be high in a patient with white coat hypertension. Ambulatory blood pressure monitoring may be used if the data from self-monitoring are unclear. Although elevated stress levels may contribute to hypertension, instructing the patient about this is unlikely to reduce BP

The registered nurse (RN) is caring for a patient with a hypertensive crisis who is receiving sodium nitroprusside (Nipride). Which nursing action can the nurse delegate to an experienced licensed practical/vocational nurse (LPN/LVN)? a. Titrate nitroprusside to decrease mean arterial pressure (MAP) to 115 mm Hg. b. Evaluate effectiveness of nitroprusside therapy on blood pressure (BP). c. Set up the automatic blood pressure machine to take BP every 15 minutes. d. Assess the patient's environment for adverse stimuli that might increase BP.

ANS: C LPN/LVN education and scope of practice include the correct use of common equipment such as automatic blood pressure machines. The other actions require advanced nursing judgment and education, and should be done by RNs

Which action should the nurse take when administering the initial dose of oral labetalol (Normodyne) to a patient with hypertension? a. Encourage the use of hard candy to prevent dry mouth. b. Instruct the patient to ask for help if heart palpitations occur. c. Ask the patient to request assistance when getting out of bed. d. Teach the patient that headaches may occur with this medication.

ANS: C Labetalol decreases sympathetic nervous system activity by blocking both á- and b-adrenergic receptors, leading to vasodilation and a decrease in heart rate, which can cause severe orthostatic hypotension. Heart palpitations, dry mouth, dehydration, and headaches are possible side effects of other antihypertensives

A 21-year-old woman is scheduled for percutaneous transluminal balloon valvuloplasty to treat mitral stenosis. Which information should the nurse include when explaining the advantages of valvuloplasty over valve replacement to the patient? a. Biologic valves will require immunosuppressive drugs after surgery. b. Mechanical mitral valves need to be replaced sooner than biologic valves. c. Lifelong anticoagulant therapy will be needed after mechanical valve replacement. d. Ongoing cardiac care by a health care provider is not necessary after valvuloplasty.

ANS: C Long-term anticoagulation therapy is needed after mechanical valve replacement, and this would restrict decisions about career and childbearing in this patient. Mechanical valves are durable and last longer than biologic valves. All valve repair procedures are palliative, not curative, and require lifelong health care. Biologic valves do not activate the immune system, and immunosuppressive therapy is not needed.

The nurse plans discharge teaching for a patient with chronic heart failure who has prescriptions for digoxin (Lanoxin) and hydrochlorothiazide. Appropriate instructions for the patient include a. limit dietary sources of potassium. b. take the hydrochlorothiazide before bedtime. c. notify the health care provider if nausea develops. d. take the digoxin if the pulse is below 60 beats/min.

ANS: C Nausea is an indication of digoxin toxicity and should be reported so that the provider can assess the patient for toxicity and adjust the digoxin dose, if necessary. The patient will need to include potassium-containing foods in the diet to avoid hypokalemia. Patients should be taught to check their pulse daily before taking the digoxin and if the pulse is less than 60 beats/min, to call their provider before taking the digoxin. Diuretics should be taken early in the day to avoid sleep disruption.

The nurse plans discharge teaching for a patient with chronic heart failure who has prescriptions for digoxin (Lanoxin) and hydrochlorothiazide (HydroDIURIL). Appropriate instructions for the patient include a. limit dietary sources of potassium. b. take the hydrochlorothiazide before bedtime. c. notify the health care provider if nausea develops. d. skip the digoxin if the pulse is below 60 beats/minute.

ANS: C Nausea is an indication of digoxin toxicity and should be reported so that the provider can assess the patient for toxicity and adjust the digoxin dose, if necessary. The patient will need to include potassium-containing foods in the diet to avoid hypokalemia. Patients should be taught to check their pulse daily before taking the digoxin and if the pulse is less than 60, to call their provider before taking the digoxin. Diuretics should be taken early in the day to avoid sleep disruption.

Propranolol (Inderal) is prescribed for a patient diagnosed with hypertension. The nurse should consult with the health care provider before giving this drug when the patient reveals a history of a. daily alcohol use. c. reactive airway disease. b. peptic ulcer disease. d. myocardial infarction (MI).

ANS: C Nonselective b-blockers block b1- and b2-adrenergic receptors and can cause bronchospasm, especially in patients with a history of asthma. b-Blockers will have no effect on the patient's peptic ulcer disease or alcohol use. b-Blocker therapy is recommended after MI.

A 54-year-old woman with acute myelogenous leukemia (AML) is considering treatment with a hematopoietic stem cell transplant (HSCT). The best approach for the nurse to assist the patient with a treatment decision is to a. emphasize the positive outcomes of a bone marrow transplant. b. discuss the need for adequate insurance to cover post-HSCT care. c. ask the patient whether there are any questions or concerns about HSCT. d. explain that a cure is not possible with any other treatment except HSCT.

ANS: C Offering the patient an opportunity to ask questions or discuss concerns about HSCT will encourage the patient to voice concerns about this treatment and also will allow the nurse to assess whether the patient needs more information about the procedure. Treatment of AML using chemotherapy is another option for the patient. It is not appropriate for the nurse to ask the patient to consider insurance needs in making this decision

A patient who has chronic heart failure tells the nurse, "I was fine when I went to bed, but I woke up in the middle of the night feeling like I was suffocating!" The nurse will document this assessment finding as a. orthopnea. b. pulsus alternans. c. paroxysmal nocturnal dyspnea. d. acute bilateral pleural effusion.

ANS: C Paroxysmal nocturnal dyspnea is caused by the reabsorption of fluid from dependent body areas when the patient is sleeping and is characterized by waking up suddenly with the feeling of suffocation. Pulsus alternans is the alteration of strong and weak peripheral pulses during palpation. Orthopnea indicates that the patient is unable to lie flat because of dyspnea. Pleural effusions develop over a longer time period.

The nurse working on the heart failure unit knows that teaching an older female patient with newly diagnosed heart failure is effective when the patient states that a. she will take furosemide (Lasix) every day at bedtime. b. the nitroglycerin patch is to be used when chest pain develops. c. she will call the clinic if her weight goes up 3 pounds in 1 week. d. an additional pillow can help her sleep if she is short of breath at night.

ANS: C Teaching for a patient with heart failure includes information about the need to weigh daily and notify the health care provider about an increase of 3 lb in 2 days or 3 to 5 lb in a week. Nitroglycerin patches are used primarily to reduce preload (not to prevent chest pain) in patients with heart failure and should be used daily, not on an "as needed" basis. Diuretics should be taken earlier in the day to avoid nocturia and sleep disturbance. The patient should call the clinic if increased orthopnea develops rather than just compensating by further elevating the head of the bed.

The nurse working on the heart failure unit knows that teaching an older female patient with newly diagnosed heart failure is effective when the patient states that a. she will take furosemide (Lasix) every day at bedtime. b. the nitroglycerin patch is applied when any chest pain develops. c. she will call the clinic if her weight goes from 124 to 128 pounds in a week. d. an additional pillow can help her sleep if she is feeling short of breath at night.

ANS: C Teaching for a patient with heart failure includes information about the need to weigh daily and notify the health care provider about an increase of 3 pounds in 2 days or 3 to 5 pounds in a week. Nitroglycerin patches are used primarily to reduce preload (not to prevent chest pain) in patients with heart failure and should be used daily, not on an "as needed" basis. Diuretics should be taken earlier in the day to avoid nocturia and sleep disturbance. The patient should call the clinic if increased orthopnea develops, rather than just compensating by further elevating the head of the bed.

A patient with chronic heart failure who is taking a diuretic and an angiotensin-converting enzyme (ACE) inhibitor and who is on a low-sodium diet tells the home health nurse about a 5-lb weight gain in the past 3 days. The nurse's priority action will be to a. have the patient recall the dietary intake for the past 3 days. b. ask the patient about the use of the prescribed medications. c. assess the patient for clinical manifestations of acute heart failure. d. teach the patient about the importance of restricting dietary sodium.

ANS: C The 5-lb weight gain over 3 days indicates that the patient's chronic heart failure may be worsening. It is important that the patient be assessed immediately for other clinical manifestations of decompensation, such as lung crackles. A dietary recall to detect hidden sodium in the diet, reinforcement of sodium restrictions, and assessment of medication compliance may be appropriate interventions but are not the first nursing actions indicated.

Which assessment finding obtained by the nurse when assessing a patient with acute pericarditis should be reported immediately to the health care provider? a. Pulsus paradoxus 8 mm Hg b. Blood pressure (BP) of 168/94 c. Jugular venous distention (JVD) to jaw level d. Level 6 (0 to 10 scale) chest pain with a deep breath

ANS: C The JVD indicates that the patient may have developed cardiac tamponade and may need rapid intervention to maintain adequate cardiac output. Hypertension would not be associated with complications of pericarditis, and the BP is not high enough to indicate that there is any immediate need to call the health care provider. A pulsus paradoxus of 8 mm Hg is normal. Level 6/10 chest pain should be treated but is not unusual with pericarditis.

Which action will the nurse include in the plan of care for a patient who has thalassemia major? a. Teach the patient to use iron supplements. b. Avoid the use of intramuscular injections. c. Administer iron chelation therapy as needed. d. Notify health care provider of hemoglobin 11g/dL.

ANS: C The frequent transfusions used to treat thalassemia major lead to iron toxicity in patients unless iron chelation therapy is consistently used. Iron supplementation is avoided in patients with thalassemia. There is no need to avoid intramuscular injections. The goal for patients with thalassemia major is to maintain a hemoglobin of 10 g/dL or greater

During change-of-shift report, the nurse obtains the following information about a hypertensive patient who received the first dose of nadolol (Corgard) during the previous shift. Which information indicates that the patient needs immediate intervention? a. The patient's pulse has dropped from 68 to 57 beats/min. b. The patient complains that the fingers and toes feel quite cold. c. The patient has developed wheezes throughout the lung fields. d. The patient's blood pressure (BP) reading is now 158/91 mm Hg.

ANS: C The most urgent concern for this patient is the wheezes, which indicate that bronchospasm (a common adverse effect of the noncardioselective b-blockers) is occurring. The nurse should immediately obtain an O2 saturation measurement, apply supplemental O2, and notify the health care provider. The mild decrease in heart rate and complaint of cold fingers and toes are associated with b-receptor blockade but do not require any change in therapy. The BP reading may indicate that a change in medication type or dose may be indicated. However, this is not as urgently needed as addressing the bronchospasm.

During change-of-shift report, the nurse obtains the following information about a hypertensive patient who received the first dose of nadolol (Corgard) during the previous shift. Which information indicates that the patient needs immediate intervention? a. The patient's most recent blood pressure (BP) reading is 158/91 mm Hg. b. The patient's pulse has dropped from 68 to 57 beats/minute. c. The patient has developed wheezes throughout the lung fields. d. The patient complains that the fingers and toes feel quite cold.

ANS: C The most urgent concern for this patient is the wheezes, which indicate that bronchospasm (a common adverse effect of the noncardioselective b-blockers) is occurring. The nurse should immediately obtain an oxygen saturation measurement, apply supplemental oxygen, and notify the health care provider. The mild decrease in heart rate and complaint of cold fingers and toes are associated with b-receptor blockade but do not require any change in therapy. The BP reading may indicate that a change in medication type or dose may be indicated. However, this is not as urgently needed as addressing the bronchospasm

During change-of-shift report, the nurse obtains this information about a hypertensive patient who received the first dose of propranolol (Inderal) during the previous shift. Which information indicates that the patient needs immediate intervention? a. The patient's most recent BP reading is 156/94 mm Hg. b. The patient's pulse has dropped from 64 to 58 beats/minute. c. The patient has developed wheezes throughout the lung fields. d. The patient complains that the fingers and toes feel quite cold.

ANS: C The most urgent concern for this patient is the wheezes, which indicate that bronchospasm (a common adverse effect of the noncardioselective β-blockers) is occurring. The nurse should immediately obtain an oxygen saturation measurement, apply supplemental oxygen, and notify the health care provider. The mild decrease in heart rate and complaint of cold fingers and toes are associated with β-receptor blockade but do not require any change in therapy. The BP reading may indicate that a change in medication type or dose may be indicated; however, this is not as urgently needed as addressing the bronchospasm.

A postoperative patient receiving a transfusion of packed red blood cells develops chills, fever, headache, and anxiety 35 minutes after the transfusion is started. After stopping the transfusion, what action should the nurse take? a. Draw blood for a new crossmatch. b. Send a urine specimen to the laboratory. c. Administer PRN acetaminophen (Tylenol). d. Give the PRN diphenhydramine (Benadryl).

ANS: C The patient's clinical manifestations are consistent with a febrile, nonhemolytic transfusion reaction. The transfusion should be stopped and antipyretics administered for the fever as ordered. A urine specimen is needed if an acute hemolytic reaction is suspected. Diphenhydramine (Benadryl) is used for allergic reactions. This type of reaction does not indicate incorrect crossmatching

A patient with a history of chronic heart failure is admitted to the emergency department (ED) with severe dyspnea and a dry, hacking cough. Which action should the nurse do first? a. Auscultate the abdomen. b. Check the capillary refill. c. Auscultate the breath sounds. d. Assess the level of orientation.

ANS: C This patient's severe dyspnea and cough indicate that acute decompensated heart failure (ADHF) is occurring. ADHF usually manifests as pulmonary edema, which should be detected and treated immediately to prevent ongoing hypoxemia and cardiac/respiratory arrest. The other assessments will provide useful data about the patient's volume status and also should be accomplished rapidly, but detection (and treatment) of pulmonary complications is the priority.

A patient with a history of chronic heart failure is admitted to the emergency department with severe dyspnea and a dry, hacking cough. Which action should the nurse do first? a. Auscultate the abdomen. b. Check the capillary refill. c. Auscultate the breath sounds. d. Ask about the patient's allergies.

ANS: C This patient's severe dyspnea and cough indicate that acute decompensated heart failure (ADHF) is occurring. ADHF usually manifests as pulmonary edema, which should be detected and treated immediately to prevent ongoing hypoxemia and cardiac/respiratory arrest. The other assessments will provide useful data about the patient's volume status and also should be accomplished rapidly, but detection (and treatment) of pulmonary complications is the priority.

Following an acute myocardial infarction, a previously healthy 63-yr-old develops clinical manifestations of heart failure. The nurse anticipates discharge teaching will include information about a. -Adrenergic blockers. b. calcium channel blockers. c. digitalis and potassium therapy regimens. d. angiotensin-converting enzyme (ACE) inhibitors.

ANS: D ACE inhibitor therapy is currently recommended to prevent the development of heart failure in patients who have had a myocardial infarction and as a first-line therapy for patients with chronic heart failure. Digoxin therapy for heart failure is no longer considered a first-line measure, and digoxin is added to the treatment protocol when therapy with other drugs such as ACE-inhibitors, diuretics, and -adrenergic blockers is insufficient. Calcium channel blockers are not generally used in the treatment of heart failure. The -adrenergic blockers are not used as initial therapy for new onset heart failure.

Following an acute myocardial infarction, a previously healthy 63-year-old develops clinical manifestations of heart failure. The nurse anticipates discharge teaching will include information about a. digitalis preparations. b. -adrenergic blockers. c. calcium channel blockers. d. angiotensin-converting enzyme (ACE) inhibitors.

ANS: D ACE inhibitor therapy is currently recommended to prevent the development of heart failure in patients who have had a myocardial infarction and as a first-line therapy for patients with chronic heart failure. Digoxin therapy for heart failure is no longer considered a first-line measure, and digoxin is added to the treatment protocol when therapy with other medications such as ACE-inhibitors, diuretics, and -adrenergic blockers is insufficient. Calcium channel blockers are not generally used in the treatment of heart failure. The -adrenergic blockers are not used as initial therapy for new onset heart failure.

A patient who has been receiving a heparin infusion and warfarin (Coumadin) for a deep vein thrombosis (DVT) is diagnosed with heparin-induced thrombocytopenia (HIT) when her platelet level drops to 110,000/µL. Which action will the nurse include in the plan of care? a. Use low-molecular-weight heparin (LMWH) only. b. Administer the warfarin (Coumadin) at the scheduled time. c. Teach the patient about the purpose of platelet transfusions. d. Discontinue heparin and flush intermittent IV lines using normal saline.

ANS: D All heparin is discontinued when the HIT is diagnosed. The patient should be instructed to never receive heparin or LMWH. Warfarin is usually not given until the platelet count has returned to 150,000/µL. The platelet count does not drop low enough in HIT for a platelet transfusion, and platelet transfusions increase the risk for thrombosis

Which statement by a patient indicates good understanding of the nurse's teaching about prevention of sickle cell crisis? a. "Home oxygen therapy is frequently used to decrease sickling." b. "There are no effective medications that can help prevent sickling." c. "Routine continuous dosage narcotics are prescribed to prevent a crisis." d. "Risk for a crisis is decreased by having an annual influenza vaccination."

ANS: D Because infection is the most common cause of a sickle cell crisis, influenza, Haemophilus influenzae, pneumococcal pneumonia, and hepatitis immunizations should be administered. Although continuous dose opioids and oxygen may be administered during a crisis, patients do not receive these therapies to prevent crisis. Hydroxyurea (Hydrea) is a medication used to decrease the number of sickle cell crises

The nurse is caring for a patient who is receiving IV furosemide (Lasix) and morphine for the treatment of acute decompensated heart failure (ADHF) with severe orthopnea. Which clinical finding is the best indicator that the treatment has been effective? a. Weight loss of 2 pounds in 24 hours b. Hourly urine output greater than 60 mL c. Reduction in patient complaints of chest pain d. Reduced dyspnea with the head of bed at 30 degrees

ANS: D Because the patient's major clinical manifestation of ADHF is orthopnea (caused by the presence of fluid in the alveoli), the best indicator that the medications are effective is a decrease in dyspnea with the head of the bed at 30 degrees. The other assessment data also may indicate that diuresis or improvement in cardiac output has occurred, but are not as specific to evaluating this patient's response.

The nurse is caring for a patient who is receiving IV furosemide (Lasix) and morphine for the treatment of acute decompensated heart failure (ADHF) with severe orthopnea. Which clinical finding is the best indicator that the treatment has been effective? a. Weight loss of 2 lb in 24 hours b. Hourly urine output greater than 60 mL c. Reduction in patient complaints of chest pain d. Reduced dyspnea with the head of bed at 30 degrees

ANS: D Because the patient's major clinical manifestation of ADHF is orthopnea (caused by the presence of fluid in the alveoli), the best indicator that the medications are effective is a decrease in dyspnea with the head of the bed at 30 degrees. The other assessment data may also indicate that diuresis or improvement in cardiac output has occurred but are not as specific to evaluating this patient's response.

The nurse is caring for a 70-year-old who uses hydrochlorothiazide (HydroDIURIL) and enalapril (Norvasc), but whose self-monitored blood pressure (BP) continues to be elevated. Which patient information may indicate a need for a change? a. Patient takes a daily multivitamin tablet. b. Patient checks BP daily just after getting up. c. Patient drinks wine three to four times a week. d. Patient uses ibuprofen (Motrin) daily to treat osteoarthritis.

ANS: D Because use of nonsteroidal antiinflammatory drugs (NSAIDs) can prevent adequate BP control, the patient may need to avoid the use of ibuprofen. A multivitamin tablet will help supply vitamin D, which may help lower BP. BP decreases while sleeping, so self-monitoring early in the morning will result in obtaining pressures that are at their lowest. The patient's alcohol intake is not excessive

The nurse obtains a health history from a 65-year-old patient with a prosthetic mitral valve who has symptoms of infective endocarditis (IE). Which question by the nurse is most appropriate? a. "Do you have a history of a heart attack?" b. "Is there a family history of endocarditis?" c. "Have you had any recent immunizations?" d. "Have you had dental work done recently?"

ANS: D Dental procedures place the patient with a prosthetic mitral valve at risk for infective endocarditis (IE). Myocardial infarction (MI), immunizations, and a family history of endocarditis are not risk factors for IE.

Which laboratory test will the nurse use to determine whether filgrastim (Neupogen) is effective for a patient with acute lymphocytic leukemia who is receiving chemotherapy? a. Platelet count b. Reticulocyte count c. Total lymphocyte count d. Absolute neutrophil count

ANS: D Filgrastim increases the neutrophil count and function in neutropenic patients. Although total lymphocyte, platelet, and reticulocyte counts also are important to monitor in this patient, the absolute neutrophil count is used to evaluate the effects of filgrastim

Which action will be included in the plan of care when the nurse is caring for a patient who is receiving nicardipine (Cardene) to treat a hypertensive emergency? a. Keep the patient NPO to prevent aspiration caused by nausea and possible vomiting. b. Organize nursing activities so that the patient has undisturbed sleep for 6 to 8 hours at night. c. Assist the patient up in the chair for meals to avoid complications associated with immobility. d. Use an automated noninvasive blood pressure machine to obtain frequent blood pressure (BP) measurements.

ANS: D Frequent monitoring of BP is needed when the patient is receiving rapid-acting IV antihypertensive medications. This can be most easily accomplished with an automated BP machine or arterial line. The patient will require frequent assessments, so allowing 6 to 8 hours of undisturbed sleep is not appropriate. When patients are receiving IV vasodilators, bed rest is maintained to prevent decreased cerebral perfusion and fainting. There is no indication that this patient is nauseated or at risk for aspiration, so an NPO status is unnecessary

Which action will be included in the plan of care when the nurse is caring for a patient who is receiving nicardipine (Cardene) to treat a hypertensive emergency? a. Organize nursing activities so that the patient has undisturbed sleep for 8 hours at night. b. Keep the patient NPO to prevent aspiration caused by nausea and possible vomiting. c. Assist the patient up in the chair for meals to avoid complications associated with immobility. d. Use an automated noninvasive blood pressure machine to obtain frequent measurements.

ANS: D Frequent monitoring of BP is needed when the patient is receiving rapid-acting IV antihypertensive medications. This can be most easily accomplished with an automated BP machine or arterial line. The patient will require frequent assessments, so allowing 8 hours of undisturbed sleep is not reasonable. When patients are receiving IV vasodilators, bed rest is maintained to prevent decreased cerebral perfusion and fainting. There is no indication that this patient is nauseated or at risk for aspiration, so an NPO status is unnecessary.

Which information should the nurse include when teaching a patient with newly diagnosed hypertension? a. Increasing physical activity will control blood pressure (BP) for most patients. b. Most patients are able to control BP through dietary changes. c. Annual BP checks are needed to monitor treatment effectiveness. d. Hypertension is usually asymptomatic until target organ damage occurs.

ANS: D Hypertension is usually asymptomatic until target organ damage has occurred. Lifestyle changes (e.g., physical activity, dietary changes) are used to help manage blood pressure, but drugs are needed for most patients. Home BP monitoring should be taught to the patient and findings checked by the health care provider frequently when starting treatment for hypertension and then every 3 months once stable

A patient who has non-Hodgkin's lymphoma is receiving combination treatment with rituximab (Rituxan) and chemotherapy. Which patient assessment finding requires the most rapid action by the nurse? a. Anorexia b. Vomiting c. Oral ulcers d. Lip swelling

ANS: D Lip swelling in angioedema may indicate a hypersensitivity reaction to the rituximab. The nurse should stop the infusion and further assess for anaphylaxis. The other findings may occur with chemotherapy, but are not immediately life threatening

A nurse assesses a client's electrocardiograph tracing and observes that not all QRS complexes are preceded by a P wave. How should the nurse interpret this observation? a. The client has hyperkalemia causing irregular QRS complexes. b. Ventricular tachycardia is overriding the normal atrial rhythm. c. The client's chest leads are not making sufficient contact with the skin. d. Ventricular and atrial depolarizations are initiated from different sites.

ANS: D Normal rhythm shows one P wave preceding each QRS complex, indicating that all depolarization is initiated at the sinoatrial node. QRS complexes without a P wave indicate a different source of initiation of depolarization. This finding on an electrocardiograph tracing is not an indication of hyperkalemia, ventricular tachycardia, or disconnection of leads.

The nurse has identified a nursing diagnosis of acute pain related to inflammatory process for a patient with acute pericarditis. The priority intervention by the nurse for this problem is to a. teach the patient to take deep, slow breaths to control the pain. b. force fluids to 3000 mL/day to decrease fever and inflammation. c. remind the patient to request opioid pain medication every 4 hours. d. place the patient in Fowler's position, leaning forward on the overbed table.

ANS: D Sitting upright and leaning forward frequently will decrease the pain associated with pericarditis. Forcing fluids will not decrease the inflammation or pain. Taking deep breaths will tend to increase pericardial pain. Opioids are not very effective at controlling pain caused by acute inflammatory conditions and are usually ordered PRN. The patient would receive scheduled doses of a nonsteroidal antiinflammatory drug (NSAID).

After the nurse teaches the patient with stage 1 hypertension about diet modifications that should be implemented, which diet choice indicates that the teaching has been effective? a. The patient avoids eating nuts or nut butters. b. The patient restricts intake of dietary protein. c. The patient has only one cup of coffee in the morning. d. The patient has a glass of low-fat milk with each meal.

ANS: D The Dietary Approaches to Stop Hypertension (DASH) recommendations for prevention of hypertension include increasing the intake of calcium-rich foods. Caffeine restriction and decreased protein intake are not included in the recommendations. Nuts are high in beneficial nutrients and 4 to 5 servings weekly are recommended in the DASH diet.

A patient has just been diagnosed with hypertension and has been started on captopril (Capoten). Which information is important to include when teaching the patient about this medication? a. Check blood pressure (BP) in both arms before taking the medication. b. Increase fluid intake if dryness of the mouth is a problem. c. Include high-potassium foods such as bananas in the diet. d. Change position slowly to help prevent dizziness and falls.

ANS: D The angiotensin-converting enzyme (ACE) inhibitors frequently cause orthostatic hypotension, and patients should be taught to change position slowly to allow the vascular system time to compensate for the position change. Increasing fluid intake may counteract the effect of the medication, and the patient is taught to use gum or hard candy to relieve dry mouth. The BP should be taken in the nondominant arm by newly diagnosed patients in the morning, before taking the medication, and in the evening. Because ACE inhibitors cause potassium retention, increased intake of high-potassium foods is inappropriate

A patient has just been diagnosed with hypertension and has a new prescription for captopril (Capoten). Which information is important to include when teaching the patient? a. Check BP daily before taking the medication. b. Increase fluid intake if dryness of the mouth is a problem. c. Include high-potassium foods such as bananas in the diet. d. Change position slowly to help prevent dizziness and falls.

ANS: D The angiotensin-converting enzyme (ACE) inhibitors frequently cause orthostatic hypotension, and patients should be taught to change position slowly to allow the vascular system time to compensate for the position change.The BP does not need to be checked at home by the patient before taking the medication. ** Increasing fluid intake may counteract the effect of the medication, and the patient is taught to use gum or hard candy to relieve dry mouth. ***Because ACE inhibitors cause potassium retention, increased intake of high-potassium foods is inappropriate.

Following successful treatment of Hodgkin's lymphoma for a 55-year-old woman, which topic will the nurse include in patient teaching? a. Potential impact of chemotherapy treatment on fertility b. Application of soothing lotions to treat residual pruritus c. Use of maintenance chemotherapy to maintain remission d. Need for follow-up appointments to screen for malignancy

ANS: D The chemotherapy used in treating Hodgkin's lymphoma results in a high incidence of secondary malignancies; follow-up screening is needed. The fertility of a 55-year-old woman will not be impacted by chemotherapy. Maintenance chemotherapy is not used for Hodgkin's lymphoma. Pruritus is a clinical manifestation of lymphoma, but should not be a concern after treatment

Which topic will the nurse plan to include in discharge teaching for a patient with heart failure with reduced ejection fraction (HFrEF)? a. Need to begin an aerobic exercise program several times weekly b. Use of salt substitutes to replace table salt when cooking and at the table c. Importance of making an annual appointment with the health care provider d. Benefits and side effects of angiotensin-converting enzyme (ACE) inhibitors

ANS: D The core measures for the treatment of heart failure established by The Joint Commission indicate that patients with an ejection fraction below 40% should receive an ACE inhibitor to decrease the progression of heart failure. Aerobic exercise may not be appropriate for a patient with this level of heart failure, salt substitutes are not usually recommended because of the risk of hyperkalemia, and the patient will need to see the primary care provider more frequently than annually.

While admitting an 82-yr-old patient with acute decompensated heart failure to the hospital, the nurse learns that the patient lives alone and sometimes confuses the "water pill" with the "heart pill." When planning for the patient's discharge the nurse will facilitate a a. plan for around-the-clock care. b. consultation with a psychologist. c. transfer to a long-term care facility. d. referral to a home health care agency.

ANS: D The data about the patient suggest that assistance in developing a system for taking medications correctly at home is needed. A home health nurse will assess the patient's home situation and help the patient develop a method for taking the two medications as directed. There is no evidence that the patient requires services such as a psychologist consult, long-term care, or around-the-clock home care.

A patient admitted with acute dyspnea is newly diagnosed with dilated cardiomyopathy. Which information will the nurse plan to teach the patient about managing this disorder? a. A heart transplant should be scheduled as soon as possible. b. Elevating the legs above the heart will help relieve dyspnea. c. Careful compliance with diet and medications will prevent heart failure. d. Notify the doctor about any symptoms of heart failure such as shortness of breath.

ANS: D The patient should be instructed to notify the health care provider about any worsening of heart failure symptoms. Because dilated cardiomyopathy does not respond well to therapy, even patients with good compliance with therapy may have recurrent episodes of heart failure. Elevation of the legs above the heart will worsen symptoms (although this approach is appropriate for a patient with hypertrophic cardiomyopathy). The patient with terminal or end-stage cardiomyopathy may consider heart transplantation.

An outpatient who has chronic heart failure returns to the clinic after 2 weeks of therapy with metoprolol (Toprol XL). Which assessment finding is most important for the nurse to report to the health care provider? a. 2+ bilateral pedal edema b. Heart rate of 56 beats/min c. Complaints of increased fatigue d. Blood pressure (BP) of 88/42 mm Hg

ANS: D The patient's BP indicates that the dose of metoprolol may need to be decreased because of hypotension. Bradycardia is a frequent adverse effect of -adrenergic blockade, but the rate of 56 is not unusual though it may need to be monitored. -Adrenergic blockade initially will worsen symptoms of heart failure in many patients and patients should be taught that some increase in symptoms, such as fatigue and edema, is expected during the initiation of therapy with this class of drugs.

A 62-year old man with chronic anemia is experiencing increased fatigue and occasional palpitations at rest. The nurse would expect the patient's laboratory findings to include a. a hematocrit (Hct) of 38%. b. an RBC count of 4,500,000/mL. c. normal red blood cell (RBC) indices. d. a hemoglobin (Hgb) of 8.6 g/dL (86 g/L).

ANS: D The patient's clinical manifestations indicate moderate anemia, which is consistent with a Hgb of 6 to 10 g/dL. The other values are all within the range of normal

A nurse prepares to defibrillate a client who is in ventricular fibrillation. Which priority intervention should the nurse perform prior to defibrillating this client? a. Make sure the defibrillator is set to the synchronous mode. b. Administer 1 mg of intravenous epinephrine. c. Test the equipment by delivering a smaller shock at 100 joules. d. Ensure that everyone is clear of contact with the client and the bed.

ANS: D To avoid injury, the rescuer commands that all personnel clear contact with the client or the bed and ensures their compliance before delivery of the shock. A precordial thump can be delivered when no defibrillator is available. Defibrillation is done in asynchronous mode. Equipment should not be tested before a client is defibrillated because this is an emergency procedure; equipment should be checked on a routine basis. Epinephrine should be administered after defibrillation.

Which action could the nurse delegate to unlicensed assistive personnel (UAP) trained as electrocardiogram (ECG) technicians working on the cardiac unit? a. Select the best lead for monitoring a patient with an admission diagnosis of Dressler syndrome. b. Obtain a list of herbal medications used at home while admitting a new patient with pericarditis. c. Teach about the need to monitor the weight daily for a patient who has hypertrophic cardiomyopathy. d. Check the heart monitor for changes in rhythm while a patient who had a valve replacement ambulates.

ANS: D Under the supervision of registered nurses (RNs), UAP check the patient's cardiac monitor and obtain information about changes in heart rate and rhythm with exercise. Teaching and obtaining information about home medications (prescribed or complementary) and selecting the best leads for monitoring patients require more critical thinking and should be done by the RN.

While caring for a 23-year-old patient with mitral valve prolapse (MVP) without valvular regurgitation, the nurse determines that discharge teaching has been effective when the patient states that it will be necessary to a. take antibiotics before any dental appointments. b. limit physical activity to avoid stressing the heart. c. take an aspirin a day to prevent clots from forming on the valve. d. avoid use of over-the-counter (OTC) medications that contain stimulant drugs.

ANS: D Use of stimulant medications should be avoided by patients with MVP because these may exacerbate symptoms. Daily aspirin and restricted physical activity are not needed by patients with mild MVP. Antibiotic prophylaxis is needed for patients with MVP with regurgitation but will not be necessary for this patient.

A nurse cares for a client with congestive heart failure who has a regular cardiac rhythm of 128 beats/min. For which physiologic alterations should the nurse assess? (Select all that apply.) a. Decrease in cardiac output b. Increase in cardiac output c. Decrease in blood pressure d. Increase in blood pressure e. Decrease in urine output f. Increase in urine output

ANS: Decrease in cardiac output Increase in blood pressure Decrease in urine output Elevated heart rates in a healthy client initially cause blood pressure and cardiac output to increase. However, in a client who has congestive heart failure or a client with long-term tachycardia, ventricular filling time, cardiac output, and blood pressure eventually decrease. As cardiac output and blood pressure decrease, urine output will fall

A nurse prepares to defibrillate a client who is in ventricular fibrillation. Which priority intervention should the nurse perform prior to defibrillating this client? a. Make sure the defibrillator is set to the synchronous mode. b. Administer 1 mg of intravenous epinephrine. c. Test the equipment by delivering a smaller shock at 100 joules. d. Ensure that everyone is clear of contact with the client and the bed.

ANS: Ensure that everyone is clear of contact with the client and the bed. To avoid injury, the rescuer commands that all personnel clear contact with the client or the bed and ensures their compliance before delivery of the shock. A precordial thump can be delivered when no defibrillator is available. Defibrillation is done in asynchronous mode. Equipment should not be tested before a client is defibrillated because this is an emergency procedure; equipment should be checked on a routine basis. Epinephrine should be administered after defibrillation.

The nurse is caring for a client on the medical-surgical unit who suddenly becomes unresponsive and has no pulse. The cardiac monitor shows the rhythm below: After calling for assistance and a defibrillator, which action should the nurse take next? a. Perform a pericardial thump. b. Initiate cardiopulmonary resuscitation (CPR). c. Start an 18-gauge intravenous line. d. Ask the client's family about code status.

ANS: Initiate cardiopulmonary resuscitation (CPR). The client's rhythm is ventricular fibrillation. This is a lethal rhythm that is best treated with immediate defibrillation. While the nurse is waiting for the defibrillator to arrive, the nurse should start CPR. A pericardial thump is not a treatment for ventricular fibrillation. If the client does not already have an IV, other members of the team can insert one after defibrillation. The client's code status should already be known by the nurse prior to this event.

A telemetry nurse assesses a client with third-degree heart block who has wide QRS complexes and a heart rate of 35 beats/min on the cardiac monitor. Which assessment should the nurse complete next? a. Pulmonary auscultation b. Pulse strength and amplitude c. Level of consciousness d. Mobility and gait stability

ANS: Level of consciousness A heart rate of 40 beats/min or less with widened QRS complexes could have hemodynamic consequences. The client is at risk for inadequate cerebral perfusion. The nurse should assess for level of consciousness, light-headedness, confusion, syncope, and seizure activity. Although the other assessments should be completed, the client's level of consciousness is the priority.

A nurse prepares to discharge a client with cardiac dysrhythmia who is prescribed home health care services. Which priority information should be communicated to the home health nurse upon discharge? a. Medication reconciliation b. Immunization history c. Religious beliefs d. Nutrition preferences

ANS: Medication reconciliation The home health nurse needs to know current medications the client is taking to ensure assessment, evaluation, and further education related to these medications. The other information will not assist the nurse to develop a plan of care for the client.

A nurse assesses a client with tachycardia. Which clinical manifestation requires immediate intervention by the nurse? a. Mid-sternal chest pain b. Increased urine output c. Mild orthostatic hypotension d. P wave touching the T wave

ANS: Mid-sternal chest pain Chest pain, possibly angina, indicates that tachycardia may be increasing the client's myocardial workload and oxygen demand to such an extent that normal oxygen delivery cannot keep pace. This results in myocardial hypoxia and pain. Increased urinary output and mild orthostatic hypotension are not life-threatening conditions and therefore do not require immediate intervention. The P wave touching the T wave indicates significant tachycardia and should be assessed to determine the underlying rhythm and cause; this is an important assessment but is not as critical as chest pain, which indicates cardiac cell death.

A nurse cares for a client with atrial fibrillation who reports fatigue when completing activities of daily living. What interventions should the nurse implement to address this client's concerns? a. Administer oxygen therapy at 2 liters per nasal cannula. b. Provide the client with a sleeping pill to stimulate rest. c. Schedule periods of exercise and rest during the day. d. Ask unlicensed assistive personnel to help bathe the client.

ANS: Schedule periods of exercise and rest during the day. Clients who have atrial fibrillation are at risk for decreased cardiac output and fatigue when completing activities of daily living. The nurse should schedule periods of exercise and rest during the day to decrease fatigue. The other interventions will not assist the client with self-care activities.

A nurse administers prescribed adenosine (Adenocard) to a client. Which response should the nurse assess for as the expected therapeutic response? a. Decreased intraocular pressure b. Increased heart rate c. Short period of asystole d. Hypertensive crisis

ANS: Short period of asystole Clients usually respond to adenosine with a short period of asystole, bradycardia, hypotension, dyspnea, and chest pain. Adenosine has no conclusive impact on intraocular pressure.

A nurse assesses a client's electrocardiogram (ECG) and observes the reading shown below: How should the nurse document this client's ECG strip? a. Ventricular tachycardia b. Ventricular fibrillation c. Sinus rhythm with premature atrial contractions (PACs) d. Sinus rhythm with premature ventricular contractions (PVCs)

ANS: Sinus rhythm with premature ventricular contractions (PVCs) Sinus rhythm with PVCs has an underlying regular sinus rhythm with ventricular depolarization that sometimes precede atrial depolarization. Ventricular tachycardia and ventricular fibrillation rhythms would not have sinus beats present. Premature atrial contractions are atrial contractions initiated from another region of the atria before the sinus node initiates atrial depolarization.

Integrated Process: Nursing Process (Analysis) 6. The client's heart rate increases slightly during inspiration and decreases slightly during expiration. What action does the nurse take? a. Evaluate for a respirator disorder. b. Assess the client for chest pain. c. Document the finding in the chart. d. Administer antidysrhythmic drugs.

C Sinus dysrhythmia is noted when the heart rate increases slightly during inspiration and decreases slightly during expiration. Sinus dysrhythmia is a variant of normal sinus rhythm that is frequently observed in healthy children and adults. No other actions are needed. DIF: Cognitive Level: Comprehension/Understanding REF: p. 718 TOP: Client Needs Category: Physiological Integrity (Physiological Adaptation—Pathophysiology)

Integrated Process: Nursing Process (Assessment) 2. The nurse is assessing the client's electrocardiography (ECG). What does the P wave on the ECG tracing represent? a. Contraction of the atria b. Contraction of the ventricles c. Depolarization of the atria d. Depolarization of the ventricles

C The ECG tracing of a P wave represents electrical changes caused by atrial depolarization. DIF: Cognitive Level: Knowledge/Remembering REF: p. 715 TOP: Client Needs Category: Physiological Integrity (Physiological Adaptation—Pathophysiology)

Health Promotion and Maintenance 13. When caring for a patient with mitral valve stenosis, it is most important that the nurse assess for a. diastolic murmur. b. peripheral edema. c. shortness of breath on exertion. d. right upper quadrant tenderness.

C The pressure gradient changes in mitral stenosis lead to fluid backup into the lungs, resulting in hypoxemia and dyspnea. The other findings also may be associated with mitral valve disease but are not indicators of possible hypoxemia. DIF: Cognitive Level: Apply (application) REF: 821 TOP: Nursing Process: Assessment MSC:

Integrated Process: Nursing Process (Assessment) 13. The nurse is caring for a client on a cardiac monitor. The monitor shows a rapid rhythm with a "saw tooth" configuration. What physical assessment findings does the nurse expect? a. Presence of a split S1 and wheezing b. Anorexia and gastric distress c. Shortness of breath and anxiety d. Hypertension and mental status changes

C The rhythm described is atrial flutter with a rapid ventricular response. Rapid atrial flutter may manifest with palpitations, shortness of breath, and anxiety. Syncope, angina, and evidence of heart failure also may be present. DIF: Cognitive Level: Application/Applying or higher REF: N/A TOP: Client Needs Category: Physiological Integrity (Physiological Adaptation—Pathophysiology)

Integrated Process: Nursing Process (Implementation) 16. The nurse is caring for a client admitted for myocardial infarction. The client's monitor shows frequent premature ventricular contractions (PVCs). What dysrhythmia does the nurse remain alert for? a. Sinus tachycardia b. Rapid atrial flutter c. Ventricular tachycardia d. Atrioventricular junctional rhythm

C With an acute myocardial infarction (MI), the onset of PVCs may be considered as a warning that could herald the onset of ventricular tachycardia or ventricular fibrillation. DIF: Cognitive Level: Application/Applying or higher REF: N/A TOP: Client Needs Category: Physiological Integrity (Reduction of Risk Potential—Potential for Complications from Surgical Procedures and Health Alterations)

After assessing a client who is receiving an amiodarone intravenous infusion for unstable ventricular tachycardia, the nurse documents the findings and compares these with the previous assessment findings: Vital Signs Nursing Assessment Time: 0800 Temperature: 98° F Heart rate: 68 beats/min Blood pressure: 135/60 mm Hg Respiratory rate: 14 breaths/min Oxygen saturation: 96% Oxygen therapy: 2 L nasal cannula Time: 1000 Temperature: 98.2° F Heart rate: 50 beats/min Blood pressure: 132/57 mm Hg Respiratory rate: 16 breaths/min Oxygen saturation: 95% Oxygen therapy: 2 L nasal cannula Time: 0800 Client alert and oriented. Cardiac rhythm: normal sinus rhythm. Skin: warm, dry, and appropriate for race. Respirations equal and unlabored. Client denies shortness of breath and chest pain. Time: 1000 Client alert and oriented. Cardiac rhythm: sinus bradycardia. Skin: warm, dry, and appropriate for race. Respirations equal and unlabored. Client denies shortness of breath and chest pain. Client voids 420 mL of clear yellow urine. Based on the assessments, which action should the nurse take? a. Stop the infusion and flush the IV. b. Slow the amiodarone infusion rate. c. Administer IV normal saline. d. Ask the client to cough and deep breathe.

ANS: Slow the amiodarone infusion rate. IV administration of amiodarone may cause bradycardia and atrioventricular (AV) block. The correct action for the nurse to take at this time is to slow the infusion, because the client is asymptomatic and no evidence reveals AV block that might require pacing. Abruptly ceasing the medication could allow fatal dysrhythmias to occur. The administration of IV fluids and encouragement of coughing and deep breathing exercises are not indicated, and will not increase the client's heart rate.

A nurse is teaching a client with premature ectopic beats. Which education should the nurse include in this client's teaching? (Select all that apply.) a. Smoking cessation b. Stress reduction and management c. Avoiding vagal stimulation d. Adverse effects of medications e. Foods high in potassium

ANS: Smoking cessation Stress reduction and management Adverse effects of medications A client who has premature beats or ectopic rhythms should be taught to stop smoking, manage stress, take medications as prescribed, and report adverse effects of medications. Clients with premature beats are not at risk for vasovagal attacks or potassium imbalances.

A nurse assesses a client with atrial fibrillation. Which manifestation should alert the nurse to the possibility of a serious complication from this condition? a. Sinus tachycardia b. Speech alterations c. Fatigue d. Dyspnea with activity

ANS: Speech alterations Clients with atrial fibrillation are at risk for embolic stroke. Evidence of embolic events includes changes in mentation, speech, sensory function, and motor function. Clients with atrial fibrillation often have a rapid ventricular response as a result. Fatigue is a nonspecific complaint. Clients with atrial fibrillation often have dyspnea as a result of the decreased cardiac output caused by the rhythm disturbance.

A nurse assesses a client's electrocardiograph tracing and observes that not all QRS complexes are preceded by a P wave. How should the nurse interpret this observation? a. The client has hyperkalemia causing irregular QRS complexes. b. Ventricular tachycardia is overriding the normal atrial rhythm. c. The client's chest leads are not making sufficient contact with the skin. d. Ventricular and atrial depolarizations are initiated from different sites.

ANS: Ventricular and atrial depolarizations are initiated from different sites. Normal rhythm shows one P wave preceding each QRS complex, indicating that all depolarization is initiated at the sinoatrial node. QRS complexes without a P wave indicate a different source of initiation of depolarization. This finding on an electrocardiograph tracing is not an indication of hyperkalemia, ventricular tachycardia, or disconnection of leads.

A nurse evaluates prescriptions for a client with chronic atrial fibrillation. Which medication should the nurse expect to find on this client's medication administration record to prevent a common complication of this condition? a. Sotalol (Betapace) b. Warfarin (Coumadin) c. Atropine (Sal-Tropine) d. Lidocaine (Xylocaine)

ANS: Warfarin (Coumadin) Atrial fibrillation puts clients at risk for developing emboli. Clients at risk for emboli are treated with anticoagulants, such as heparin, enoxaparin, or warfarin. Sotalol, atropine, and lidocaine are not appropriate for this complication.

For which dysrhythmia is defibrillation primarily indicated? A. Ventricular fibrillation B. Third-degree AV block C. Uncontrolled atrial fibrillation D. Ventricular tachycardia with a pulse

ANS:A Defibrillation is always indicated in the treatment of ventricular fibrillation. Drug treatments are normally used in the treatment of uncontrolled atrial fibrillation and for ventricular tachycardia with a pulse (if the patient is stable). Otherwise, synchronized cardioversion is used (as long as the patient has a pulse). Pacemakers are the treatment of choice for third-degree heart block. Reference: 833

You prepare a patient for synchronized cardioversion knowing that cardioversion differs from defibrillation in that A. defibrillation requires lower dose of electrical energy. B. cardioversion is indicated for treatment of atrial bradydysrhythmias. C. defibrillation is synchronized to deliver a shock during the QRS complex. D. patients should be sedated if cardioversion is done on a nonemergent basis.

ANS:D Synchronized cardioversion is the therapy of choice for the patient with hemodynamically unstable ventricular or supraventricular tachydysrhythmias. A synchronized circuit in the defibrillator delivers a countershock that is programmed to occur on the R wave of the QRS complex seen on the ECG tracing. The synchronizer switch must be turned on when cardioversion is planned. The procedure for synchronized cardioversion is the same as for defibrillation, with the following exceptions. If synchronized cardioversion is done on a nonemergent basis, the patient is sedated before the procedure. The initial energy needed for synchronized cardioversion is less than the energy needed for defibrillation. Reference: 833

A patient has no pulse and the cardiac monitor shows VF. Which drugs does the nurse prepare to administer during the resuscitation? (select all that apply) a. Lidocaine b. Epinephrine c. Calcium chloride d. Amiodarone hydrochloride (Cordarone) e. Dopamine hydrochloride (Intropin) f. Magnesium sulfate

Abdf

Traditionally, what medications will most likely be ordered for a patient with AF? (select all that apply) a Diltiazem hydrochloride (Cardizem) b. Furosemide (Lasix) c. Heparin d. Enoxaparin (Lovenox) e. Sodium warfarin (Coumadin)

Acde

The nurse is teaching a patient with a permanent pacemaker. What information about the pacemaker does the nurse tell the patient? (select all that apply) a. Report any pulse rate lower than what is set on the pacemaker. b. If the surgical incision is near the shoulder, avoid overextending the joint. c. Keep handheld cellular phones at least 6 inches away from the generator. d. Avoid sources of strong electro magnetic fields, such as magnets. e. Avoid strenuous activities that may cause the device to discharge inappropriately f. Carry a pacemaker identification card and wear a medical alert bracelet

Acdf

Which clinical manifestations are reflections of sustained tachdysrhythmias and bradydysrhythmias? (select all that apply) a. Weakness and fatigue b. Warm, dry skin c. Dyspnea d. Hypertension e. Decreased urine output

Ace

A client is transported to the emergency department in respiratory distress after eating peanuts. The following interventions are ordered by the health care provider. Which intervention should the nurse complete first?

Administer epinephrine (adrenaline).

A patient admitted with heart failure is anxious and reports shortness of breath. Which nursing actions would be appropriate to alleviate this patient's anxiety (select all that apply.)? Administer ordered morphine sulfate. Position patient in a semi-Fowler's position. Position patient on left side with head of bed flat. Instruct patient on the use of relaxation techniques. Use a calm, reassuring approach while talking to patient.

Administer ordered morphine sulfate. -Morphine sulfate reduces anxiety and may assist in reducing dyspnea. The patient should be positioned in semi-Fowler's position to improve ventilation that will reduce anxiety. Relaxation techniques and a calm reassuring approach will also serve to reduce anxiety. Position patient in a semi-Fowler's position. -Morphine sulfate reduces anxiety and may assist in reducing dyspnea. The patient should be positioned in semi-Fowler's position to improve ventilation that will reduce anxiety. Relaxation techniques and a calm reassuring approach will also serve to reduce anxiety Instruct patient on the use of relaxation techniques. -Morphine sulfate reduces anxiety and may assist in reducing dyspnea. The patient should be positioned in semi-Fowler's position to improve ventilation that will reduce anxiety. Relaxation techniques and a calm reassuring approach will also serve to reduce anxiety Use a calm, reassuring approach while talking to patient. -Morphine sulfate reduces anxiety and may assist in reducing dyspnea. The patient should be positioned in semi-Fowler's position to improve ventilation that will reduce anxiety. Relaxation techniques and a calm reassuring approach will also serve to reduce anxiety.

A patient admitted with heart failure is anxious and reports shortness of breath. Which nursing actions would be appropriate to alleviate (giảm bớt) this patient's anxiety (select all that apply.)? Administer ordered morphine sulfate. Position patient in a semi-Fowler's position. Position patient on left side with head of bed flat. Instruct patient on the use of relaxation techniques. Use a calm, reassuring approach while talking to patient.

Administer ordered morphine sulfate. Position patient in a semi-Fowler's position. Instruct patient on the use of relaxation techniques. Use a calm, reassuring approach while talking to patient. Morphine sulfate reduces anxiety and may assist in reducing dyspnea. The patient should be positioned in semi-Fowler's position to improve ventilation that will reduce anxiety. Relaxation techniques and a calm reassuring approach will also serve to reduce anxiety.

D ) The catheter used to place the stent is usually inserted in the femoral artery. The affected leg is maintained in extension to prevent bleeding and hematoma formation. Chest tubes are not needed for this surgery and would not be secured to bed linens in any case. IV lines would be maintained as a saline lock in the event a line is needed for IV medications. Because the stent re-establishes blood flow to the myocardium, narcotic analgesics are rarely needed.

An African American male client with a history of diabetes and stroke returns from coronary angioplasty with stent placement. Which priority action will the nurse perform at this time? A) Discontinuing intravenous lines when taking oral fluids B) Securing chest tubes to the bedding C) Treating chest pain with intravenous morphine as needed D) Maintaining leg extension on the affected side

The nurse is administering a dose of digoxin to a patient with heart failure (HF). The nurse would become concerned with the possibility of digitalis toxicity if the patient reported which symptom? Muscle aches Constipation Pounding headache Anorexia and nausea (Biếng ăn và buồn nôn)

Anorexia (ˌanəˈreksēə) and nausea Anorexia, nausea, vomiting, blurred or yellow vision, and cardiac dysrhythmias are all signs of digitalis toxicity. The nurse would become concerned and notify the health care provider if the patient exhibited any of these symptoms.

The nurse is administering a dose of digoxin to a patient with heart failure (HF). The nurse would become concerned with the possibility of digitalis toxicity if the patient reported which symptom? Muscle aches Constipation Pounding headache Anorexia and nausea

Anorexia and nausea Anorexia, nausea, vomiting, blurred or yellow vision, and cardiac dysrhythmias are all signs of digitalis toxicity. The nurse would become concerned and notify the health care provider if the patient exhibited any of these symptoms.

A patient who had bladder surgery 2 days ago develops acute decompensated heart failure (ADHF) with severe dyspnea. Which action by the nurse would be indicated first? Perform a bladder scan to assess for urinary retention. Restrict the patient's oral fluid intake to 500 mL per day. Assist the patient to a sitting position with arms on the overbed table. Instruct the patient to use pursed-lip breathing until the dyspnea subsides.

Assist the patient to a sitting position with arms on the overbed table. The nurse should place the patient with ADHF in a high Fowler's position with the feet horizontal in the bed or dangling at the bedside. This position helps decrease venous return because of the pooling of blood in the extremities. This position also increases the thoracic capacity, allowing for improved ventilation. Pursed-lip breathing helps with obstructive air trapping but not with acute pulmonary edema. Restricting fluids takes considerable time to have an effect.

The nurse is watching the cardiac monitor, and a patient's rhythm suddenly changes. There are no P waves. Instead there are fine, wavy lines between the QRS complexes. The QRS complexes measure 0.08 sec (narrow), but they occur irregularly with a rate of 120 beats/min. The nurse correctly interprets this rhythm as what? Sinus tachycardia Atrial fibrillation Ventricular fibrillation Ventricular tachycardia

Atrial fibrillation Atrial fibrillation is represented on the cardiac monitor by irregular R-R intervals and small fibrillatory (f) waves. There are no normal P waves because the atria are not truly contracting, just fibrillating. Sinus tachycardia is a sinus rate above 100 beats/minute with normal P waves. Ventricular fibrillation is seen on the ECG without a visible P wave; an unmeasurable heart rate, PR or QRS; and the rhythm is irregular and chaotic. Ventricular tachycardia is seen as three or more premature ventricular contractions (PVCs) that have distorted QRS complexes with regular or irregular rhythm, and the P wave is usually buried in the QRS complex without a measurable PR interval.

Integrated Process: Nursing Process (Implementation) 12. The nurse is assessing clients at a community clinic. Which client does the nurse assess most carefully for atrial fibrillation? a. Middle-aged client who takes an aspirin daily b. Client who is dismissed after coronary artery bypass surgery c. Older adult client after a carotid endarterectomy d. Client with chronic obstructive pulmonary disease

B Atrial fibrillation occurs commonly in clients with cardiac disease and is a common occurrence after coronary artery bypass graft (CABG) surgery. The other conditions do not place a client at higher risk for atrial fibrillation. DIF: Cognitive Level: Application/Applying or higher REF: N/A TOP: Client Needs Category: Physiological Integrity (Reduction of Risk Potential—Potential for Complications from Surgical Procedures and Health Alterations)

Integrated Process: Nursing Process (Assessment) 15. The nurse is caring for a client with chronic atrial fibrillation. Which drug does the nurse expect to administer to prevent a common complication of this condition? a. Sotalol (Betapace) b. Warfarin (Coumadin) c. Atropine (Atropine) d. Lidocaine (Xylocaine)

B Atrial fibrillation puts clients at risk for developing emboli. Clients at risk for emboli are treated with anticoagulants, such as heparin, enoxaparin, or warfarin. The other drugs are not appropriate for this complication. DIF: Cognitive Level: Application/Applying or higher REF: N/A TOP: Client Needs Category: Physiological Integrity (Pharmacological and Parenteral Therapies—Expected Actions/Outcomes)

Integrated Process: Nursing Process (Implementation) 11. A client has a heart rate averaging 56 beats/min with no adverse symptoms. What activity modifications does the nurse suggest to avoid further slowing of the heart rate? a. "Make certain that your bath water is warm (100° F)." b. "Avoid bearing down or straining while having a bowel movement." c. "Avoid strenuous exercise, such as running, during the late afternoon." d. "Limit your intake of caffeinated drinks to no more than 2 cups per day."

B Bearing down strenuously during a bowel movement is one type of Valsalva maneuver, which stimulates the vagus nerve and results in slowing of the heart rate. Such a response is not desirable in a person who has bradycardia. The other instructions are not appropriate for this condition. DIF: Cognitive Level: Application/Applying or higher REF: N/A TOP: Client Needs Category: Health Promotion and Maintenance (Self-Care)

Physiological Integrity 3. The nurse identifies the nursing diagnosis of decreased cardiac output related to valvular insufficiency for the patient with infective endocarditis (IE) based on which assessment finding(s)? a. Fever, chills, and diaphoresis b. Urine output less than 30 mL/hr c. Petechiae on the inside of the mouth and conjunctiva d. Increase in heart rate of 15 beats/minute with walking

B Decreased renal perfusion caused by inadequate cardiac output will lead to decreased urine output. Petechiae, fever, chills, and diaphoresis are symptoms of IE, but are not caused by decreased cardiac output. An increase in pulse rate of 15 beats/minute is normal with exercise. DIF: Cognitive Level: Apply (application) REF: 812 TOP: Nursing Process: Diagnosis MSC:

Integrated Process: Nursing Process (Analysis) 17. A client with myocardial ischemia is having frequent early, wide ventricular complexes seen on the cardiac monitor. Which medication does the nurse administer? a. Lanoxin (Digoxin) b. Amiodarone (Cordarone) c. Dobutamine (Dobutamine) d. Atropine sulfate (Atropisol)

B Early, wide ventricular complexes are premature ventricular contractions (PVCs). Amiodarone, an antidysrhythmic, is the treatment of choice for frequent PVCs. The other medications are not appropriate for this condition. DIF: Cognitive Level: Application/Applying or higher REF: N/A TOP: Client Needs Category: Physiological Integrity (Pharmacological and Parenteral Therapies—Expected Actions/Outcomes)

Integrated Process: Nursing Process (Implementation) 20. A client with unstable ventricular tachycardia is receiving amiodarone by intravenous infusion. The nurse notes that the client's heart rate has decreased from 68 to 50 beats/min. The client is asymptomatic. What is the nurse's priority intervention? a. Stop the infusion and flush the IV. b. Slow the amiodarone infusion rate. c. Administer a precordial thump. d. Place the client in a side-lying position.

B IV administration of amiodarone may cause bradycardia and atrioventricular (AV) block. The correct action for the nurse to take at this time is to slow the infusion, because the client is asymptomatic and no evidence reveals AV block that might require pacing. Abruptly ceasing the medication could allow fatal dysrhythmias to occur. A precordial thump is not required at this time because the client still has a heart rate. A side-lying position will not increase the client's heart rate. DIF: Cognitive Level: Application/Applying or higher REF: N/A TOP: Client Needs Category: Physiological Integrity (Pharmacological and Parenteral Therapies—Adverse Effects/Contraindications/Side Effects/Interactions)

Physiological Integrity 6. To assess the patient with pericarditis for evidence of a pericardial friction rub, the nurse should a. listen for a rumbling, low-pitched, systolic murmur over the left anterior chest. b. auscultate by placing the diaphragm of the stethoscope on the lower left sternal border. c. ask the patient to cough during auscultation to distinguish the sound from a pleural friction rub. d. feel the precordial area with the palm of the hand to detect vibrations with cardiac contraction.

B Pericardial friction rubs are heard best with the diaphragm at the lower left sternal border. The nurse should ask the patient to hold his or her breath during auscultation to distinguish the sounds from a pleural friction rub. Friction rubs are not typically low pitched or rumbling and are not confined to systole. Rubs are not assessed by palpation. DIF: Cognitive Level: Understand (comprehension) REF: 815 TOP: Nursing Process: Assessment MSC:

Integrated Process: Teaching/Learning 32. A nurse assesses the following electrocardiography (ECG) strip from a client's telemetry monitor. What does the nurse chart as the client's ventricular heart rate? a. 40 beats/min b. 80 beats/min c. 120 beats/min d. 160 beats/min

B Precisely 6 seconds is represented by 150 small blocks on ECG paper. The number of R-R intervals, representing ventricular depolarization episodes present in 6 seconds, can be multiplied by 10 to calculate the ventricular heart rate. DIF: Cognitive Level: Application/Applying or higher REF: N/A TOP: Client Needs Category: Physiological Integrity (Physiological Adaptation-Pathophysiology)

Integrated Process: Nursing Process (Analysis) 5. The nurse observes a prominent U wave on the client's electrocardiograph (ECG) tracing. What is the most appropriate action for the nurse to take? a. Document the finding as a normal variant. b. Review the client's daily electrolyte results. c. Move the crash cart closer to the client's room. d. Call for an immediate electrocardiogram.

B Prominent U waves may be the result of hypokalemia. The nurse should review the client's daily electrolyte results. Although documentation is important, this is not a normal variant. Moving the crash cart closer to the room may or may not be warranted. The client does not need an immediate ECG. DIF: Cognitive Level: Application/Applying or higher REF: N/A TOP: Client Needs Category: Physiological Integrity (Physiological Adaptation—Pathophysiology)

14. The nurse identifies the collaborative problem of potential complication: pulmonary edema for a patient in ADHF. When assessing the patient, the nurse will be most concerned about a. an apical pulse rate of 106 beats/min. b. an oxygen saturation of 88% on room air. c. weight gain of 1 kg (2.2 lb) over 24 hours. d. decreased hourly patient urinary output.

B Rationale: A decrease in oxygen saturation to less than 92% indicates hypoxemia. The nurse should administer supplemental oxygen immediately to the patient. An increase in apical pulse rate, 1-kg weight gain, and decreases in urine output also indicate worsening heart failure and require rapid nursing actions, but the low oxygen saturation rate requires the most immediate nursing action. Cognitive Level: Analysis Text Reference: pp. 829-830 Nursing Process: Assessment NCLEX: Physiological Integrity

22. A patient who is receiving dobutamine (Dobutrex) for the treatment of ADHF has all of the following nursing actions included in the plan of care. Which action will be best for the RN to delegate to an experienced LPN/LVN? a. Teach the patient the reasons for remaining on bed rest. b. Monitor the patient's BP every hour. c. Adjust the drip rate to keep the systolic BP >90 mm Hg. d. Call the health care provider about a decrease in urine output.

B Rationale: An experienced LPN/LVN would be able to monitor BP and would know to report significant changes to the RN. Teaching patients and making adjustments to the drip rate for vasoactive medications are RN-level skills. Because the health care provider may order changes in therapy based on the decrease in urine output, the RN should call the health care provider about the decreased urine output. Cognitive Level: Application Text Reference: pp. 827-829 Nursing Process: Planning NCLEX: Safe and Effective Care Environment

5. When the nurse is developing a teaching plan to prevent the development of heart failure in a patient with stage 1 hypertension, the information that is most likely to improve compliance with antihypertensive therapy is that a. hypertensive crisis may lead to development of acute heart failure in some patients. b. hypertension eventually will lead to heart failure by overworking the heart muscle. c. high BP increases risk for rheumatic heart disease. d. high systemic pressure precipitates papillary muscle rupture.

B Rationale: Hypertension is a primary cause of heart failure because the increase in ventricular afterload leads to ventricular hypertrophy and dilation. Hypertensive crisis may precipitate acute heart failure is some patients, but this patient with stage 1 hypertension may not be concerned about a crisis that happens only to some patients. Hypertension does not directly cause rheumatic heart disease (which is precipitated by infection with group A -hemolytic streptococcus) or papillary muscle rupture (which is caused by myocardial infarction/necrosis of the papillary muscle). Cognitive Level: Application Text Reference: p. 822 Nursing Process: Planning NCLEX: Health Promotion and Maintenance

8. A patient admitted to the hospital with an exacerbation of chronic heart failure tells the nurse, "I felt fine when I went to bed, but I woke up in the middle of the night feeling like I was suffocating!" The nurse can best document this assessment information as a. pulsus alternans. b. paroxysmal nocturnal dyspnea. c. two-pillow orthopnea. d. acute bilateral pleural effusion.

B Rationale: Paroxysmal nocturnal dyspnea is caused by the reabsorption of fluid from dependent body areas when the patient is sleeping and is characterized by waking up suddenly with the feeling of suffocation. Pulsus alternans is the alternation of strong and weak peripheral pulses during palpation. Orthopnea indicates that the patient is unable to lie flat because of dyspnea. Pleural effusions develop over a longer time period. Cognitive Level: Comprehension Text Reference: p. 825 Nursing Process: Assessment NCLEX: Physiological Integrity

A 54-year-old male patient who had bladder surgery 2 days ago develops acute decompensated heart failure (ADHF) with severe dyspnea. Which action by the nurse would be indicated first? A Perform a bladder scan to assess for urinary retention. B Restrict the patient's oral fluid intake to 500 mL per day. C Assist the patient to a sitting position with arms on the overbed table. D Instruct the patient to use pursed-lip breathing until the dyspnea subsides.

C Assist the patient to a sitting position with arms on the overbed table. The nurse should place the patient with ADHF in a high Fowler's position with the feet horizontal in the bed or dangling at the bedside. This position helps decrease venous return because of the pooling of blood in the extremities. This position also increases the thoracic capacity, allowing for improved ventilation. Pursed-lip breathing helps with obstructive air trapping but not with acute pulmonary edema. Restricting fluids takes considerable time to have an effect.

A patient who has a history of pulmonary valve stenosis tells the healthcare provider, "I don't have a lot of energy anymore, and both of my feet get swollen in the late afternoon." Which of these problems does the healthcare provider conclude is the likely cause of these clinical findings? A Acute pericarditis B Peripheral artery disease C Right ventricular failure D Deep vein thrombosis (DVT)

C Right ventricular failure

The nurse prepares to administer digoxin (Lanoxin) 0.125 mg to an 82-year-old man admitted with influenza and a history of chronic heart failure. What should the nurse assess before giving the medication? A Prothrombin time B Urine specific gravity C Serum potassium level D Hemoglobin and hematocrit

C Serum potassium level Serum potassium should be monitored because hypokalemia increases the risk for digoxin toxicity. Changes in prothrombin time, urine specific gravity, and hemoglobin or hematocrit would not require holding the digoxin dose.

10. The nurse working in the heart failure clinic will know that teaching for a 74-year-old patient with newly diagnosed heart failure has been effective when the patient a. says that the nitroglycerin patch will be used for any chest pain that develops. b. calls when the weight increases from 124 to 130 pounds in a week. c. tells the home care nurse that furosemide (Lasix) is taken daily at bedtime. d. makes an appointment to see the doctor at least once yearly.

B Rationale: Teaching for a patient with heart failure includes information about the need to weigh daily and notify the health care provider about an increase of 3 pounds in 2 days or 5 pounds in a week. Nitroglycerin patches are used primarily to reduce preload (not to prevent chest pain) in patients with heart failure and should be used daily, not on an "as necessary" basis. Diuretics should be taken earlier in the day to avoid nocturia and sleep disturbance. Heart failure is a chronic condition that will require frequent follow-up rather than an annual health care provider examination. Cognitive Level: Application Text Reference: pp. 826, 833-834, 838 Nursing Process: Evaluation NCLEX: Health Promotion and Maintenance

15. While admitting an 80-year-old patient with heart failure to the medical unit, the nurse obtains the information that the patient lives alone and sometimes confuses the "water pill" with the "heart pill." The nurse makes a note that discharge planning for the patient will need to include a. transfer to a dementia care service. b. referral to a home health care agency. c. placement in a long-term-care facility. d. arrangements for around-the-clock care.

B Rationale: The data about the patient suggest that assistance in developing a system for taking medications correctly at home is needed. A home health nurse will assess the patient's home situation and help the patient to develop a method for taking the two medications as directed. There is no evidence that the patient requires services such as dementia care, long-term-care, or around-the-clock home care. Cognitive Level: Application Text Reference: pp. 836-837 Nursing Process: Assessment NCLEX: Health Promotion and Maintenance

Physiological Integrity 9. The nurse is admitting a patient with possible rheumatic fever. Which question on the admission health history will be most pertinent to ask? a. "Do you use any illegal IV drugs?" b. "Have you had a recent sore throat?" c. "Have you injured your chest in the last few weeks?" d. "Do you have a family history of congenital heart disease?"

B Rheumatic fever occurs as a result of an abnormal immune response to a streptococcal infection. Although illicit IV drug use should be discussed with the patient before discharge, it is not a risk factor for rheumatic fever, and would not be as pertinent when admitting the patient. Family history is not a risk factor for rheumatic fever. Chest injury would cause musculoskeletal chest pain rather than rheumatic fever. DIF: Cognitive Level: Apply (application) REF: 819 TOP: Nursing Process: Assessment MSC:

Physiological Integrity 24. Which assessment finding in a patient who is admitted with infective endocarditis (IE) is most important to communicate to the health care provider? a. Generalized muscle aching b. Sudden onset right flank pain c. Janeway's lesions on the palms d. Temperature 100.7° F (38.1° C)

B Sudden onset of flank pain indicates possible embolization to the kidney and may require diagnostic testing such as a renal arteriogram and interventions to improve renal perfusion. The other findings are typically found in IE, but do not require any new interventions. DIF: Cognitive Level: Apply (application) REF: 812 OBJ: Special Questions: Prioritization TOP: Nursing Process: Assessment MSC:

Integrated Process: Teaching/Learning 31. The nurse is providing discharge instructions for a client with an implantable cardioverter-defibrillator (ICD). What statement by the client indicates a good understanding of the instructions? a. "I should wear a snug-fitting shirt over the ICD." b. "I will avoid sources of strong electromagnetic fields." c. "I can't perform activities that increase my heart rate." d. "Now I can discontinue my antidysrhythmic medication."

B The client being discharged with an ICD is instructed to avoid strong sources of electromagnetic fields. Clients should avoid tight clothing, which could cause irritation over the ICD generator. The client should not engage in strenuous activities that cause the heart rate to meet or exceed the ICD cutoff point because the ICD can discharge inappropriately. The client should continue all prescribed medications. DIF: Cognitive Level: Application/Applying or higher REF: N/A TOP: Client Needs Category: Health Promotion and Maintenance (Self-Care)

Integrated Process: Nursing Process (Evaluation) 30. A client was admitted for a permanent pacemaker insertion. What priority instruction does the nurse include in the client's discharge teaching? a. "Do not submerge your pacemaker, take only showers." b. "Report pulse rates lower than your pacemaker setting." c. "If you feel weak, apply pressure over your generator." d. "Have your pacemaker turned off before having an MRI."

B The client should be instructed to report changes in heart rate or rhythm, such as rates lower than the pacemaker setting or greater than 100 beats/min. DIF: Cognitive Level: Application/Applying or higher REF: N/A TOP: Client Needs Category: Health Promotion and Maintenance (Self-Care)

Physiological Integrity 12. When developing a community health program to decrease the incidence of rheumatic fever, which action would be most important for the community health nurse to include? a. Vaccinate high-risk groups in the community with streptococcal vaccine. b. Teach community members to seek treatment for streptococcal pharyngitis. c. Teach about the importance of monitoring temperature when sore throats occur. d. Teach about prophylactic antibiotics to those with a family history of rheumatic fever.

B The incidence of rheumatic fever is decreased by treatment of streptococcal infections with antibiotics. Family history is not a risk factor for rheumatic fever. There is no immunization that is effective in decreasing the incidence of rheumatic fever. Teaching about monitoring temperature will not decrease the incidence of rheumatic fever. DIF: Cognitive Level: Apply (application) REF: 820 TOP: Nursing Process: Planning MSC:

Safe and Effective Care Environment 33. After receiving the following information about four patients during change-of-shift report, which patient should the nurse assess first? a. Patient with acute pericarditis who has a pericardial friction rub b. Patient who has just returned to the unit after balloon valvuloplasty c. Patient who has hypertrophic cardiomyopathy and a heart rate of 116 d. Patient with a mitral valve replacement who has an anticoagulant scheduled

B The patient who has just arrived after balloon valvuloplasty will need assessment for complications such as bleeding and hypotension. The information about the other patients is consistent with their diagnoses and does not indicate any complications or need for urgent assessment or intervention. DIF: Cognitive Level: Analyze (analysis) REF: 824 OBJ: Special Questions: Prioritization; Multiple Patients TOP: Nursing Process: Planning MSC:

Physiological Integrity 10. A patient with rheumatic fever has subcutaneous nodules, erythema marginatum, and polyarthritis. Based on these findings, which nursing diagnosis would be most appropriate? a. Pain related to permanent joint fixation b. Activity intolerance related to arthralgia c. Risk for infection related to open skin lesions d. Risk for impaired skin integrity related to pruritus

B The patient's joint pain will lead to difficulty with activity. The skin lesions seen in rheumatic fever are not open or pruritic. Although acute joint pain will be a problem for this patient, joint inflammation is a temporary clinical manifestation of rheumatic fever and is not associated with permanent joint changes. DIF: Cognitive Level: Apply (application) REF: 819-820 TOP: Nursing Process: Diagnosis MSC:

Physiological Integrity 4. When planning care for a patient hospitalized with a streptococcal infective endocarditis (IE), which intervention is a priority for the nurse to include? a. Monitor labs for streptococcal antibodies. b. Arrange for placement of a long-term IV catheter. c. Teach the importance of completing all oral antibiotics. d. Encourage the patient to begin regular aerobic exercise.

B Treatment for IE involves 4 to 6 weeks of IV antibiotic therapy in order to eradicate the bacteria, which will require a long-term IV catheter such as a peripherally inserted central catheter (PICC) line. Rest periods and limiting physical activity to a moderate level are recommended during the treatment for IE. Oral antibiotics are not effective in eradicating the infective bacteria that cause IE. Blood cultures, rather than antibody levels, are used to monitor the effectiveness of antibiotic therapy. DIF: Cognitive Level: Apply (application) REF: 814 TOP: Nursing Process: Planning MSC:

Physiological Integrity 23. Which admission order written by the health care provider for a patient admitted with infective endocarditis (IE) and a fever would be a priority for the nurse to implement? a. Administer ceftriaxone (Rocephin) 1 g IV. b. Order blood cultures drawn from two sites. c. Give acetaminophen (Tylenol) PRN for fever. d. Arrange for a transesophageal echocardiogram.

B Treatment of the IE with antibiotics should be started as quickly as possible, but it is essential to obtain blood cultures before initiating antibiotic therapy to obtain accurate sensitivity results. The echocardiogram and acetaminophen administration also should be implemented rapidly, but the blood cultures (and then administration of the antibiotic) have the highest priority. DIF: Cognitive Level: Apply (application) REF: 812-813 OBJ: Special Questions: Prioritization TOP: Nursing Process: Implementation MSC:

When obtaining a health history of a patient admitted with a diagnosis of heart failure, which statement made by the patient supports the diagnosis of heart failure? A "I get hot and break out in a sweat during the night." B "I get out of breath when I go up a flight of stairs." C "I sometimes feel pain in the middle of my chest during exercise." D "I often feel pain in my lower legs when I take my walk."

B "I get out of breath when I go up a flight of stairs."

A patient diagnosed with mild heart failure is prescribed hydrochlorothiazide (Microzide). The healthcare provider should determine the teaching about the medication has been successful if the patient makes which of these statements? A "I might experience swelling in my legs when taking this medication." B "It is important for me to change positions slowly because I might become dizzy." C "This medication might cause me to have a decrease in my appetite." D "I should not worry if I experience a dry cough when taking this medication."

B "It is important for me to change positions slowly because I might become dizzy."

A patient with a diagnosis of heart failure has been started on a nitroglycerin patch by his primary care provider. What should this patient be taught to avoid? A High-potassium foods B Drugs to treat erectile dysfunction C Nonsteroidal antiinflammatory drugs D Over-the-counter H2 -receptor blockers

B Drugs to treat erectile dysfunction The use of erectile drugs concurrent with nitrates creates a risk of severe hypotension and possibly death. High-potassium foods, NSAIDs, and H2-receptor blockers do not pose a risk in combination with nitrates.

What should the nurse recognize as an indication for the use of dopamine (Intropin) in the care of a patient with heart failure? A Acute anxiety B Hypotension and tachycardia C Peripheral edema and weight gain D Paroxysmal nocturnal dyspnea (PND)

B Hypotension and tachycardia Dopamine is a β-adrenergic agonist whose inotropic action is used for treatment of severe heart failure accompanied by hemodynamic instability. Such a state may be indicated by tachycardia accompanied by hypotension. PND, anxiety, edema, and weight gain are common signs and symptoms of heart failure, but these do not necessarily warrant the use of dopamine.

A patient is being assessed for possible heart failure. Which of these laboratory results will provide support this diagnosis? A Decreased serum sodium B Increased brain natriuretic peptide (BNP) C Decreased C-reactive protein D Increased creatine kinase

B Increased brain natriuretic peptide (BNP)

When assessing a patient with chronic heart failure, the healthcare provider would expect to identify which of these clinical manifestations? A Expiratory wheezing B Inspiratory crackles C Subcutaneous crepitus D Asymmetrical chest expansion

B Inspiratory crackles

The nurse is preparing to administer digoxin to a patient with heart failure. In preparation, laboratory results are reviewed with the following findings: sodium 139 mEq/L, potassium 5.6 mEq/L, chloride 103 mEq/L, and glucose 106 mg/dL. What should the nurse do next? A Withhold the daily dose until the following day. B Withhold the dose and report the potassium level. C Give the digoxin with a salty snack, such as crackers. D Give the digoxin with extra fluids to dilute the sodium level.

B Withhold the dose and report the potassium level The normal potassium level is 3.5 to 5.0 mEq/L. The patient is hyperkalemic, which makes the patient more prone to digoxin toxicity. For this reason, the nurse should withhold the dose and report the potassium level. The physician may order the digoxin to be given once the potassium level has been treated and decreases to within normal range.

The patient had aortic aneurysm repair. What priority nursing action will the nurse use to maintain graft patency? A. Assess output for renal dysfunction. B. Use IV fluids to maintain adequate BP. C. Use oral antihypertensives to maintain cardiac output. D. Maintain a low BP to prevent pressure on surgical site

B) The priority is to maintain an adequate BP (determined by the surgeon) to maintain graft patency. A prolonged low BP may result in graft thrombosis, and hypertension may cause undue stress on arterial anastomoses resulting in leakage of blood or rupture at the suture lines, which is when IV antihypertensives may be used. Renal output will be assessed when the aneurysm repair is above the renal arteries to assess graft patency, not maintain it.

A female patient with critical limb ischemia has had peripheral artery bypass surgery to improve her circulation. What care should the nurse provide on postoperative day 1? A) Keep the patient on bed rest. B) Assist the patient with walking several times. C) Have the patient sit in the chair several times. D) Place the patient on her side with knees flexed.

B) To avoid blockage of the graft or stent, the patient should walk several times on postoperative day 1 and subsequent days. Having the patient's knees flexed for sitting in a chair or in bed increase the risk of venous thrombosis and may place stress on the suture lines.

A postoperative patient asks the nurse why the physician ordered daily administration of enoxaparin (Lovenox). Which reply by the nurse is most appropriate? A. "This medication will help prevent breathing problems after surgery, such as pneumonia." B. "This medication will help lower your blood pressure to a safer level, which is very important after surgery." C. "This medication will help prevent blood clots from forming in your legs until your level of activity, such as walking, returns to normal." D. "This medication is a narcotic pain medication that will help take away any muscle aches caused by positioning on the operating room table."

C. "This medication will help prevent blood clots from forming in your legs until your level of activity, such as walking, returns to normal." Enoxaparin is an anticoagulant that is used to prevent DVTs postoperatively. All other explanations/options do not describe the action/purpose of enoxaparin.

The nurse performs discharge teaching for a 68-year-old man who is newly diagnosed with infective endocarditis with a history of IV substance abuse. Which statement by the patient indicates to the nurse that teaching was successful? A."I will need antibiotics before having any invasive procedure or surgery." B. "I will inform my dentist about my hospitalization for infective endocarditis." C."I should not be alarmed if I have difficulty breathing or pink-tinged sputum." D. "An elevated temperature is expected and can be managed by taking acetaminophen."

B. "I will inform my dentist about my hospitalization for infective endocarditis." Patients with infective endocarditis should inform their dental providers of their health history. Antibiotic prophylaxis is recommended for patients with a history of infective endocarditis who have certain dental procedures performed. Antibiotics are not indicated before genitourinary or gastrointestinal procedures unless an infection is present. Patients should immediately report the presence of fever or clinical manifestations indicating heart failure to their health care provider.

A patient with varicose veins has been prescribed compression stockings. How should the nurse teach the patient to use these? A. "Try to keep your stockings on 24 hours a day, as much as possible." B. "While you're still lying in bed in the morning, put on your stockings." C. "Dangle your feet at your bedside for 5 minutes before putting on your stockings." D. "Your stockings will be most effective if you can remove them for a few minutes several times a day."

B. "While you're still lying in bed in the morning, put on your stockings." The patient with varicose veins should apply stockings in bed, before rising in the morning. Stockings should not be worn continuously, but they should not be removed several times daily. Dangling at the bedside prior to application is likely to decrease their effectiveness.

A nurse is caring for a patient with a diagnosis of deep venous thrombosis (DVT). The patient has an order to receive 30 mg enoxaparin (Lovenox). Which injection site should the nurse use to administer this medication safely? A. Buttock, upper outer quadrant B. Abdomen, anterior-lateral aspect C. Back of the arm, 2 inches away from a mole D. Anterolateral thigh, with no scar tissue nearby

B. Abdomen, anterior-lateral aspect Enoxaparin (Lovenox) is a low-molecular-weight (LMW) heparin that is given as a deep subcutaneous injection in the right and left anterolateral abdomen. All subcutaneous injections should be given away from scars, lesions, or moles.

The patient had a history of rheumatic fever and has been diagnosed with mitral valve stenosis. The patient is planning to have a biologic valve replacement. What protective mechanisms should the nurse teach the patient about using after the valve replacement? A. Long-term anticoagulation therapy B. Antibiotic prophylaxis for dental care C. Exercise plan to increase cardiac tolerance D. Take β-adrenergic blockers to control palpitations.

B. Antibiotic prophylaxis for dental care The patient will need to use antibiotic prophylaxis for dental care to prevent endocarditis. Long-term anticoagulation therapy is not used with biologic valve replacement unless the patient has atrial fibrillation. An exercise plan to increase cardiac tolerance is needed for a patient with heart failure. Taking β-adrenergic blockers to control palpitations is prescribed for mitral valve prolapse, not valve replacement.

The patient had a history of rheumatic fever and has been diagnosed with mitral valve stenosis. The patient is planning to have a biologic valve replacement. What protective mechanisms should the nurse teach the patient about using after the valve replacement? A. Long-term anticoagulation therapy B. Antibiotic prophylaxis for dental care C. Exercise plan to increase cardiac tolerance D. Take β-adrenergic blockers to control palpitations.

B. Antibiotic prophylaxis for dental care The patient will need to use antibiotic prophylaxis for dental care to prevent endocarditis. Long-term anticoagulation therapy is not used with biologic valve replacement unless the patient has atrial fibrillation. An exercise plan to increase cardiac tolerance is needed for a patient with heart failure. Taking β-adrenergic blockers to control palpitations is prescribed for mitral valve prolapse, not valve replacement.

The nurse is caring for a newly admitted patient with vascular insufficiency. The patient has a new order for enoxaparin (Lovenox) 30 mg subcutaneously. What should the nurse do to correctly administer this medication? A. Spread the skin before inserting the needle. B. Leave the air bubble in the prefilled syringe. C. Use the back of the arm as the preferred site. D. Sit the patient at a 30-degree angle before administration.

B. Leave the air bubble in the prefilled syringe. The nurse should not expel the air bubble from the prefilled syringe because it should be injected to clear the needle of medication and avoid leaving medication in the needle track in the tissue.

The nurse conducts a complete physical assessment on a patient admitted with infective endocarditis. Which finding is significant? A. Respiratory rate of 18 and heart rate of 90 B. Regurgitant murmur at the mitral valve area C. Heart rate of 94 and capillary refill time of 2 seconds D. Point of maximal impulse palpable in fourth intercostal space

B. Regurgitant murmur at the mitral valve area A regurgitant murmur of the aortic or mitral valves would indicate valvular disease, which is a complication of endocarditis. All the other findings are within normal limits.

The nurse conducts a complete physical assessment on a patient admitted with infective endocarditis. Which finding is significant? A. Respiratory rate of 18 and heart rate of 90 B. Regurgitant murmur at the mitral valve area C. Heart rate of 94 and capillary refill time of 2 seconds D. Point of maximal impulse palpable in fourth intercostal space

B. Regurgitant murmur at the mitral valve area A regurgitant murmur of the aortic or mitral valves would indicate valvular disease, which is a complication of endocarditis. All the other findings are within normal limits.

The home care nurse visits a 73-year-old Hispanic woman with chronic heart failure. Which clinical manifestations, if assessed by the nurse, would indicate acute decompensated heart failure (pulmonary edema)? A. Fatigue, orthopnea, and dependent edema B. Severe dyspnea and blood-streaked, frothy sputum C. Temperature is 100.4o F and pulse is 102 beats/minute D. Respirations 26 breaths/minute despite oxygen by nasal cannula

B. Severe dyspnea and blood-streaked, frothy sputum Clinical manifestations of pulmonary edema include anxiety, pallor, cyanosis, clammy and cold skin, severe dyspnea, use of accessory muscles of respiration, a respiratory rate > 30 breaths per minute, orthopnea, wheezing, and coughing with the production of frothy, blood-tinged sputum. Auscultation of the lungs may reveal crackles, wheezes, and rhonchi throughout the lungs. The heart rate is rapid, and blood pressure may be elevated or decreased.

A 25-year-old patient with a group A streptococcal pharyngitis does not want to take the antibiotics prescribed. What should the nurse tell the patient to encourage the patient to take the medications and avoid complications of the infection? A. "The complications of this infection will affect the skin, hair, and balance." B. "You will not feel well if you do not take the medicine and get over this infection." C. "Without treatment, you could get rheumatic fever, which can lead to rheumatic heart disease." D. "You may not want to take the antibiotics for this infection, but you will be sorry if you do not."

C. "Without treatment, you could get rheumatic fever, which can lead to rheumatic heart disease." Rheumatic fever (RF) is not common because of effective use of antibiotics to treat streptococcal infections. Without treatment, RF can occur and lead to rheumatic heart disease, especially in young adults. The complications do not include hair or balance. Saying that the patient will not feel well or that the patient will be sorry if the antibiotics are not taken is threatening to the patient and inappropriate for the nurse to say.

A 25-year-old patient with a group A streptococcal pharyngitis does not want to take the antibiotics prescribed. What should the nurse tell the patient to encourage the patient to take the medications and avoid complications of the infection? A. "The complications of this infection will affect the skin, hair, and balance." B. "You will not feel well if you do not take the medicine and get over this infection." C. "Without treatment, you could get rheumatic fever, which can lead to rheumatic heart disease." D. "You may not want to take the antibiotics for this infection, but you will be sorry if you do not."

C. "Without treatment, you could get rheumatic fever, which can lead to rheumatic heart disease." Rheumatic fever (RF) is not common because of effective use of antibiotics to treat streptococcal infections. Without treatment, RF can occur and lead to rheumatic heart disease, especially in young adults. The complications do not include hair or balance. Saying that the patient will not feel well or that the patient will be sorry if the antibiotics are not taken is threatening to the patient and inappropriate for the nurse to say.

The nurse obtains a 6-second rhythm strip and charts the following analysis: Tab 1 Atrial data Rate: 70, regular Variable PR interval Independent beats Tab 2 Ventricular data Rate: 40, regular Isolated escape beats Tab 3 Additional data QRS: 0.04 sec P wave and QRS complexes unrelated What is the correct interpretation of this rhythm strip? A. Sinus arrhythmias B. Third-degree heart block C. Wenckebach phenomenon D. Premature ventricular contractions

B. Third-degree heart block Third-degree heart block represents a loss of communication between the atrium and ventricles from AV node dissociation. This is depicted on the rhythm strip as no relationship between the P waves (representing atrial contraction) and QRS complexes (representing ventricular contraction). The atria are beating totally on their own at 70 beats/min, whereas the ventricles are pacing themselves at 40 beats/min. Sinus dysrhythmia is seen with a slower heart rate with exhalation and an increased heart rate with inhalation. In Wenckebach heart block, there is a gradual lengthening of the PR interval until an atrial impulse is nonconducted and a QRS complex is blocked or missing. Premature ventricular contractions (PVCs) are the early occurrence of a wide, distorted QRS complex.

The nurse is preparing to administer digoxin to a patient with heart failure. In preparation, laboratory results are reviewed with the following findings: sodium 139 mEq/L, potassium 5.6 mEq/L, chloride 103 mEq/L, and glucose 106 mg/dL. What should the nurse do next? A. Withhold the daily dose until the following day. B. Withhold the dose and report the potassium level. C. Give the digoxin with a salty snack, such as crackers. D. Give the digoxin with extra fluids to dilute the sodium level.

B. Withhold the dose and report the potassium level. The normal potassium level is 3.5 to 5.0 mEq/L. The patient is hyperkalemic, which makes the patient more prone to digoxin toxicity. For this reason, the nurse should withhold the dose and report the potassium level. The physician may order the digoxin to be given once the potassium level has been treated and decreases to within normal range.

Which statement best describes the electrical activity of the heart represented by measuring the PR interval on the ECG? A. The length of time it takes to depolarize the atrium B. The length of time it takes for the atria to depolarize and repolarize C. The length of time for the electrical impulse to travel from the SA node to the Purkinje fibers D. The length of time it takes for the electrical impulse to travel from the SA node to the AV node

C. The length of time for the electrical impulse to travel from the SA node to the Purkinje fibers The electrical impulse in the heart must travel from the SA node through the AV node and into the Purkinje fibers in order for synchronous atrial and ventricular contraction to occur. When measuring the PR interval (the time from the beginning of the P wave to the beginning of the QRS), the nurse is identifying the length of time it takes for the electrical impulse to travel from the SA node to the Purkinje fibers. The P wave represents the length of time it takes for the impulse to travel from the SA node through the atrium causing depolarization of the atria (atrial contraction). Atrial repolarization occurs during ventricular depolarization and is hidden by the QRS complex. The length of time it takes for the electrical impulse to travel from the SA node to the AV node is the flat line between the end of the P wave and the beginning of the Q wave on the ECG and is not usually measured.

A patient is scheduled for a heart transplant. Beyond the first year after a heart transplant, the nurse knows that what is a major cause of death? Infection Acute rejection Immunosuppression Cardiac vasculopathy

Cardiac vasculopathy Beyond the first year after a heart transplant, malignancy (ác tính) (especially lymphoma) and cardiac vasculopathy (accelerated coronary artery disease) are the major causes of death. During the first year after transplant, infection and acute rejection are the major causes of death. Immunosuppressive therapy will be used for posttransplant management to prevent rejection and increases the

The nurse is caring for a client who has just experienced an acute myocardial infarction and is diagnosed with "pump failure." The nurse is aware that the client is experiencing which type of shock?

Cardiogenic

A patient with a long-standing history of heart failure recently qualified for hospice care. What measure should the nurse now prioritize when providing care for this patient? Taper the patient off his current medications. Continue education for the patient and his family. Pursue experimental therapies or surgical options. Choose interventions to promote comfort and prevent suffering.

Choose interventions to promote comfort and prevent suffering. The central focus of hospice care is the promotion of comfort and the prevention of suffering. Patient education should continue, but providing comfort is paramount. Medications should be continued unless they are not tolerated. Experimental therapies and surgeries are not commonly used in the care of hospice patients.

Which of the following statements regarding heart failure is true?

Compensated congestive heart failure may be clinically asymptomatic.

Physiological Integrity 21. The nurse is obtaining a health history from a 24-year-old patient with hypertrophic cardiomyopathy (HC). Which information obtained by the nurse is most important? a. The patient has a history of a recent upper respiratory infection. b. The patient has a family history of coronary artery disease (CAD). c. The patient reports using cocaine a "couple of times" as a teenager. d. The patient's 29-year-old brother died from a sudden cardiac arrest.

D About half of all cases of HC have a genetic basis, and it is the most common cause of sudden cardiac death in otherwise healthy young people. The information about the patient's brother will be helpful in planning care (such as an automatic implantable cardioverter-defibrillator [AICD]) for the patient and in counseling other family members. The patient should be counseled against the use of stimulant drugs, but the limited past history indicates that the patient is not at current risk for cocaine use. Viral infections and CAD are risk factors for dilated cardiomyopathy, but not for HC. DIF: Cognitive Level: Apply (application) REF: 828 TOP: Nursing Process: Assessment MSC:

1. The nurse obtains a health history from a 65-year-old patient with a prosthetic mitral valve who has symptoms of infective endocarditis (IE). Which question by the nurse is most appropriate? a. "Do you have a history of a heart attack?" b. "Is there a family history of endocarditis?" c. "Have you had any recent immunizations?" d. "Have you had dental work done recently?"

D Dental procedures place the patient with a prosthetic mitral valve at risk for infective endocarditis (IE). Myocardial infarction (MI), immunizations, and a family history of endocarditis are not risk factors for IE. DIF: Cognitive Level: Apply (application) REF: 812 TOP: Nursing Process: Assessment MSC:

Integrated Process: Nursing Process (Implementation) 29. The nurse is recovering a client after insertion of an implantable cardioverter-defibrillator (ICD). What complication must the nurse intervene for immediately? a. 2/4 bilateral peripheral edema b. Heart rate of 56 beats/min c. Temperature of 96° F (35.5° C) d. Muffled heart sounds

D In the postimplantation period, the nurse should be alert for complications of cardiac tamponade, bleeding, and dysrhythmias. Muffled heart sounds are a manifestation of cardiac tamponade. Edema and a lower temperature would not be indicative of a complication of this procedure. Bradycardia might need intervention, but this client's heart rate is not critically low. DIF: Cognitive Level: Application/Applying or higher REF: N/A TOP: Client Needs Category: Physiological Integrity (Reduction of Risk Potential—Potential for Complications from Diagnostic Tests/Treatments/Procedures)

17. Following an acute myocardial infarction, a previously healthy 67-year-old patient develops clinical manifestations of heart failure. The nurse anticipates discharge teaching will include information about a. digitalis preparations, such as digoxin (Lanoxin). b. calcium-channel blockers, such as diltiazem (Cardizem). c. -adrenergic agonists, such as dobutamine (Dobutrex). d. angiotensin-converting enzyme (ACE) inhibitors, such as captopril (Capoten).

D Rationale: ACE-inhibitor therapy is currently recommended to prevent the development of heart failure in patients who have had a myocardial infarction and as a first-line therapy for patients with chronic heart failure. Digoxin therapy for heart failure is no longer considered a first-line measure, and digoxin is added to the treatment protocol when therapy with other medications such as ACE-inhibitors, diuretics, and -adrenergic blockers is insufficient. Calcium-channel blockers are not generally used in the treatment of heart failure. The -adrenergic agonists such as dobutamine are administered through the IV route and are not used as initial therapy for heart failure. Cognitive Level: Application Text Reference: p. 832 Nursing Process: Implementation NCLEX: Physiological Integrity

23. A hospitalized patient with heart failure has a new order for captopril (Capoten) 12.5 mg PO. After administering the first dose and teaching the patient about captopril, which statement by the patient indicates that teaching has been effective? a. "I will need to include more high-potassium foods in my diet." b. "I will expect to feel more short of breath for the next few days." c. "I will be sure to take the medication after eating something." d. "I will call for help when I need to get up to the bathroom."

D Rationale: Captopril can cause hypotension, especially after the initial dose, so it is important that the patient not get up out of bed without assistance until the nurse has had a chance to evaluate the effect of the first dose. The ACE inhibitors are potassium sparring, and the nurse should not teach the patient to increase sources of dietary potassium. Increased shortness of breath is expected with initiation of -blocker therapy for heart failure, not for ACE-inhibitor therapy. ACE inhibitors are best absorbed when taken an hour before eating. Cognitive Level: Application Text Reference: p. 832 Nursing Process: Evaluation NCLEX: Physiological Integrity

12. When teaching the patient with heart failure about a 2000-mg sodium diet, the nurse explains that foods to be restricted include a. eggs and other high-cholesterol foods. b. canned and frozen fruits. c. fresh or frozen vegetables. d. milk, yogurt, and other milk products.

D Rationale: Milk and yogurt naturally contain a significant amount of sodium, and intake of these should be limited for patients on a diet that limits sodium to 2000 mg daily. Other milk products, such as processed cheeses, have very high levels of sodium and are not appropriate for a 2000-mg sodium diet. The other foods listed have minimal levels of sodium and can be eaten without restriction. Cognitive Level: Application Text Reference: p. 833 Nursing Process: Implementation NCLEX: Health Promotion and Maintenance

Physiological Integrity 8. The nurse has identified a nursing diagnosis of acute pain related to inflammatory process for a patient with acute pericarditis. The priority intervention by the nurse for this problem is to a. teach the patient to take deep, slow breaths to control the pain. b. force fluids to 3000 mL/day to decrease fever and inflammation. c. remind the patient to request opioid pain medication every 4 hours. d. place the patient in Fowler's position, leaning forward on the overbed table.

D Sitting upright and leaning forward frequently will decrease the pain associated with pericarditis. Forcing fluids will not decrease the inflammation or pain. Taking deep breaths will tend to increase pericardial pain. Opioids are not very effective at controlling pain caused by acute inflammatory conditions and are usually ordered PRN. The patient would receive scheduled doses of a nonsteroidal antiinflammatory drug (NSAID). DIF: Cognitive Level: Apply (application) REF: 817 OBJ: Special Questions: Prioritization TOP: Nursing Process: Implementation MSC:

Safe and Effective Care Environment 35. The nurse is caring for a patient with mitral regurgitation. Referring to the figure below, where should the nurse listen to best hear any murmur that the patient has? a. 1 b. 2 c. 3 d. 4

D Sounds from the mitral valve are best heard at the apex of the heart, fifth intercostal space, midclavicular line. DIF: Cognitive Level: Understand (comprehension) REF: 821 TOP: Nursing Process: Assessment MSC:

Integrated Process: Nursing Process (Assessment) 3. A nurse notes that the PR interval on a client's electrocardiograph (ECG) tracing is 0.14 second. What action does the nurse take? a. Assess serum cardiac enzymes. b. Administer 1 mg epinephrine IV. c. Administer oxygen via nasal cannula. d. Document the finding in the client's chart.

D The PR interval normally ranges from 0.12 to 0.20 second. This is a normal finding, so the nurse simply documents this. No further action is required. DIF: Cognitive Level: Application/Applying or higher REF: N/A TOP: Client Needs Category: Physiological Integrity Physiological Integrity (Physiological Adaptation—Pathophysiology)

Physiological Integrity 20. A patient admitted with acute dyspnea is newly diagnosed with dilated cardiomyopathy. Which information will the nurse plan to teach the patient about managing this disorder? a. A heart transplant should be scheduled as soon as possible. b. Elevating the legs above the heart will help relieve dyspnea. c. Careful compliance with diet and medications will prevent heart failure. d. Notify the doctor about any symptoms of heart failure such as shortness of breath.

D The patient should be instructed to notify the health care provider about any worsening of heart failure symptoms. Because dilated cardiomyopathy does not respond well to therapy, even patients with good compliance with therapy may have recurrent episodes of heart failure. Elevation of the legs above the heart will worsen symptoms (although this approach is appropriate for a patient with hypertrophic cardiomyopathy). The patient with terminal or end-stage cardiomyopathy may consider heart transplantation. DIF: Cognitive Level: Apply (application) REF: 828 TOP: Nursing Process: Planning MSC:

Integrated Process: Nursing Process (Assessment) 28. The nurse is assisting with resuscitation of a client. What priority intervention does the nurse perform before defibrillating a client? a. Make sure the defibrillator is set to the synchronous mode. b. Deliver a precordial thump to the upper portion of the sternum. c. Test the equipment by delivering a smaller shock at 100 J. d. Ensure that all personnel are clear of contact with the client and the bed.

D To avoid injury, the rescuer commands that all personnel clear contact with the client or the bed and ensures their compliance before delivery of the shock. A precordial thumb can be delivered when no defibrillator is available. Equipment should not be tested before a client is defibrillated because this is an emergency procedure; equipment should be checked on a routine basis. Defibrillation is done in asynchronous mode. DIF: Cognitive Level: Application/Applying or higher REF: N/A TOP: Client Needs Category: Safe and Effective Care Environment (Safety and Infection Control—Safe Use of Equipment)

Physiological Integrity 15. While caring for a 23-year-old patient with mitral valve prolapse (MVP) without valvular regurgitation, the nurse determines that discharge teaching has been effective when the patient states that it will be necessary to a. take antibiotics before any dental appointments. b. limit physical activity to avoid stressing the heart. c. take an aspirin a day to prevent clots from forming on the valve. d. avoid use of over-the-counter (OTC) medications that contain stimulant drugs.

D Use of stimulant medications should be avoided by patients with MVP because these may exacerbate symptoms. Daily aspirin and restricted physical activity are not needed by patients with mild MVP. Antibiotic prophylaxis is needed for patients with MVP with regurgitation but will not be necessary for this patient. DIF: Cognitive Level: Apply (application) REF: 822 TOP: Nursing Process: Evaluation MSC:

A stable patient with acute decompensated heart failure (ADHF) suddenly becomes dyspneic. Before positioning the patient on the bedside, what should the nurse assess first? A Urine output B Heart rhythm C Breath sounds D Blood pressure

D Blood pressure The nurse should evaluate the blood pressure before dangling the patient on the bedside because the blood pressure can decrease as blood pools in the periphery and preload decreases. If the patient's blood pressure is low or marginal, the nurse should put the patient in the semi-Fowler's position and use other measures to improve gas exchange.

Beyond the first year after a heart transplant, the nurse knows that what is a major cause of death? A Infection B Acute rejection C Immunosuppression D Cardiac vasculopathy

D Cardiac vasculopathy Beyond the first year after a heart transplant, malignancy (especially lymphoma) and cardiac vasculopathy (accelerated CAD) are the major causes of death. During the first year after transplant, infection and acute rejection are the major causes of death. Immunosuppressive therapy will be used for posttransplant management to prevent rejection and increases the patient's risk of an infection.

A male patient with a long-standing history of heart failure has recently qualified for hospice care. What measure should the nurse now prioritize when providing care for this patient? A Taper the patient off his current medications. B Continue education for the patient and his family. C Pursue experimental therapies or surgical options. D Choose interventions to promote comfort and prevent suffering.

D Choose interventions to promote comfort and prevent suffering. The central focus of hospice care is the promotion of comfort and the prevention of suffering. Patient education should continue, but providing comfort is paramount. Medications should be continued unless they are not tolerated. Experimental therapies and surgeries are not commonly used in the care of hospice patients.

When considering physiological changes common in geriatric patients, the healthcare provider understands that which of these factors may increase the risk of heart failure in older patients? A Decreased sympathetic activity B Increased stroke volume C Increased myocardial contractility D Impaired diastolic filling

D Impaired diastolic filling

A patient has been admitted to the cardiac unit with a diagnosis of right ventricular failure. Which of the following assessment findings is most likely to be observed by the healthcare provider? A Bradycardia and circumoral cyanosis B Fatigue and hemoptysis C Dyspnea and pulmonary crackles D Peripheral edema and jugular vein distension

D Peripheral edema and jugular vein distension

After having an MI, the nurse notes the patient has jugular venous distention, gained weight, developed peripheral edema, and has a heart rate of 108/minute. What should the nurse suspect is happening? A ADHF B Chronic HF C Left-sided HF D Right-sided HF

D Right-sided HF An MI is a primary cause of heart failure. The jugular venous distention, weight gain, peripheral edema, and increased heart rate are manifestations of right-sided heart failure.

What is a priority nursing intervention in the care of a patient with a diagnosis of chronic venous insufficiency (CVI)? A) Application of topical antibiotics to venous ulcers B) Maintaining the patient's legs in a dependent position C) Administration of oral and/or subcutaneous anticoagulants D) Teaching the patient the correct use of compression stockings

D) CVI requires conscientious and consistent application of compression stockings. Anticoagulants are not necessarily indicated and antibiotics, if required, are typically oral or IV, not topical. The patient should avoid prolonged positioning with the limb in a dependent position.

The patient reports tenderness when she touches her leg over a vein. The nurse assesses warmth and a palpable cord in the area. The nurse knows the patient needs treatment to prevent which sequelae? A. Pulmonary embolism B. Pulmonary hypertension C. Post-thrombotic syndrome D. Venous thromboembolism

D) The clinical manifestations are characteristic of a superficial vein thrombosis. If untreated, the clot may extend to deeper veins, and venous thromboembolism may occur. Pulmonary embolism, pulmonary hypertension, and post-thrombotic syndrome are the sequelae of venous thromboembolism.

A stable patient with acute decompensated heart failure (ADHF) suddenly becomes dyspneic. Before positioning the patient on the bedside, what should the nurse assess first? A. Urine output B. Heart rhythm C. Breath sounds D. Blood pressure

D. Blood pressure The nurse should evaluate the blood pressure before dangling the patient on the bedside because the blood pressure can decrease as blood pools in the periphery and preload decreases. If the patient's blood pressure is low or marginal, the nurse should put the patient in the semi-Fowler's position and use other measures to improve gas exchange.

After having an MI, the nurse notes the patient has jugular venous distention, gained weight, developed peripheral edema, and has a heart rate of 108/minute. What should the nurse suspect is happening? A. ADHF B. Chronic HF C. Left-sided HF D. Right-sided HF

D. Right-sided HF An MI is a primary cause of heart failure. The jugular venous distention, weight gain, peripheral edema, and increased heart rate are manifestations of right-sided heart failure.

The acronym FACES is used to help educate patients to identify symptoms of heart failure. what does this acronym mean?

Fatigue Limitation of activities chest congestion/Cough edema SOB

Lisinopril (Prinivil) is part of the treatment regimen for a client with HF. The nurse monitors the client for which electrolyte imbalance of this drug? Hyponatremia Hyperkalemia Hypokalemia Hypernatremia

Hyperkalemia Rationale: ACE inhibitors block aldosterone secretion, which results in sodium loss and potassium retention. Hyperkalemia may occur, especially when the drug is taken concurrently with potassium-sparing diuretics.

What should the nurse recognize as an indication for the use of dopamine in the care of a patient with heart failure? Acute anxiety Hypotension and tachycardia Peripheral edema and weight gain Paroxysmal nocturnal dyspnea (PND)

Hypotension and tachycardia Dopamine is a β-adrenergic agonist whose inotropic action is used for treatment of severe heart failure accompanied by hemodynamic instability. Such a state may be indicated by tachycardia accompanied by hypotension. PND, anxiety, edema, and weight gain are common signs and symptoms of heart failure, but these do not necessarily warrant the use of dopamine.

A 20-year-old college student has a pelvic fracture and a severed leg from a motorcycle accident. She lost several units of blood. When the student arrived in the emergency department, her blood pressure was very low, her pulse was high, and her skin was pale. The nurse knows that this patient has developed which of the following types of shock?

Hypovolemic

Acute HF Collab Care

Improve Cardiac Function positive inotropic drugs (dobutamine) - increases force of contraction Dopamine (raises HR) vs dobutamine (does NOT raise HR) If on dobutamine you need to be on BP lowering drugs (because dobutamine raises BP) Improve gas exchange and oxygenation Probably 100% O2 They usually vomit and can't breath well - INTUBATE (gets pulled out 24 hr later) Reducing Anxiety Explain everything you're doing to help calm the pt

A) Normal ejection fraction is 60%; 25% ejection indicates severe dysfunction. The percentage represents the amount of blood ejected from the ventricle, not the amount retained. Cardiac output is decreased in heart failure.

In reviewing the physician's admitting notes for a client with heart failure, the nurse notes that the client has an ejection fraction of 25%. What is the appropriate interpretation of the nurse's findings? A) Ventricular function is severely impaired. B) Cardiac output is greater than normal. C) The amount of blood ejected from the ventricles is within normal limits. D) 25% of the blood in the ventricle remains after systole.

D The most accurate area on the body to assess dependent edema in a bed-ridden client is the sacral area. Sacral, or dependent, edema is secondary to right-sided heart failure.

In which of the following disorders would the nurse expect to assess sacral edema in a bedridden client? A Diabetes B Pulmonary emboli C Renal failure D Right-sided heart failure

The nurse developing a teaching plan for a client receiving thiazide diuretics should include the following. Teaching the client to take apical pulse. Decreasing potassium-rich foods in the diet. Including citrus fruits, melons, and vegetables in the diet. Teaching the client to check blood pressure t.i.d.

Including citrus fruits, melons, and vegetables in the diet. Rationale: Thiazide diuretics are potassium wasting, and levels should be closely monitored. Encouraging foods rich in potassium could help maintain potassium levels. Taking an apical pulse is indicated before administering cardiac glycosides and beta blockers. It would not be necessary to check blood pressure TID unless the client was experiencing hypotension.

The patient has heart failure (HF) with an ejection fraction of less than 40%. What core measures should the nurse expect to include in the plan of care for this patient (select all that apply.)? Left ventricular function is documented. Controlling dysrhythmias will eliminate HF. Prescription for digoxin (Lanoxin) at discharge Prescription for angiotensin-converting enzyme inhibitor at discharge Education materials about activity, medications, weight monitoring, and what to do if symptoms worsen

Left ventricular function is documented. -The Joint Commission has identified these three core measures for heart failure patients. Although controlling dysrhythmias will improve CO and workload, it will not eliminate HF. Prescribing digoxin for all HF patients is no longer done because there are newer effective drugs and digoxin toxicity occurs easily related to electrolyte levels and the therapeutic range must be maintained. Prescription for angiotensin-converting enzyme inhibitor at discharge -The Joint Commission has identified these three core measures for heart failure patients. Although controlling dysrhythmias will improve CO and workload, it will not eliminate HF. Prescribing digoxin for all HF patients is no longer done because there are newer effective drugs and digoxin toxicity occurs easily related to electrolyte levels and the therapeutic range must be maintained. Education materials about activity, medications, weight monitoring, and what to do if symptoms worsen -The Joint Commission has identified these three core measures for heart failure patients. Although controlling dysrhythmias will improve CO and workload, it will not eliminate HF. Prescribing digoxin for all HF patients is no longer done because there are newer effective drugs and digoxin toxicity occurs easily related to electrolyte levels and the therapeutic range must be maintained.

The client is prescribed captopril (Capoten) for treatment of HF. The nurse teaches that the primary action of the drug is to Prevent influx of calcium. Lower peripheral resistance and reduce blood volume. Increase strength of ventricular contractions. Increase heart rate.

Lower peripheral resistance and reduce blood volume. Rationale: ACE inhibitors decrease peripheral resistance and reduce blood volume by enhancing the excretion of sodium and water. This results in decreased afterload and increased cardiac output.

When caring for a client receiving triamterene (Dyrenium), the nurse places priority on which nursing intervention? Monitor electrolytes for hyperkalemia. Monitor electrolytes for hypernatremia. Monitor heart rate closely. Monitor blood pressure closely.

Monitor electrolytes for hyperkalemia. Rationale: Dyrenium is a potassium-sparing diuretic. The client should be monitored for signs of hyperkalemia. Hyponatremia, not hypernatremia may occur. BP and heart rate should be monitored, but are not priority with this drug.

A client is being started on lisinopril (Zestril). Nursing interventions during initial therapy with this medication must include Monitoring blood pressure. Monitoring intake and output. Monitoring EKG. Monitoring serum levels.

Monitoring blood pressure. Rationale: Lisinopril is an ACE inhibitor, which can cause severe hypotension with initial doses. The nurse should monitor the client closely for several hours.

A patient has developed cardiogenic shock. The most frequent cause of this type of shock is which of the following?

Myocardial infarction

The patient is admitted with acute coronary syndrome (ACS). The ECG shows ST-segment depression and T-wave inversion. What should the nurse know that this indicates? Myocardia injury Myocardial ischemia Myocardial infarction A pacemaker is present.

Myocardial ischemia The ST depression and T wave inversion on the ECG of a patient diagnosed with ACS indicate myocardial ischemia from inadequate supply of blood and oxygen to the heart. Myocardial injury is identified with ST-segment elevation. Myocardial infarction is identified with ST-segment elevation and a widened and deep Q wave. A pacemaker's presence is evident on the ECG by a spike leading to depolarization and contraction.

Which ECG characteristic is consistent with a diagnosis of ventricular tachycardia (VT)? Unmeasurable rate and rhythm Rate 150 beats/min; inverted P wave Rate 200 beats/min; P wave not visible Rate 125 beats/min; normal QRS complex

Rate 200 beats/min; P wave not visible VT is associated with a rate of 150 to 250 beats/min; the P wave is not normally visible. Rate and rhythm are not measurable in ventricular fibrillation. P wave inversion and a normal QRS complex are not associated with VT.

After having a myocardial infarction (MI), the nurse notes the patient has jugular venous distention, gained weight, developed peripheral edema, and has a heart rate of 108 beats/min. What should the nurse suspect is happening? ADHF Chronic HF Left-sided HF Right-sided HF

Right-sided HF An MI is a primary cause of heart failure. The jugular venous distention, weight gain, peripheral edema, and increased heart rate are manifestations of right-sided heart failure.

The nurse prepares to administer digoxin 0.125 mg to a patient admitted with influenza and a history of chronic heart failure. What should the nurse assess before giving the medication? Prothrombin time Urine specific gravity Serum potassium level Hemoglobin and hematocrit

Serum potassium level Serum potassium should be monitored because hypokalemia increases the risk for digoxin toxicity. Changes in prothrombin time, urine specific gravity, and hemoglobin or hematocrit would not require holding the digoxin dose.

The nurse prepares to administer digoxin 0.125 mg to a patient admitted with influenza and a history of chronic heart failure. What should the nurse assess before giving the medication? Prothrombin time Urine specific gravity Serum potassium level Hemoglobin and hematocrit

Serum potassium level Serum potassium should be monitored because hypokalemia increases the risk for digoxin toxicity. Changes in prothrombin time, urine specific gravity, and hemoglobin or hematocrit would not require holding the digoxin dose.

The nurse reviews lab studies of a client receiving digoxin (Lanoxin). Intervention by the nurse is required if the results include a Serum sodium level of 140 mEq/L. Serum digoxin level of 1.2 ng/dL. Serum potassium level of 3.0 mEq/L. Hemoglobin 14.4 g/dL.

Serum potassium level of 3.0 mEq/L. Rationale: Normal serum potassium level is 3.5-5.0 mEq/L. Hypokalemia may predispose the client to digitalis toxicity. The other lab values are WNL.

The home care nurse visits a patient with chronic heart failure. Which clinical manifestations, assessed by the nurse, would indicate acute decompensated heart failure (pulmonary edema)? Fatigue, orthopnea, and dependent edema Severe dyspnea and blood-streaked, frothy sputum Temperature is 100.4oF and pulse is 102 beats/min Respirations 26 breaths/min despite oxygen by nasal cannula

Severe dyspnea and blood-streaked, frothy sputum Clinical manifestations of pulmonary edema include anxiety, pallor, cyanosis, clammy and cold skin, severe dyspnea, use of accessory muscles of respiration, a respiratory rate greater than 30 breaths/min, orthopnea, wheezing, and coughing with the production of frothy, blood-tinged sputum. Auscultation of the lungs may reveal crackles, wheezes, and rhonchi throughout the lungs. The heart rate is rapid, and blood pressure may be elevated or decreased.

The home care nurse visits a patient with chronic heart failure. Which clinical manifestations, assessed by the nurse, would indicate acute decompensated heart failure (pulmonary edema)? Fatigue, orthopnea, and dependent edema Severe dyspnea and blood-streaked, frothy sputum Temperature is 100.4oF and pulse is 102 beats/min Respirations 26 breaths/min despite oxygen by nasal cannula

Severe dyspnea and blood-streaked, frothy sputum (Khó thở nặng và có đốm máu, đờm bọt) Clinical manifestations of pulmonary edema include anxiety, pallor, cyanosis, clammy and cold skin, severe dyspnea, use of accessory muscles of respiration, a respiratory rate greater than 30 breaths/min, orthopnea, wheezing, and coughing with the production of frothy, blood-tinged sputum. Auscultation of the lungs may reveal crackles, wheezes, and rhonchi throughout the lungs. The heart rate is rapid, and blood pressure may be elevated or decreased.

Sally's height is 70 inches, and her weight is 145 lb. Her resting HR is 110 bpm and her SV is 35 mL/beat. Which of the following is NOT true about Sally? Her EF is probably low (but you can't calculate it with this information) She is bradycardic Her CO is low, which will result in a low BP and tissue hypoxia Her CO is 3.85 L/min

She is bradycardic

An increased serum lactate level (lactic acidosis) is commonly used as a marker of anaerobic metabolism.

TRUE

In systolic dysfunction, there is a decreased ejection fraction.

TRUE

Left-sided HF may lead to right-sided HF owing to increased right ventricular afterload.

TRUE

Patients with right-sided HF may present with jugular venous distention and peripheral edema.

TRUE

Severe HTN (which increases afterload) is a primary cause of left-sided heart failure.

TRUE

The patient with chronic heart failure is being discharged from the hospital. What information should the nurse emphasize in the patient's discharge teaching to prevent progression of the disease to acute decompensated heart failure (ADHF)? Take medications as prescribed. Use oxygen when feeling short of breath. Only ask the physician's office questions. Encourage most activity in the morning when rested.

Take medications as prescribed. The goal for the patient with chronic HF is to avoid exacerbations and hospitalization. Taking the medications as prescribed along with nondrug therapies such as alternating activity with rest will help the patient meet this goal. If the patient needs to use oxygen at home, it will probably be used all the time or with activity to prevent respiratory acidosis. Many HF patients are monitored by a care manager or in a transitional program to assess the patient for medication effectiveness and monitor for patient deterioration and encourage the patient. This nurse manager can be asked questions or can contact the health care provider if there is evidence of worsening HF.

The patient with chronic heart failure is being discharged from the hospital. What information should the nurse emphasize in the patient's discharge teaching to prevent progression of the disease to acute decompensated heart failure (ADHF)? Take medications as prescribed. Use oxygen when feeling short of breath. Only ask the physician's office questions. Encourage most activity in the morning when rested.

Take medications as prescribed. The goal for the patient with chronic HF is to avoid exacerbations and hospitalization. Taking the medications as prescribed along with nondrug therapies such as alternating activity with rest will help the patient meet this goal. If the patient needs to use oxygen at home, it will probably be used all the time or with activity to prevent respiratory acidosis. Many HF patients are monitored by a care manager or in a transitional program to assess the patient for medication effectiveness and monitor for patient deterioration and encourage the patient. This nurse manager can be asked questions or can contact the health care provider if there is evidence of worsening HF.

ANS: D A premature ventricular contraction (PVC) is a contraction originating in an ectopic focus in the ventricles. When every other beat is a PVC, the rhythm is called ventricular bigeminy. PVCs are usually a benign finding in the patient with a normal heart. In heart disease, PVCs may reduce the cardiac output and precipitate angina and heart failure, depending on the frequency. Because PVCs in coronary artery disease or acute MI indicate ventricular irritability, the patient's physiologic response to PVCs must be monitored. Assessment of the patient's hemodynamic status is important to determine whether treatment with drug therapy is needed. Reference: 830

The ECG monitor of a patient in the cardiac care unit after a myocardial infarction (MI) indicates ventricular bigeminy with a rate of 50 beats/minute. You anticipate A. performing defibrillation. B. treatment with IV lidocaine. C. insertion of a temporary, transvenous pacemaker. D. assessing the patient's response to the dysrhythmia.

ANS: A Failure to capture occurs when the electrical charge to the myocardium is insufficient to produce atrial or ventricular contraction. It can result in serious bradycardia, and treatment includes increasing the electrical charge. Failure to sense occurs when the pacemaker fails to recognize spontaneous atrial or ventricular activity and fires inappropriately. This can result in ventricular tachycardia. Reference: 836

The charge nurse is explaining the concept of pacemaker failure to capture to the new graduate. What information should the charge nurse give? A. It occurs when the electrical charge is insufficient. B. It occurs when the pacemaker does not recognize spontaneous heart activity. C. A complication is ventricular tachycardia. D. First-line treatment when this occurs is to turn down the electrical charge.

ANS: B A change in the level of consciousness should always have glucose and oxygen (and cardiac) assessed first. Hypoglycemia is a noncardiovascular cause that can be easily treated. It takes priority over the other assessments. Although those who have syncope have a 30% chance of recurrence, ruling out a simple treatable cause should be done first. Reference: 839

The female patient presents to the emergency department just after a syncope episode. What should you assess first? A. History of syncope B. Capillary glucose level C. Last menstrual period D. Allergies

Answer: B. This finding would be expected upon administration of adenosine. The rhythm should then begin again in some other rhythm, hopefully normal sinus rhythm. It would be important to document the exact time of this change and continue to monitor the change back to NSR. If this change does not occur, or if another rhythm is produced, appropriate action would then be taken based on the result.

The nurse has just administered adenosine via IV push and sees the following rhythm on the monitor. What is the nurse's priority intervention? a) Apply conductive gel and defibrillate the patient b) Document the findings and continue to monitor c) Administer another mg of the medication d) Begin chest compressions

A, E The client's edema is unlikely to be due to fluid retention if daily weights have been stable, so the nurse's interventions are aimed at promoting venous return to the heart by having the client elevate the legs and applying antiembolism stockings. While reviewing the diet and lab values is appropriate, it is unlikely the client is experiencing fluid retention if daily weights are not increasing. There is no need to increase the client's diuretic dosage.

The nurse in a long-term care facility is talking with the family of a client diagnosed with heart failure, diabetes, hypertension, and chronic renal failure. The nurse notes mild edema of the ankles while the client is sitting in the chair. Breath sounds are clear, equal, and with good chest excursion, and the client denies any feeling of shortness of breath. The nurse reviews the medical record and sees no significant change in the client's daily weights over the last week. What are the nurse's priority interventions for this client? (Select all that apply.) A) Encourage the client to elevate feet when sitting. B) Review the client's BUN and creatinine. C) Call the doctor for an order to increase the client's diuretic. D) Review the client's diet to determine sodium intake. E) Apply antiembolism stockings.

B Rationale: Stable angina is predictable and is associated with increased activity, and is relieved by rest and nitrates. ECG changes, nocturnal pain, and weak peripheral pulses are not findings associated with stable angina.

The nurse in the clinic assesses a client with stable angina. What expectations does the nurse have for this client? A) Increasing nocturnal pain B) Correlation between activity level and pain C) Weak peripheral pulses D) Persistent ECG changes

Answer: d) We cannot defibrillate asystole. A

The nurse in the intensive care unit (ICU) hears an alarm sound in the patient's room. Arriving in the room, the patient is unresponsive, without a pulse, and a flat line on the monitor. What is the first action by the nurse? a) Administer atropine 0.5 mg b) Administer epinephrine c) Defibrillate with 360 joules d) Begin cardiopulmonary resuscitation (CPR)

Answer: A, B, and D. Adenosine is administered as a very quick IV push. The physician must be present in the room and the crash cart must be on hand. An ekg monitor should be in the room to monitor the effectiveness of the medication.

The nurse is preparing to administer adenosine to the patient with the following rhythm which is symptomatic. What should the nurse plan on having in the patient room? a) Physician b) Crash cart c) IV pump d) EKG monitor e) Lidocaine

ANS: A In third-degree atrioventricular (AV) block, there is no correlation between the impulse from the atrium to the ventricles and the ventricular rhythm seen. A pacemaker eventually is required. Action must be taken because this usually results in reduced cardiac output with subsequent ischemia if untreated. Carotid massage is vagal stimulation, and it can cause bradycardia. There is a problem in conduction, not abnormal contraction, and defibrillation is not used. Reference: 830

The patient has a heart rate of 40 beats/minute. The P waves are regular, and the Q waves are regular, but there is no relationship between the P wave and QRS complex. What treatment do you anticipate? A. Pacemaker B. Continue to monitor C. Carotid massage D. Defibrillation

ANS: B These pacer spikes show that the pacemaker is firing and the atrium is responding to the impulse. It is a normal, expected finding in this situation. Reference: 834-835

The patient has a permanent cardiac pacemaker. On the electrocardiographic tracing, you notice a spike before each P wave. What action should you take? A. Assess the patient for syncope. B. Document the findings. C. Notify the physician. D. Take blood pressure in both arms.

ANS: C The rhythm described is sinus bradycardia. Treatment depends on the patient's response and whether adequate perfusion is occurring. If the patient tolerates the rhythm, no treatment is given. Reference: 824

The patient has an electrocardiographic (ECG) tracing that is 50 beats/minute, the rhythm is regular, and there is a P wave before every QRS complex. The QRS has a normal shape and duration, and the PR interval is normal. What is your response? A. Administer atropine by intravenous push (IVP). B. Administer epinephrine by IVP. C. Monitor the patient for syncope. D. Attach an external pacemaker.

ANS: D The chaotic atrial activity results in blood stasis that can lead to embolic events. Patients with chronic AF are given an anticoagulant, most often warfarin (Coumadin), to prevent the formation of emboli. There is no PR interval in AF because the P wave is absent, replaced by chaotic fibrillatory waves. Defibrillation is an elective procedure in chronic AF and is performed at lower levels of electricity. Pulse deficit is a higher-level skill and is not taught to the patient. Reference: 827

The patient has chronic atrial fibrillation (AF). What action do you anticipate? A. Monitoring the PR interval B. Defibrillation with 360 joule C. Teaching the patient to monitor the pulse deficit D. Teaching the patient to take an anticoagulant daily

Heart failure and circulatory shock are both conditions of circulatory system failure. Which of the following statements regarding these conditions is correct?

They have the same compensatory mechanisms.

Hydrochlorothiazide, metolozone

Thiazide and thiazide like duiretics

Reduce Afterload

Vasodilator (IV Nitroprusside) Nitroprusside - IV continuous - arterial vasodilator They HAVE to have a high BP to be on this Morphine Help with the breathlessness Helps with respiratory problems

While teaching a client with new-onset right-sided heart failure, the nurse should educate the client to monitor for fluid accumulation by:

Weighing every day at the same time with same type of clothing

A patient is diagnosed with right-sided heart failure. The nurse knows that a frequent sign of this type of failure is peripheral edema, evidenced by which of the following?

Weight gain

ANS: C The head-up tilt test is used to see whether there is cardioneurogenic syncope with increased venous pooling that occurs in the upright position. This reduces the venous return to the heart. Reference: 839

What is the purpose of the head-up tilt test? A. To determine whether a fluid volume deficit exists B. To assess for dysrhythmias when under stress C. To determine whether there is positional decreased venous return to the heart D. To evaluate for peripheral vascular disease

ANS: C Overdrive pacing involves pacing the atrium at a rate of 200 to 500 impulses per minute in an attempt to terminate atrial tachycardias (e.g., atrial flutter with a rapid ventricular response). A temporary pacemaker is a category of pacemakers used temporarily with the power source outside the body. Antitachycardia pacing delivers a stimulus to the ventricle to terminate tachydysrhythmias. Cardiac resynchronization therapy is a technique to resynchronize the cardiac cycle by pacing both ventricles. Reference: 835

What term is applied to a pacemaker that is implanted for the purpose of terminating atrial tachycardias? A. Temporary pacemaker B. Antitachycardia pacing C. Overdriving pacing D. Cardiac resynchronization therapy

B Left-sided failure results in backflow of blood from the pulmonary system resulting in pulmonary edema and shortness of breath. Edema of the feet and ankles and liver enlargement would be seen in clients with right-sided failure, due to backup of blood return from the body. Abdominal distention is not usually a symptom of heart failure.

When assessing a client diagnosed with left-sided heart failure, the nurse anticipates which finding? A) Abdominal distention B) Shortness of breath C) Liver enlargement D) Edema of the feet and ankles

ANS: C Because each small block on the ECG paper represents 0.04 second, 1500 of them represent 1 minute. By dividing the number of small blocks (15 in this case) into 1500, you can calculate the heart rate in a patient whose rhythm is regular (in this case, 100). Reference: 821

When computing a heart rate from the ECG tracing, you count 15 of the small blocks between the R waves of a patient whose rhythm is regular. From these data, you calculate the patient's heart rate to be what? A. 60 beats/minute B. 75 beats/minute C. 100 beats/minute D. 150 beats/minute

D) ("Generally, left heart failure causes you to be short of breath (breathlessness). This is generally worse when you exert yourself (for example, walking up hill) or when you are lying flat in bed")

When obtaining a health history of a patient admitted with a diagnosis of heart failure, which statement made by the patient supports the diagnosis of heart failure? A) "I often feel pain in my lower legs when I take my walk." B) "I sometimes feel pain in the middle of my chest during exercise." C) "I get hot and break out in a sweat during the night." D) "I get out of breath when I go up a flight of stairs

ANS: C VT is associated with a rate of 150 to 250 beats/minute, and the P wave is not normally visible. P-wave inversion and a normal QRS complex are not associated with VT. Rate and rhythm are not measurable in ventricular fibrillation. Reference: 831

Which ECG characteristics are consistent with a diagnosis of ventricular tachycardia (VT)? A. Unmeasurable rate and rhythm B. Rate of 150 beats/minute; inverted P wave C. Rate of 200 beats/minute; P wave not visible D. Rate of 125 beats/minute; normal QRS complex

ANS: C The electrical impulse in the heart must travel from the SA node through the AV node and into the Purkinje fibers for synchronous atrial and ventricular contraction to occur. The P wave represents atrial contraction, and the R wave is part of the QRS complex that represents ventricular contraction. When measuring the time from the beginning of the P wave to the beginning of the QRS (PR interval), you are identifying the length of time it takes for the electrical impulse to travel from the SA node to the Purkinje fibers. Reference: 818-819

Which statement best describes the electrical activity of the heart represented by measuring the PR interval on the ECG? A. The length of time it takes to depolarize the atrium B. The length of time it takes for the atria to depolarize and repolarize C. The length of time for the electrical impulse to travel from the sinoatrial (SA) node to the Purkinje fibers D. The length of time it takes for the electrical impulse to travel from the SA node to the AV node

ANS: B,C,D Air travel is not restricted. Inform airport security about the pacemaker because it may set off the metal detector. If a hand-held screening wand is used, it should not be placed directly over the pacemaker. Manufacturer information may vary regarding the effect of metal detectors on the function of the pacemaker. The patient should monitor the pulse and inform the primary care provider if it drops below predetermined rate. A Medic Alert ID or bracelet should be worn at all times. The patient should avoid lifting the arm on the pacemaker side above the shoulder until it is approved by the primary care provider. Microwave ovens are safe to use and do not interfere with pacemaker function. Table 36-13 provides additional discharge teaching guidelines for a patient with a pacemaker. Reference: 837

Which teaching points should you include when providing discharge instructions to a patient with a new permanent pacemaker and to the caregiver (select all that apply)? A. Avoid or limit air travel. B. Take and record a pulse rate daily. C. Obtain and wear a Medic Alert ID or bracelet at all times. D. Avoid lifting the arm on the side of the pacemaker above the shoulder. E. Avoid microwave ovens because they interfere with pacemaker function.

The nurse is preparing to administer digoxin to a patient with heart failure. In preparation, laboratory results are reviewed with the following findings: sodium 139 mEq/L, potassium 5.6 mEq/L, chloride 103 mEq/L, and glucose 106 mg/dL. What is the priority action by the nurse? Withhold the daily dose until the following day. Withhold the dose and report the potassium level. Give the digoxin with a salty snack, such as crackers. Give the digoxin with extra fluids to dilute the sodium level.

Withhold the dose and report the potassium level. The normal potassium level is 3.5 to 5.0 mEq/L. The patient is hyperkalemic, which makes the patient more prone to digoxin toxicity. For this reason, the nurse should withhold the dose and wait for the potassium level to normalize. The physician may order the digoxin to be given once the potassium level has been treated and decreases to within normal range.

The nurse is preparing to administer digoxin to a patient with heart failure. In preparation, laboratory results are reviewed with the following findings: sodium 139 mEq/L, potassium 5.6 mEq/L, chloride 103 mEq/L, and glucose 106 mg/dL. What is the priority action by the nurse? Withhold the daily dose until the following day. Withhold the dose and report the potassium level. Give the digoxin with a salty snack, such as crackers. Give the digoxin with extra fluids to dilute the sodium level.

Withhold the dose and report the potassium level. The normal potassium level is 3.5 to 5.0 mEq/L. The patient is hyperkalemic, which makes the patient more prone to digoxin toxicity. For this reason, the nurse should withhold the dose and wait for the potassium level to normalize. The physician may order the digoxin to be given once the potassium level has been treated and decreases to within normal range.

ANS: C After pacemaker insertion, it is important for the patient to limit activity of the involved arm to minimize pacemaker lead displacement. The nonpressure dressing is kept dry until removed, usually 24 hours postoperatively. It is important for you to observe signs of infection by assessing for redness, swelling, or discharge from the incision site. Reference: 836-837

You are caring for a patient 24 hours after pacemaker insertion. Which nursing intervention is most appropriate at this time? A. Reinforcing the pressure dressing as needed B. Encouraging range-of-motion exercises of the involved arm C. Assessing the incision for any redness, swelling, or discharge D. Applying wet-to-dry dressings every 4 hours to the insertion site

ANS: C The typical ECG changes seen in myocardial ischemia include ST-segment depression or T-wave inversion, or both. Reference: 837

You are monitoring the electrocardiogram of a patient admitted with ACS. Which ECG characteristics most suggest ischemia? A. Sinus rhythm with a pathologic Q wave B. Sinus rhythm with an elevated ST segment C. Sinus rhythm with a depressed ST segment D. Sinus rhythm with premature atrial contractions

Answer: D. A pulse would be assessed for first. Then the patient will be defibrillized and chest compressions will begin immediately.

You are the nurse working on the telemetry unit and you have finally gotten to sit down to work on some charting. Suddenly the heart monitors at the station start beeping. Patient in room 18 is showing this rhythm on the monitor. The medical team advances together into the room and finds them unconscious. What is priority action by the nurse? A) Prepare to administer adenosine to the patient B) Begin chest compressions C) Prepare for defibrillation D) Check for a pulse

ANS: B Atrial fibrillation is represented on the cardiac monitor by irregular R-R intervals and small fibrillatory (F) waves. There are no normal P waves because the atria are not contracting, just fibrillating. Reference: 827

You are watching the cardiac monitor, and a patient's rhythm suddenly changes. There are no P waves. Instead, there are fine, wavy lines between the QRS complexes. The QRS complexes each measure 0.08 second (narrow), but they occur irregularly with a rate of 120 beats/minute. You correctly interpret that this rhythm is A. sinus tachycardia. B. atrial fibrillation. C. ventricular fibrillation. D. ventricular tachycardia.

ANS: B Third-degree heart block represents a loss of communication between the atrium and ventricles. This is depicted on the rhythm strip because there is no relationship between the P waves, representing atrial contraction, and QRS complexes, representing ventricular contraction. The atria are beating totally on their own at 70 beats/minute, whereas the ventricles are pacing themselves at 40 beats/minute. Reference: 830

You obtain a 6-second rhythm strip, and document the following analysis: atrial rate of 70 beats/minute, regular; ventricular rate of 40 beats/minute, regular; QRS of 0.04 second; no relationship between P waves and QRS complexes; and atria and ventricles beating independently of each other. What is the correct interpretation of this rhythm strip? A. Sinus dysrhythmias B. Third-degree heart block C. Wenckebach phenomenon D. Premature ventricular contractions

ANS:D Synchronized cardioversion is the therapy of choice for the patient with hemodynamically unstable ventricular or supraventricular tachydysrhythmias. A synchronized circuit in the defibrillator delivers a countershock that is programmed to occur on the R wave of the QRS complex seen on the ECG tracing. The synchronizer switch must be turned on when cardioversion is planned. The procedure for synchronized cardioversion is the same as for defibrillation, with the following exceptions. If synchronized cardioversion is done on a nonemergent basis, the patient is sedated before the procedure. The initial energy needed for synchronized cardioversion is less than the energy needed for defibrillation. Reference: 833

You prepare a patient for synchronized cardioversion knowing that cardioversion differs from defibrillation in that A. defibrillation requires lower dose of electrical energy. B. cardioversion is indicated for treatment of atrial bradydysrhythmias. C. defibrillation is synchronized to deliver a shock during the QRS complex. D. patients should be sedated if cardioversion is done on a nonemergent basis.

Answer: B. With SVT (Supraventricular tachycardia), the first thing to do would be to instruct the pt. to perform the Valsalva maneuver by bearing down.

You, the nurse, note the following rhythm on the EKG monitor for a patient named billy. What is the first thing the nurse should do? a) Check for a pulse b) Tell Billy to try to poop c) Prepare to defibrillate billy d) Prepare to administer Amiodorone

A patient is found pulseless and the cardiac monitor shows a rhythm that has no recognizable deflections, but instead has coarse "waves" of varying amplitudes. What is the priority ACLS intervention for this rhythm? a. Immediate defibrillation b. Administration of epinephrine IVP c. Administration of lidocaine IVP d. Noninvasive temporary pacing

a

A patient with AF suddenly develops shortness of breath, chest pain, hemoptysis, and a feeling of impending doom. The nurse recognizes these symptoms as which complication? a. Pulmonary embolism b. Embolic stroke c. Absence of atrial kick d. Increased cardiac output

a

A patient with atrial fibrillation is scheduled to have an elective cardioversion. The nurse ensures that the patient has a prescription for 4-6 week supply of which ytpe of medication? a. Anticoagulants b. Digitalis c. Diuretics d. Potassium supplements

a

Assessment of a patient's peripheral IV site reveals that phlebitis has developed over the past several hours. Which intervention should the nurse implement first? a. Remove the patient's IV catheter. b. Apply an ice pack to the affected area. c. Decrease the IV rate to 20 to 30 mL/hr. d. Administer prophylactic anticoagulants.

a

Based on the prevalence and risk factors for atrial fibrillation (AF), which patient group is at highest risk for AF? a. Older adults b. Diabetics c. Substance abusers d. Pediatric cardiology patients

a

In a patient's record, the nurse notes frequent episodes of bradycardia and hypotension related to unintended vagal stimulation. Which instruction for this patient's care does the nurse relay to the unlicensed assistive personnel (UAP)? a. Avoid raising the patient's arms above the head during hygiene. b. Ambulate the patient slowly and stop frequently for brief rests c. Generously lubricate rectal thermometer probes and insert very cautiously d. Monitor the heart rate and rhythm if the patient is vomiting

a

The nurse is interviewing a patient who suddenly becomes faint, immediately loses consciousness, and becomes pulseless and apneic. There is no blood pressure, and heart sounds are absent. What does the nurse do next? a. Begin compressions. b. Defibrillate the patient c. Establish or ensure IV access d. Give supplemental oxygen

a

The nurse is performing a 12-lead ECG on a patient with chest pain. Because the positioning of the electrodes is crucial, how does the nurse place the ECG components? a. Four leads are placed on the limbs and six are places on the chest n. The negative electrode is placed on the left arm and the positive electrode is placed on the right leg c. Four leads are placed on the limbs and four are placed on the chest d. The negative electrode is placed on the right arm and the positive electrode is placed on the left leg

a

The nurse is providing care for a patient who has decreased cardiac output related to heart failure. What should the nurse recognize about cardiac output? a. Calculated by multiplying the patient's stroke volume by the heart rate b. The average amount of blood ejected during one complete cardiac cycle c. Determined by measuring the electrical activity of the heart and the patient's heart rate d. The patient's average resting heart rate multiplied by the patient's mean arterial blood pressure

a

The nurse is reviewing preliminary ECG results of a patient admitted for mental status changes. The nurse alerts the health care provider about ST elevation or depression in the patient because it is an indication of which condition? a. Myocardial injury or ischemia b. Ventricular irritability c. Subarachnoid hemorrhage d. Prinzmetal's angina

a

The nurse teaches the patient with any venous disorder that the best way to prevent venous stasis and increase venous return is to a. walk. b. sit with the legs elevated. c. frequently rotate the ankles. d. continuously wear elastic compression stockings.

a

What is the QRS complex in an ECG normally? a Less than 0.12 second b. 0.10 to 0.16 second c. 0.12 to 0.20 second d. 0.16 to 0.24 second

a

What is the ST segment in an ECG normally? a. Isoelectric b. Elevated c. Depressed d. Biphasic

a

When the patient is being examined for venous thromboembolism (VTE) in the calf, what diagnostic test should the nurse expect to teach the patient about first? a. Duplex ultrasound b. Contrast venography c. Magnetic resonance venography Incorrect d. Computed tomography venography

a

With the speed set for 25 mm/second, the segment between the dark lines on a monitor ECG strip represents how many seconds? a 3 b. 6 c. 10 d. 20

a

The patient has chronic venous insufficiency and a venous ulcer. The unlicensed assistive personnel (UAP) decides to apply compression stockings because that is what patients 'always' have ordered. Which assessment finding would indicate the application of compression stockings could harm the patient? a. Rest pain Correct High blood pressureThe patient has chronic venous insufficiency and a venous ulcer. The unlicensed assistive personnel (UAP) decides to apply compression stockings because that is what patients 'always' have ordered. Which assessment finding would indicate the application of compression stockings could harm the patient? a. Rest pain b. High blood pressure c. Elevated blood sugar d. Dry, itchy, flaky skin

a Rest pain occurs as peripheral artery disease (PAD) progresses and involves multiple arterial segments. Compression stockings should not be used on patients with PAD.

A patient with chronic HF and atrial fibrillation is treated with a digitalis glycoside and a loop diuretic. To prevent possible complications of this combination of drugs, what does the nurse need to do (select all that apply)? a. Monitor serum potassium levels b. teach the patient how to take a pulse rate. c. keep an accurate measure of intake and output d. Teach the patient about dietary restriction or potassium e. Withhold digitalis and notify health care provider if heart rate is irregular

a & b Rationale: Hypokalemia, which can be caused by the use of potassium-depleting diuretics (e.g., thiazides, loop diuretics), is one of the most common causes of digitalis toxicity. Low serum levels of potassium enhance the action of digitalis, causing a therapeutic dose to achieve toxic levels. Hypokalemia can also precipitate dysrhythmias. Monitoring the serum potassium levels of patients receiving digitalis preparations and potassium-depleting diuretics is essential. Patients taking digitalis preparations should be taught how to measure their pulse rate because bradycardia and atrioventricular blocks are late signs of digitalis toxicity. In addition, patients should know what pulse rate would necessitate a call to the health care provider.

Despite a high dosage, a male patient who is taking nifedipine (Procardia XL) for antihypertensive therapy continues to have blood pressures over 140/90 mmHg. What should the nurse do next? a) Assess his adherence to therapy. b) Ask him to make an exercise plan. c) Instruct him to use the DASH diet. d) Request a prescription for a thiazide diuretic.

a) Assess his adherence to therapy. A long-acting calcium-channel blocker such as nifedipine causes vascular smooth muscle relaxation resulting in decreased SVR and arterial BP and related side effects. The patient data the nurse has about this patient is very limited, so the nurse needs to assess his adherence to therapy.

The nurse teaches a patient with hypertension that uncontrolled hypertension may damage organs in the body primarily by which mechanism? a) Hypertension promotes atherosclerosis and damage to the walls of the arteries. b) Hypertension causes direct pressure on organs, resulting in necrosis and replacement of cells with scar tissue. c) Hypertension causes thickening of the capillary membranes, leading to hypoxia of organ systems. d) Hypertension increases blood viscosity, which contributes to intravascular coagulation and tissue necrosis distal to occlusions.

a) Hypertension promotes atherosclerosis and damage to the walls of the arteries. Hypertension is a major risk factor for the development of atherosclerosis by mechanisms not yet fully known. However, once atherosclerosis develops, it damages the walls of arteries and reduces circulation to target organs and tissues.

The nurse is caring for a patient with hypertension who is scheduled to receive a dose of esmolol (Brevibloc). The nurse should withhold the dose and consult the prescribing physician for which vital sign taken just before administration? a) Pulse 48 b) Respirations 24 c) Blood pressure 118/74 d) Oxygen saturation 93%

a) Pulse 48 Because esmolol is a β1-adrenergic blocking agent, it can cause hypotension and bradycardia as adverse effects. The nurse should withhold the dose and consult with the health care provider for parameters regarding pulse rate limits.

A patient admitted with heart failure appears very anxious and complains of shortness of breath. Which nursing actions would be appropriate to alleviate this patient's anxiety (select all that apply)? a. Administer ordered morphine sulfate. b. Position patient in a semi-Fowler's position. c. Position patient on left side with head of bed flat. d. Instruct patient on the use of relaxation techniques. e. Use a calm, reassuring approach while talking to patient.

a, b, d, & e Morphine sulfate reduces anxiety and may assist in reducing dyspnea. The patient should be positioned in semi-Fowler's position to improve ventilation that will reduce anxiety. Relaxation techniques and a calm reassuring approach will also serve to reduce anxiety.

Which statements accurately describe heart failure (HF) (select all that apply)? a. A common cause of diastolic failure is left ventricular dysfunction. b. A primary risk factor for heart failure is coronary artery disease (CAD). c. Systolic heart failure results in a normal left ventricular ejection fraction. d. Systolic failure is characterized by abnormal resistance to ventricular filling. e. Hypervolemia precipitates heart failure by decreasing cardiac output and increasing oxygen consumption.

a, b. Diastolic failure is characterized by abnormal resistance to ventricular filling. Hypertension, coronary artery disease (CAD), advanced age, and diabetes are all risk factors for heart failure (HF). Ejection fraction (EF) is decreased in systolic HF. Decreased cardiac output (CO) and increased workload and oxygen requirements of the myocardium precipitate HF due to left ventricle dysfunction.

When caring for elderly patients with hypertension, which information should the nurse consider when planning care (select all that apply.)? a. Systolic blood pressure increases with aging. b. Blood pressures should be maintained near 120/80 mm Hg. c. White coat syndrome is prevalent in elderly patients. d. Volume depletion contributes to orthostatic hypotension. e. Blood pressure drops 1 hour postprandially in many older patients. f. Older patients will require higher doses of antihypertensive medications.

a, c, d, e

A patient is admitted to the hospital in hypertensive emergency (BP 244/142 mm Hg). Sodium nitroprusside is started to treat the elevated BP. Which management strategy(ies) would be most appropriate for this patient (select all that apply)? a. Measuring hourly urine output b. Decreasing the MAP by 50% within the first hour c. Continuous BP monitoring with an arterial line d. Maintaining bed rest and providing tranquilizers to lower the BP e. Assessing the patient for signs and symptoms of heart failure and changes in mental status

a, c, e

Which BP-regulating mechanism(s) can result in the development of hypertension if defective (select all that apply)? a. Release of norepinephrine b. Secretion of prostaglandins c. Stimulation of the sympathetic nervous system d. Stimulation of the parasympathetic nervous system e. Activation of the renin-angiotensin-aldosterone system

a, c, e

Which BP-regulating mechanism(s) can result in the development of hypertension if defective (select all that apply)? a. Release of norepinephrine b. Secretion of prostaglandins c. Stimulation of the sympathetic nervous system d. Stimulation of the parasympathetic nervous system e. Activation of the renin-angiotensin-aldosterone system

a, c, e

The patient has heart failure (HF) with an ejection fraction of less than 40%. What core measures should the nurse expect to include in the plan of care for this patient (select all that apply)? a. Left ventricular function is documented. b. Controlling dysrhythmias will eliminate HF. c. Prescription for digoxin (Lanoxin) at discharge d. Prescription for angiotensin-converting enzyme (ACE) inhibitor at discharge e. Education materials about activity, medications, weight monitoring, and what to do if symptoms worsen

a, d, & e The Joint Commission has identified these three core measures for heart failure patients. Although controlling dysrhythmias will improve CO and workload, it will not eliminate HF. Prescribing digoxin for all HF patients is no longer done because there are newer effective drugs and digoxin toxicity occurs easily related to electrolyte levels and the therapeutic range must be maintained.

A patient is admitted to the hospital in hypertensive emergency (BP 244/142 mmHg). Sodium nitroprusside is started to treat the elevated BP. Which management strategy(ies) would be appropriate for this patient (select all that apply)? a. Measuring hourly urine output b. Decreasing the MAP by 50% within the first hour c. Continuous BP monitoring with an intraarterial line d. Maintaining bed rest and providing tranquilizers to lower the BP e. Assessing the patient for signs and symptoms of heart failure and changes in mental status

a,c, & e

What are nonmodifiable risk factors for primary hypertension (select all that apply)? a. Age b. Obesity c. Gender d. Genetic link e. Ethnicity

a,c,d,e Hypertension progresses with increasing age. It is more prevalent in men up to age 45 and above the age of 64 in women. African Americans have a higher incidence of hypertension than do white Americans. Children and siblings of patients with hypertension should be screened and taught about healthy lifestyles.

A 78-year-old patient is admitted with a BP of 180/98 mm Hg. Which age-related physical changes may contribute to this patient's hypertension (select all that apply)? a. Decreased renal function b. Increased adrenergic receptor sensitivity c. Increased baroreceptor reflexes d. Increased collagen and stiffness of the myocardium e. Increased peripheral vascular resistance f. Loss of elasticity in large arteries from arteriosclerosis

a,d,e,f The age-related changes that contribute to hypertension include decreased renal function, increased peripheral vascular resistance, increased collagen and stiffness of the myocardium, and decreased elasticity in large arteries from arteriosclerosis.

The nurse teaches a patient with hypertension that uncontrolled hypertension may damage organs in the body primarily by which mechanism? a. Hypertension promotes atherosclerosis and damage to the walls of the arteries. b. Hypertension causes direct pressure on organs, resulting in necrosis and replacement of cells with scar tissue. c. Hypertension causes thickening of the capillary membranes, leading to hypoxia of organ systems. d. Hypertension increases blood viscosity, which contributes to intravascular coagulation and tissue necrosis distal to occlusions.

a.

The patient with chronic heart failure is being discharged with a diuretic, a renin-angiotensin-aldosterone system (RAAS) inhibitor, and a β-adrenergic blocker. When received from the pharmacy, which medication should not be included for this patient? a. Dopamine b. Losartan (Cozaar) c. Carvedilol (Coreg) d. Hydrochlorothiazide

a. Dopamine is a β-adrenergic agonist that is a positive inotrope given IV, not orally, and used for acute HF. Losartan (Cozaar) is an angiotensin II receptor blocker used for patients who do not tolerate angiotensin- converting enzyme (ACE) inhibitors. Carvedilol (Coreg) is the β-adrenergic blocker that blocks the sympathetic nervous system's negative effects on the failing heart. Hydrochlorothiazide is the diuretic.

Which initial physical assessment finding would the nurse expect to be present in a patient with acute left-sided heart failure? a. Bubbling crackles and tachycardia. b. Hepatosplenomegaly and tachypnea. c. Peripheral edema and cool, diaphoretic skin. d. Frothy, blood-tinged sputum and distended jugular veins.

a. Early clinical manifestations of acute left-sided HF are those of interstitial edema, with bubbling crackles and tachycardia, as well as tachypnea. Later frothy, blood- tinged sputum; severe dyspnea; and orthopnea develop with alveolar edema. Severe tachycardia and cool, clammy skin are present as a result of stimulation of the SNS from hypoxemia. Systemic edema reflected by jugular vein distention, peripheral edema, and hepatosplenomegaly are characteristic of right-sided heart failure.

What is the patient with primary hypertension likely to report? a. No symptoms b. Dyspnea on exertion c. Cardiac palpitations d. Dizziness and vertigo

a. Hypertension is often asymptomatic, especially if it is mild or moderate, and has been called the "silent killer."

The nurse plans long-term goals for the patient who has had a heart transplant with the knowledge that what is the most common cause of death in heart transplant patients during the first year? a. Infection b. Heart failure c. Embolization d. Malignant conditions

a. In the first year after transplant, with the need for long-term immunosuppressant therapy to prevent rejection, the patient with a transplant is at high risk for infection, a leading cause of death in transplant patients. Acute rejection episodes may also cause death in patients with transplants, but many can be treated successfully with augmented immunosuppressive therapy. Malignancies occur in patients with organ transplants after taking immunosuppressants for a number of years.

The health care provider prescribes spironolactone (Aldactone) for the patient with chronic heart failure. What diet modifications related to the use of this drug should the nurse include in the patient teaching? a. Decrease both sodium and potassium intake. b. Increase calcium intake and decrease sodium intake. c. Decrease sodium intake and increase potassium intake. d. Decrease sodium intake by using salt substitutes for seasoning.

a. Spironolactone is a potassium-sparing diuretic, and when it is the only diuretic used in the treatment of heart failure, moderate to low levels of potassium intake should be maintained to prevent development of hyperkalemia. Sodium intake is usually reduced to at least 2400 mg/day in patients with heart failure, but salt substitutes cannot be freely used because many contain high concentrations of potassium. Calcium intake is not increased.

10] A 60-year-old male client comes into the emergency department with a complaint of crushing substernal chest pain that radiates to his shoulder and left arm. The admitting diagnosis is acute myocardial infarction (MI). Immediate admission orders include oxygen by nasal cannula at 4L/minute, blood work, a chest radiograph, a 12-lead electrocardiogram (ECG), and 2mg of morphine sulfate given IV. The nurse should first: a. Administer the morphine b. Obtain a 12-lead ECG c. Obtain the blood work d. Order the chest radiograph

a. Although obtaining the ECG, chest radiograph, and blood work are all important, the nurse's priority action should be to relieve the crushing chest pain. Therefore, administering morphine sulfate is the priority action.

The patient is in chronic junctional escape rhythm with no atrial activity noted. Studies have demonstrated normal AV node function. This patient may be a candidate for which type of pacing? a. Atrial pacing b. Ventricular pacing c. Dual-chamber pacing d. Transcutaneous pacing

a. Atrial pacing

Sinus bradycardia is a symptom of which of the following? (Select all that apply.) a. Calcium channel blocker medication b. Beta-blocker medication c. Athletic conditioning d. Hypothermia e. Hyperthyroidism

a. Calcium channel blocker medication b. Beta-blocker medication c. Athletic conditioning d. Hypothermia

28. A 46-year-old is diagnosed with thromboangiitis obliterans (Buergers disease). When the nurse is developing a discharge teaching plan for the patient, which outcome has the highest priority for this patient? a. Cessation of all tobacco use b. Control of serum lipid levels c. Maintenance of appropriate weight d. Demonstration of meticulous foot care

a. Cessation of all tobacco use

A 42-year-old man has been diagnosed with primary hypertension with an average BP of 162/92 mm Hg on three consecutive clinic visits. What are four priority lifestyle modifications that should be explored in the initial treatment of the patient? a. b. c. d.

a. Dietary modifications to restrict sodium, cholesterol, and saturated fat; maintain intake of potassium, calcium, and magnesium; and promote weight reduction if overweight b. Daily moderate-intensity physical activity for at least 30 minutes on most days of the week c. Cessation of smoking (if a smoker) d. Moderation or cessation of alcohol intake; usually medications and monitor BP at home. Also, psychosocial risk factors must be addressed.

Upon admission assessment, the nurse notes clubbing of the patient's fingers. Based on this finding, the nurse will question the patient about which of the following disease processes? a. Endocarditis b. Acute renal failure c. Myocardial infarction d. Chronic thrombophlebitis

a. Endocarditis Rationale: Clubbing of the fingers is a loss of the normal angle between the base of the nail and the skin. This finding can be found in endocarditis, congenital defects, and/or prolonged oxygen deficiency.

Which of the following are common causes of sinus tachycardia? (Select all that apply.) a. Hyperthyroidism b. Hypovolemia c. Hypothyroidism d. Heart Failure e. Sleep

a. Hyperthyroidism b. Hypovolemia d. Heart Failure

16. A 67-year-old patient is admitted to the hospital with a diagnosis of venous insufficiency. Which patient statement is most supportive of the diagnosis? a. I cant get my shoes on at the end of the day. b. I cant seem to ever get my feet warm enough. c. I have burning leg pains after I walk two blocks. d. I wake up during the night because my legs hurt.

a. I cant get my shoes on at the end of the day.

A patient with newly discovered high BP has an average reading of 158/98 mmHg after 3 months of exercise and diet modifications. Which management strategy will be a priority for this patient? a. Medication will be required because the BP is still not at goal b. BP monitoring should continue for another 3 months to confirm a diagnosis of hypertension c. Lifestyle changes are less important, since they were not effective, and medications will be started d. More vigorous changes in the patient's lifestyle are needed for a longer time before starting medications

a. Medication will be required because the BP is still not at goal

Because of the location of the AV node, the possible P waveforms that are associated with junctional rhythms include which of the following? (Select all that apply.) a. No P wave b. Inverted P wave c. Shortened PR interval d. P wave after the QRS complex e. Normal P wave and PR interval

a. No P wave b. Inverted P wave c. Shortened PR interval d. P wave after the QRS complex

24. The nurse is caring for a patient immediately after repair of an abdominal aortic aneurysm. On assessment, the patient has absent popliteal, posterior tibial, and dorsalis pedis pulses. The legs are cool and mottled. Which action should the nurse take first? a. Notify the surgeon and anesthesiologist. b. Wrap both the legs in a warming blanket. c. Document the findings and recheck in 15 minutes. d. Compare findings to the preoperative assessment of the pulses.

a. Notify the surgeon and anesthesiologist.

22. An 80-year-old patient with a history of an abdominal aortic aneurysm arrives at the emergency department (ED) with severe back pain and absent pedal pulses. Which actions should the nurse take first? a. Obtain the blood pressure. b. Obtain blood for laboratory testing. c. Assess for the presence of an abdominal bruit. d. Determine any family history of kidney disease.

a. Obtain the blood pressure.

While admitting a patient with pericarditis, the nurse will assess for which of the following signs, symptoms, and complications of this disorder? a. Pulsus paradoxus b. Prolonged PR intervals c. Widened pulse pressure d. Clubbing of the fingers

a. Pulsus paradoxus Rationale: Pericarditis can lead to cardiac tamponade, an emergency situation. Pulsus paradoxus >10 mm Hg is a sign of cardiac tamponade that should be assessed at least every 4 hours in a patient with pericarditis.

Assessment of an IV cocaine user with infective endocarditis should focus on which signs and symptoms (select all that apply) a. Retinal hemorrhages b. splinter hemorrhages c. presence of Osler's nodes d. Painless nodules over bony prominences e. painless erythematous macules on the palms and soles

a. Retinal hemorrhages b. splinter hemorrhages c. presence of Osler's nodes e. painless erythematous macules on the palms and soles Rationale: Clinical manifestations of infective endocarditis may include hemorrhagic retinal lesions (Roth's spots), splinter hemorrhages (black, longitudinal streaks) that may occur in the nail beds, Osler's nodes (painful, tender, red or purple, pea-size lesions) on the fingertips or toes, and Janeway's lesions (flat, painless, small, red spots) on the palms and soles.

Priority Decision: The nurse reviews the following vital signs recorded by an unlicensed assistive personnel (UAP) on a patient with acute decompensated heart failure (ADHF): BP 98/60, HR 102 bpm, RR 24, temp 98.2° F (36.7° C), SpO2 84% on 2 L/min via nasal cannula. a. Which of these findings is of highest priority? b. What should the nurse do next?

a. SpO2 of 84% on 2 L/min via nasal cannula indicates impaired oxygen saturation. The patient is having trouble with gas exchange. Airway and breathing are the priority (follow ABCs). b. The nurse should place the patient in high Fowler's position, assess the patient immediately, recheck SpO2, auscultate breath sounds, assess level of consciousness (LOC), check the oxygen connection and rate setting (2 L/min), and talk with the patient about her or his breathing.

4. A patient in the outpatient clinic has a new diagnosis of peripheral artery disease (PAD). Which group of medications will the nurse plan to include when providing patient teaching about PAD management? a. Statins b. Antibiotics c. Thrombolytics d. Anticoagulants

a. Statins

The nurse is caring for a patient who has atrial fibrillation. Sequelae that place the patient at greater risk for mortality/morbidity include which of the following? (Select all that apply.) a. Stroke b. Ashman beats c. Pulmonary emboli d. Prolonged PR interval e. Decreased cardiac output

a. Stroke c. Pulmonary emboli e. Decreased cardiac output

The patient with chronic heart failure is being discharged from the hospital. What information should the nurse emphasize in the patient's discharge teaching to prevent progression of the disease to ADHF? a. Take medications as prescribed. b. Use oxygen when feeling short of breath. c. Only ask the physician's office questions. d. Encourage most activity in the morning when rested.

a. Take medications as prescribed. The goal for the patient with chronic HF is to avoid exacerbations and hospitalization. Taking the medications as prescribed along with nondrug therapies such as alternating activity with rest will help the patient meet this goal. If the patient needs to use oxygen at home, it will probably be used all the time or with activity to prevent respiratory acidosis. Many HF patients are monitored by a care manager or in a transitional program to assess the patient for medication effectiveness and monitor for patient deterioration and encourage the patient. This nurse manager can be asked questions or can contact the health care provider if there is evidence of worsening HF.

27. A patient who is 2 days post-femoral-popliteal bypass graft to the right leg is being cared for on the vascular unit. Which action by a licensed practical/vocational nurse (LPN/LVN) caring for the patient requires the registered nurse (RN) to intervene? a. The LPN/LVN has the patient sit in a chair for 90 minutes. b. The LPN/LVN assists the patient to walk 40 feet in the hallway. c. The LPN/LVN gives the ordered aspirin 160 mg after breakfast. d. The LPN/LVN places the patient in a Fowlers position for meals.

a. The LPN/LVN has the patient sit in a chair for 90 minutes.

Which of the following is true about a patient diagnosed with sinus arrhythmia? a. The heart rate varies, dependent on vagal tone and respiratory pattern. b. Immediate treatment is essential to prevent death. c. Sinus arrhythmia is not well tolerated by most patients. d. PR and QRS interval measurements are prolonged.

a. The heart rate varies, dependent on vagal tone and respiratory pattern.

9. After teaching a patient with newly diagnosed Raynauds phenomenon about how to manage the condition, which action by the patient demonstrates that the teaching has been effective? a. The patient exercises indoors during the winter months. b. The patient places the hands in hot water when they turn pale. c. The patient takes pseudoephedrine (Sudafed) for cold symptoms. d. The patient avoids taking nonsteroidal antiinflammatory drugs (NSAIDs).

a. The patient exercises indoors during the winter months.

A patient is admitted with myocarditis. While performing the initial assessment, which clinical signs and symptoms might the nurse find (select all that apply)? a. angina b. pleuritic chest pain c. splinter hemorrhages d. pericardial friction rub e. presence of Osler's nodes

a. angina b. pleuritic chest pain d. pericardial friction rub Rationale: Clinical manifestations of myocarditis may include early systemic manifestations (i.e., fever, fatigue, malaise, myalgias, pharyngitis, dyspnea, lymphadenopathy, and nausea and vomiting), early cardiac manifestations (i.e., pleuritic chest pain with a pericardial friction rub and effusion), and late cardiac signs (i.e., S3 heart sound, crackles, jugular venous distention [JVD], syncope, peripheral edema, and angina).

The nurse is reading the cardiac monitor and notes that the patient's heart rhythm is extremely irregular and there are no discernible P waves. The ventricular rate is 90 beats per minute, and the patient is hemodynamically stable. The nurse realizes that the patient's rhythm is: a. atrial fibrillation. b. atrial flutter. c. atrial flutter with rapid ventricular response. d. junctional escape rhythm.

a. atrial fibrillation.

When performing discharge teaching for the patient with any type of cardiomyopathy, the nurse instructs the patient to (select all that apply) a. eat a low-sodium diet b. suggest that caregivers learn CPR c. engage in stress reduction activities d. abstain from alcohol and caffeine intake e. avoid strenuous activity and allow for periods of rest

a. eat a low-sodium diet b. suggest that caregivers learn CPR c. engage in stress reduction activities d. abstain from alcohol and caffeine intake e. avoid strenuous activity and allow for periods of rest Rationale: These are all points that can apply to any cardiomyopathy.

Which of the following diagnostic study best differentiates the various types of cardiomyopathy? a. echocardiography b. arterial blood gases c. cardiac catheterization d. endomyocardial biopsy

a. echocardiography Rationale: The echocardiogram is the primary diagnostic tool used to differentiate between the different types of cardiomyopathies and other structural cardiac abnormalities.

The patient has an irregular heart rhythm. To determine an accurate heart rate, the nurse first: a. identifies the markers on the ECG paper that indicate a 6-second strip. b. counts the number of large boxes between two consecutive P waves. c. counts the number of small boxes between two consecutive QRS complexes. d. divides the number of complexes in a 6-second strip by 10.

a. identifies the markers on the ECG paper that indicate a 6-second strip.

The patient's heart rhythm shows an inverted P wave with a PR interval of 0.06 seconds. The heart rate is 54 beats per minute. The nurse recognizes the rhythm as a junctional escape rhythm, and understands that the rhythm is due to the: a. loss of sinus node activity. b. increased rate of the AV node. c. increased rate of the SA node. d. decreased rate of the AV node.

a. loss of sinus node activity.

A patient is diagnosed with mitral stenosis and new-onset atrial fibrillation. Which interventions could the nurse delegate to nursing assistant personnel (NAP) (select all that apply)? a. obtain and record daily weight b. determine apical-radial pulse rate c. observe for overt signs of bleeding d. obtain and record vital signs, including pulse oximetry e. teach the patient how to purchase a Medic Alert bracelet

a. obtain and record daily weight c. observe for overt signs of bleeding d. obtain and record vital signs, including pulse oximetry Rationale: The nurse may delegate routine procedures such as obtaining weights and vital signs. The nurse may give specific directions to the nursing assistive personnel (NAP) to observe and report obvious signs of bleeding. The nurse cannot delegate teaching, assessment, or activities that require clinical judgment. Obtaining an apical-radial pulse rate is an assessment.

A 20-year old patient has acute infective endocarditis. While obtaining a nursing history, the nurse should ask the patient about which of the following (select all that apply) a. renal dialysis b. IV drug abuse c. recent dental work d. cardiac catheterization e. recent urinary tract infection

a. renal dialysis b. IV drug abuse c. recent dental work d. cardiac catheterization e. recent urinary tract infection Rationale: recent dental, urologic, surgical, or gynecologic procedures and history of IV drug abuse, heart disease, cardiac catheterization or surgery, renal dialysis, and infections all increase the risk of IE.

The patient has a permanent pacemaker inserted. The provider has set the pacemaker to the demand mode at a rate of 60 beats per minute. The nurse realizes that: a. the pacemaker will pace only if the patient's intrinsic heart rate is less than 60 beats per minute. b. the demand mode often competes with the patient's own rhythm. c. the demand mode places the patient at risk for the R-on-T phenomenon. d. the fixed rate mode is safer and is the mode of choice.

a. the pacemaker will pace only if the patient's intrinsic heart rate is less than 60 beats per minute.

The patient is having premature ventricular contractions (PVCs). The nurse's greatest concern should be: a. the proximity of the R wave of the PVC to the T wave of a normal beat. b. the fact that PVCs are occurring, because they are so rare. c. if the number of PVCs are decreasing. d. if the PVCs are wider than 0.12 seconds.

a. the proximity of the R wave of the PVC to the T wave of a normal beat.

When teaching a patient about the long-term consequences of rheumatic fever, the nurse should discuss the possibility of a. valvular heart disease b. pulmonary hypertension c. superior vena cava syndrome d. hypertrophy of the right ventricle

a. valvular heart disease Rationale: Rheumatic heart disease is a chronic condition resulting from rheumatic fever that is characterized by scarring and deformity of the heart valves.

The nurse is assessing a patient's ECG rhythm strip and analyzing the P waves. Which questions does the nurse use to evaluate the P waves? (SATA) a. Are P waves present? b. Are the P waves occurring regularly? c. Does one P wave follow each QRS complex? d. Are the P waves greater than 0.20 second? e. Do all the P waves look similar? f. Are the P waves smooth, rounded, and upright in appearance?

abef

A 67-yr-old man with peripheral artery disease is seen in the primary care clinic. Which symptom reported by the patient would indicate to the nurse that the patient is experiencing intermittent claudication? a.Patient complains of chest pain with strenuous activity. b.Patient says muscle leg pain occurs with continued exercise. c. Patient has numbness and tingling of all his toes and both feet. d. Patient states the feet become red if he puts them in a dependent position.

b

A nurse is caring for a patient with a diagnosis of deep venous thrombosis (DVT). The patient has an order to receive 30 mg enoxaparin (Lovenox). Which injection site should the nurse use to administer this medication safely? a. Buttock, upper outer quadrant b. Abdomen, anterior-lateral aspect c. Back of the arm, 2 inches away from a mole d. Anterolateral thigh, with no scar tissue nearby

b

A patient is in VF. The nurse sets the biphasic defibrillator to deliver how many joules? a. 100 b. 200 c. 300 d. 360

b

A patient is in full cardiac arrest and CPR is in progress. The ECG monitor shows ventricular asystole. What does the nurse do next? a. Assist with or administer defibrillation b. Assess another ECG lead to ensure the rhythm is asystole and not fine VF c. Assist the provider with noninvasive pacing or invasive transvenous pacing d. Encourage the family's presence during the resuscitation

b

A patient reports chest pain and dizziness after exertion, and the family reports a concurrent new onset of mild confusion in the patient, as well as difficulty concentrating. What is the priority problem for this patient? a. Activity intolerance b. Decreased cardiac output c. Acute confusion d. Inadequate oxygenation

b

A patient scheduled to have elective cardioversion for AF will receive drug therapy for about 6 weeks before the procedure. What information about the drug therapy does the nurse teach the patient? a. Managing orthostatic hypotension b. Watching for bleeding signs c. Eating potassium-rich food sources d. Reporting muscle weakness or tremors

b

The patient with VTE is receiving therapy with heparin and asks the nurse whether the drug will dissolve the clot in her leg. What is the best response by the nurse? a. "This drug will break up and dissolve the clot so that circulation in the vein can be restored." b. "The purpose of the heparin is to prevent growth of the clot or formation of new clots where the circulation is slowed." c. "Heparin won't dissolve the clot but it will inhibit the inflammation around the clot and delay the development of new clots." d. "The heparin will dilate the vein, preventing turbulence of blood flow around the clot that may cause it to break off and travel to the lungs."

b

The primary pacemaker of the heart, the sinoatrial (SA) node, is functional if a patient's pulse is at what regular rate? a. Fewer than 60 beats/min b. 60 to 100 beats/min c. 80 to 100 beats/min d. Greater than 100 beats/min

b

The remote telemetry technician calls the nurse to report that a patient's ECG signal transmission is not very clear. What does the nurse do to enhance the transmission? a. Clean the skin with povidone-iodine solution before applying the electrodes b. Ensure that the area for the electrode placement is dry and nonhairy c. Apply tincture of benzoin to the electrode sites and allow it to dry d. Abrade the skin by rubbing briskly with a rough washcloth

b

The respiratory therapist (RT) and the medical student are ventilating a patient in cardiac arrest, while the nurse and provider are preparing the patient and equipment for intubation. At which point does the nurse intervene? a. The RT inserts an oropharyngeal airway b. The medical student sets the oxygen flow meter at 2 L/min c. The RT ventilates with a manual resuscitation bag and mask d. The medical student uses the chin-lift position on the patient

b

What is most organ damage in hypertension related to? a. Increased fluid pressure exerted against organ tissue b. Atherosclerotic changes in vessels that supply the organs c. Erosion and thinning of blood vessels from constant pressure d. Increased hydrostatic pressure causing leakage of plasma into organ interstitial spaces

b

What is the most important measure in the treatment of venous stasis ulcers? a. Elevation of the limb b. Elastic compression stockings c. Application of moist dressings d. Application of topical antibiotics

b

When teaching a patient about dietary management of stage 1 hypertension, which instruction is most appropriate? a. Restrict all caffeine. b. Restrict sodium intake. c. Increase protein intake. d. Use calcium supplements.

b

When teaching the patient with PAD about modifying risk factors associated with the condition, what should the nurse emphasize? a. Amputation is the ultimate outcome if the patient does not alter lifestyle behaviors. b. Modifications will reduce the risk of other atherosclerotic conditions such as stroke. c. Risk-reducing behaviors initiated after angioplasty can stop the progression of the disease. d. Maintenance of normal body weight is the most important factor in controlling arterial disease

b

Which definition best describes the synchronous (demand) pacing mode? a. The pacemaker continues to fire at a fixed rate as set on the generator b. The pacemaker's sensitivity is set to sense the patient's own beats c. Electrical pulses are transmitted through two large external electrodes then transcutaneously to stimulate ventricular depolarization d. External battery-operated pulse generator on one end and wires in contact with the heart on the other end

b

A patient with critical limb ischemia had peripheral artery bypass surgery to improve circulation. What nursing care should be provided on postoperative day 1? a. Keep patient on bed rest. b. Assist patient to walk several times. c. Have patient sit in the chair several times. d. Place patient on their side with knees flexed.

b To avoid blockage of the graft or stent, the patient should walk several times on postoperative day 1 and subsequent days. Having the patient's knees flexed for sitting in a chair or in bed increase the risk of venous thrombosis and may place stress on the suture lines.

The nurse is caring for a patient admitted with a history of hypertension. The patient's medication history includes hydrochlorothiazide (Hydrodiuril) daily for the past 10 years. Which parameter would indicate the optimal intended effect of this drug therapy? a) Weight loss of 2 lb b) Blood pressure 128/86 c) Absence of ankle edema d) Output of 600 mL per 8 hours

b) Blood pressure 128/86 Hydrochlorothiazide may be used alone as monotherapy to manage hypertension or in combination with other medications if not effective alone. After the first few weeks of therapy, the diuretic effect diminishes, but the antihypertensive effect remains. Since the patient has been taking this medication for 10 years, the most direct measurement of its intended effect would be the blood pressure.

When teaching a patient about dietary management of stage 1 hypertension, which instruction is most appropriate? a) Restrict all caffeine. b) Restrict sodium intake. c) Increase protein intake. d) Use calcium supplements.

b) Restrict sodium intake. The patient should decrease intake of sodium. This will help to control hypertension, which can be aggravated by excessive salt intake, which in turn leads to fluid retention. Caffeine and protein intake do not affect hypertension. Calcium supplements are not recommended to lower BP.

When providing dietary instruction to a patient with hypertension, the nurse would advise the patient to restrict intake of which meat? a) Broiled fish b) Roasted duck c) Roasted turkey d) Baked chicken breast

b) Roasted duck Roasted duck is high in fat, which should be avoided by the patient with hypertension. Weight loss may slow the progress of atherosclerosis and overall CVD risk. The other meats are lower in fat and are therefore acceptable in the diet.

When assessing the patient for orthostatic hypotension, after taking the blood pressure (BP) and pulse (P) in the supine position, what should the nurse do next? a) Repeat BP and P in this position. b) Take BP and P with patient sitting. c) Record the BP and P measurements. d) Take BP and P with patient standing.

b) Take BP and P with patient sitting. When assessing for orthostatic changes in BP after measuring BP in the supine position, the patient is placed in a sitting position and BP is measured within 1 to 2 minutes and then repositioned to the standing position with BP measured again, within 1 to 2 minutes. The results are then recorded with a decrease of 20 mm Hg or more in SBP, a decrease of 10 mm Hg or more in DBP, and/or an increase in pulse of greater than or equal to 20 beats/minute from supine to standing indicating orthostatic hypotension.

Patients with a heart transplantation are at risk for which complications in the first year after transplantation (select all that apply)? a. cancer b. infection c. rejection d. vasculopathy e. sudden cardiac death

b, c, & e Rationale: A variety of complications can occur after heart transplantation. In the first year after transplantation, the major causes of death are acute rejection and infection. Heart transplant recipients also are at risk for sudden cardiac death. Later, malignancy (especially lymphoma) and cardiac vasculopathy (accelerated CAD) are major causes of death.

Which conditions characterize critical limb ischemia (select all that apply)? a. Cold feet b. Arterial leg ulcers c. Venous leg ulcers d. Gangrene of the leg e. No palpable peripheral pulses f. Rest pain lasting more than 2 weeks

b, d, f

In the patient with heart failure, which medications or treatments require careful monitoring of the patient's serum potassium level to prevent further cardiac dysfunction (select all that apply)? a. Enalapril (Vasotec) b. Furosemide (Lasix) c. Nesiritide (Natrecor) d. Spironolactone (Aldactone) e. Metoprolol CR/XL (Toprol XL)

b, d. Furosemide is a diuretic that eliminates potassium and spironolactone is a potassium-sparing diuretic that retains potassium. The other treatments and medications are used for patients with HF, but they do not directly affect serum potassium levels.

Which classification of drugs used to treat hypertension prevents the action of angiotensin II and promotes increased salt and water excretion? a. Thiazide diuretics b. Angiotensin II receptor blockers (ARBs) c. Direct vasodilators d. Angiotensin-converting enzyme (ACE) inhibitors

b. Angiotensin II receptor blockers (ARBs) prevent the action of angiotensin II and produce vasodilation and increased salt and water excretion.

Priority Decision: A patient with chronic heart failure is treated with hydrochlorothiazide, digoxin, and lisinopril. To prevent the risk of digitalis toxicity with these drugs, what is most important that the nurse monitor for this patient? a. Heart rate (HR) b. Potassium levels c. Blood pressure (BP) d. Gastrointestinal function

b. Hypokalemia is one of the most common causes of digitalis toxicity because low serum potassium levels enhance ectopic pacemaker activity. When a patient is receiving potassium-losing diuretics, such as hydrochlorothiazide or furosemide, it is essential to monitor the patient's serum potassium levels to prevent digitalis toxicity. Monitoring the heart rate (HR) assesses for complications related to digoxin but does not prevent toxicity.

What is the pathophysiologic mechanism that results in the pulmonary edema of left-sided heart failure? a. Increased right ventricular preload. b. Increased pulmonary hydrostatic pressure. c. Impaired alveolar oxygen and carbon dioxide exchange. d. Increased lymphatic flow of pulmonary extravascular fluid

b. In left-sided HF, blood backs up into the pulmonary veins and capillaries. This increased hydrostatic pressure in the vessels causes fluid to move out of the vessels and into the pulmonary interstitial space. When increased lymphatic flow cannot remove enough fluid from the interstitial space, fluid moves into the alveoli, resulting in pulmonary edema and impaired alveolar oxygen and carbon dioxide exchange. Initially the right side of the heart is not involved.

The nurse is calculating the rate for a regular rhythm. There are 20 small boxes between each P wave and 20 small boxes between each R wave. What is the ventricular rate? a. 50 beats/min b. 75 beats/min c. 85 beats/min d. 100 beats/min

b. 75 beats/min

7] The nurse has completed an assessment on a client with a decreased cardiac output. Which findings should receive highest priority? a. BP 110/62, atrial fibrillation with HR 82, bibasilar crackles b. Confusion, urine output 15mL over the last 2 hours, orthopnea. c. SpO2 92 on 2 liters nasal cannula, respirations 20, 1+ edema of lower extremities. d. Weight gain of 1kg in 3 days, BP 130/80, mild dyspnea with exercise.

b. A low urine output and confusion are signs of decreased tissue perfusion. Orthopnea is a sign of left-sided heart failure. Crackles, edema and weight gain should be monitored closely, but the levels are not as high a priority. With atrial fibrillation there is a loss of atrial kick, but the blood pressure and heart rate are stable

33. Which actions could the nurse delegate to unlicensed assistive personnel (UAP) who are providing care for a patient who is at risk for venous thromboembolism? a. Monitor for any bleeding after anticoagulation therapy is started. b. Apply sequential compression device whenever the patient is in bed. c. Ask the patient about use of herbal medicines or dietary supplements. d. Instruct the patient to call immediately if any shortness of breath occurs.

b. Apply sequential compression device whenever the patient is in bed.

20] Which of the following is an expected outcome when a client is receiving an IV administration of furosemide? a. Increased blood pressure b. Increased urine output c. Decreased pain d. Decreased premature ventricular contractions

b. Furosemide is a loop diuretic that acts to increase urine output. Furosemide does not increase blood pressure, decrease pain, or decrease arrhythmias

What should the nurse recognize as an indication for the use of dopamine (Intropin) in the care of a patient with heart failure? a. Acute anxiety b. Hypotension and tachycardia c. Peripheral edema and weight gain d. Paroxysmal nocturnal dyspnea (PND)

b. Hypotension and tachycardia Dopamine is a β-adrenergic agonist whose inotropic action is used for treatment of severe heart failure accompanied by hemodynamic instability. Such a state may be indicated by tachycardia accompanied by hypotension. PND, anxiety, edema, and weight gain are common signs and symptoms of heart failure, but these do not necessarily warrant the use of dopamine.

13. The nurse has started discharge teaching for a patient who is to continue warfarin (Coumadin) following hospitalization for venous thromboembolism (VTE). The nurse determines that additional teaching is needed when the patient says which of the following? a. I should get a Medic Alert device stating that I take Coumadin. b. I should reduce the amount of green, leafy vegetables that I eat. c. I will need routine blood tests to monitor the effects of the Coumadin. d. I will check with my health care provider before I begin any new medications.

b. I should reduce the amount of green, leafy vegetables that I eat.

21. When developing a teaching plan for a 76-year-old patient newly diagnosed with peripheral artery disease (PAD), which instructions should the nurse include? a. Exercise only if you do not experience any pain. b. It is very important that you stop smoking cigarettes. c. Try to keep your legs elevated whenever you are sitting. d. Put elastic compression stockings on early in the morning.

b. It is very important that you stop smoking cigarettes.

40] Which of the following is not a risk factor for the development of atherosclerosis? a. Family history of early heart attack b. Late onset of puberty c. Total blood cholesterol level greater than 220 mg/dL d. Elevated fasting blood glucose concentration

b. Late onset of puberty is not generally considered to be a risk factor for the development of atherosclerosis. Risk factors for atherosclerosis include family history of atherosclerosis, cigarette smoking, hypertension, high blood cholesterol level, male gender, diabetes mellitus, obesity, and physical inactivity

17. Which nursing action should be included in the plan of care after endovascular repair of an abdominal aortic aneurysm? a. Record hourly chest tube drainage. b. Monitor fluid intake and urine output. c. Check the abdominal incision for any redness. d. Teach the reason for a prolonged recovery period.

b. Monitor fluid intake and urine output.

26. The nurse is caring for a patient with critical limb ischemia who has just arrived on the nursing unit after having percutaneous transluminal balloon angioplasty. Which action should the nurse perform first? a. Begin oral intake. b. Obtain vital signs. c. Assess pedal pulses. d. Start discharge teaching.

b. Obtain vital signs.

34. The nurse who works in the vascular clinic has several patients with venous insufficiency scheduled today. Which patient should the nurse assign to an experienced licensed practical/vocational nurse (LPN/LVN)? a. Patient who has been complaining of increased edema and skin changes in the legs b. Patient who needs wound care for a chronic venous stasis ulcer on the right lower leg c. Patient who has a history of venous thromboembolism and is complaining of some dyspnea d. Patient who needs teaching about the use of elastic compression stockings for venous insufficiency

b. Patient who needs wound care for a chronic venous stasis ulcer on the right lower leg

Which patient teaching points should the nurse include when providing discharge instructions to a patient with a new permanent pacemaker and the caregiver (select all that apply)? a. Avoid or limit air travel b. Take and record a daily pulse rate c. Obtain and wear a Medic Alert ID device at all times d. Avoid lifting arm on the side of the pacemaker above shoulder e. Avoid microwave ovens because they interfere with pacemaker function

b. Take and record a daily pulse rate c. Obtain and wear a Medic Alert ID device at all times d. Avoid lifting arm on the side of the pacemaker above shoulder Rationale: Pacemaker discharge teaching should include the following instructions: First, air travel is not restricted. The patient should inform airport security of the presence of a pacemaker because it may set off the metal detector. If a hand-held screening wand is used, it should not be placed directly over the pacemaker. Manufacturer information may vary with regard to the effect of metal detectors on the function of the pacemaker. Second, the patient should monitor the pulse and inform the cardiologist if it drops below a predetermined rate. Third, the patient should obtain and wear a Medic Alert ID device at all times. Fourth, the patient must avoid lifting the arm on the pacemaker side above the shoulder until this is approved by the cardiologist. Fifth, microwave ovens are safe to use, and they do not interfere with pacemaker function. Table 35-13 provides additional discharge teaching guidelines for a patient with a pacemaker.

The nurse is preparing to administer digoxin to a patient with heart failure. In preparation, laboratory results are reviewed with the following findings: sodium 139 mEq/L, potassium 5.6 mEq/L, chloride 103 mEq/L, and glucose 106 mg/dL. What should the nurse do next? a. Withhold the daily dose until the following day. b. Withhold the dose and report the potassium level. c. Give the digoxin with a salty snack, such as crackers. d. Give the digoxin with extra fluids to dilute the sodium level.

b. Withhold the dose and report the potassium level. The normal potassium level is 3.5 to 5.0 mEq/L. The patient is hyperkalemic, which makes the patient more prone to digoxin toxicity. For this reason, the nurse should withhold the dose and report the potassium level. The physician may order the digoxin to be given once the potassium level has been treated and decreases to within normal range.

11. The health care provider prescribes an infusion of heparin (Hep-Lock) and daily partial thromboplastin time (PTT) testing for a patient with venous thromboembolism (VTE). The nurse will plan to a. decrease the infusion when the PTT value is 65 seconds. b. avoid giving any IM medications to prevent localized bleeding. c. monitor posterior tibial and dorsalis pedis pulses with the Doppler. d. have vitamin K available in case reversal of the heparin is needed.

b. avoid giving any IM medications to prevent localized bleeding.

The nurse notices ventricular tachycardia on the heart monitor. When the patient is assessed, the patient is found to be unresponsive with no pulse. The nurse should: a. treat with intravenous amiodarone or lidocaine. b. begin cardiopulmonary resuscitation and advanced life support. c. provide electrical cardioversion. d. ignore the rhythm since it is benign.

b. begin cardiopulmonary resuscitation and advanced life support.

A patient with acute pericarditis has markedly distended jugular veins, decreased BP, tachycardia, tachypnea, and muffled heart sounds. The nurse recognizes that these symptoms occur when a. the pericardial space is obliterated with scar tissue and thickened pericardium b. excess pericardial fluid compresses the heart and prevents adequate diastolic filling c. the parietal and visceral pericardial membranes adhere to each other, preventing normal myocardial contraction d. fibrin accumulation on the visceral pericardium infiltrates into the myocardium, creating generalized myocardial dysfunction.

b. excess pericardial fluid compresses the heart and prevents adequate diastolic filling Rationale: The patient is experiencing a cardiac tamponade that consists of excess fluid in the pericardial sac, which compresses the heart and the adjoining structures, preventing normal filling and cardiac output. Fibrin accumulation, a scarred and thickened pericardium, and adherent pericardial membranes occur in chronic constrictive pericarditis.

While obtaining subjective assessment date from a patient with hypertension, the nurse recognizes that a modifiable risk factor for the development of hypertension is: a. a low-calcium diet b. excessive alcohol consumption c. a family history of hypertension d. consumption of a high-protein diet

b. excessive alcohol consumption

The rhythm on the cardiac monitor is showing numerous pacemaker spikes, but no P waves or QRS complexes following the spikes. The nurse realizes this as: a. normal pacemaker function. b. failure to capture. c. failure to pace. d. failure to sense.

b. failure to capture.

The nurse is examining the patient's cardiac rhythm strip in lead II and notices that all of the P waves are upright and look the same except one that has a different shape and is inverted. The nurse realizes that the P wave with the abnormal shape is probably: a. from the SA node since all P waves come from the SA node. b. from some area in the atria other than the SA node. c. indicative of ventricular depolarization. d. normal even though it is inverted in lead II.

b. from some area in the atria other than the SA node.

A patient with symptomatic mitral valve prolapse has atrial and ventricular dysrhythmias. In addition to monitoring for decreased cardiac output related to the dysrhythmias, an appropriate nursing diagnosis related to the dysthymias identified by the nurse is a. ineffective breathing pattern related to hypervolemia b. risk for injury related to dizziness and lightheadedness c. disturbed sleep pattern related to paroxysmal nocturnal dyspnea d. ineffective self-health management related to lack of knowledge of prevention and treatment strategies

b. risk for injury related to dizziness and lightheadedness Rationale: Dysrhythmias frequently cause palpitations, lightheadedness, and dizziness, and the patient should be carefully attended to prevent falls. Hypervolemia and paroxysmal nocturnal dyspnea (PND) would be apparent in the patient with heart failure.

One of the functions of the atrioventricular (AV) node is to: a. pace the heart if the ventricles fail. b. slow the impulse arriving from the SA node. c. send the impulse to the SA node. d. allow for ventricular filling during systole.

b. slow the impulse arriving from the SA node.

2. A patient has a 6-cm thoracic aortic aneurysm that was discovered during a routine chest x-ray. When obtaining an admission history from the patient, it will be most important for the nurse to ask about a. low back pain. b. trouble swallowing. c. abdominal tenderness. d. changes in bowel habits.

b. trouble swallowing.

8. When evaluating the discharge teaching for a patient with chronic peripheral artery disease (PAD), the nursedetermines a need for further instruction when the patient says, I will a. have to buy some loose clothes that do not bind across my legs or waist. b. use a heating pad on my feet at night to increase the circulation and warmth in my feet. c. change my position every hour and avoid long periods of sitting with my legs crossed. d. walk to the point of pain, rest, and walk again until the pain returns for at least 30 minutes 3 times a week.

b. use a heating pad on my feet at night to increase the circulation and warmth in my feet.

A patient with rheumatic heart disease with carditis asks the nurse how long his activity will be restricted. The best answer by the nurse is that he a. can perform non- strenuous activities as soon as antibiotics are started b. will be confined to bed until symptoms of heart failure are controlled c. will be able to have full activity as soon as cute symptoms have subsided d. must be on bed rest until anti-inflammatory therapy has been discontinued

b. will be confined to bed until symptoms of heart failure are controlled Rationale: When carditis is present in the patient with rheumatic fever, ambulation is postponed until any symptoms of heart failure are controlled with treatment, and full activity cannot be resumed until anti-inflammatory therapy has been discontinued. In the patient without cardiac involvement, ambulation may be permitted as soon as acute symptoms have subsided, and normal activity can be resumed when anti-inflammatory therapy is discontinued.

A patient with infective endocarditis develops sudden left leg pain with pallor, paresthesia, and a loss of peripheral pulses. The nurse's initial action should be to a. elevate the leg to promote venous return. b. start anticoagulant therapy with IV heparin. c. notify the HCP of the change in peripheral perfusion. d. place the bed in reverse Trendelenburg to promote perfusion.

c

AED electrodes are placed on a patient who is unconscious and pulseless. The nurse prepares to immediately defibrillate if the monitor shows which cardiac anomaly? a. Third-degree heart block b. Pulseless electrical activity c. VF d. Idioventricular rhythm

c

Cardiac dysrhythmias are abnormal rhythms of the heart's electrical system. How does this affect the heart's function? a. It cannot oxygenate the blood throughout the body b. It cannot remove carbon dioxide from the body c. It cannot effectively pump oxygenated blood throughout the body d. It cannot effectively conduct impulses with increased acitivyt

c

Delegation Decision: The nursing care area is very busy with new surgical patients. Which care could the RN delegate to the unlicensed assistive personnel (UAP) for a patient with VTE? a. Assess the patient's use of herbs. b. Measure the patient for elastic compression stockings. c. Remind the patient to flex and extend the legs and feet every 2 hours d. Teach the patient to call emergency medical services (EMS) with signs of pulmonary embolus

c

Following teaching about medications for PAD, the nurse determines that additional instruction is necessary when the patient makes which statement? a. "I should take one aspirin a day to prevent clotting in my legs." b. "The lisinopril I use for my blood pressure may help me walk further without pain." c. "I will need to have frequent blood tests to evaluate the effect of the Coumadin I will be taking." d. "Pletal should help me increase my walking distance and help prevent clots from forming in my legs."

c

Rest pain is a manifestation of PAD that occurs due to a chronic a. vasospasm of small cutaneous arteries in the feet. b. increase in retrograde venous blood flow in the legs. c. decrease in arterial blood flow to the nerves of the feet. d. constriction in arterial blood flow to the leg muscles during exercise.

c

The nurse in the telemetry unit must perform transcutaneous pacing. How does the nurse position the electrodes? a. One over breast tissue on the right side and one over breast tissue on left side b. One on the upper chest to the left of the sternum and one beneath the left scapula c. One on the upper chest to the right of the sternum and one over the heart apex. d. One over the sternum and one on the left anterior lateral chest

c

The nurse is caring for a patient with coronary artery disease (CAD). The patient reports palpitations and chest discomfort and the nurse notes a tachydysrhythmia on the EG monitor. What does the nurse do next? c. Give supplemental oxygen

c

The nurse is caring for several patients who have dysrhythmia. What does the nurse instruct these patients to do? a. Stay at least 4 feet away from a microwave oven that is operating b. Avoid electronic metal detectors, such as those at airports c. Learn the procedure for assessing the pulse. d. Purchase an automatic external defibrillator (AED) for home use

c

The nurse is interviewing a patient with spontaneous VT who may be a possible candidate for an ICD. The nurse senses that the patient is anxious. What is the nurse's most therapeutic response? a "Your feelings are natural; patients report psychological distress related to ICD." b. "ICD is similar to defibrillation, which saved your life during the last episode" c. "You see anxious. What are your concerns about having this treatment?" d. "Would you like to talk to the doctor about the details of the procedure?"

c

The nurse is performing external defibrillation. Which step is most vital in this procedure? a. Place the gel pads anterior over the apex and posterior for better conduction b. Do not administer a second shock fro 1 minute to allow for recharging c. No-one must touch the patient at the time a shock is delivered d. Continuously ventilate the patient via endotracheal tube during the defibrillation

c

The nurse is performing the shift assessment on a cardiac patient. In order to determine if the patient has a pulse deficit, what does the nurse do? Assess the apical and radial pulses for a full minute and observe for differences.

c

The nurse is placing the electrodes on a patient for cardioversion. What is the correct placement for the electrodes? a. One electrode is placed on the upper left chest and the other is placed on the lower left chest in a midaxillary line b. One electrode is placed the upper right chest below the clavicle and the other is placed on the back c. One electrode is placed to the left of the precordium, and the other is placed on the right next to the sternum and below the clavicle. d. One electrode is placed on the sternum and the other is placed on the lower left chest in a midaxillary line

c

The nurse is reviewing ECG results of a patient admitted for fluid and electrolyte imbalances. The T waves are tall and peaked. The nurse reports this finding to the provider and obtains an order for which serum level tests? a. Sodium b. Glucose c. Potassium d. Phosphorus

c

The nurse is reviewing the monitored rhythms of several patients in the cardiac stepdown unit. The patient with which cardiac anomaly has the greatest need of immediate attention? a. Chronic atrial fibrillation b. Paroxysmal supraventricular tachycardia (SVT) that is suddenly terminated c. Sustained rapid ventricular response d. Sinus tachycardia with premature atrial complexes

c

The nurse is taking vital signs ad reviewing the electrocardiogram (ECG) of a patient who is training for a marathon. The heart rate is 45 beats/min and the ECG shows sinus bradycardia. How does the nurse interpret this data? a. A rapid filling rate that lengthens diastolic filling time and leads to decreased cardiac output b. The body's attempt to compensate for a decreased stroke volume by decreasing the heart rate c. An adequate stroke volume that is associated with cardiac conditioning d. A common finding in the health adult that would be considered normal

c

The nurse is teaching a patient with an ICD. What instruction does the nurse emphasize to the patient? a. Rest for several hours after an internal defibrillator shock before resuming activities b. Have family members step away during the internal defibrillator shock for safety c. Expect that the shock may feel like a thud or a painful kick in the chest. d. Report any pulse rate higher than what is set on the pacemaker

c

The nurse teaches a 28-yr-old man newly diagnosed with hypertension about lifestyle modifications to reduce his blood pressure. Which patient statement requires reinforcement of teaching? a. "I will avoid adding salt to my food during or after cooking." b. "If I lose weight, I might not need to continue taking medications." c. "I can lower my blood pressure by switching to smokeless tobacco." d. "Diet changes can be as effective as taking blood pressure medications.

c

What is the heart rate shown on a 6-second ECG strip when the number of R-R intervals is 5? What is this rhythm? a. 30/minute bradycardia b. 40/minute bradycardia c. 50/minute bradycardia d. 60/minute normal

c

Which dysrhythmia results in asynchrony of atrial contraction and decreased cardiac output? a. Sinus tachycardia b. Atrial flutter c. Atrial fibrillation d. First-degree atrioventricular block

c

How is secondary hypertension differentiated from primary hypertension? a. Has a more gradual onset than primary hypertension b. Does not cause the target organ damage that occurs with primary hypertension c. Has a specific cause, such as renal disease, that often can be treated by medicine or surgery d. Is caused by age-related changes in BP regulatory mechanisms in people over 65 years of age

c Secondary hypertension has an underlying cause that can often be treated, in contrast to primary or essential hypertension,which has no single known cause.

You are caring for a patient with ADHF who is receiving IV dobutamine (Dobutrex). You know that this drug is ordered because it (select all that apply): a. incerases SVR b. produces diuresis c. improves contractility d. dilates renal blood vessels e. works on the B1-receptors in the heart.

c & e Rationale: Dobutamine (Dobutrex) has a positive chronotropic effect and increases heart rate and improves contractility. It is a selective β-adrenergic agonist and works primarily on the β1-adrenergic receptors in the heart. It is frequently used in the short-term management of acute decompensated heart failure (ADHF).

The nurse is caring for a patient admitted with chronic obstructive pulmonary disease (COPD), angina, and hypertension. Before administering the prescribed daily dose of atenolol 100 mg PO, the nurse assesses the patient carefully. Which adverse effect is this patient at risk for, given the patient's health history? a) Hypocapnia b) Tachycardia c) Bronchospasm d) Nausea and vomiting

c) Bronchospasm Atenolol is a cardioselective β1-adrenergic blocker that reduces blood pressure and could affect the β2-receptors in the lungs with larger doses or with drug accumulation. Although the risk of bronchospasm is less with cardioselective β-blockers than nonselective β-blockers, atenolol should be used cautiously in patients with COPD.

When teaching how lisinopril (Zestril) will help lower the patient's blood pressure, which mechanism of action should the nurse use to explain it? a) Blocks β-adrenergic effects. b) Relaxes arterial and venous smooth muscle. c) Inhibits conversion of angiotensin I to angiotensin II. d) Reduces sympathetic outflow from central nervous system.

c) Inhibits conversion of angiotensin I to angiotensin II. Lisinopril is an angiotensin-converting enzyme (ACE) inhibitor that inhibits the conversion of angiotensin I to angiotensin II, which reduces angiotensin II-mediated vasoconstriction and sodium and water retention. Beta blockers result in vasodilation and decreased heart rate. Direct vasodilators relax arterial and venous smooth muscle. Central acting α-adrenergic antagonists reduce sympathetic outflow from the CNS to produce vasodilation and decreased SVR and BP.

In caring for a patient admitted with poorly controlled hypertension, which laboratory test result should the nurse understand as indicating the presence of target organ damage? a) BUN of 15 mg/dL b) Serum uric acid of 3.8 mg/dL c) Serum creatinine of 2.6 mg/dL d) Serum potassium of 3.5 mEq/L

c) Serum creatinine of 2.6 mg/dL The normal serum creatinine level is 0.6-1.3 mg/dL. This elevated level indicates target organ damage to the kidneys. The other lab results are within normal limits.

The nurse is teaching a women's group about prevention of hypertension. What information should be included in the teaching for all the women (select all that apply)? a) Lose weight. b) Limit nuts and seeds. c) Limit sodium and fat intake. d) Increase fruits and vegetables. e) Exercise 30 minutes most days.

c, d, & e Primary prevention of hypertension is to make lifestyle modifications that prevent or delay the increase in BP. Along with exercise for 30 minutes on most days, the DASH eating plan is a healthy way to lower BP by limiting sodium and fat intake, increasing fruits and vegetables, and increasing nutrients that are associated with lowering BP. Nuts and seeds and dried beans are used for protein intake. Weight loss may or may not be necessary for the individual.

The nurse is teaching a women's group about prevention of hypertension. What information should be included in the teaching for all the women (select all that apply)? a. Lose weight. b. Limit nuts and seeds. c. Limit sodium and fat intake. d. Increase fruits and vegetables. e. Exercise 30 minutes most days.

c, d, e

What describes the action of the natriuretic peptides and nitric oxide in their counterregulatory processes in response to heart failure (HF)? a. Excretion of potassium b. Increased release of ADH c. Vasodilation and decreased blood pressure (BP) d. Decreased glomerular filtration rate and edema

c. Both the natriuretic peptides and nitric oxide contribute to vasodilation, decreased BP, and decreased afterload. The natriuretic peptides also increase excretion of sodium by increasing glomerular filtration rate and diuresis (renal effects) as well as interfere with ADH release and inhibit aldosterone and renin secretion (hormonal effects).

Which manifestation is an indication that a patient is having a hypertensive emergency? a. Symptoms of a stroke with an elevated BP b. A systolic BP >200 mm Hg and a diastolic BP >120 mm Hg c. A sudden rise in BP accompanied by neurologic impairment d. A severe elevation of BP that occurs over several days or weeks

c. Hypertensive emergency, a type of hypertensive crisis, is a situation that develops over hours or days in which a patient's BP is severely elevated with evidence of acute target organ disease (e.g., cerebrovascular, cardiovascular, renal, or retinal). The neurologic manifestations are often similar to the presentation of a stroke but do not show the focal or lateralizing symptoms of stroke.

Priority Decision: A patient is admitted to the emergency department with ADHF. Which IV medication would the nurse expect to administer first? a. Digoxin (Lanoxin) b. Morphine sulfate c. Nesiritide (Natrecor) d. Bumetanide (Bumex)

c. Nesiritide (Natrecor) is a recombinant form of a natriuretic peptide that decreases preload and afterload by reducing pulmonary artery wedge pressure (PAWP) and systolic BP, which decreases the workload of the heart for short-term emergency treatment of acute decompensated heart failure (ADHF). Digoxin requires a loading dose and time to work, so it is not recommended for emergency treatment of ADHF. Morphine sulfate relieves dyspnea but has more adverse effects. Bumetanide (Bumex) will decrease fluid volume but also will decrease potassium levels and activate the sympathetic nervous system and RAAS, which can exacerbate HF symptoms.

The nurse determines that treatment of heart failure has been successful when the patient experiences a. weight loss and diuresis. b. warm skin and less fatigue. c. clear lung sounds and decreased HR. d. absence of chest pain and improved level of consciousness (LOC).

c. Successful treatment of heart failure is indicated by an absence of symptoms of pulmonary edema and hypoxemia, such as clear lung sounds and a normal HR. Weight loss and diuresis, warm skin, less fatigue, and improved LOC may occur without resolution of pulmonary symptoms. Chest pain is not a common finding in heart failure unless coronary artery perfusion is impaired.

Which diagnostic test is most useful in differentiating dyspnea related to pulmonary effects of heart failure from dyspnea related to pulmonary disease? a. Exercise stress testing. b. Cardiac catheterization. c. B-type natriuretic peptide (BNP) levels. d. Determination of blood urea nitrogen (BUN)

c. b-type natriuretic peptide (BNP) is released from the ventricles in response to increased blood volume in the heart and is a good marker for heart failure. If BNP is elevated, shortness of breath is due to heart failure; if BNP is normal, dyspnea is due to pulmonary disease. BNP opposes the actions of the RAAS, resulting in vasodilation and reduction in blood volume. Exercise stress testing and cardiac catheterization are more important tests to diagnose coronary artery disease, and although the blood urea nitrogen (BUN) may be elevated in heart failure, it is a reflection of decreased renal perfusion. (See Table 31-6.)

The patient is scheduled to have a permanent pacemaker implanted. The patient asks the nurse, "How long will the battery in this thing last?" The nurse should answer, a. "Life expectancy is about 1 year. Then it will need to be replaced." b. "Pacemaker batteries can last up to 25 years with constant use." c. "Battery life varies depending on usage, but it can last up to 10 years." d. "Pacemakers are used to treat temporary problems so the batteries don't last long."

c. "Battery life varies depending on usage, but it can last up to 10 years."

23. After receiving report, which patient admitted to the emergency department should the nurse assess first? a. 67-year-old who has a gangrenous left foot ulcer with a weak pedal pulse b. 58-year-old who is taking anticoagulants for atrial fibrillation and has black stools c. 50-year-old who is complaining of sudden sharp and worst ever upper back pain d. 39-year-old who has right calf tenderness, redness, and swelling after a long plane ride

c. 50-year-old who is complaining of sudden sharp and worst ever upper back pain

25] If a client displays risk factors for coronary artery disease, such as smoking cigarettes, eating a diet high in saturated fat, or leading a sedentary lifestyle, techniques of behavior modification may be used to help the client change the behavior. The nurse can best reinforce new adaptive behaviors by: a. Explaining how the old behavior leads to poor health b. Withholding praise until the new behavior is well established c. Rewarding the client whenever the acceptable behavior is performed d. Instilling mild fear into the client to extinguish the behavior

c. A basic principle of behavior modification is that behavior that is learned and continued is behavior that has been rewarded. Other reinforcement techniques have not been found to be as effective as reward.

What is the priority assessment by the nurse caring for a patient receiving IV nesiritide (Natrecor) to treat heart failure? a. Urine output b. Lung sounds c. Blood pressure d. Respiratory rate

c. Blood pressure Although all identified assessments are appropriate for a patient receiving IV nesiritide, the priority assessment would be monitoring for hypotension, the main adverse effect of nesiritide.

31. The nurse is caring for a patient with a descending aortic dissection. Which assessment finding is most important to report to the health care provider? a. Weak pedal pulses b. Absent bowel sounds c. Blood pressure 137/88 mm Hg d. 25 mL urine output over last hour

c. Blood pressure 137/88 mm Hg

24] A 58-year-old female with a family history of CAD is being seen for her annual physical exam. Fasting lab test results include: Total cholesterol 198; LDL cholesterol 120; HDL cholesterol 58; Triglycerides 148; Blood sugar 102; and C-reactive protein (CRP) 4.2. The health care provider informs the client that she will be started on a statin medication and aspirin. The client asks the nurse why she needs to take these medications. Which is the best response by the nurse? a. "The labs indicate severe hyperlipidemia and the medications will lower your LDL, along with a low-fat diet." b. "The triglycerides are elevated and will not return to normal without these medications." c. "The CRP is elevated indicating inflammation seen in cardiovascular disease, which can be lowered by the medications ordered." d. "The medications are not indicated since your lab values are all normal."

c. CRP is a marker of inflammation and is elevated in the presence of cardiovascular disease. The high sensitivity CRP (hs-CRP) is the blood test for greater accuracy in measuring the CRP to evaluate cardiovascular risk. The family history, post-menopausal age, LDL above optimum levels and elevated CRP place the client at risk of CAD. Statin medications can decrease LDL, whereas statins and aspirin can reduce CRP and decrease the risk of MI and stroke.

14. A 46-year-old service-counter worker undergoes sclerotherapy for treatment of superficial varicose veins at an outpatient center. Which instructions should the nurse provide to the patient before discharge? a. Sitting at the work counter, rather than standing, is recommended. b. Exercise, such as walking or jogging, can cause recurrence of varicosities. c. Elastic compression stockings should be applied before getting out of bed. d. Taking an aspirin daily will help prevent clots from forming around venous valves.

c. Elastic compression stockings should be applied before getting out of bed.

12. A patient with a venous thromboembolism (VTE) is started on enoxaparin (Lovenox) and warfarin (Coumadin). The patient asks the nurse why two medications are necessary. Which response by the nurse is mostappropriate? a. Taking two blood thinners reduces the risk for another clot to form. b. Lovenox will start to dissolve the clot, and Coumadin will prevent any more clots from forming. c. Lovenox will work right away, but Coumadin takes several days to have an effect on preventing clots. d. Because of the risk for a blood clot in the lungs, it is important for you to take more than one blood thinner.

c. Lovenox will work right away, but Coumadin takes several days to have an effect on preventing clots.

41] As an initial step in treating a client with angina, the physician prescribes nitroglycerin tablets, 0.3 mg given sublingually. This drug's principal effects are produced by: a. Antispasmodic effects on the pericardium b. Causing an increased myocardial oxygen demand c. Vasodilation of peripheral vasculature d. Improved conductivity in the myocardium

c. Nitroglycerin produces peripheral vasodilation, which reduces myocardial oxygen consumption and demand. Vasodilation in coronary arteries and collateral vessels may also increase blood flow to the ischemic areas of the heart. Nitroglycerin does not have an effect on pericardial spasticity or conductivity in the myocardium.

36] The client who experiences angina has been told to follow a low-cholesterol diet. Which of the following meals should the nurse tell the client would be best on her low-cholesterol diet? a. Hamburger, salad, and milkshake b. Baked liver, green beans, and coffee c. Spaghetti with tomato sauce, salad, and coffee d. Fried chicken, green beans, and skim milk

c. Pasta, tomato sauce, salad, and coffee would be the best selection for the client following a low-cholesterol diet. Hamburgers, milkshakes, liver, and fried foods tend to be high in cholesterol.

The nurse is monitoring the ECG of a patient admitted with ACS. Which ECG characteristics would be most suggestive of myocardial ischemia? a. Sinus rhythm with a pathologic Q wave b. Sinus rhythm with an elevated ST segment c. Sinus rhythm with a depressed ST segment d. Sinus rhythm with premature atrial contractions

c. Sinus rhythm with a depressed ST segment Rationale: Typical electrocardiographic (ECG) changes that are seen in myocardial ischemia include ST-segment depression and T-wave inversion.

Which of the following nursing actions should the nurse prioritize during the care of a patient who has recently recovered from rheumatic fever? a. Teach the patient how to manage his or her physical activity. b. Teach the patient about the need for ongoing anticoagulation. c. Teach the patient about his or her need for continuous antibiotic prophylaxis. d. Teach the patient about the need to maintain standard infection control procedures.

c. Teach the patient about his or her need for continuous antibiotic prophylaxis. Rationale: Patients with a history of rheumatic fever frequently require ongoing antibiotic prophylaxis, an intervention that necessitates education. This consideration is more important than activity management in preventing recurrence. Anticoagulation and standard precautions are not indicated in this patient population.

43] Sublingual nitroglycerine tablets begin to work within 1 to 2 minutes. How should the nurse instruct the client to use the drug when chest pain occurs? a. Take one tablet every 2 to 5 minutes until the pains stops b. Take one tablet and rest for 10 minutes. Call the physician if pain persists after 10 minutes c. Take one tablet, then an additional tablet every 5 minutes for a total of three tablets. Call the physician if pain persists after three tablets d. Take one tablet. If pain persists after 5 minutes, take two tablets. If pain still persists 5 minutes later, call the physician

c. The correct protocol for nitroglycerin use involves immediate administration, with subsequent doses taken at 5-minute intervals as needed, for a total dose of three tablets. Sublingual nitroglycerin appears in the bloodstream within 2 to 3 minutes and is metabolized within about 10 minutes.

18. Which action by a nurse who is giving fondaparinux (Arixtra) to a patient with a lower leg venous thromboembolism (VTE) indicates that more education about the drug is needed? a. The nurse avoids rubbing the injection site after giving the drug. b. The nurse injects the drug into the abdominal subcutaneous tissue. c. The nurse ejects the air bubble in the syringe before giving the drug. d. The nurse fails to assess the partial thromboplastin time (PTT) before giving the drug.

c. The nurse ejects the air bubble in the syringe before giving the drug.

The patient is asymptomatic but is diagnosed with second-degree heart block Mobitz I. The patient is on digitalis medication at home. The nurse should expect that: a. the patient has had an anterior wall myocardial infarction. b. the physician will order the digitalis to be continued in the hospital. c. a digitalis level would be ordered upon admission. d. the patient will require a transcutaneous pacemaker.

c. a digitalis level would be ordered upon admission.

The patient is admitted with sinus pauses causing periods of loss of consciousness. The patient is asymptomatic, awake and alert, but fatigued. He answers questions appropriately. When admitting this patient, the nurse should first: a. prepare the patient for temporary pacemaker insertion. b. prepare the patient for permanent pacemaker insertion. c. assess the patient's medication profile. d. apply transcutaneous pacemaker paddles.

c. assess the patient's medication profile.

19. A 23-year-old patient tells the health care provider about experiencing cold, numb fingers when running during the winter and Raynauds phenomenon is suspected. The nurse will anticipate teaching the patient about tests for a. hyperglycemia. b. hyperlipidemia. c. autoimmune disorders. d. coronary artery disease.

c. autoimmune disorders.

The patient has a permanent pacemaker in place with a demand rate set at 60 beats/min. The cardiac monitor is showing a heart rate of 44 beats/min with no pacemaker spikes. The nurse realizes this as: a. normal pacemaker function. b. failure to capture. c. failure to pace. d. failure to sense.

c. failure to pace.

The QT interval is the total time taken for ventricular depolarization and repolarization. Prolongation of the QT interval: a. decreases the risk of lethal dysrhythmias. b. usually occurs when heart rate increases. c. increases the risk of lethal dysrhythmias. d. can only be measured with irregular rhythms.

c. increases the risk of lethal dysrhythmias.

The nurse is caring for a patient with chronic constrictive pericarditis. Which assessment finding reflects a more serious complication of this condition? a. fatigue b. peripheral edema c. jugular venous distention d. thickened pericardium on echocardiography

c. jugular venous distention Rationale: Cardiac tamponade is a serious complication of pericarditis. As the compression of the heart increases, decreased left atrial filling decreases cardiac output. Neck veins usually are markedly distended because of jugular venous pressure elevation.

An RN is working with an LPN in caring for a group of patients on a cardiac telemetry unit. A patient with aortic stenosis has the nursing diagnosis of activity intolerance related to fatigue and exertional dyspnea. Which of these nursing activities could be delegated to the LPN? a. explain the reason for planning frequent periods of rest b. evaluate the patient's understanding of his disease process c. monitor BP, HR, RR, and SpO2 before, during, and after ambulation d. teach the patient which activities to choose that will gradually increase endurance

c. monitor BP, HR, RR, and SpO2 before, during, and after ambulation Rationale: Monitoring VS before and after ambulation is the collection of data. Instructions should be provided to the LPN regarding what changes in these should be reported to the RN. Other actions listed are RN responsibilities.

The nurse is caring for a patient newly admitted with heart failure secondary to dilated cardiomyopathy. Which of the following interventions would be a priority? a. encourage caregivers to learn CPR b. consider a consultation with hospice for palliative care c. monitor the patient's response to prescribed medications d. arrange for the patient to enter a cardiac rehabilitation program

c. monitor the patient's response to prescribed medications Rationale: Observing for signs and symptoms of worsening heart failure, dysrhythmias, and embolic formation in patients with dilated cardiomyopathy is essential, as is monitoring drug responsiveness. The goal of therapy is to keep the patient at an optimal level of functioning and out of the hospital. The priority intervention is to manage the acute symptoms with medications. The caregivers should learn cardiopulmonary resuscitation (CPR) before hospital discharge, and the patient may be referred to cardiac rehabilitation. Patients with dilated cardiomyopathy with progression to class IV stage D heart failure are candidates for palliative care.

A compensatory mechanism involved in HF that leads to inappropriate fluid retention and additional workload of the heart is: a. ventricular dilation b. ventricular hypertrophy c. neurohormonal response d. sympathetic nervous system activation

c. neurohormonal response Rationale: The following mechanisms in heart failure lead to inappropriate fluid retention and additional workload of the heart: activation of the renin-angiotensin-aldosterone system (RAAS) cascade and release of antidiuretic hormone from the posterior pituitary gland in response to low cerebral perfusion pressure that results from low cardiac output.

The nurse understands that in a third-degree AV block: a. every P wave is conducted to the ventricles. b. some P waves are conducted to the ventricles. c. none of the P waves are conducted to the ventricles. d. the PR interval is prolonged.

c. none of the P waves are conducted to the ventricles.

A patient admitted with ACS has continuous ECG monitoring. An examination of the rhythm strip reveals the following characteristics: atrial rate 74 beats/min and regular; ventricular rate 62 beats/min and irregular; P wave normal shape; PR interval lengthens progressively until a P wave is not conducted; QRS normal shape. The priority nursing intervention would be to a. perform synchronized cardioversion b. administer epinephrine 1 mg IV push c. observe for symptoms of hypotension or angina d. apply transcutaneous pacemaker pads on the patient

c. observe for symptoms of hypotension or angina Rationale: The rhythm is a second-degree atrioventricular (AV) block, type I (i.e., Mobitz I or Wenckebach heart block). The rhythm is characterized by a gradual lengthening of the PR interval. Type I AV block is usually a result of myocardial ischemia or infarction and typically is transient and well tolerated. The nurse should assess for bradycardia, hypotension, and angina. If the patient experiences symptoms, atropine or a temporary pacemaker may be needed.

A patient is scheduled for a percutaneous transluminal valvuloplasty. The nurse understands that this procedure is indicated for a. any patient with aortic regurgitation b. older patients with aortic regurgitation c. older patients with stenosis of any valve d. young adult patients with mild mitral valve stenosis

c. older patients with stenosis of any valve Rationale: This procedure has been used for repair of pulmonic, tricuspid, and mitral stenosis, but usually for those patients that are poor surgical risks.

7. The nurse performing an assessment with a patient who has chronic peripheral artery disease (PAD) of the legs and an ulcer on the right second toe would expect to find a. dilated superficial veins. b. swollen, dry, scaly ankles. c. prolonged capillary refill in all the toes. d. a serosanguineous drainage from the ulcer.

c. prolonged capillary refill in all the toes.

When obtaining a nursing history for a patient with myocarditis, the nurse specifically questions the patient about a. a history of CAD with or without an MI b. prior use of digoxin for treatment of cardiac problems c. recent symptoms of a viral illness, such as fever or malaise d. a recent streptococcal infection requiring treatment with penicillin

c. recent symptoms of a viral illness, such as fever or malaise Rationale: Viruses are the most common cause of myocarditis in the US, and early manifestations of myocarditis are often those of systemic viral infections. Myocarditis may also be associated with systemic inflammatory and metabolic disorders as well as with other microorganisms, drugs, or toxins. The heart has increased sensitivity to digoxin in myocarditis, and it is used very cautiously, if at all, in treatment of the condition.

A patient is scheduled for an open surgical valvuloplasty of the mitral valve. In preparing the patient for surgery, the nurse recognizes that a. cardiopulmonary bypass is not required with this procedure b. valve repair is a palliative measure, whereas valve replacement is curative c. the operative mortality rate is lower in valve repair than in valve replacement d. patients with valve repair do not require postoperative anticoagulation as they do with valve replacement

c. the operative mortality rate is lower in valve repair than in valve replacement Rationale: Repair of mitral or tricuspid valves has a lower operative mortality rate than does replacement and is becoming the surgical procedure of choice for these valvular diseases. Open repair is more precise than closed repair and requires cardiopulmonary bypass during surgery. All types of valve surgery are palliative, not curative, and patients require lifelong health care. Anticoagulation therapy is used for all valve surgery for at least some time postoperatively.

Important teaching for the patient scheduled for a radiofrequency catheter ablation procedure includes explaining that a. ventricular bradycardia may be induced and treated during the procedure b. a catheter will be placed in both femoral arteries to allow double-catheter use c. the procedure will destroy areas of the conduction system that are causing rapid heart rhythms d. a general anesthetic will be given to prevent the awareness of any "sudden cardiac death" experiences

c. the procedure will destroy areas of the conduction system that are causing rapid heart rhythms Rationale: Radiofrequency catheter ablation therapy involves the use of electrical energy to "burn" or ablate areas of the conduction system as definitive treatment of tachydysrhythmias.

A patient is admitted to the hospital with a suspected acute pericarditis. To establish the presence of a pericardial friction rub, the nurse listens to the patient's chest a. while timing the sound with the respiratory pattern b. with the bell of the stethoscope at the apex of the heart c. with the diaphragm of the stethoscope at the lower left sternal bored of the chest d. with the diaphragm of the stethoscope to auscultate a high-pitched continuous rumbling sound

c. with the diaphragm of the stethoscope at the lower left sternal bored of the chest Rationale: The stethoscope diaphragm at the left sternal border is the best method to use to hear the high-pitched, grating sound of a pericardial friction rub. The sound does not radiate widely and occurs with the heartbeat.

A 39-yr-old woman with a history of smoking and oral contraceptive use is admitted with a venous thromboembolism (VTE) and prescribed unfractionated heparin. What laboratory test should the nurse review to evaluate the expected effect of the heparin? a. Platelet count b. Activated clotting time (ACT) c. International normalized ratio (INR) d. Activated partial thromboplastin time (APTT)

d

A patient is about to undergo elective cardioversion. The nurse sets the defibrillator for synchronized mode so that the electrical shock is not delivered on the T wave. This is done to avoid which complication? a. Electrical burns to the skin b. Ventricular standstill c. Arcing from the electrodes d VF

d

A patient is diagnosed with recurrent SVT. What does the nurse do in order to accomplish the preferred treatment? a. Place the patient on the cardiac monitor and perform carotid massage b. Give oxygen and establish IV access for antidysrhythmic drugs c. Assist the provider in attempting atrial overdrive pacing d. Provide information about radiofrequency catheter ablation therapy.

d

A priority consideration in the management of the older adult with hypertension is to a. prevent primary hypertension from converting to secondary hypertension. b. recognize that the older adult is less likely to adhere to the drug therapy regimen than a younger adult. c. ensure that the patient receives larger initial doses of antihypertensive drugs because of impaired absorption. d. use careful technique in assessing the BP of the patient because of the possible presence of an auscultatory gap.

d

In planning care and patient teaching for the patient with venous leg ulcers, the nurse recognizes that the most important intervention in healing and control of this condition is a. discussing activity guidelines. b. using moist environment dressings. c. taking horse chestnut seed extract daily. d. applying graduated compression stockings.

d

In teaching a patient with hypertension about controlling the illness, the nurse recognizes that a. all patients with elevated BP require medication. b. obese persons must achieve a normal weight to lower BP. c. it is not necessary to limit salt in the diet if taking a diuretic. d. lifestyle modifications are indicated for all persons with elevated BP.

d

The blood pressure of a 71-year-old patient admitted with pneumonia is 160/70 mm Hg. What is an age-related change that contributes to this finding? a. Stenosis of the heart valves Incorrect b. Decreased adrenergic sensitivity c. Increased parasympathetic activity d. Loss of elasticity in arterial vessels

d

The nurse is assessing a patient's ECG rhythm strip and notes that occasionally the QRS complex is missing. How does the nurse interpret this finding? a. A junctional impulse b. A supraventricular impulse c. Ventricular tachycardia d. A dysrhythmia

d

The nurse is caring for several patients in the telemetry unit who are being remotely watched by a monitor technician. What is the nurse's primary responsibility in the monitoring process of these patients? a. Watching the bank of monitors on the unit b. Printing ECG rhythm strips routinely and as needed c. Interpreting rhythm d. Assessment and management

d

The nurse is notified y the telemetry monitor technician about a patient's heart rate. Which method does the nurse use to confirm the technician's report? a. Count QRS complexes in a 6-second strip and multiply by 10 b. Analyze an ECG rhythm strip by using an ECG cliper c. Run an ECG rhythm strip and use the memory emthod d. Assess the patient's heart rate directly by taking an apical pulse

d

The nurse is taking a history and vital signs on a patient who has come to the clinic for a routine checkup. The patient has a pulse rate of 50 beats/min, but denies any distress. What does the nurse do next? a. Give supplemental oxygen b. Establish IV access c. Complete the health history d. Check the blood pressure

d

The patient reports tenderness when she touches her leg over a vein. The nurse assesses warmth and a palpable cord in the area. The nurse knows the patient needs treatment to prevent which sequela? a. Pulmonary embolism b. Pulmonary hypertension c. Post-thrombotic syndrome d. Venous thromboembolism

d

The recommended treatment for an initial VTE in an otherwise healthy person with no significant co-morbidities would include a. IV argatroban (Acova) as an inpatient. b. IV unfractionated heparin as an inpatient. c. subcutaneous unfractionated heparin as an outpatient. d. subcutaneous low-molecular-weight heparin as an outpatient.

d

To help prevent embolization of the thrombus in a patient with a VTE, what should the nurse teach the patient to do? a. Dangle the feet over the edge of the bed q2-3hr. b. Ambulate around the bed three to four times a day. c. Keep the affected leg elevated above the level of the heart. d. Maintain bed rest until edema is relieved and anticoagulation is established

d

What is a priority nursing intervention in the care of a patient with a diagnosis of chronic venous insufficiency (CVI)? a. Application of topical antibiotics to venous ulcers b. Maintaining the patient's legs in a dependent position c. Administration of oral and/or subcutaneous anticoagulants d. Teaching the patient the correct use of compression stockings

d

What is the primary significance of ventricular tachycardia (VT) in a cardiac patient? a. It increases the ventricular filling time, therefore increaseing cardiac output b. It signals that the patient needs potassium supplement for replacement c. It warrants immediate initiation of cardiopulmonary resuscitation. d. It is commonly the initial rhythm before deterioration into ventricular fibrillation (VF)

d

What is the total time required for ventricular depolarization and repolarization as represented on the ECG? a. PR interval b. QRS complex c. ST segment d. QT interval

d

Which components measure ECG waveforms? a. BP and cardiac output (CO) b. Seconds and minutes c. Heart rate per minute d. Amplitude (voltage) and duration (time)

d

Which medication does an adult patient with VG or pulseless VT receive? a. Propranolol (Inderal) b. Adenosine (Adenocard) c. Diltiazem hydrochloride (Cardizem) d. Epinephrine (Adrenalin chloride)

d

In reviewing medication instructions with a patient being discharged on antihypertensive medications, which statement would be most appropriate for the nurse to make when discussing guanethidine (Ismelin)? a) "A fast heart rate is a side effect to watch for while taking guanethidine." b) "Stop the drug and notify your doctor if you experience any nausea or vomiting." c) "Because this drug may affect the lungs in large doses, it may also help your breathing." d) "Make position changes slowly, especially when rising from lying down to a standing position."

d) "Make position changes slowly, especially when rising from lying down to a standing position." Guanethidine is a peripheral-acting α-adrenergic antagonist and can cause marked orthostatic hypotension. For this reason, the patient should be instructed to rise slowly, especially when moving from a recumbent to a standing position. Support stockings may also be helpful. Tachycardia or lung effects are not evident with guanethidine.

The nurse monitors the patient receiving treatment for acute decompensated heart failure with the knowledge that marked hypotension is most likely to occur with the IV administration of which medication? a. Milrinone b. Furosemide c. Nitroglycerin d. Nitroprusside

d. Although all of these drugs may cause hypotension, nitroprusside is a potent dilator of both arteries and veins and may cause such marked hypotension that an inotropic agent (e.g., dobutamine) administration may be necessary to maintain the BP during its administration. Furosemide may cause hypotension because of diuretic-induced depletion of intravascular fluid volume. Milrinone has a positive inotropic effect in addition to peripheral vasodilation. Nitroglycerin is a vasodilator and can decrease BP but not as severely as nitroprusside. It primarily dilates veins and increases myocardial oxygen supply.

Which statement by the patient with chronic heart failure should cause the nurse to determine that additional discharge teaching is needed? a. "I will call my health clinic if I wake up breathless at night." b. "I will look for sodium content on labels of foods and over-the-counter medicines." c. "I plan to organize my household tasks so I don't have to constantly go up and down the stairs." d. "I should weigh myself every morning and go on a diet if I gain more than 2 or 3 pounds in 2 days."

d. Further teaching is needed if the patient believes a weight gain of 2 to 3 pounds in 2 days is an indication for dieting. In a patient with heart failure, this type of weight gain reflects fluid retention and is a sign of heart failure that should be reported to the HCP. The other options show patient understanding of the heart failure management teaching (see Table 34-10).

What does the nursing responsibility in the management of the patient with hypertensive urgency often include? a. Monitoring hourly urine output for drug effectiveness b. Titrating IV drug dosages based on BP measurements every 2 to 3 minutes c. Providing continuous electrocardiographic (ECG) monitoring to detect side effects of the drugs d. Instructing the patient to follow up with a health care professional within 24 hours after outpatient treatment

d. Hypertensive urgencies are often treated with oral drugs on an outpatient basis but it is important for the patient to be seen by a health care professional within 24 hours to evaluate the effectiveness of the treatment. Hourly urine measurements, titration of IV drugs, and ECG monitoring are indicated for hypertensive emergencies.

Which medication shows improvement in HF for African American patients for hypertension and angina? a. Captopril b. Nitroglycerin c. Spironolactone (Aldactone) d. Isosorbide dinitrate and hydralazine (BiDil)

d. Isosorbide dinitrate and hydralazine (BiDil) is recommended for use in African American patients with HF to treat hypertension and angina. Captopril is used for hypertension by all patients. Nitroglycerin is used with hydralazine for patients who cannot tolerate RAAS inhibitors (ACE inhibitors or ARBs) for heart failure management. Spironolactone (Aldactone) is used for hypertension.

The nurse assesses the patient with chronic biventricular heart failure for paroxysmal nocturnal dyspnea (PND) by questioning the patient regarding a. the presence of restlessness and confusion. b. frequent awakening to void during the night. c. the presence of swelling in dependent body areas. d. waking in a panic with a feeling of suffocation.

d. Paroxysmal nocturnal dyspnea (PND) is awakening from sleep with a feeling of suffocation and a need to sit up to be able to breathe. Patients learn that sleeping with the upper body elevated on several pillows helps to prevent PND. Behavior changes are seen in late stages of HF. Nocturia occurs with HF as fluid moves back into the vascular system during recumbency, increasing renal blood flow. Dependent edema does not indicate PND.

The evaluation team for cardiac transplantation is evaluating patients. Which patient is most likely to receive the most benefit from a new heart? a. A 24-year-old man with Down syndrome who has received excellent care from parents in their 60s b. A 46-year-old single woman with a limited support system who has alcohol-induced cardiomyopathy c. A 60-year-old man with inoperable coronary artery disease who has not been compliant with lifestyle changes and rehabilitation programs d. A 52-year-old woman with end-stage coronary artery disease who has limited financial resources but is emotionally stable and has strong social support

d. The 52-yr-old woman does not have any contraindications for cardiac transplantation, even though she lacks the indication of adequate financial resources. The 24-yr-old man does not have a current cardiac diagnosis. The postoperative transplant regimen is complex and rigorous, and patients who have not been compliant with other treatments or who may not have the means to understand the care would not be good candidates. A history of drug or alcohol abuse is usually a contraindication to heart transplant.

The nurse is administering a dose of digoxin (Lanoxin) to a patient with heart failure (HF). The nurse would become concerned with the possibility of digitalis toxicity if the patient reported which symptom(s)? a. Muscle aches b. Constipation c. Pounding headache d. Anorexia and nausea

d. Anorexia and nausea Anorexia, nausea, vomiting, blurred or yellow vision, and cardiac dysrhythmias are all signs of digitalis toxicity. The nurse would become concerned and notify the health care provider if the patient exhibited any of these symptoms.

A stable patient with acute decompensated heart failure (ADHF) suddenly becomes dyspneic. Before positioning the patient on the bedside, what should the nurse assess first? a. Urine output b. Heart rhythm c. Breath sounds d. Blood pressure

d. Blood pressure The nurse should evaluate the blood pressure before dangling the patient on the bedside because the blood pressure can decrease as blood pools in the periphery and preload decreases. If the patient's blood pressure is low or marginal, the nurse should put the patient in the semi-Fowler's position and use other measures to improve gas exchange.

Beyond the first year after a heart transplant, the nurse knows that what is a major cause of death? a. Infection b. Acute rejection c. Immunosuppression d. Cardiac vasculopathy

d. Cardiac vasculopathy Beyond the first year after a heart transplant, malignancy (especially lymphoma) and cardiac vasculopathy (accelerated CAD) are the major causes of death. During the first year after transplant, infection and acute rejection are the major causes of death. Immunosuppressive therapy will be used for posttransplant management to prevent rejection and increases the patient's risk of an infection.

32. A patient is being evaluated for post-thrombotic syndrome. Which assessment will the nurse perform? a. Ask about leg pain with exercise. b. Determine the ankle-brachial index. c. Assess capillary refill in the patients toes. d. Check for presence of lipodermatosclerosis.

d. Check for presence of lipodermatosclerosis.

A male patient with a long-standing history of heart failure has recently qualified for hospice care. What measure should the nurse now prioritize when providing care for this patient? a. Taper the patient off his current medications. b. Continue education for the patient and his family. c. Pursue experimental therapies or surgical options. d. Choose interventions to promote comfort and prevent suffering.

d. Choose interventions to promote comfort and prevent suffering. The central focus of hospice care is the promotion of comfort and the prevention of suffering. Patient education should continue, but providing comfort is paramount. Medications should be continued unless they are not tolerated. Experimental therapies and surgeries are not commonly used in the care of hospice patients.

35. The nurse reviews the admission orders shown in the accompanying figure for a patient newly diagnosed with peripheral artery disease. Which admission order should the nurse question? a. Use of treadmill for exercise b. Referral for dietary instruction c. Exercising to the point of discomfort d. Combined clopidogrel and omeprazole therapy

d. Combined clopidogrel and omeprazole therapy

31] Crackles heard on lung auscultation indicate which of the following? a. Cyanosis b. Bronchospasm c. Airway narrowing d. Fluid-filled alveoli

d. Crackles are auscultated over fluid-filled alveoli. Crackles heard on lung auscultation do not have to be associated with cyanosis. Bronchospasm and airway narrowing generally are associated with wheezing sounds.

30. Which nursing intervention for a patient who had an open repair of an abdominal aortic aneurysm 2 days previously is appropriate for the nurse to delegate to unlicensed assistive personnel (UAP)? a. Monitor the quality and presence of the pedal pulses. b. Teach the patient the signs of possible wound infection. c. Check the lower extremities for strength and movement. d. Help the patient to use a pillow to splint while coughing.

d. Help the patient to use a pillow to splint while coughing.

20. While working in the outpatient clinic, the nurse notes that a patient has a history of intermittent claudication. Which statement by the patient would support this information? a. When I stand too long, my feet start to swell. b. I get short of breath when I climb a lot of stairs. c. My fingers hurt when I go outside in cold weather. d. My legs cramp whenever I walk more than a block.

d. My legs cramp whenever I walk more than a block.

10. The health care provider has prescribed bed rest with the feet elevated for a patient admitted to the hospital with venous thromboembolism. Which action by the nurse to elevate the patients feet is best? a. The patient is placed in the Trendelenburg position. b. Two pillows are positioned under the affected leg. c. The bed is elevated at the knee and pillows are placed under the feet. d. One pillow is placed under the thighs and two pillows are placed under the lower legs.

d. One pillow is placed under the thighs and two pillows are placed under the lower legs.

23] The nurse is assessing clients at a health fair. Which client is at greatest risk for coronary artery disease? a. a 32-year-old female with mitral valve prolapse who quit smoking 10 years ago. b. a 43-year-old male with a family history of CAD and cholesterol level of 158 c. A 56-year-old male with an HDL of 60 who takes atorvastatin (Lipitor) d. A 65-year-old female who is obese with an LDL of 188

d. The woman who is 65-years-old, over weight and has an elevated LDL is at greatest risk. Total cholesterol >200, LDL >100, HDL <40 in men, HDL <50 in women, men 45-years and older, women 55-years and older, smoking and obesity increase the risk of CAD. Atorvastatin is a medication to reduce LDL and decrease risk of CAD. The combination of postmenopausal, obesity and high LDL cholesterol places this client at greatest risk.

A patient hospitalized for 1 week with subacute infective endocarditis is afebrile and has no signs of heart damage. Discharge with outpatient antibiotic therapy is planned. During discharge planning with the patient, it is most important for the nurse to a. plan how his needs will be met while he continues on bed rest b. teach the patient to avoid crowds and exposure to upper respiratory infections c. encourage the use of diversional activities to relieve boredom and restlessness d. assess the patient's home environment in terms of family assistance and hospital access

d. assess the patient's home environment in terms of family assistance and hospital access Rationale: The patient with outpatient antibiotic therapy requires vigilant home nursing care, and it is most important to determine the adequacy of the home environment for successful management of the patient. The patient is at risk for life-threatening complications, such as embolization and pulmonary edema, and must be able to access a hospital if needed. Bed rest will not be necessary for the patient without heart damage. Avoiding infections and planning diversional activities are indicated for the patient but are not the most important step while he is on outpatient antibiotic therapy.

The ECG monitor of a patient in the cardiac care unit after an MI indicates ventricular bigeminy with a rate of 50 beats/min. The nurse would anticipate a. performing defibrillation b. treating with IV amiodarone c. inserting a temporary transvenous pacemaker d. assessing the patient's response to the dysrhythmia

d. assessing the patient's response to the dysrhythmia Rationale: A premature ventricular contraction (PVC) is a contraction originating in an ectopic focus in the ventricles. When every other beat is a PVC, the rhythm is called ventricular bigeminy. PVCs are usually a benign finding in patients with a normal heart. In patients with heart disease, PVCs may reduce the cardiac output and precipitate angina and heart failure, depending on the frequency. Because PVCs in coronary artery disease (CAD) or acute myocardial infarction indicate ventricular irritability, the patient's physiologic response to PVCs must be monitored. Assessment of the patient's hemodynamic status is important for determining whether treatment with drug therapy is needed.

3. Several hours after an open surgical repair of an abdominal aortic aneurysm, the UAP reports to the nurse that urinary output for the past 2 hours has been 40 mL. The nurse notifies the health care provider and anticipates an order for a(n) a. hemoglobin count. b. additional antibiotic. c. decrease in IV infusion rate. d. blood urea nitrogen (BUN) level.

d. blood urea nitrogen (BUN) level.

The patient is admitted with a condition that requires cardiac rhythm monitoring. To apply the monitoring electrodes, the nurse must first: a. apply a moist gel to the chest. b. make certain that the electrode gel is dry. c. avoid soaps to avoid skin irritation. d. clip chest hair if needed.

d. clip chest hair if needed.

The patient's heart rate is 165 beats per minute. His cardiac monitor shows a rapid rate with narrow QRS complexes. The P waves cannot be seen, but the rhythm is regular. The patient's blood pressure has dropped from 124/62 to 78/30. His skin is cold and diaphoretic and he is complaining of nausea. The nurse prepares the patient for: a. administration of beta-blockers. b. administration of atropine. c. transcutaneous pacemaker insertion. d. emergent cardioversion.

d. emergent cardioversion.

5. A 73-year-old patient with chronic atrial fibrillation develops sudden severe pain, pulselessness, pallor, and coolness in the right leg. The nurse should notify the health care provider and immediately a. apply a compression stocking to the leg. b. elevate the leg above the level of the heart. c. assist the patient in gently exercising the leg. d. keep the patient in bed in the supine position.

d. keep the patient in bed in the supine position.

In teaching a patient with hypertension about controlling the condition, the nurse recognizes that: a. all patients with elevated BP require medication b. obese persons must achieve a normal weight to lower BP c. It is not necessary to limit salt in the diet if taking a diuretic d. lifestyle modifications are indicated for all persons with elevated BP

d. lifestyle modifications are indicated for all persons with elevated BP

When teaching a patient with endocarditis how to prevent recurrence of the infection, the nurse instructs the patient to a. start on antibiotic therapy when exposed to persons with infections b. take one aspirin a day to prevent vegetative lesions from forming around the valves c. always maintain continuous antibiotic therapy to prevent the development of any systemic infection d. obtain prophylactic antibiotic therapy before certain invasive medical or dental procedures (e.g. dental cleaning)

d. obtain prophylactic antibiotic therapy before certain invasive medical or dental procedures (e.g. dental cleaning) Rationale: Prophylactic antibiotic therapy should be initiated before invasive dental, medical, or surgical procedures to prevent recurrence of endocarditis. Continuous antibiotic therapy is indicated only in patients with implanted devices or ongoing invasive procedures. Symptoms of infection should be treated promptly, but antibiotics are not used for exposure to infection.

Priority nursing management for a patient with myocarditis includes interventions related to a. meticulous skin care b. antibiotic prophylaxis c. tight glycemic control d. oxygenation and ventilation

d. oxygenation and ventilation Rationale: General supportive measures for management of myocarditis include interventions to improve ventilation and oxygenation (i.e., oxygen therapy, bed rest, and restricted activity).

A patient with acute pericarditis has a nursing diagnosis of pain related to pericardial inflammation. An appropriate nursing intervention for the patient is a. administering opioids as prescribed on an around the clock schedule b. promoting progressive relaxation exercises with the use of deep, slow breathing c. positioning the patient on the right side with the head of the bed elevated 15 degrees d. positioning the patient in Fowler's position with a padded over the bed table for the patient to lean on

d. positioning the patient in Fowler's position with a padded over the bed table for the patient to lean on Rationale: Relief from pericardial pain is often obtained by sitting up and leaning forward. Pain is increased by lying flat. The pain has a sharp, pleuritic quality that changes with respiration, and patients take shallow breaths. Anti-inflammatory medications may also be used to help control pain, but opioids are not usually indicated.

The most important role of the nurse in preventing rheumatic fever is to a. teach patients with infective endocarditis to adhere to antibiotic prophylaxis b. identify patients with valvular heart disease who are at risk for rheumatic fever c. encourage the use of antibiotics for treatment of all infections involving a sore throat d. promote the early diagnosis and immediate treatment of group A streptococcal pharyngitis

d. promote the early diagnosis and immediate treatment of group A streptococcal pharyngitis Rationale: Initial attacks of rheumatic fever and the development of rheumatic heart disease can be prevented by adequate treatment of group A streptococcal pharyngitis. Because streptococcal infection accounts for only about 20% of acute pharyngitis, cultures should be done to identify the organism and direct antibiotic therapy. Viral infections should not be treated with antibiotics. Prophylactic therapy is indicated in those who have valvular heart disease or have has rheumatic heart disease.

Which clinical finding would most likely indicate decreased cardiac output in a patient with aortic valve regurgitation? a. reduction in peripheral edema and weights b. carotid venous distention and new-onset atrial fibrillation c. significant pulses paradoxus and diminished peripheral pulses d. shortness of breath on minimal exertion and a diastolic murmur

d. shortness of breath on minimal exertion and a diastolic murmur Rationale: Clinical manifestations of aortic regurgitation (AR) that indicate decreased cardiac output include severe dyspnea, chest pain, and hypotension. Other manifestations of chronic AR include water-hammer pulse (i.e., a strong, quick beat that collapses immediately), soft or absent S1, presence of S3 or S4, and soft, high-pitched diastolic murmur. A low-pitched diastolic murmur may be heard in severe AR. Early manifestations may include exertional dyspnea, orthopnea, and paroxysmal nocturnal dyspnea.

When performing discharge teaching for the patient following a mechanical valve replacement, the nurse determines that further instruction is needed when the patient says, a. I may begin an exercise program to gradually increase my cardiac tolerance b. I will always need to have my blood checked once a month for its clotting function c. I should wear a medic alert bracelet to identify my valve and anticoagulant therapy d. the biggest risk I have during invasive health procedures is bleeding because of my anticoagulants

d. the biggest risk I have during invasive health procedures is bleeding because of my anticoagulants Rationale: The greatest risk to a patient who has an artificial valve is the development of endocarditis with invasive medical or dental procedures; before any of these procedures, antibiotic prophylaxis is necessary to prevent infection. Health care providers must be informed of the presence of the valve and the anticoagulation therapy, but the most important factor is using antibiotic prophylaxis before invasive procedures.

A major consideration in the management of the other adult with hypertension is to: a. prevent primary hypertension from converting to secondary hypertension b. recognize that the older adult is less likely to adhere to the drug therapy regimen than a younger adult c. ensure that the patient receives larger initial doses of antihypertensive drugs because of impaired absorption d. use careful technique in assessing the BP of the patient because of the possible presence of ab auscultatory gap

d. use careful technique in assessing the BP of the patient because of the possible presence of ab auscultatory gap

The nurse includes the definition of HF in the teaching plan for the client. An accurate description of the mechanism of HF is that [Hint] there is too much fluid in the heart. the heart cannot get oxygen. the heart is unable to pump effectively. there is too much fluid in the lungs.

the heart is unable to pump effectively.

___________ ___________ is another name for distributive shock.

vascular shock

____________ _____________ ___________ are mechanical pumps placed within the thorax to support ventricular pumping.

ventricular assist devices

The nurse is doing discharge teaching with the patient and spouse of the patient who just received an implantable cardioverter-defibrillator (ICD) in the left side. Which statement by the patient indicates to the nurse that the patient needs more teaching? "I will call the cardiologist if my ICD fires." "I cannot fly because it will damage the ICD." "I cannot move my left arm until it is approved." "I cannot drive until my cardiologist says it is okay."

"I cannot fly because it will damage the ICD." The patient statement that flying will damage the ICD indicates misunderstanding about flying. The patient should be taught that informing TSA about the ICD can be done because it may set off the metal detector and if a hand-held screening wand is used, it should not be placed directly over the ICD. The other options indicate the patient understands the teaching.

At a clinic visit, the nurse provides dietary teaching for a patient recently hospitalized with an exacerbation of chronic heart failure. The nurse determines that teaching is successful if the patient makes which statement? "I will limit the amount of milk and cheese in my diet." "I can add salt when cooking foods but not at the table." "I will take an extra diuretic pill when I eat a lot of salt." "I can have unlimited amounts of foods labeled as reduced sodium."

"I will limit the amount of milk and cheese in my diet." Milk products should be limited to 2 cups per day for a 2500-mg sodium-restricted diet. Salt should not be added during food preparation or at the table. Diuretics should be taken as prescribed (usually daily) and not based on sodium intake. Foods labeled as reduced sodium contain at least 25% less sodium than regular.

At a clinic visit, the nurse provides dietary teaching for a patient recently hospitalized with an exacerbation of chronic heart failure. The nurse determines that teaching is successful if the patient makes which statement? "I will limit the amount of milk and cheese in my diet." "I can add salt when cooking foods but not at the table." "I will take an extra diuretic pill when I eat a lot of salt." "I can have unlimited amounts of foods labeled as reduced sodium."

"I will limit the amount of milk and cheese in my diet." Milk products should be limited to 2 cups per day for a 2500-mg sodium-restricted diet. Salt should not be added during food preparation or at the table. Diuretics should be taken as prescribed (usually daily) and not based on sodium intake. Foods labeled as reduced sodium contain at least 25% less sodium than regular.

A nurse is performing client health education with a 68-year-old man who has recently been diagnosed with heart failure. Which of the following statements demonstrates an accurate understanding of his new diagnosis?

"I'm trying to think of ways that I can cut down the amount of salt that I usually eat."

A client with heart failure asks, "Why am I taking a 'water pill9 when it's my heart that is having a problem?" While educating the client about the Frank-Starling mechanism, which of the following explanations is most appropriate to share?

"Since your heart is not pumping efficiently, the kidneys are getting less blood flow; therefore, the kidneys are holding on to sodium and water."

A patient in the nursing home wonders why he is having these signs and symptoms of left-sided failure. Which of the following explanations will the nurse give the patient?

"The left ventricle is having problems pumping blood forward, and this is causing blood to back up into your lungs."

A client awaiting a heart transplant is experiencing decompensation of her left ventricle that will not respond to medications. The physicians suggest placing the client on a ventricular assist device (VAD). The client asks what this equipment will do. The health care providers respond:

"This device will decrease the workload of the myocardium while maintaining cardiac output and systemic arterial pressure."

***

***

What are the physiologic signs and symptoms of cardiogenic shock? (Select all that apply.)

- Rise in central venous pressure (CVP) - Increased extraction of O2 from hemoglobin - Decrease in mean arterial blood pressures

The sympathetic (adrenergic) nervous system is an important compensatory mechanism in heart failure. Which of the following statements regarding the sympathetic nervous system response to heart failure are correct? Select all that apply.

- The sympathetic nervous system responds rapidly to a fall in cardiac output. - The sympathetic nervous system increase in cardiac workload and oxygen use can cause ischemia and worsening of heart failure. - The sympathetic nervous system increase in heart rate and force of contraction support cardiac output.

A 75-year-old patient is being evaluated for heart failure. Which questions will the nurse ask to confirm common signs and symptoms observed in older adults experiencing heart failure? Select all that apply.

-"Do you easily get tired?" -"Do you have swelling in your ankles?" -"Are you feeling depressed?" -"Do you get up often during the night to urinate?"

enalapril (Vasotec)

-ACE inhibitor -administer 1 hour ac or 2 hours pc -empty stomach enhances absorption

Circulatory shock is characterized by an inability of the circulatory system to provide adequate oxygen to body tissues. Which of the following damaging effects at the cellular level can cause hypoxia? Select all that apply.

-Pyruvate converted to lactic acid -Cellular edema -Deranged sodium/potassium balance -Impaired cellular production of adenosine triphosphate (ATP)

loop diuretics

-furosemide (Lasix), bumetanide (Bumex) -give in the early AM -assess DW, I&O, breath sounds, edema -monitor K+, Na+ -monitor for signs of dehydration -note ringing in ears -urine increases 6-8 hours after oral dose -weight is the most accurate noninvasive measurement of volume status

for the patient and in counseling other family members. The patient should be counseled against the use of stimulant drugs, but the limited past history indicates that the patient is not at current risk for cocaine use. Viral infections and CAD are risk factors for dilated cardiomyopathy, but not for HC.

...

The nurse is caring for a patient newly diagnosed with heart failure. The patient is to receive a first dose of digoxin 0.125 mg IV push. An ampule containing 0.25 mg/mL is available. Calculate the number of milliliters the nurse should draw up to administer this dose. ______ mL

0.5

The nurse is caring for a patient newly diagnosed with heart failure. The patient is to receive a first dose of digoxin 0.125 mg IV push. An ampule containing 0.25 mg/mL is available. Calculate the number of milliliters the nurse should draw up to administer this dose. ______mL

0.5

Anaphylactic shock is most frequently due to a Type______Correct Response(50 %)hypersensitivity reaction, which involves______Correct Response(50 %)antibodies.

1 IgE

A nurse provides education to a hypertensive patient related to lifestyle modifications to reduce cardiovascular risks associated with high blood pressure (BP). Which statement made by the patient indicates effective learning? Select all that apply. 1. "I should achieve and maintain a healthy weight." 2. "I can continue to smoke, because nicotine does not affect blood pressure." 3. "I should exercise for at least 30 minutes daily." 4. "I can have up to five alcoholic drinks per day." 5. "I should restrict my salt intake to less than 1500 mg/day."

1. "I should achieve and maintain a healthy weight." 3. "I should exercise for at least 30 minutes daily." 5. "I should restrict my salt intake to less than 1500 mg/day." Lifestyle modifications play a vital role in reducing blood pressure and cardiovascular risk. Overweight people are at higher risk of cardiovascular disease. A weight loss of 22 lb may decrease systolic blood pressure by approximately 5 to 20 mm Hg. Being physically active is essential to maintain good health. It decreases the cardiovascular risk of hypertension. Sodium reduction helps to control blood pressure. A hypertensive patient should lower salt intake to 1500 mg/day. The nicotine in tobacco causes vasoconstriction and increases blood pressure. Therefore, smokers who are hypertensive should stop smoking. Excessive alcohol consumption increases the risk of hypertension. Consuming three or more drinks per day increases the risk of cardiovascular disease and stroke. Text Reference - p. 715

A patient is diagnosed with primary hypertension and asks the nurse what caused this condition. Which is the best response by the nurse? 1. "There is no one identifiable reason." 2. "Kidney disease is the most common reason." 3. "It is caused by a decrease in plasma renin levels." 4. "There is too much plaque in the blood vessels."

1. "There is no one identifiable reason." There is not one exact cause of primary hypertension; there are several contributing factors. Renal or kidney disease is a cause of secondary hypertension. An increase, not a decrease, in plasma renin levels is a contributing factor in the development of primary hypertension. Hypertension speeds up the process of atherosclerosis in the peripheral blood vessels. Text Reference - p. 712

A nurse is monitoring the digoxin level for a client who has been taking a daily dose of digoxin for 1 month. the digoxin level is 0.25 ng/mL. The nurse should notify the provider and anticipate which of the following: 1. An increase in the client's digoxin dose. 2. A decrease in the client's digoxin dose. 3. No change in the client's digoxin dose. 4. Dicontinuation of the client's digoxin dose.

1. An increase in the client's digoxin dose.

The nurse providing dietary instruction to a patient with hypertension would advise the patient to cut down on the intake of which foods? Select all that apply. 1. Canned vegetables 2. Red meat 3. Baked chicken 4. Canned fruit 5. Processed cheeses

1. Canned vegetables 2. Red meat 5. Processed cheeses Foods high in fat and sodium—including canned vegetables, red meat, and processed cheeses—should be avoided by the patient with hypertension. Baked chicken and canned fruit are low in sodium and fat. Text Reference - p. 716

A patient has a prescription for nadolol 50 mg by mouth (PO) daily. The nurse questions the prescription after noting which medical diagnosis in the patient's health record? 1. Chronic obstructive pulmonary disease (COPD) 2. Renal insufficiency 3. Diabetes mellitus 4. Hypertension

1. Chronic obstructive pulmonary disease (COPD) Nadolol is a nonselective β1-adrenergic-blocking agent that reduces blood pressure and could affect the β2 receptors in the lungs with larger doses or with drug accumulation. It should be used cautiously in patients with COPD, because it could trigger bronchospasm, a potentially life-threatening adverse effect. Nadolol will not worsen renal insufficiency and diabetes and will treat, not worsen, hypertension. Text Reference - p. 718

A nurse is monitoring the blood pressure (BP) of a patient visiting the health care facility. What should the nurse ensure when recording the BP? Select all that apply. 1. Ensure the patient has not exercised within 30 minutes. 2. Seat the patient and begin measurement. 3. Support the patient's arm at heart level. 4. Palpate the radial pulse for auscultatory measurement. 5. Deflate the cuff at the rate of 5 mm Hg/sec.

1. Ensure the patient has not exercised within 30 minutes. 3. Support the patient's arm at heart level. 4. Palpate the radial pulse for auscultatory measurement. The nurse should ensure that the patient has not exercised, smoked, or ingested caffeine within 30 minutes before measurement. The patient's arm should be supported at heart level. The radial pulse is palpated for auscultatory measurement. The nurse should begin measurement only after the patient has rested patiently for 5 minutes after sitting. The cuff should be deflated at a rate of 2 to 3 mm Hg/sec. Text Reference - p. 723

Lisinopril (Prinivil) is part of the treatment regimen for a client with HF. The nurse monitors the client for the development of which of teh following adverse effects of this drug? SELECT ALL THAT APPLY 1. Hyperkalemia 2.Hypocalcemia 3. Cough 4. Dizziness 5. Heartburn

1. Hyperkalemia 2.Hypocalcemia 3. Cough 4. Dizziness

A patient arrives at a medical clinic for a check-up. The patient's blood pressure (BP) is 150/94 mm Hg. All other assessment findings are within normal limits. The nurse reviews the patient's file from previous visits, and there is no history of elevated blood pressure. What could be the reason for a falsely high blood pressure? 1. The blood pressure cuff might have been too small. 2. There may be atherosclerosis in the subclavian artery. 3. The patient may have smoked the day before the BP measurement. 4. The patient may have engaged in strenuous exercises the day before the BP measurement.

1. The blood pressure cuff might have been too small. BP measurements should be performed using proper technique to get an accurate reading. BP measurements may be falsely high if the BP cuff is too small as it puts undue pressure on the artery. If the subclavian artery has atherosclerosis, the BP measurement would be falsely low. Smoking and engaging in strenuous exercise should be avoided 30 minutes before the BP measurement, because they can alter the measurement. Smoking or engaging in strenuous exercise one day before a BP measurement will not affect the readings. Text Reference - p. 723

10. The nurse is caring for a patient newly admitted with heart failure secondary to dilated cardiomyopathy. Which intervention would be a priority? a. Encourage caregivers to learn CPR. b. Consider a consultation with hospice for palliative care. c. Monitor the patient's response to prescribed medications. d. Arrange for the patient to enter a cardiac rehabilitation program.

10. Correct answer: c Rationale: Observing for signs and symptoms of worsening heart failure, dysrhythmias, and embolus formation in patients with dilated cardiomyopathy is essential, as is monitoring drug responsiveness. The goal of therapy is to keep the patient at an optimal level of functioning and out of the hospital. The priority intervention is to manage the acute symptoms with medications. The caregivers should learn cardiopulmonary resuscitation (CPR) before hospital discharge, and the patient may be referred to cardiac rehabilitation. Patients with dilated cardiomyopathy with progression to class IV stage D heart failure are candidates for palliative care.

10. After defibrillation, the advanced cardiac life support (ACLS) nurse says that the patient has pulseless electrical activity (PEA). What is most important for the nurse to understand about this rhythm? a. The heart rate is 40 to 60 bpm. b. Hypoxemia and hypervolemia are common with PEA. c. There is dissociated activity of the ventricle and atrium. d. There is electrical activity with no mechanical response.

10. d. Pulseless electrical activity (PEA) occurs when there is electrical activity on the ECG but no mechanical activity on assessment and therefore no heart rate. PEA is the most common dysrhythmia seen after defibrillation and may be caused by hypovolemia, hypoxia, metabolic acidosis, altered potassium level, hypoglycemia, hypothermia, toxins, cardiac tamponade, thrombosis, tension pneumothorax, and trauma. Dissociated atria and ventricles is third-degree AV block.

When computing a heart rate from the ECG tracing, the nurse counts 15 of the small blocks between the R waves of a patient whose rhythm is regular. From these data, the nurse calculates the patient's heart rate to be 60 beats/min. 75 beats/min. 100 beats/min. 150 beats/min.

100 beats/min. Since each small block on the ECG paper represents 0.04 seconds, 1500 of these blocks represents 1 minute. By dividing the number of small blocks (15, in this case) into 1500, the nurse can calculate the heart rate in a patient whose rhythm is regular (in this case, 100).

12. A patient with an acute myocardial infarction (MI) develops the following ECG pattern: atrial rate of 82 and regular; ventricular rate of 46 and regular; P wave and QRS complex are normal but there is no relationship between the P wave and the QRS complex. What dysrhythmia does the nurse identify this as and what treatment is expected? a. Sinus bradycardia treated with atropine b. Third-degree heart block treated with a pacemaker c. Atrial fibrillation treated with electrical cardioversion d. Type I second-degree AV block treated with observation

12. b. Third-degree or complete heart block is recognized with the atrial and ventricular dissociation and treated with a pacemaker. Sinus bradycardia does not have atrial and ventricular dissociation. Atrial fibrillation does not have normal P waves, as they are stimulated by ectopic foci. In type 1 second-degree AV heart block the P-R interval gradually lengthens and a QRS complex is dropped. Then the cycle begins again.

13. Which rhythm abnormality has an increased risk of ventricular tachycardia and ventricular fibrillation? a. PAC b. PVC on the T wave c. Accelerated idioventricular rhythm d. Premature ventricular contraction (PVC) couplet

13. b. When premature ventricular contraction (PVC) falls on the T wave of the preceding beat, R-on-T phenomenon occurs. Because the ventricle is repolarizing and there is increased excitability of cardiac cells, there is an increased risk of ventricular tachycardia or ventricular fibrillation. The other options do not increase this risk.

14. A patient with an acute MI has sinus tachycardia of 126 bpm. The nurse recognizes that if this dysrhythmia is not treated, the patient is likely to experience a. hypertension. b. escape rhythms. c. ventricular tachycardia. d. an increase in infarct size.

14. d. Although many factors can cause a sinus tachycardia, in the patient who has had an acute MI, tachycardia increases myocardial oxygen need in a heart that already has impaired circulation and may lead to increasing angina and further ischemia and necrosis.

17. In the patient with a dysrhythmia, which assessment indicates decreased cardiac output (CO)? a. Hypertension and bradycardia b. Chest pain and decreased mentation c. Abdominal distention and hepatomegaly d. Bounding pulses and a ventricular heave

17. b. Symptoms of decreased cardiac output (CO) related to cardiac dysrhythmias include a sudden drop in BP and symptoms of hypoxemia, such as decreased mentation, chest pain, and dyspnea. Peripheral pulses are weak and the HR may be increased or decreased, depending on the type of dysrhythmia present.

18. Priority Decision: A patient with an acute MI is having multifocal PVCs and ventricular couplets. He is alert and has a BP of 118/78 mm Hg with an irregular pulse of 86 bpm. What is the priority nursing action at this time? a. Continue to assess the patient. b. Ask the patient to perform Valsalva maneuver. c. Prepare to administer antidysrhythmic drugs per protocol. d. Be prepared to administer cardiopulmonary resuscitation (CPR).

18. c. Multifocal PVCs in a patient with an MI indicate significant ventricular irritability that may lead to ventricular tachycardia or ventricular fibrillation. Antidysrhythmics, such as β-adrenergic blockers, procainamide, amiodarone, or lidocaine, may be used to control the dysrhythmias. Valsalva maneuver may be used to treat paroxysmal supraventricular tachycardia. The nurse must always be ready to perform cardiopulmonary resuscitation (CPR).

In reviewing medication instructions with a patient prescribed lisinopril, the nurse should include which statement? 1. "You should not take this medication if you have asthma." 2. "You may develop a dry cough while taking this medication." 3. "Never take this medication on an empty stomach." 4. "Discontinue use of this medication if you develop a drop in your blood pressure."

2. "You may develop a dry cough while taking this medication." Lisinopril is an ACE-inhibitor. A common side effect is a dry cough. This medication is safe for use with asthma, can be taken on an empty stomach, and should not be discontinued unless instructed to do so by a health care provider. Text Reference - p. 719

The nurse records normal blood pressure (BP) for a patient with a family history of hypertension and diabetes. What should the nurse teach the patient to specifically address the risks of hypertension? Select all that apply. 1. Increase caloric intake. 2. Avoid foods high in sodium. 3. Reduce the use of tobacco products. 4. Take brisk walks. 5. Avoid overexertion with muscle-strengthening activities

2. Avoid foods high in sodium. 4. Take brisk walks. The nurse should teach the patient to adopt lifestyle changes, such as avoiding foods high in sodium and taking brisk walks. A decrease in caloric intake helps to reduce weight and prevent hypertension. The patient should completely avoid use of tobacco products, because the nicotine contained in tobacco causes vasoconstriction and increases BP. All adults should perform muscle-strengthening activities to maintain and increase endurance and strength of muscles. Text Reference - p. 716

The nurse is caring for a patient admitted with a history of hypertension. The patient's medication history includes hydrochlorothiazide daily for the past 10 years. Which parameter would indicate the optimal intended effect of this drug therapy? 1. Weight loss of 2 lb 2. Blood pressure 128/86 3. Absence of ankle edema 4. Output of 600 mL per eight hours

2. Blood pressure 128/86 Hydrochlorothiazide may be used alone as monotherapy to manage hypertension or in combination with other medications if not effective alone. After the first few weeks of therapy, the diuretic effect diminishes, but the antihypertensive effect remains. Because the patient has been taking this medication for 10 years, the most direct measurement of its intended effect would be the blood pressure. Text Reference - p. 717

2. The nurse is caring for a patient with chronic constrictive pericarditis. Which assessment finding reflects a more serious complication of this condition? a. Fatigue b. Peripheral edema c. Jugular venous distention d. Thickened pericardium on echocardiography

2. Correct answer: c Rationale: Cardiac tamponade is a serious complication of pericarditis. As the compression of the heart increases, decreased left atrial filling results in decreased cardiac output. Neck veins usually are markedly distended as a result of jugular venous pressure elevation related to compression of the right side of the heart. Heart sounds become muffled secondary to sound distortion by the fluid causing compression of the heart.

A nurse is administering a dopamine infusion at a moderate dose to a client who has severe HF. Which of the following is an expected effect? 1. Lowered heart rate 2. Increased myocardial contractility 3. Decreased conduction through the AV node D. Vasoconstriction of the renal blood vessels

2. Increased myocardial contractility -- thus increasing CO

A nurse is caring for a patient admitted to the health care facility with acute ischemic stroke. The patient is receiving IV antihypertensive drugs. Which interventions should the nurse perform for this patient? Select all that apply. 1. Assess blood pressure (BP) and pulse every 30 minutes. 2. Titrate drug according to mean arterial pressure (MAP) or BP as prescribed. 3. Measure hourly urine output. 4. Provide assistance to get up as patient desires. 5. Perform frequent neurologic checks.

2. Titrate drug according to mean arterial pressure (MAP) or BP as prescribed. 3. Measure hourly urine output. 5. Perform frequent neurologic checks. Drugs should be titrated according to MAP or BP as prescribed. The nurse should measure hourly urine output to assess renal perfusion and should perform frequent neurologic checks. Antihypertensive IV drugs have a rapid onset of action; hence BP and pulse should be assessed EVERY 2-3 MINUTES using a noninvasive BP machine. The patient should be restricted to bed; severe cerebral ischemia or fainting may result if the patient tries to get up. Text Reference - p. 272

A nurse is caring for an older adult client who has a new prescription for digoxin and takes multiple other medications. concurrent use of which of the following medications places the client at risk for dig toxicity? 1. Phenytoin (Dilantin) 2. Verapamil (Calan) 3. Warfarin (Coumadin) 4. Aluminum hydroxide (Amphojel)

2. Verapamil (Calan) -- CCB, can increase digoxin levels. if used together dig dosage may need to be lowered.

In hypovolemic shock, renal perfusion and urinary output decline. The nurse will monitor urinary output and knows that output below which of the following levels indicates inadequate renal perfusion?

20 mL/hour

The nurse is monitoring hourly urine output of a client diagnosed with hypovolemic shock. The nurse is most concerned if the client's output is:

20 mL/hour

b. Furosemide is a loop diuretic that acts to increase urine output. Furosemide does not increase blood pressure, decrease pain, or decrease arrhythmias

20] Which of the following is an expected outcome when a client is receiving an IV administration of furosemide? a. Increased blood pressure b. Increased urine output c. Decreased pain d. Decreased premature ventricular contractions

c. A basic principle of behavior modification is that behavior that is learned and continued is behavior that has been rewarded. Other reinforcement techniques have not been found to be as effective as reward.

25] If a client displays risk factors for coronary artery disease, such as smoking cigarettes, eating a diet high in saturated fat, or leading a sedentary lifestyle, techniques of behavior modification may be used to help the client change the behavior. The nurse can best reinforce new adaptive behaviors by: a. Explaining how the old behavior leads to poor health b. Withholding praise until the new behavior is well established c. Rewarding the client whenever the acceptable behavior is performed d. Instilling mild fear into the client to extinguish the behavior

26. A patient with chest pain that is unrelieved by nitroglycerin is admitted to the coronary care unit for observation and diagnosis. While the patient has continuous ECG monitoring, what finding would most concern the nurse? a. Occasional PVCs b. An inverted T wave c. ST segment elevation d. A PR interval of 0.18 second

26. c. ST segment elevation indicates injury or infarction of an area of the heart. An inverted T wave is most often associated with ischemia and resolves when blood flow is restored. Occasional PVCs may be normal or may be the result of electrolyte imbalance or hypoxia. They require continued observation. A PR interval of 0.18 second is within normal range.

The nurse is teaching the patient about the Dietary Approaches to Stop Hypertension (DASH) diet. Which statement indicates that the patient understood the teaching? 1. "I should eat more red meat, such as pork or beef." 2. "I should drink no more than three glasses of whole milk per day." 3. "I should include four to five servings of fruits and vegetables daily." 4. "I should consume whole grain products no more than once per week."

3. "I should include four to five servings of fruits and vegetables daily." The DASH diet encourages consumption of fruits and vegetables. Pork and beef are high in fat and therefore have to be restricted according to the DASH diet; poultry and fish have to be consumed instead of red meat. Fat-free or low-fat milk has to be used instead of whole milk according to the DASH recommendations. The DASH diet recommends a few servings of whole grain products daily. Text Reference - p. 715

3. A patient with a regular heart rate (HR) has four QRS complexes between every 3-second marker on the ECG paper. Calculate the patient's heart rate. _________ bpm

3. 4 (beats per 3 seconds) + 4 = 8 × 10 = 80 bpm

3. A patient is admitted with myocarditis. While performing the initial assessment, the nurse may find which clinical signs and symptoms (select all that apply)? a. Angina b. Pleuritic chest pain c. Splinter hemorrhages d. Pericardial friction rub e. Presence of Osler's nodes

3. Correct answers: a, b, d Rationale: Clinical manifestations of myocarditis may include early systemic manifestations (i.e., fever, fatigue, malaise, myalgias, pharyngitis, dyspnea, lymphadenopathy, and nausea and vomiting), early cardiac manifestations (e.g., pleuritic chest pain with a pericardial friction rub and effusion), and late cardiac signs (e.g., S3 heart sound, crackles, jugular venous distention [JVD], syncope, peripheral edema, and angina).

Joe has an EDV of 145 mL and an ESV of 100 mL. What is his ejection fraction? Give your answer as a % with no decimals - i.e. round your answer. Do NOT include the % sign.______Correct Response(50 %)Would this be consistent with systolic dysfunction or diastolic dysfunction? Just give the term systolic or diastolic in your response.______Correct Response(50 %)

31 systolic

d. Crackles are auscultated over fluid-filled alveoli. Crackles heard on lung auscultation do not have to be associated with cyanosis. Bronchospasm and airway narrowing generally are associated with wheezing sounds.

31] Crackles heard on lung auscultation indicate which of the following? a. Cyanosis b. Bronchospasm c. Airway narrowing d. Fluid-filled alveoli

c. Pasta, tomato sauce, salad, and coffee would be the best selection for the client following a low-cholesterol diet. Hamburgers, milkshakes, liver, and fried foods tend to be high in cholesterol.

36] The client who experiences angina has been told to follow a low-cholesterol diet. Which of the following meals should the nurse tell the client would be best on her low-cholesterol diet? a. Hamburger, salad, and milkshake b. Baked liver, green beans, and coffee c. Spaghetti with tomato sauce, salad, and coffee d. Fried chicken, green beans, and skim milk

4. The ECG pattern of a patient with a regular HR reveals 20 small squares between each R-R interval. What is the patient's heart rate? ________ bpm

4. 1500 ÷ 20 = 75 bpm

The nurse is checking blood pressure for people at a health fair. Which patient is at higher risk to develop primary hypertension? 1. 65-year-old retired Caucasian with a body mass index (BMI) of 15 2. 60-year-old who has chronic pain caused by cancer 3. 45-year-old blue collar worker who smokes one pack of cigarettes per day 4. 59-year-old African American with a BMI of 35 who has a high stress job

4. 59-year-old African American with a BMI of 35 who has a high stress job The patient has four risk factors for primary hypertension: advanced age, African American race, morbid obesity with a BMI of 35, and a high level of stress. All of the other patients have fewer risk factors for primary hypertension: in the 45-year-old smoker, smoking is the only risk factor; in the 60-year-old with cancer, advanced age and pain are the only risk factors; and in the 65-year-old retiree, the only risk factor is advanced age. Text Reference - p. 713

Which test result would indicate the presence of target organ damage resulting from uncontrolled hypertension? 1. Check for history of depression. 2. Do not give with grapefruit juice. 3. Monitor for cardiac dysrhythmias. 4. Assess for orthostatic hypotension

4. Assess for orthostatic hypotension Low blood pressure or postural hypotension can cause a fall from dizziness. The peripheral-acting alpha-adrenergic antagonist reserpine is contraindicated in patients with a history of depression. Administrating grapefruit juice with certain calcium channel blockers may increase the serum concentrations, resulting in toxicity. The direct vasodilator minoxidil may cause EKG changes of flattened and inverted T waves. Text Reference - p. 718

The patient with osteoporosis and hypertension understands dietary teaching when the patient selects which meal for dinner? 1. Ham and Swiss cheese sandwich on whole-wheat bread, steamed broccoli, and an apple 2. Baked chicken with one cup of yogurt and steamed rice 3. A two-egg omelet with 2 oz. of American cheese, one slice of whole-wheat toast, and half a grapefruit 4. Baked salmon with one cup of spinach and steamed carrots

4. Baked salmon with one cup of spinach and steamed carrots The highest calcium content is present in the dinner containing salmon and spinach, also taking into account fat and sodium restrictions required to manage hypertension. Ham and cheese are both high in sodium and should be avoided in the patient with hypertension. Eggs are not a large source of calcium, and chicken, yogurt, and rice, although lower in sodium, do not have the highest calcium content. Text Reference - p. 716

Which item on the patient's dinner tray should not be taken in large quantities by the patient prescribed furosemide for hypertension? 1. Coffee 2. Ice cream 3. Grapefruit juice 4. Chicken noodle soup

4. Chicken noodle soup Furosemide, a diuretic, causes fluid loss to decrease blood pressure. Chicken noodle soup is high in sodium and may cause increased fluid retention, negating the effects of the medication and increasing the blood pressure. Ice cream, grapefruit juice, and coffee will not decrease the effectiveness of furosemide. Text Reference - p. 717

8. A patient is diagnosed with mitral stenosis and new-onset atrial fibrillation. Which interventions could the nurse delegate to unlicensed assistive personnel (UAP) (select all that apply)? a. Obtain and record daily weight. b. Determine apical-radial pulse rate. c. Observe for overt signs of bleeding. d. Obtain and record vital signs, including pulse oximetry. e. Teach the patient how to purchase a Medic Alert device.

8. Correct answers: a, c, d Rationale: The nurse may delegate routine procedures such as measuring weights and vital signs. The nurse may give specific directions to the unlicensed assistive personnel (UAP) to observe and report obvious signs of bleeding. The nurse cannot delegate teaching, assessment, or activities that require clinical judgment. Obtaining an apical-radial pulse rate is an assessment.

9. Which diagnostic study best differentiates the various types of cardiomyopathy? a. Echocardiography b. Arterial blood gases c. Heart catheterization d. Endomyocardial biopsy

9. Correct answer: a Rationale: Echocardiography is the primary diagnostic tool used to differentiate between the different types of cardiomyopathies and other structural heart abnormalities.

The nurse is assessing the blood pressure of a client who is experiencing cardiogenic shock. Which of the following blood pressure readings is most likely?

90/75

Integrated Process: Nursing Process (Analysis) 8. A client is experiencing sinus bradycardia with hypotension and dizziness. What medication does the nurse administer? a. Atropine (Atropine) b. Digoxin (Lanoxin) c. Lidocaine (Xylocaine) d. Metoprolol (Lopressor)

A Atropine is a cholinergic antagonist that inhibits parasympathetically-induced hyperpolarization of the sinoatrial node. This inhibition results in an increased heart rate. The other medications are not appropriate. DIF: Cognitive Level: Application/Applying or higher REF: N/A TOP: Client Needs Category: Physiological Integrity (Pharmacological and Parenteral Therapies—Expected Actions/Outcomes)

Integrated Process: Nursing Process (Implementation) 27. A client has an epicardial pacemaker. The nurse observes the presence of a pacing spike but no QRS complex on the client's electrocardiograph (ECG) tracing. How does the nurse interpret this event? a. Loss of capture b. Ventricular fibrillation c. Failure to sense d. A normal tracing

A In epicardial pacing, the wires are threaded onto the epicardial surface of the heart and exit through the chest wall. The pacemaker spike should be followed immediately by a QRS complex. Pacing spikes seen without subsequent QRS complexes imply loss of capture. DIF: Cognitive Level: Application/Applying or higher REF: N/A TOP: Client Needs Category: Physiological Integrity (Reduction of Risk Potential—Potential for Complications from Diagnostic Tests/Treatments/Procedures)

A patient diagnosed with heart failure has a pulmonary artery catheter (PAC) in place. What information about the patient's hemodynamic functioning will the healthcare provider obtain from this monitoring device? A Left ventricular functioning B Pulmonary valve function C Coronary artery patency D Stroke volume

A Left ventricular functioning

Which statements accurately describe heart failure

A common cause of diastolic failure is left ventricular hypertrophy A primary risk factor for heart failure is CAD

The patient has CVI and a venous ulcer. The unlicensed assistive personnel (UAP) decides to apply compression stockings because that is what these patients always have ordered. What assessment by the nurse would cause the application of compression stockings to harm the patient? A) Rest pain B) High blood pressure C) Elevated blood sugar D) Dry, itchy, flaky skin

A) Rest pain occurs as peripheral artery disease (PAD) progresses and involves multiple arterial segments. Compression stockings should not be used on patients with PAD. Elevated blood glucose, possibly indicating uncontrolled diabetes mellitus, and hypertension may or may not indicate arterial problems. Dry, itchy, flaky skin indicates venous insufficiency. The RN should be the one to obtain the order and instruct the UAP to apply compression stockings if they are ordered.

When the patient is being examined for venous thromboembolism (VTE) in the calf, what diagnostic test should the nurse expect to teach the patient about first? A) Duplex ultrasound B) Contrast venography C) Magnetic resonance venography D) Computed tomography venography

A) The duplex ultrasound is the most widely used test to diagnose VTE. Contrast venography is rarely used now. Magnetic resonance venography is less accurate for calf veins than pelvic and proximal veins. Computed tomography venography may be used but is invasive and much more expensive than the duplex ultrasound.

Lisinopril, captopril, enalapril, ramipril

ACE - inhibitors

The nurse obtains a blood pressure of 176/83 mm Hg for a patient. What is the patient's mean arterial pressure (MAP)?

ANS: 114 mm Hg MAP = (SBP + 2 DBP)/3

A patient is to receive an infusion of 250 mL of platelets over 2 hours through tubing that is labeled: 1 mL equals 10 drops. How many drops per minute will the nurse infuse?

ANS: 21 To infuse 250 mL over 2 hours, the calculated drip rate is 20.8 drops/minute or 21 drops/minute

A nurse supervises an unlicensed assistive personnel (UAP) applying electrocardiographic monitoring. Which statement should the nurse provide to the UAP related to this procedure? a. "Clean the skin and clip hairs if needed." b. "Add gel to the electrodes prior to applying them." c. "Place the electrodes on the posterior chest." d. "Turn off oxygen prior to monitoring the client."

ANS: "Clean the skin and clip hairs if needed." To ensure the best signal transmission, the skin should be clean and hairs clipped. Electrodes should be placed on the anterior chest, and no additional gel is needed. Oxygen has no impact on electrocardiographic monitoring.

Which action should the nurse include in the plan of care when caring for a patient admitted with acute decompensated heart failure (ADHF) who is receiving nesiritide (Natrecor)? a. Monitor blood pressure frequently. b. Encourage patient to ambulate in room. c. Titrate nesiritide slowly before stopping. d. Teach patient about home use of the drug.

ANS: A Nesiritide is a potent arterial and venous dilator, and the major adverse effect is hypotension. Because the patient is likely to have orthostatic hypotension, the patient should not be encouraged to ambulate. Nesiritide does not require titration and is used for ADHF but not in a home setting.

Which nursing action should the nurse take first in order to assist a patient with newly diagnosed stage 1 hypertension in making needed dietary changes? a. Collect a detailed diet history. b. Provide a list of low-sodium foods. c. Help the patient make an appointment with a dietitian. d. Teach the patient about foods that are high in potassium.

ANS: A The initial nursing action should be assessment of the patient's baseline dietary intake through a thorough diet history. The other actions may be appropriate, but assessment of the patient's baseline should occur first

The female patient presents to the emergency department just after a syncope episode. What should you assess first? A. History of syncope B. Capillary glucose level C. Last menstrual period D. Allergies

ANS: B A change in the level of consciousness should always have glucose and oxygen (and cardiac) assessed first. Hypoglycemia is a noncardiovascular cause that can be easily treated. It takes priority over the other assessments. Although those who have syncope have a 30% chance of recurrence, ruling out a simple treatable cause should be done first. Reference: 839

Which assessment finding in a patient who is admitted with infective endocarditis (IE) is most important to communicate to the health care provider? a. Generalized muscle aching b. Sudden onset right flank pain c. Janeway's lesions on the palms d. Temperature 100.7° F (38.1° C)

ANS: B Sudden onset of flank pain indicates possible embolization to the kidney and may require diagnostic testing such as a renal arteriogram and interventions to improve renal perfusion. The other findings are typically found in IE, but do not require any new interventions.

The nurse obtains this information from a patient with prehypertension. Which finding is most important to address with the patient? a. Low dietary fiber intake b. No regular aerobic exercise c. Weight 5 pounds above ideal weight d. Drinks wine with dinner once a week

ANS: B The recommendations for preventing hypertension include exercising aerobically for 30 minutes most days of the week. A weight that is 5 pounds over the ideal body weight is not a risk factor for hypertension. The Dietary Approaches to Stop Hypertension (DASH) diet is high in fiber, but increasing fiber alone will not prevent hypertension from developing. The patient's alcohol intake will not increase the hypertension risk.

When computing a heart rate from the ECG tracing, you count 15 of the small blocks between the R waves of a patient whose rhythm is regular. From these data, you calculate the patient's heart rate to be what? A. 60 beats/minute B. 75 beats/minute C. 100 beats/minute D. 150 beats/minute

ANS: C Because each small block on the ECG paper represents 0.04 second, 1500 of them represent 1 minute. By dividing the number of small blocks (15 in this case) into 1500, you can calculate the heart rate in a patient whose rhythm is regular (in this case, 100). Reference: 821

What term is applied to a pacemaker that is implanted for the purpose of terminating atrial tachycardias? A. Temporary pacemaker B. Antitachycardia pacing C. Overdriving pacing D. Cardiac resynchronization therapy

ANS: C Overdrive pacing involves pacing the atrium at a rate of 200 to 500 impulses per minute in an attempt to terminate atrial tachycardias (e.g., atrial flutter with a rapid ventricular response). A temporary pacemaker is a category of pacemakers used temporarily with the power source outside the body. Antitachycardia pacing delivers a stimulus to the ventricle to terminate tachydysrhythmias. Cardiac resynchronization therapy is a technique to resynchronize the cardiac cycle by pacing both ventricles. Reference: 835

Which statement best describes the electrical activity of the heart represented by measuring the PR interval on the ECG? A. The length of time it takes to depolarize the atrium B. The length of time it takes for the atria to depolarize and repolarize C. The length of time for the electrical impulse to travel from the sinoatrial (SA) node to the Purkinje fibers D. The length of time it takes for the electrical impulse to travel from the SA node to the AV node

ANS: C The electrical impulse in the heart must travel from the SA node through the AV node and into the Purkinje fibers for synchronous atrial and ventricular contraction to occur. The P wave represents atrial contraction, and the R wave is part of the QRS complex that represents ventricular contraction. When measuring the time from the beginning of the P wave to the beginning of the QRS (PR interval), you are identifying the length of time it takes for the electrical impulse to travel from the SA node to the Purkinje fibers. Reference: 818-819

The patient has an electrocardiographic (ECG) tracing that is 50 beats/minute, the rhythm is regular, and there is a P wave before every QRS complex. The QRS has a normal shape and duration, and the PR interval is normal. What is you response? A. Administer atropine by intravenous push (IVP). B. Administer epinephrine by IVP. C. Monitor the patient for syncope. D. Attach an external pacemaker.

ANS: C The rhythm described is sinus bradycardia. Treatment depends on the patient's response and whether adequate perfusion is occurring. If the patient tolerates the rhythm, no treatment is given. Reference: 824

Which action for a patient with neutropenia is appropriate for the registered nurse (RN) to delegate to a licensed practical/vocational nurse (LPN/LVN)? a. Assessing the patient for signs and symptoms of infection b. Teaching the patient the purpose of neutropenic precautions c. Administering subcutaneous filgrastim (Neupogen) injection d. Developing a discharge teaching plan for the patient and family

ANS: C Administration of subcutaneous medications is included in LPN/LVN education and scope of practice. Patient education, assessment, and developing the plan of care require RN level education and scope of practice

During discharge teaching with a 68-year-old patient who had a mitral valve replacement with a mechanical valve, the nurse instructs the patient on the a. use of daily aspirin for anticoagulation. b. correct method for taking the radial pulse. c. need for frequent laboratory blood testing. d. need to avoid any physical activity for 1 month.

ANS: C Anticoagulation with warfarin (Coumadin) is needed for a patient with mechanical valves to prevent clotting on the valve. This will require frequent international normalized ratio (INR) testing. Daily aspirin use will not be effective in reducing the risk for clots on the valve. Monitoring of the radial pulse is not necessary after valve replacement. Patients should resume activities of daily living as tolerated.

A nurse administers prescribed adenosine (Adenocard) to a client. Which response should the nurse assess for as the expected therapeutic response? a. Decreased intraocular pressure b. Increased heart rate c. Short period of asystole d. Hypertensive crisis

ANS: C Clients usually respond to adenosine with a short period of asystole, bradycardia, hypotension, dyspnea, and chest pain. Adenosine has no conclusive impact on intraocular pressure.

Which instruction will the nurse plan to include in discharge teaching for the patient admitted with a sickle cell crisis? a. Take a daily multivitamin with iron. b. Limit fluids to 2 to 3 quarts per day. c. Avoid exposure to crowds when possible. d. Drink only two caffeinated beverages daily.

ANS: C Exposure to crowds increases the patient's risk for infection, the most common cause of sickle cell crisis. There is no restriction on caffeine use. Iron supplementation is generally not recommended. A high-fluid intake is recommended

After the nurse teaches the patient with stage 1 hypertension about diet modifications that should be implemented, which diet choice indicates that the teaching has been *most* effective? a. The patient avoids eating nuts or nut butters. b. The patient restricts intake of chicken and fish. c. The patient drinks low-fat milk with each meal. d. The patient has two cups of coffee in the morning.

ANS: C For the prevention of hypertension, the Dietary Approaches to Stop Hypertension (DASH) recommendations include increasing the intake of calcium-rich foods. Caffeine restriction and decreased protein intake are not included in the recommendations. Nuts are high in beneficial nutrients and 4 to 5 servings weekly are recommended in the DASH diet.

Which information is *most* important for the nurse to include when teaching a patient with newly diagnosed hypertension? a. Most people are able to control BP through dietary changes. b. Annual BP checks are needed to monitor treatment effectiveness. c. Hypertension is usually asymptomatic until target organ damage occurs. d. Increasing physical activity alone controls blood pressure (BP) for most people.

ANS: C Hypertension is usually asymptomatic until target organ damage has occurred. Lifestyle changes (e.g., physical activity, dietary changes) are used to help manage BP, but drugs are needed for most patients. Home BP monitoring should be taught to the patient and findings checked by the health care provider frequently when starting treatment for hypertension and then every 3 months when stable.

A patient has recently started on digoxin (Lanoxin) in addition to furosemide (Lasix) and captopril for the management of heart failure. Which assessment finding by the home health nurse is a priority to communicate to the health care provider? a. Presence of 1+ to 2+ edema in the feet and ankles b. Palpable liver edge 2 cm below the ribs on the right side c. Serum potassium level 3.0 mEq/L after 1 week of therapy d. Weight increase from 120 pounds to 122 pounds over 3 days

ANS: C Hypokalemia can predispose the patient to life-threatening dysrhythmias (e.g., premature ventricular contractions) and potentiate the actions of digoxin. Hypokalemia also increases the risk for digoxin toxicity, which can also cause life-threatening dysrhythmias. The other data indicate that the patient's heart failure requires more effective therapies, but they do not require nursing action as rapidly as the low serum potassium level.

A 53-yr-old patient with stage D heart failure and type 2 diabetes asks the nurse whether heart transplant is a possible therapy. Which response by the nurse is most accurate? a. "Your heart failure has not reached the end stage yet." b. "You could not manage the multiple complications of that surgery." c. "The suitability of a heart transplant for you depends on many factors." d. "Because you have diabetes, you would not be a heart transplant candidate."

ANS: C Indications for a heart transplant include end-stage heart failure (stage D), but other factors such as coping skills, family support, and patient motivation to follow the rigorous posttransplant regimen are also considered. Patients with diabetes who have well-controlled blood glucose levels may be candidates for heart transplant. Although heart transplants can be associated with many complications, there are no data to suggest that the patient could not manage the care.

After giving a patient the initial dose of oral labetalol (Normodyne) for treatment of hypertension, which action should the nurse take? a. Encourage oral fluids to prevent dry mouth or dehydration. b. Instruct the patient to ask for help if heart palpitations occur. c. Ask the patient to request assistance when getting out of bed. d. Teach the patient that headaches may occur with this medication.

ANS: C Labetalol decreases sympathetic nervous system activity by blocking both α- and β-adrenergic receptors, leading to vasodilation and a decrease in heart rate, which can cause severe orthostatic hypotension. Heart palpitations, dehydration, and headaches are possible side effects of other antihypertensives.

Which statement by a patient with restrictive cardiomyopathy indicates that the nurse's discharge teaching about self-management has been most effective? a. "I will avoid taking aspirin or other antiinflammatory drugs." b. "I will need to limit my intake of salt and fluids even in hot weather." c. "I will take antibiotics when my teeth are cleaned at the dental office." d. "I should begin an exercise program that includes things like biking or swimming."

ANS: C Patients with restrictive cardiomyopathy are at risk for infective endocarditis and should use prophylactic antibiotics for any procedure that may cause bacteremia. The other statements indicate a need for more teaching by the nurse. Dehydration and vigorous exercise impair ventricular filling in patients with restrictive cardiomyopathy. There is no need to avoid salt (unless ordered), aspirin, or NSAIDs.

IV sodium nitroprusside is ordered for a patient with acute pulmonary edema. During the first hours of administration, the nurse will need to titrate the nitroprusside rate down if the patient develops a. ventricular ectopy. b. a dry, hacking cough. c. a systolic BP below 90 mm Hg. d. a heart rate below 50 beats/min.

ANS: C Sodium nitroprusside is a potent vasodilator and the major adverse effect is severe hypotension. Coughing and bradycardia are not adverse effects of this medication. Nitroprusside does not cause increased ventricular ectopy.

IV sodium nitroprusside (Nipride) is ordered for a patient with acute pulmonary edema. During the first hours of administration, the nurse will need to titrate the nitroprusside rate if the patient develops a. ventricular ectopy. b. a dry, hacking cough. c. a systolic BP <90 mm Hg. d. a heart rate <50 beats/minute.

ANS: C Sodium nitroprusside is a potent vasodilator, and the major adverse effect is severe hypotension. Coughing and bradycardia are not adverse effects of this medication. Nitroprusside does not cause increased ventricular ectopy.

The charge nurse observes a new RN doing discharge teaching for a hypertensive patient who has a new prescription for enalapril (Vasotec). The charge nurse will need to intervene if the new RN tells the patient to a. check the BP with a home BP monitor every day. b. move slowly when moving from lying to standing. c. increase the dietary intake of high-potassium foods. d. make an appointment with the dietitian for teaching.

ANS: C The ACE inhibitors cause retention of potassium by the kidney, so hyperkalemia is a possible adverse effect. The other teaching by the new RN is appropriate for a patient with newly diagnosed hypertension who has just started therapy with enalapril.

Which finding about a patient with polycythemia vera is most important for the nurse to report to the health care provider? a. Hematocrit 55% b. Presence of plethora c. Calf swelling and pain d. Platelet count 450,000/mL

ANS: C The calf swelling and pain suggest that the patient may have developed a deep vein thrombosis, which will require diagnosis and treatment to avoid complications such as pulmonary embolus. The other findings will also be reported to the health care provider but are expected in a patient with this diagnosis

Which topic will the nurse plan to include in discharge teaching for a patient with systolic heart failure and an ejection fraction of 33%? a. Need to begin an aerobic exercise program several times weekly b. Use of salt substitutes to replace table salt when cooking and at the table c. Benefits and side effects of angiotensin-converting enzyme (ACE) inhibitors d. Importance of making an annual appointment with the primary care provider

ANS: C The core measures for the treatment of heart failure established by The Joint Commission indicate that patients with an ejection fraction (EF) <40% receive an ACE inhibitor to decrease the progression of heart failure. Aerobic exercise may not be appropriate for a patient with this level of heart failure, salt substitutes are not usually recommended because of the risk of hyperkalemia, and the patient will need to see the primary care provider more frequently than annually

While assessing a 68-yr-old with ascites, the nurse also notes jugular venous distention (JVD) with the head of the patient's bed elevated 45 degrees. The nurse knows this finding indicates a. decreased fluid volume. b. jugular vein atherosclerosis. c. increased right atrial pressure. d. incompetent jugular vein valves.

ANS: C The jugular veins empty into the superior vena cava and then into the right atrium, so JVD with the patient sitting at a 45-degree angle reflects increased right atrial pressure. JVD is an indicator of excessive fluid volume (increased preload), not decreased fluid volume. JVD is not caused by incompetent jugular vein valves or atherosclerosis.

Which action will the nurse in the hypertension clinic take in order to obtain an accurate baseline blood pressure (BP) for a new patient? a. Obtain a BP reading in each arm and average the results. b. Deflate the BP cuff at a rate of 5 to 10 mm Hg per second. c. Have the patient sit in a chair with the feet flat on the floor. d. Assist the patient to the supine position for BP measurements.

ANS: C The patient should be seated with the feet flat on the floor. The BP is obtained in both arms, but the results of the two arms are not averaged. The patient does not need to be in the supine position. The cuff should be deflated at 2 to 3 mm Hg per second.

An outpatient who has chronic heart failure returns to the clinic after 2 weeks of therapy with metoprolol (Toprol XL). Which assessment finding is most important for the nurse to report to the health care provider? a. 2+ pedal edema b. Heart rate of 56 beats/minute c. Blood pressure (BP) of 88/42 mm Hg d. Complaints of fatigue

ANS: C The patient's BP indicates that the dose of metoprolol may need to be decreased because of hypotension. Bradycardia is a frequent adverse effect of -adrenergic blockade, but the rate of 56 is not unusual with â-adrenergic blocker therapy. -Adrenergic blockade initially will worsen symptoms of heart failure in many patients, and patients should be taught that some increase in symptoms, such as fatigue and edema, is expected during the initiation of therapy with this class of drugs

A patient with possible disseminated intravascular coagulation arrives in the emergency department with a blood pressure of 82/40, temperature 102° F (38.9° C), and severe back pain. Which physician order will the nurse implement first? a. Administer morphine sulfate 4 mg IV. b. Give acetaminophen (Tylenol) 650 mg. c. Infuse normal saline 500 mL over 30 minutes. d. Schedule complete blood count and coagulation studies.

ANS: C The patient's blood pressure indicates hypovolemia caused by blood loss and should be addressed immediately to improve perfusion to vital organs. The other actions also are appropriate and should be rapidly implemented, but improving perfusion is the priority for this patient

When caring for a patient with mitral valve stenosis, it is most important that the nurse assess for a. diastolic murmur. b. peripheral edema. c. shortness of breath on exertion. d. right upper quadrant tenderness.

ANS: C The pressure gradient changes in mitral stenosis lead to fluid backup into the lungs, resulting in hypoxemia and dyspnea. The other findings also may be associated with mitral valve disease but are not indicators of possible hypoxemia.

A nurse performs an admission assessment on a 75-year-old client with multiple chronic diseases. The client's blood pressure is 135/75 mm Hg and oxygen saturation is 94% on 2 liters per nasal cannula. The nurse assesses the client's rhythm on the cardiac monitor and observes the reading shown below: Which action should the nurse take first? a. Begin external temporary pacing. b. Assess peripheral pulse strength. c. Ask the client what medications he or she takes. d. Administer 1 mg of atropine.

ANS: C This client is stable and therefore does not require any intervention except to determine the cause of the bradycardia. Bradycardia is often caused by medications. Clients who have multiple chronic diseases are often on multiple medications that can interact with each other. The nurse should assess the client's current medications first.

The ECG monitor of a patient in the cardiac care unit after a myocardial infarction (MI) indicates ventricular bigeminy with a rate of 50 beats/minute. You anticipate A. performing defibrillation. B. treatment with IV lidocaine. C. insertion of a temporary, transvenous pacemaker. D. assessing the patient's response to the dysrhythmia.

ANS: D A premature ventricular contraction (PVC) is a contraction originating in an ectopic focus in the ventricles. When every other beat is a PVC, the rhythm is called ventricular bigeminy. PVCs are usually a benign finding in the patient with a normal heart. In heart disease, PVCs may reduce the cardiac output and precipitate angina and heart failure, depending on the frequency. Because PVCs in coronary artery disease or acute MI indicate ventricular irritability, the patient's physiologic response to PVCs must be monitored. Assessment of the patient's hemodynamic status is important to determine whether treatment with drug therapy is needed. Reference: 830

The patient has chronic atrial fibrillation (AF). What action do you anticipate? A. Monitoring the PR interval B. Defibrillation with 360 joule C. Teaching the patient to monitor the pulse deficit D. Teaching the patient to take an anticoagulant daily

ANS: D The chaotic atrial activity results in blood stasis that can lead to embolic events. Patients with chronic AF are given an anticoagulant, most often warfarin (Coumadin), to prevent the formation of emboli. There is no PR interval in AF because the P wave is absent, replaced by chaotic fibrillatory waves. Defibrillation is an elective procedure in chronic AF and is performed at lower levels of electricity. Pulse deficit is a higher-level skill and is not taught to the patient. Reference: 827

A 52-year-old patient who has no previous history of hypertension or other health problems suddenly develops a BP of 188/106 mm Hg. After reconfirming the BP, it is appropriate for the nurse to tell the patient that a. a BP recheck should be scheduled in a few weeks. b. the dietary sodium and fat content should be decreased. c. there is an immediate danger of a stroke and hospitalization will be required. d. more diagnostic testing may be needed to determine the cause of the hypertension.

ANS: D A sudden increase in BP in a patient over age 50 with no previous hypertension history or risk factors indicates that the hypertension may be secondary to some other problem. The BP will need rapid treatment and ongoing monitoring. If the patient has no other risk factors, a stroke in the immediate future is unlikely. There is no indication that dietary salt or fat intake have contributed to this sudden increase in BP, and reducing intake of salt and fat alone will not be adequate to reduce this BP to an acceptable level.

A 56-year-old patient who has no previous history of hypertension or other health problems suddenly develops a blood pressure (BP) of 198/110 mm Hg. After reconfirming the BP, it is appropriate for the nurse to tell the patient that a. a BP recheck should be scheduled in a few weeks. b. dietary sodium and fat content should be decreased. c. there is an immediate danger of a stroke and hospitalization will be required. d. diagnosis of a possible cause, treatment, and ongoing monitoring will be needed.

ANS: D A sudden increase in BP in a patient over age 50 with no previous hypertension history or risk factors indicates that the hypertension may be secondary to some other problem. The BP will need treatment and ongoing monitoring. If the patient has no other risk factors, a stroke in the immediate future is unlikely. There is no indication that dietary salt or fat intake have contributed to this sudden increase in BP, and reducing intake of salt and fat alone will not be adequate to reduce this BP to an acceptable level

The nurse is obtaining a health history from a 24-year-old patient with hypertrophic cardiomyopathy (HC). Which information obtained by the nurse is most important? a. The patient has a history of a recent upper respiratory infection. b. The patient has a family history of coronary artery disease (CAD). c. The patient reports using cocaine a "couple of times" as a teenager. d. The patient's 29-year-old brother died from a sudden cardiac arrest.

ANS: D About half of all cases of HC have a genetic basis, and it is the most common cause of sudden cardiac death in otherwise healthy young people. The information about the patient's brother will be helpful in planning care (such as an automatic implantable cardioverter-defibrillator [AICD

A patient who has just been admitted with pulmonary edema is scheduled to receive the following medications. Which medication should the nurse question before giving? a. captopril 25 mg b. furosemide (Lasix) 60 mg c. digoxin (Lanoxin) 0.125 mg d. carvedilol (Coreg) 3.125 mg

ANS: D Although carvedilol is appropriate for the treatment of chronic heart failure, it is not used for patients with acute decompensated heart failure (ADHF) because of the risk of worsening the heart failure. The other drugs are appropriate for the patient with ADHF.

A patient who has just been admitted with pulmonary edema is scheduled to receive the following medications. Which medication should the nurse question before giving? a. Furosemide (Lasix) 60 mg b. Captopril (Capoten) 25 mg c. Digoxin (Lanoxin) 0.125 mg d. Carvedilol (Coreg) 3.125 mg

ANS: D Although carvedilol is appropriate for the treatment of chronic heart failure, it is not used for patients with acute decompensated heart failure (ADHF) because of the risk of worsening the heart failure. The other medications are appropriate for the patient with ADHF.

The nurse has just finished teaching a hypertensive patient about the newly prescribed quinapril (Accupril). Which patient statement indicates that more teaching is needed? a. "The medication may not work as well if I take any aspirin." b. "The doctor may order a blood potassium level occasionally." c. "I will call the doctor if I notice that I have a frequent cough." d. "I won't worry if I have a little swelling around my lips and face."

ANS: D Angioedema occurring with angiotensin-converting enzyme (ACE) inhibitor therapy is an indication that the ACE inhibitor should be discontinued. The patient should be taught that if any swelling of the face or oral mucosa occurs, the health care provider should be immediately notified because this could be life threatening. The other patient statements indicate that the patient has an accurate understanding of ACE inhibitor therapy.

Which patient information is most important for the nurse to monitor when evaluating the effectiveness of deferoxamine (Desferal) for a patient with hemochromatosis? a. Skin color b. Hematocrit c. Liver function d. Serum iron level

ANS: D Because iron chelating agents are used to lower serum iron levels, the most useful information will be the patient's iron level. The other parameters will also be monitored, but are not the most important to monitor when determining the effectiveness of deferoxamine

A patient in the emergency department complains of back pain and difficulty breathing 15 minutes after a transfusion of packed red blood cells is started. The nurse's first action should be to a. administer oxygen therapy at a high flow rate. b. obtain a urine specimen to send to the laboratory. c. notify the health care provider about the symptoms. d. disconnect the transfusion and infuse normal saline.

ANS: D The patient's symptoms indicate a possible acute hemolytic reaction caused by the transfusion. The first action should be to disconnect the transfusion and infuse normal saline. The other actions also are needed but are not the highest priority

A nurse assists with the cardioversion of a client experiencing acute atrial fibrillation. Which action should the nurse take prior to the initiation of cardioversion? a. Administer intravenous adenosine. b. Turn off oxygen therapy. c. Ensure a tongue blade is available. d. Position the client on the left side.

ANS: Turn off oxygen therapy. For safety during cardioversion, the nurse should turn off any oxygen therapy to prevent fire. The other interventions are not appropriate for a cardioversion. The client should be placed in a supine position.

The nurse is taking the initial history and vital signs on a patient with fatigue. The nurse notes a regular apical pulse of 130 beats/min. Which contributing factors does the nurse assess for? (select all that apply) a. Anxiety or stress b. Fever c. Hypovolemia d. Anemia or hypoxemia e. Hypothyroidism f. Constipation

Abcd

What other actions are essential interventions for the patient in question 65? (select all that apply) a. Providing effective CPR b. Supplying a temporary pacemaker c. Administrating epinephrine, vasopressin, and atropine, as appropriate d. Identifying and correcting the cause of the pulseless rhythm e. Providing continuous ECG monitoring

Abcd

A patient has an invasive temporary pace-maker. In what ways does the nurse ensure the patient's safety related to electrical issues with the pacemaker? (select all that apply) a. Ensure that external ends of the lead wires are insulated with rubber loves. b. Loop the wire ends and cover with non-conductive tape. c. Ensure that no electrical equipment is used in the patient's room. d. Report frayed wire to the biomedical engineering department. e. Wash hands before touching any of the wires.

Abd

Which descriptions are characteristic of a class III antidysrhythmic? (select all that apply) a. Lengthens the absolute refractory period b. Is a beta blocker c. Includes hypertension as a side effect d. Prolongs repolarization e. Includes bradycardia as a side effect

Ade

A client is transported to the emergency department in respiratory distress after eating peanuts. The following interventions are ordered by the health care provider. Which intervention should the nurse complete first?

Administer epinephrine (adrenaline)

ANS: A Although all options will be assessed eventually, determining a cardiac cause for this brief lapse of consciousness is most important. Reference: 839

An elderly patient presents to the emergency department after a fall. She states she does not remember the incident. What is most important to assess first? A. Heart rate and rhythm B. Hemoglobin C. Home environment D. Alcohol consumption

A client is rushed to the emergency department with symptoms of urticaria, wheezing, chest tightness, and difficulty in breathing. The client is most likely experiencing which type of shock?

Anaphylactic

A 20-year-old male client is experiencing a severe immunologically mediated reaction in which histamines have been released into the blood. Select the type of reaction most likely occurring with this client.

Anaphylatic shock

Candesartan, Ibesartan, Losartan, valsartan

Angiotension Receptor blockers (ARBS)

Digitalis Toxicity

Anorexia, nausea, visual changes, confusion, bradycardia

Other medications for heart failure

Anticoagulants, antiarrhythmic drugs, statins Contraindicated: NSAIDs (ibuprofen, Motrin)

Regarding heart failure, the nurse knows that which of the following statements are correct? Select all that apply.

Aortic stenosis can cause left-sided failure. Cardiomyopathy is a common cause of heart failure. Compensated heart failure may be clinically asymptomatic.

A patient who had bladder surgery 2 days ago develops acute decompensated heart failure (ADHF) with severe dyspnea. Which action by the nurse would be indicated first? Perform a bladder scan to assess for urinary retention. Restrict the patient's oral fluid intake to 500 mL per day. Assist the patient to a sitting position with arms on the overbed table. Instruct the patient to use pursed-lip breathing until the dyspnea subsides.

Assist the patient to a sitting position with arms on the overbed table The nurse should place the patient with ADHF in a high Fowler's position with the feet horizontal in the bed or dangling at the bedside. This position helps decrease venous return because of the pooling of blood in the extremities. This position also increases the thoracic capacity, allowing for improved ventilation. Pursed-lip breathing helps with obstructive air trapping but not with acute pulmonary edema. Restricting fluids takes considerable time to have an effect.

Dysrhythmias can occur in patients with heart failure. The dysrhythmia that occurs most frequently in heart failure is which of the following?

Atrial fibrillation

What is the heart rate from an ECG strip when there are 25 small blocks from one R wave to the next R wave? a. 50/minute b. 60/minute c. 70/minute d. 80/minute

B

Integrated Process: Nursing Process (Implementation) 26. A client with ventricular tachycardia (VT) is unresponsive and has no pulse. The nurse calls for assistance and a defibrillator. What is the nurse's priority intervention while waiting for the defibrillator to arrive? a. Perform a pericardial thump. b. Initiate cardiopulmonary resuscitation. c. Start an 18-gauge IV in the antecubital. d. Ask the client's family about code status.

B A client with pulseless VT should be defibrillated immediately. If the defibrillator is not available, the nurse should initiate cardiopulmonary resuscitation (CPR) and then should defibrillate as soon as possible. Basic life support (BLS) is the basis of emergency cardiac care; if the client does not have an IV already, this can wait until others have arrived to help. Providing good quality CPR is vital. The client should have already been assessed for code status. DIF: Cognitive Level: Application/Applying or higher REF: N/A TOP: Client Needs Category: Physiological Integrity (Physiological Adaptation—Medical Emergencies)

11. When developing a plan to decrease preload in the patient with heart failure, the nurse will include actions such as a. administering sedatives to promote rest and decrease myocardial oxygen demand. b. positioning the patient in a high-Fowler's position with the feet horizontal in the bed. c. administering oxygen per mask or nasal cannula. d. encouraging leg exercises to improve venous return.

B Rationale: Positioning the patient in a high-Fowler's position with the legs dependent will reduce preload by decreasing venous return to the right atrium. The other interventions may also be appropriate for patients with heart failure but will not help in decreasing preload. Cognitive Level: Application Text Reference: pp. 827-828 Nursing Process: Planning NCLEX: Physiological Integrity

A patient with a diagnosis of heart failure has been started on a nitroglycerin patch by his primary care provider. What should this patient be taught to avoid? A. High-potassium foods B. Drugs to treat erectile dysfunction C. Nonsteroidal antiinflammatory drugs D. Over-the-counter H2-receptor blockers

B. Drugs to treat erectile dysfunction The use of erectile drugs concurrent with nitrates creates a risk of severe hypotension and possibly death. High-potassium foods, NSAIDs, and H2-receptor blockers do not pose a risk in combination with nitrates.

A 73-year-old man with dementia has a venous ulcer related to chronic venous insufficiency. The nurse should provide education on which type of diet for this patient and his caregiver? A. Low-fat diet B. High-protein diet C. Calorie-restricted diet D. High-carbohydrate diet

B. High-protein diet A patient with a venous ulcer should have a balanced diet with adequate protein, calories, and micronutrients; this type of diet is essential for healing. Nutrients most important for healing include protein, vitamins A and C, and zinc. Foods high in protein (e.g., meat, beans, cheese, tofu), vitamin A (green leafy vegetables), vitamin C (citrus fruits, tomatoes, cantaloupe), and zinc (meat, seafood) must be provided. Restricting fat or calories is not helpful for wound healing or in patients of normal weight. For overweight individuals with no active venous ulcer, a weight-loss diet should be considered.

What should the nurse recognize as an indication for the use of dopamine (Intropin) in the care of a patient with heart failure? A. Acute anxiety B. Hypotension and tachycardia C. Peripheral edema and weight gain D. Paroxysmal nocturnal dyspnea (PND)

B. Hypotension and tachycardia Dopamine is a β-adrenergic agonist whose inotropic action is used for treatment of severe heart failure accompanied by hemodynamic instability. Such a state may be indicated by tachycardia accompanied by hypotension. PND, anxiety, edema, and weight gain are common signs and symptoms of heart failure, but these do not necessarily warrant the use of dopamine.

A patient was just diagnosed with acute arterial ischemia in the left leg secondary to atrial fibrillation. Which early clinical manifestation must be reported to the physician immediately to save the patient's limb? A. Paralysis B. Paresthesia C. Crampiness D. Referred pain

B. Paresthesia The physician must be notified immediately if any of the six Ps of acute arterial ischemia occur to prevent ischemia from quickly progressing to tissue necrosis and gangrene. The six Ps are paresthesia, pain, pallor, pulselessness, and poikilothermia, with paralysis being a very late sign indicating the death of nerves to the extremity. Crampy leg sensation is more common with varicose veins. The pain is not referred.

67-year-old man with peripheral artery disease is seen in the primary care clinic. Which symptom reported by the patient would indicate to the nurse that the patient is experiencing intermittent claudication? A. Patient complains of chest pain with strenuous activity. B. Patient says muscle leg pain occurs with continued exercise. C. Patient has numbness and tingling of all his toes and both feet. D. Patient states the feet become red if he puts them in a dependent position.

B. Patient says muscle leg pain occurs with continued exercise. Intermittent claudication is an ischemic muscle ache or pain that is precipitated by a consistent level of exercise, resolves within 10 minutes or less with rest, and is reproducible. Angina is the term used to describe chest pain with exertion. Paresthesia is the term used to describe numbness or tingling in the toes or feet. Reactive hyperemia is the term used to describe redness of the foot; if the limb is in a dependent position the term is dependent rubor.

A 62-year-old Hispanic male patient with diabetes mellitus has been diagnosed with peripheral artery disease (PAD). The patient is a smoker and has a history of gout. What should the nurse focus her teaching on to prevent complications for this patient? A. Gender B. Smoking C. Ethnicity D. Co-morbidities

B. Smoking Smoking is the most significant factor for this patient. PAD is a marker of advanced systemic atherosclerosis. Therefore tobacco cessation is essential to reduce PAD progression, CVD events, and mortality. Diabetes mellitus and hyperuricemia are also risk factors. Being male or Hispanic are not risk factors for PAD.

Which diagnostic test is most useful in differentiating dyspnea related to pulmonary effects of heart failure from dyspnea related to pulmonary disease?

BNP levels

What effects does dobutamine (Dobutrex) have on the heart? (select all that apply) a. Decreases myocardial contractility b. Stimulates beta-adrenergic receptors c. Improves myocardial contractility d. Depresses stimulation of beta-adrenergic receptors e. May cause dysrhythmias

Bce

Carvedilol (Coreg)

Beta Blocker Used for HR and chest pain Takes about 3 weeks to get into system

Carvedilol, Metoprolol

Beta blockers

Indicated for diastolic dysfunction

Beta-blockers

What is the priority assessment by the nurse caring for a patient receiving IV nesiritide to treat heart failure? Urine output Lung sounds Blood pressure Respiratory rate

Blood pressure Although all identified assessments are appropriate for a patient receiving IV nesiritide, the priority assessment would be monitoring for hypotension, the main adverse effect of nesiritide.

Which initial physical assessment finding would the nurse expect to be present in a patient with acute left-sided heart failure

Bubbling crackles and tachycardia

Physiological Integrity 25. Which assessment finding obtained by the nurse when assessing a patient with acute pericarditis should be reported immediately to the health care provider? a. Pulsus paradoxus 8 mm Hg b. Blood pressure (BP) of 168/94 c. Jugular venous distention (JVD) to jaw level d. Level 6 (0 to 10 scale) chest pain with a deep breath

C The JVD indicates that the patient may have developed cardiac tamponade and may need rapid intervention to maintain adequate cardiac output. Hypertension would not be associated with complications of pericarditis, and the BP is not high enough to indicate that there is any immediate need to call the health care provider. A pulsus paradoxus of 8 mm Hg is normal. Level 6/10 chest pain should be treated but is not unusual with pericarditis. DIF: Cognitive Level: Apply (application) REF: 815 OBJ: Special Questions: Prioritization TOP: Nursing Process: Assessment MSC:

Integrated Process: Nursing Process (Planning) 23. The nurse is caring for a client with a complete heart block (third-degree atrioventricular [AV] block). What is the nurse's priority intervention? a. Perform a cardioversion. b. Assist with carotid massage. c. Begin external pacing. d. Administer adenosine (Adenocard) IV.

C The nurse would expect the client with complete heart block or third-degree AV block to be paced externally until the client can be scheduled for a permanent pacemaker. DIF: Cognitive Level: Application/Applying or higher REF: N/A TOP: Client Needs Category: Physiological Integrity (Physiological Adaptation—Medical Emergencies)

Physiological Integrity 18. A patient recovering from heart surgery develops pericarditis and complains of level 6 (0 to 10 scale) chest pain with deep breathing. Which ordered PRN medication will be the most appropriate for the nurse to give? a. Fentanyl 1 mg IV b. IV morphine sulfate 4 mg c. Oral ibuprofen (Motrin) 600 mg d. Oral acetaminophen (Tylenol) 650 mg

C The pain associated with pericarditis is caused by inflammation, so nonsteroidal antiinflammatory drugs (NSAIDs) (e.g., ibuprofen) are most effective. Opioid analgesics are usually not used for the pain associated with pericarditis. DIF: Cognitive Level: Apply (application) REF: 816 TOP: Nursing Process: Implementation MSC:

What is the priority assessment by the nurse caring for a patient receiving IV nesiritide (Natrecor) to treat heart failure? A Urine output B Lung sounds C Blood pressure D Respiratory rate

C Blood pressure Although all identified assessments are appropriate for a patient receiving IV nesiritide, the priority assessment would be monitoring for hypotension, the main adverse effect of nesiritide.

A patient has been diagnosed with early left ventricular heart failure (HF). The nurse knows that the following changes occur as the disease progresses. In which order do the changes involved in the development of dyspnea associated with left ventricular HF occur? (Answer with a letter followed by a comma and a space (e.g. A, B, C, D).) A. Inadequate alveolar gas exchange B. Elevated pressure in the left atrium C. Ineffective ventricular contractility D. Fluid leaking into interstitial spaces

C, B, D, A In left ventricular HF, ineffective ventricular contractions impair the normal forward flow of blood to the body. As a result, fluid returning to the heart from the lungs backs up in the heart and increases the pressure in the left atrium. If contractility continues to falter, the blood continues to back up into the pulmonary vasculature and eventually can result in fluid leaking into the interstitial spaces and alveoli of the lungs. This abnormal fluid in the parenchyma and alveoli impairs gas exchange, which causes dyspnea.

What is the priority assessment by the nurse caring for a patient receiving IV nesiritide (Natrecor) to treat heart failure? A. Urine output B. Lung sounds C. Blood pressure D. Respiratory rate

C. Blood pressure Although all identified assessments are appropriate for a patient receiving IV nesiritide, the priority assessment would be monitoring for hypotension, the main adverse effect of nesiritide.

A 32-year-old female is prescribed diltiazem (Cardizem) for Raynaud's phenomenon. To evaluate the patient's expected response to this medication, what is most important for the nurse to assess? A. Improved skin turgor B. Decreased cardiac rate C. Improved finger perfusion D. Decreased mean arterial pressure

C. Improved finger perfusion Raynaud's phenomenon is an episodic vasospastic disorder of small cutaneous arteries, most frequently involving the fingers and toes. Diltiazem (Cardizem) is a calcium channel blocker that relaxes smooth muscles of the arterioles by blocking the influx of calcium into the cells, thus reducing the frequency and severity of vasospastic attacks. Perfusion to the fingertips is improved and vasospastic attacks reduced. Diltiazem may decrease heart rate and blood pressure, but that is not the purpose in Raynaud's phenomenon. Skin turgor is most often a reflection of hydration status.

The nurse is caring for a preoperative patient who has an order for vitamin K by subcutaneous injection. The nurse should verify that which laboratory study is abnormal before administering the dose? A. Hematocrit (Hct) B. Hemoglobin (Hgb) C. Prothrombin time (PT) D. Partial thromboplastin time (PTT)

C. Prothrombin time (PT) Vitamin K counteracts hypoprothrombinemia and/or reverses the effects of warfarin (Coumadin) and thus decreases the risk of bleeding. High values for either the prothrombin time (PT) or the international normalized ratio (INR) demonstrates the need for this medication.

What nursing action should the nurse prioritize during the care of a patient who has recently recovered from rheumatic fever? A. Teach the patient how to manage his or her physical activity. B. Teach the patient about the need for ongoing anticoagulation. C. Teach the patient about the need for continuous antibiotic prophylaxis. D. Teach the patient about the need to maintain standard infection control procedures.

C. Teach the patient about the need for continuous antibiotic prophylaxis. Patients with a history of rheumatic fever frequently require ongoing antibiotic prophylaxis, an intervention that necessitates education. This consideration is more important than activity management in preventing recurrence. Anticoagulation is not indicated in this patient population. Standard precautions are indicated for all patients.

Blood pressure = ______ x _______

CO x SVR

The health care provider is reviewing diagnostic tests that were ordered for a client diagnosed with heart failure. Select the test that would provide information about the client's ejection fraction and ventricular preload.

Cardiac computerized axial tomography (CCT)

A patient is scheduled for a heart transplant. Beyond the first year after a heart transplant, the nurse knows that what is a major cause of death? Infection Acute rejection Immunosuppression Cardiac vasculopathy

Cardiac vasculopathy Beyond the first year after a heart transplant, malignancy (especially lymphoma) and cardiac vasculopathy (accelerated coronary artery disease) are the major causes of death. During the first year after transplant, infection and acute rejection are the major causes of death. Immunosuppressive therapy will be used for posttransplant management to prevent rejection and increases the patient's risk of an infection.

ACE inhibitors

Cause agitative cough=non compliant Cannot give to asthma pt causes bronchospasm Treats mild HTN Teach pt to change position slowly

beta blockers (lol)

Check BP and HR first Make pt feel disconnected, hard to get use to Drop BP and HR to orthostatic point Erectile disfunction, depression on men

A client with congestive heart failure, CHF, is prescribed digoxin (Lanoxin) and furosemide (Lasix). Nursing interventions will include: (Select all that apply.) Encourage intake of water and fruit juices. Restrict intake of green, leafy vegetables. Checking apical pulse before administering medication. Monitor hemoglobin and hematocrit levels. Monitor serum electrolytes.

Checking apical pulse before administering medication. Monitor serum electrolytes. Rationale: Digoxin is a cardiac glycoside. which can slow heart rate, and an apical heart rate is checked prior to administration. Lasix is a loop diuretic used in treatment of CHF, which promotes not only water loss, but also loss of electrolytes. A low potassium level increases risk of digoxin toxicity. Fluids are often restricted with CHF. H and H level do not need to be checked, and green, leafy vegetables would not need to be restricted.

A patient with a long-standing history of heart failure recently qualified for hospice care. What measure should the nurse now prioritize when providing care for this patient? Taper the patient off his current medications. Continue education for the patient and his family. Pursue experimental therapies or surgical options. Choose interventions to promote comfort and prevent suffering.

Choose interventions to promote comfort and prevent suffering. The central focus of hospice care is the promotion of comfort and the prevention of suffering. Patient education should continue, but providing comfort is paramount. Medications should be continued unless they are not tolerated. Experimental therapies and surgeries are not commonly used in the care of hospice patients.

Nitrates (isosorbide dinitrate) & hydralazine

Combo is an alternative for HF treatment--preferred for African Americans (ACE inhibitors not always as effective) Nitrates: vasodilation Hydralazine: lowers vascular resistance & afterload Monitor: hypotension

Chronic HF - Collab Care 1

Correct Na and H2O retention, volume overload and preload diuretics (thiazide, loop, K sparing): - Hydrochlorothiazide (HCTZ) - it's not as potent as Lasix, inhibits Na reabsorption in the distal tubules monitor for K+ - Lasix - Spironolactone (K+ sparing) - will help retain potassium ACE inhibitors - Block angiotensin - vasodilation and will block aldosterone to lower BP FIRST LINE DRUG - Side effects - dry and tickling cough (some people can't tolerate), may switch to an ARB Can drop BP - monitor! Nitrates With high BP Reduction of cardiac workload - Beta Blockers - will block the SNS to slow down the heart - Reduces O2 consumption and workload of the heart

The nurse must achieve which of the following objectives for a patient in cardiogenic shock? Select all that apply.

Correct pulmonary edema Increase coronary perfusion Improve cardiac output Regulate blood volume

Which manifestation of left-sided heart failure can be diagnosed by examination of the lips and mucous membranes?

Cyanosis

Integrated Process: Nursing Process (Assessment) 36. The nurse is alerted to a client's telemetry monitor. After assessing the following ECG, what is the nurse's priority intervention? a. Start a large-bore IV. b. Administer atropine. c. Prepare for intubation. d. Perform defibrillation.

D The client's rhythm is ventricular fibrillation. This is a lethal rhythm that is best treated with immediate defibrillation. If the client does not already have an IV , other members of the team can insert one after defibrillation. Likewise, intubation can occur later if necessary. Atropine is not given for ventricular fibrillation. DIF: Cognitive Level: Application/Applying or higher REF: N/A TOP: Client Needs Category: Physiological Integrity (Physiological Adaptation-Pathophysiology)

The nurse is administering a dose of digoxin (Lanoxin) to a patient with heart failure (HF). The nurse would become concerned with the possibility of digitalis toxicity if the patient reported which symptom(s)? A Muscle aches B Constipation C Pounding headache D Anorexia and nausea

D Anorexia and nausea Anorexia, nausea, vomiting, blurred or yellow vision, and cardiac dysrhythmias are all signs of digitalis toxicity. The nurse would become concerned and notify the health care provider if the patient exhibited any of these symptoms.

A patient is diagnosed with heart failure and is prescribed digoxin (Lanoxin) and furosemide (Lasix). Before administering the furosemide to the patient, which laboratory result should the healthcare provider to review? A Blood urea nitrogen (BUN) B Serum sodium C Serum troponin D Serum potassium

D Serum potassium

A male patient was admitted for a possible ruptured aortic aneurysm, but had no back pain. Ten minutes later his assessment includes the following: sinus tachycardia at 138, BP palpable at 65 mm Hg, increasing waist circumference, and no urine output. How should the nurse interpret this assessment about the patient's aneurysm? A) Tamponade will soon occur. B) The renal arteries are involved. C) Perfusion to the legs is impaired. D) He is bleeding into the abdomen.

D) The lack of back pain indicates the patient is most likely exsanguinating into the abdominal space, and the bleeding is likely to continue without surgical repair. A blockade of the blood flow will not occur in the abdominal space as it would in the retroperitoneal space where surrounding anatomic structures may control the bleeding. The lack of urine output does not indicate renal artery involvement, but that the bleeding is occurring above the renal arteries, which decreases the blood flow to the kidneys. There is no assessment data indicating decreased perfusion to the legs.

The nurse is caring for a patient with a recent history of deep vein thrombosis (DVT). The patient now needs to undergo surgery for appendicitis. The nurse is reviewing the laboratory results for this patient before administering an ordered dose of vitamin K. The nurse determines that the medication is both safe to give and is most needed when the international normalized ratio (INR) is which result? A) 1.0 B) 1.2 C) 1.6 D) 2.2

D) Vitamin K is the antidote to warfarin (Coumadin), which the patient has most likely been taking before admission for treatment of DVT. Warfarin is an anticoagulant that impairs the ability of the blood to clot. Therefore it is necessary to give vitamin K before surgery to reduce the risk of hemorrhage. The largest value of the INR indicates the greatest impairment of clotting ability, making 2.2 the correct selection.

A 39-year-old woman with a history of smoking and oral contraceptive use is admitted with a venous thromboembolism (VTE) and prescribed unfractionated heparin. What laboratory test should the nurse review to evaluate the expected effect of the heparin? A. Platelet count B. Activated clotting time (ACT) C. International normalized ratio (INR) D. Activated partial thromboplastin time (APTT)

D. Activated partial thromboplastin time (APTT) Unfractionated heparin can be given by continuous IV for VTE treatment. When given IV, heparin requires frequent laboratory monitoring of clotting status as measured by activated partial thromboplastin time (aPTT). Platelet counts can decrease as an adverse reaction to heparin, but that is not the expected effect.

The nurse is administering a dose of digoxin (Lanoxin) to a patient with heart failure (HF). The nurse would become concerned with the possibility of digitalis toxicity if the patient reported which symptom(s)? A. Muscle aches B. Constipation C. Pounding headache D. Anorexia and nausea

D. Anorexia and nausea Anorexia, nausea, vomiting, blurred or yellow vision, and cardiac dysrhythmias are all signs of digitalis toxicity. The nurse would become concerned and notify the health care provider if the patient exhibited any of these symptoms.

Beyond the first year after a heart transplant, the nurse knows that what is a major cause of death? A. Infection B. Acute rejection C. Immunosuppression D. Cardiac vasculopathy

D. Cardiac vasculopathy Beyond the first year after a heart transplant, malignancy (especially lymphoma) and cardiac vasculopathy (accelerated CAD) are the major causes of death. During the first year after transplant, infection and acute rejection are the major causes of death. Immunosuppressive therapy will be used for posttransplant management to prevent rejection and increases the patient's risk of an infection.

A male patient with a long-standing history of heart failure has recently qualified for hospice care. What measure should the nurse now prioritize when providing care for this patient? A. Taper the patient off his current medications. B. Continue education for the patient and his family. C. Pursue experimental therapies or surgical options. D. Choose interventions to promote comfort and prevent suffering.

D. Choose interventions to promote comfort and prevent suffering. The central focus of hospice care is the promotion of comfort and the prevention of suffering. Patient education should continue, but providing comfort is paramount. Medications should be continued unless they are not tolerated. Experimental therapies and surgeries are not commonly used in the care of hospice patients.

A 55-year-old female patient develops acute pericarditis after a myocardial infarction. It is most important for the nurse to assess for which clinical manifestation of a possible complication? A. Presence of a pericardial friction rub B. Distant and muffled apical heart sounds C. Increased chest pain with deep breathing D. Decreased blood pressure with tachycardia

D. Decreased blood pressure with tachycardia Cardiac tamponade is a serious complication of acute pericarditis. Signs and symptoms indicating cardiac tamponade include narrowed pulse pressure, tachypnea, tachycardia, a decreased cardiac output, and decreased blood pressure. The other symptoms are consistent with acute pericarditis.

The patient with pericarditis is complaining of chest pain. After assessment, which intervention should the nurse expect to implement to provide pain relief? A. Corticosteroids B. Morphine sulfate C. Proton pump inhibitor D. Nonsteroidal antiinflammatory drugs

D. Nonsteroidal antiinflammatory drugs Nonsteroidal antiinflammatory drugs (NSAIDs) will control pain and inflammation. Corticosteroids are reserved for patients already taking corticosteroids for autoimmune conditions or those who do not respond to NSAIDs. Morphine is not necessary. Proton pump inhibitors are used to decrease stomach acid to avoid the risk of GI bleeding from the NSAIDs.

The patient with pericarditis is complaining of chest pain. After assessment, which intervention should the nurse expect to implement to provide pain relief? A. Corticosteroids B. Morphine sulfate C. Proton pump inhibitor D. Nonsteroidal antiinflammatory drugs

D. Nonsteroidal antiinflammatory drugs Nonsteroidal antiinflammatory drugs (NSAIDs) will control pain and inflammation. Corticosteroids are reserved for patients already taking corticosteroids for autoimmune conditions or those who do not respond to NSAIDs. Morphine is not necessary. Proton pump inhibitors are used to decrease stomach acid to avoid the risk of GI bleeding from the NSAIDs.

An 80-year-old patient with uncontrolled type 1 diabetes mellitus is diagnosed with aortic stenosis. When conservative therapy is no longer effective, the nurse knows that the patient will need to do or have what done? A. Aortic valve replacement B. Take nitroglycerin for chest pain. C. Open commissurotomy (valvulotomy) procedure D. Percutaneous transluminal balloon valvuloplasty (PTBV) procedure

D. Percutaneous transluminal balloon valvuloplasty (PTBV) procedure

An 80-year-old patient with uncontrolled type 1 diabetes mellitus is diagnosed with aortic stenosis. When conservative therapy is no longer effective, the nurse knows that the patient will need to do or have what done? A. Aortic valve replacement B. Take nitroglycerin for chest pain. C. Open commissurotomy (valvulotomy) procedure D. Percutaneous transluminal balloon valvuloplasty (PTBV) procedure

D. Percutaneous transluminal balloon valvuloplasty (PTBV) procedure The percutaneous transluminal balloon valvuloplasty (PTBV) procedure is best for this older adult patient who is a poor surgery candidate related to the uncontrolled type 1 diabetes mellitus. Aortic valve replacement would probably not be tolerated well by this patient, although it may be done if the PTBV fails and the diabetes is controlled in the future. Nitroglycerin is used cautiously for chest pain because it can reduce BP and worsen chest pain in patients with aortic stenosis. Open commissurotomy procedure is used for mitral stenosis.

The nurse is preparing to administer a scheduled dose of enoxaparin (Lovenox) 30 mg subcutaneously. What should the nurse do to administer this medication correctly? A. Remove the air bubble in the prefilled syringe. B. Aspirate before injection to prevent IV administration. C. Rub the injection site after administration to enhance absorption. D. Pinch the skin between the thumb and forefinger before inserting the needle.

D. Pinch the skin between the thumb and forefinger before inserting the needle. The nurse should gather together or "bunch up" the skin between the thumb and the forefinger before inserting the needle into the subcutaneous tissue. The nurse should not remove the air bubble in the prefilled syringe, aspirate, nor rub the site after injection.

The diagnosis is septic shock. The nurse knows that administration of insulin to this patient will provide which of the following beneficial effects? Select all that apply.

Decreased risk of infection Better wound healing

A client has developed constrictive pericarditis and myocardial hypertrophy. Select the most likely cause.

Diastolic dysfunction

Lanoxin

Digoxin

The client's serum digoxin level is 2.2 ng/dL and the heart rate is 120 and irregular. The nurse expects to administer which of the following drugs? Digoxin immune Fab (Digibind) Furosemide (Lasix) 60 mg I.V. Digoxin 0.5 mg bolus I.V. Potassium 40 mEq added to I.V. fluids

Digoxin immune Fab (Digibind) Rationale: Digibind binds and removes digoxin from the body and prevents toxic effects of digoxin overdose. A serum level of 2.2 is elevated, and the client is exhibiting signs of digoxin toxicity. The question does not indicate that the potassium level is low. Giving additional digoxin would exacerbate the toxicity. Giving Lasix may reduce potassium levels and contribute to increased toxicity.

A 70-year-old woman presents at a hospital after fainting at the funeral of a loved one. She is diagnosed as being in shock. Which signs and symptoms is she most likely to display? Select all that apply.

Dry, warm skin Bradycardia

At the cellular level, cardiac muscle cells respond to an increase in ventricular volume to the point of overload by: Select all that apply.

Elongating the cardiac muscle cells Decreasing the ventricular wall thickness

A patient in asystole is likely to receive which drug treatment? Epinephrine and atropine Lidocaine and amiodarone Digoxin and procainamide β-adrenergic blockers and dopamine

Epinephrine and atropine Normally the patient in asystole cannot be successfully resuscitated. However, administration of epinephrine and atropine may prompt the return of depolarization and ventricular contraction. Lidocaine and amiodarone are used for PVCs. Digoxin and procainamide are used for ventricular rate control. β-adrenergic blockers are used to slow heart rate, and dopamine is used to increase heart rate.

The nurse will monitor which of the following respiratory manifestations of heart failure? Select all that apply.

Exertional dyspnea Orthopnea Cheyne-Stokes respiration Chronic dry cough

A patient with Class I heart failure is likely to need an immediate heart transplant.

FALSE. A patient with Class IV heart failure is likely to need an immediate heart transplant

Diastolic dysfunction occurs due to a decrease in contractility leading to decreased stroke volume.

FALSE. Diastolic dysfunction occurs due to a decrease in preload leading to decreased stroke volume.

Myocardial hypertrophy is a useful compensatory mechanism in the later stages of HF.

FALSE. Myocardial hypertrophy is a useful compensatory mechanism in the later stages of HF. It unfortunately increases the oxygen demand of the heart.

Septic shock involves inappropriate vasodilation due to a histamine response.

FALSE. Septic shock involves inappropriate vasodilation due to inflammatory cytokines released in response to gram-negative bacterial endotoxins.

The Renin-angiotensin-aldosterone mechanism results in increased preload and increased afterload, which are helpful during end stage heart failure.

FALSE. The Renin-angiotensin-aldosterone mechanism results in increased preload and increased afterload, which are NOT helpful during end stage heart failure. Not helpful! The heart can't pump against a high afterload, and the heart is already too full of fluid, so more preload will worsen the HF.

The patient has hypokalemia, and the nurse obtains the following measurements on the rhythm strip: Heart rate of 86 with a regular rhythm; the P wave is 0.06 seconds (sec) and normal shape; the PR interval is 0.24 sec; the QRS is 0.09 sec. How should the nurse document this rhythm? First-degree AV block Second-degree AV block Premature atrial contraction (PAC) Premature ventricular contraction (PVC)

First-degree AV block In first-degree AV block there is prolonged duration of AV conduction that lengthens the PR interval above 0.20 sec. In type I second-degree AV block the PR interval continues to increase in duration until a QRS complex is blocked. In Type II the PR interval may be normal or prolonged, the ventricular rhythm may be irregular, and the QRS is usually greater than 0.12 sec. PACs cause an irregular rhythm with a different-shaped P wave than the rest of the beats, and the PR interval may be shorter or longer. PVCs cause an irregular rhythm, and the QRS complex is wide and distorted in shape.

Dopamine

For acute pain pt Excess preload Doesn't affect BP

Dobutrex (dobutamine)

For acute pain pt Excess preload Good for BP

Primacor and Inocor

For pt who are chronic Pt will get an infusion for a couple hours a couple days a week Palliative care Breath better and feel better

Natrecor

For pt who have >900 BMP Aren't going to get any better, just used to keep them alive

ANS:A Defibrillation is always indicated in the treatment of ventricular fibrillation. Drug treatments are normally used in the treatment of uncontrolled atrial fibrillation and for ventricular tachycardia with a pulse (if the patient is stable). Otherwise, synchronized cardioversion is used (as long as the patient has a pulse). Pacemakers are the treatment of choice for third-degree heart block. Reference: 833

For which dysrhythmia is defibrillation primarily indicated? A. Ventricular fibrillation B. Third-degree AV block C. Uncontrolled atrial fibrillation D. Ventricular tachycardia with a pulse

Digoxin toxicity

Halos, ringing in the ear, n/v Give digibind

Anaphylactic shock causes severe hypoxia very quickly because of which of the following reasons?

Histamine release causes massive vasodilation

Apresoline

Hydralazine

Increased cardiac workload with left heart failure can result in which of the following change to the myocardial cells?

Hypertrophy

A client has just returned from his surgical procedure. During initial vital sign measurements, the nurse notes that the client's heart rate is 111 beats/minute and the BP is 100/78 (borderline low). In this early postoperative period, the nurse should be diligently monitoring the client for the development of:

Hypovolemic shock due to acute intravascular volume loss

A 2400 mg sodium diet is prescribed for a patient with chronic heart failure, the nurse recognizes that additional teaching is necessary when the patient makes which statement?

I can eat most foods as long as I do not add salt when cooking or at the table?

which statement by the patient with chronic heart failure should cause the nurse to determine that additional discharge teaching is needed?

I should weigh myself every morning and go on a diet if I gain more than 2 or 3 pounds in 2 days.

IV Infusions

IV inotropes (milrinone--Primacor, dobutamine--Dobutrex) Acute decompensated HF Increase force of myocardial contraction IV vasodilators (nitropusside--Nipride, nitroglycerin, nesitiride--Natrecor)

The shortness of breath and cyanosis that occur in clients experiencing acute heart failure syndrome are primarily caused by: Select all that apply.

Impaired gas exchange Accumulation of fluid in the alveoli and airways Lung stiffness

ANS: C Radiofrequency catheter ablation therapy uses electrical energy to "burn" or ablate areas of the conduction system as definitive treatment for tachydysrhythmias. Reference: 837

Important teaching for the patient scheduled for a radiofrequency catheter ablation procedure includes explaining that A. ventricular bradycardia may be induced and treated during the procedure. B. catheters will be placed in both femoral arteries to allow double-catheter use. C. the procedure will destroy areas of the conduction system that are causing rapid heart rhythms. D. a general anesthetic will be given to prevent the awareness of any "sudden cardiac death" experiences.

Chronic HF - Collab Care 2

Improve Cardiac Contractility - cardiac glycosides - DIGOXIN - increase force of contraction and slows the heart down - improved cardiac output - kidneys work better to clean out body - monitor for DIGOXIN TOXICITY - halos around lights - brady arrythmias - anorexia and N/V **Also used for Atrail Fib (will also be on blood thinners to reduce chance of having a stroke)**

The health care team is developing a plan of care for a client diagnosed with congestive heart failure (CHF). The primary treatment goal would be:

Improving quality of life by relieving symptoms

A Interdisciplinary goals for the client with heart failure are to reduce cardiac workload and improve pump effectiveness. Loss of fluid, as indicated by weight loss, reduces cardiac workload. Decrease in heart rate and reduced pulmonary congestion indicate improved pump effectiveness. The client's condition has improved and there is not a need for more aggressive treatment. However, the heart rate remains higher than normal, and there are still crackles in the lungs, indicating the need for continued treatment.

In assessing a client admitted 24 hours ago with heart failure, the nurse notes that the client has lost 2.5 pounds, heart rate is down from 105 to 88, and there are fine crackles only in the bases of the lungs. The nurse correctly interprets these data as indicating: A) The treatment regimen is achieving the desired effect. B)The client's condition is unchanged. C)A need for more aggressive treatment. D) Heart failure has resolved.

An 86-year-old male client is disappointed to learn that he has class II heart failure despite a lifelong commitment to exercise and healthy eating. Which of the following age-related changes predisposes older adults to developing heart failure?

Increased vascular stiffness

A patient diagnosed with diastolic heart failure asks the nurse to explain why this has developed. The nurse knows that which of the following are often associated with diastolic failure? Select all that apply.

Ischemic heart disease Constrictive pericarditis Myocardial hypertrophy

Isordil

Isosorbide Dinitrate

Imdur

Isosorbide Mononitrate

Spironolactone

K sparing diuretic

Nitrate Teaching

Keep meds in dark area Replace every 3-6 months Check BP first Take 2-3 before calling 911 Causes GI upset

Anaphylactic shock is the most severe form of systemic allergic reaction. Immunologically medicated substances are released into the blood, causing vasodilation and an increase in capillary permeability. What physiologic response often accompanies the vascular response in anaphylaxis?

Laryngeal edema

The patient has heart failure (HF) with an ejection fraction of less than 40%. What core measures should the nurse expect to include in the plan of care for this patient (select all that apply.)? Left ventricular function is documented. Controlling dysrhythmias will eliminate HF. Prescription for digoxin (Lanoxin) at discharge Prescription for angiotensin-converting enzyme inhibitor at discharge Education materials about activity, medications, weight monitoring, and what to do if symptoms worsen

Left ventricular function is documented. Prescription for angiotensin-converting enzyme inhibitor at discharge. Education materials about activity, medications, weight monitoring, and what to do if symptoms worsen. The Joint Commission has identified these three core measures for heart failure patients. Although controlling dysrhythmias will improve CO and workload, it will not eliminate HF. Prescribing digoxin for all HF patients is no longer done because there are newer effective drugs and digoxin toxicity occurs easily related to electrolyte levels and the therapeutic range must be maintained.

best antibiotic for pneumonia

Levoquin

Bumex (bumetanide)

Loop diuretic Cerebral edema

A 30-year-old male brought to the emergency department has the following admission data: Blood pressure 50/30, pulse 100, respiratory rate 12, temperature 101°F. The nurse does not know the exact cause but does know that patient is in shock because of which of the following readings?

Low blood pressure

Zaroxylyn

Mild diuretic Does not have a lot of K+ change

Valsartan (Diovan)

Most common ARB Given to pt who can't have ACE Used for HF and HTN Pt take huge amounts, causes a decrease in BP

Metoprolol (Lopressor)

Most common beta blocker -check BP and HR

Lasix (furosemide)

Most common loop diuretic Depletes K+

Spironolactone

Most common potassium-sparing Treatment of edema and HTN

Which of the following health problems can contribute to diastolic dysfunction heart failure?

Myocardial hypertrophy

Natrecor

Nesiritide

A compensatory mechanism involved in HF that leads to inappropriate fluid retention and additional workload of the heart is

Neurohormonal response

Vasodilators

Nitric Oxide (fluffs alveoli) Flolan (pulmonary htn) Flolan is used with Viagra with cystic fibrosis kids -monitor for hypotension

Nitrostat

Nitroglycerin

Nitrates

Nitroglycerin -Always check BP before giving it -Decreases BP

A client who developed a deep vein thrombosis during a prolonged period of bed rest has deteriorated as the clot has dislodged and resulted in a pulmonary embolism. Which of the following types of shock is this client at risk of experiencing?

Obstructive shock

A client with heart failure tells the nurse that he is frustrated and is unable to get "a good night's rest." The client relates that he falls asleep and is suddenly awakened and feels as though he is having a hard time breathing and is suffocating. The nurse recognizes this assessment as:

Paroxysmal nocturnal dyspnea

On a holiday trip home, the nurse's mother states that the nurse's father was diagnosed with right heart failure. Which of the following manifestations exhibited by the father does the nurse know might have preceded this diagnosis?

Peripheral edema weight gain

Milrinone

Phosphodiesterase inhibitor

when caring for the patient with heart failure is treated with hydrochlorothiazide, digoxin and lisinopril. To prevent the risk of digitalis toxicity with these drugs, what is most important that the nurse monitor for this patient?

Potassium levles

A patient has sought care following a syncopal episode of unknown etiology. Which nursing action should the nurse prioritize in the patient's subsequent diagnostic workup? Preparing to assist with a head-up tilt-test Preparing an IV dose of a β-adrenergic blocker Assessing the patient's knowledge of pacemakers Teaching the patient about the role of antiplatelet aggregators

Preparing to assist with a head-up tilt-test In patients without structural heart disease, the head-up tilt-test is a common component of the diagnostic workup following episodes of syncope. IV β-blockers are not indicated although an IV infusion of low-dose isoproterenol may be started in an attempt to provoke a response if the head-up tilt-test did not have a response. Addressing pacemakers is premature and inappropriate at this stage of diagnosis. Patient teaching surrounding antiplatelet aggregators is not directly relevant to the patient's syncope at this time.

Heart failure can have which of the following consequences? Select all that apply.

Pulmonary congestion Systemic congestion Low cardiac output

A nurse preceptor is evaluating the skills of a new registered nurse (RN) caring for clients experiencing shock. Which action by the new RN indicates a need for more education?

Raising the head of the bed to a high Fowler's position

The nurse knows that the basic pathophysiology of heart failure is best described as which of the following?

Reduced ventricular efficiency

A patient with a recent diagnosis of heart failure has been prescribed furosemide. What outcome does the nurse anticipate will occur that demonstrates medication effectiveness? Promote vasodilation. Reduction of preload. Decrease in afterload. Increase in contractility.

Reduction of preload. Diuretics such as furosemide are used in the treatment of heart failure to mobilize edematous fluid, reduce pulmonary venous pressure, and reduce preload. They do not directly influence afterload, contractility, or vessel tone.

A patient with a recent diagnosis of heart failure has been prescribed furosemide. What outcome does the nurse anticipate will occur that demonstrates medication effectiveness? Promote vasodilation. Reduction of preload. Decrease in afterload. Increase in contractility.

Reduction of preload. Diuretics such as furosemide are used in the treatment of heart failure to mobilize edematous fluid, reduce pulmonary venous pressure, and reduce preload. They do not directly influence afterload, contractility, or vessel tone.

Diuretics

Remove excess extracellular fluid Different types differ in site of action All should be admin. in the morning to avoid disruption of sleep cycle Need may be avoided by diet & fluid intake Low Na diet (<2 g/day) & fluid intake (<2 qt/day)

After having a myocardial infarction (MI), the nurse notes the patient has jugular venous distention, gained weight, developed peripheral edema, and has a heart rate of 108 beats/min. What should the nurse suspect is happening? ADHF Chronic HF Left-sided HF Right-sided HF

Right-sided HF An MI is a primary cause of heart failure. The jugular venous distention, weight gain, peripheral edema, and increased heart rate are manifestations of right-sided heart failure.

Assessment of an elderly female client reveals the presence of bilateral pitting edema of the client's feet and ankles and pedal pulses that are difficult to palpate. Auscultation of the client's lungs reveals clear air entry to bases, and the client's oxygen saturation level is 93%, and vital signs are within reference ranges. What is this client's most likely health problem?

Right-sided heart failure

CO = __________ x _________

SV x HR

Which of the following does the nurse know can lead to right-sided heart failure?

Severe pneumonia

An important factor in the mortality of severe shock is acute renal failure. What impacts the extent of renal damage in shock?

Severity and duration of shock

The client is prescribed a beta-blocker as adjunct therapy to treatment of heart failure. The nurse recognizes that beta blockers act by Increasing contractility and cardiac output. Decreasing preload. Slowing the heart and decreasing afterload. Decreasing peripheral resistance

Slowing the heart and decreasing afterload. Rationale: Beta-blockers improve symptoms of HF by slowing heart rate and decreasing blood pressure. The decreased afterload causes decreased workload on the heart.

Corticosteroids

Solumedrol/Solucortef Prednisone -decreases agitation and calms cilia -makes pt gain weight, agitated, immunocompromised -make sure pt doesn't develop thrush

The nurse reviews the following vital signs recorded by an unlicensed assistive personnel (UAP) on a patient with acute decompensated heart failure: BP 98/60, HR 102 bpm, Temp 98.2 F, SpO2 84 % on 2 L/min via nasal cannula. what is the highest priority? what should the nurse due next?

SpO2 Place patient in high fowlers

Aldosterone antagonists (K-sparing diuretics)

Spironolactone (Aldactone) Blocks aldosterone in distal tubule & collecting duct Reduces mortality & morbidity in HF Monitor: hyperkalemia, hyponatremia

Digoxin (Lanoxin)

Strengthens ventricular pump Causes BP to drop, hold <60 Give IV if HR is really high Labs drawn q3months

What is the primary cause of heart failure in infants and children?

Structural heart defects

The patient has atrial fibrillation with a rapid ventricular response. The nurse knows to prepare for which treatment if an electrical treatment is planned for this patient? Defibrillation Synchronized cardioversion Automatic external defibrillator (AED) Implantable cardioverter-defibrillator (ICD)

Synchronized cardioversion Synchronized cardioversion is planned for a patient with supraventricular tachydysrhythmias (atrial fibrillation with a rapid ventricular response). Defibrillation or AEDs are the treatment of choice to end ventricular fibrillation and pulseless ventricular tachycardia (VT). An ICD is used with patients who have survived sudden cardiac death (SCD), have spontaneous sustained VT, and are at high risk for future life-threatening dysrhythmias.

A common finding in all types of circulatory shock is tissue hypoxia.

TRUE

The health care provider is reviewing lab results of a client diagnosed with heart failure. The provider notes that the client's ANP and BNP levels have been increasing and remain significantly elevated. These results would be interpreted as:

The condition is getting progressively worse

ANS: B PSVT is a dysrhythmia originating in an ectopic focus anywhere above the bifurcation of the bundle of His. Treatment includes vagal stimulation (e.g., Valsalva maneuver, coughing) and intravenous (IV) adenosine as the first drug of choice. The drug has a short half-life and is given rapid IVP. Other drugs are β-adrenergic blockers, calcium channel blockers, and amiodarone. Defibrillation is used if the vagal stimulation and drug therapy are ineffective and the patient becomes hemodynamically unstable. Digoxin is not used for this dysrhythmia but typically is used for atrial fibrillation. Reference: 826

The emergency department patient is in paroxysmal supraventricular tachycardia (PSVT) at a rate of 170 beats/minute. Which treatment do you anticipate first? A. Sotalol (Betapace) by slow IVP B. Adenosine (Adenocard) by fast IVP C. Defibrillation D. Digoxin (Lanoxin)

A, B, C, D, F (HDL is good)

The healthcare provider is teaching a group of senior citizens about risk factors for heart failure. Which of these factors will the healthcare provider include in the teaching? Select all that apply. A) High sodium intake B) History of preeclampsia C) Obesity D) Sleep apnea E) Increased high density lipoproteins (HDL) F) Hypertension

A) Stopping smoking can decrease the risk of CAD by 50%. While obesity, diet, and, stress impact the risk for CAD, stopping smoking has the greatest impact.

The nurse determines that teaching has been effective when a client with coronary artery disease (CAD) identifies which priority modifiable risk factor? A) Smoking B) Obesity C) Stress D) Diet

A) Peptic ulcer disease does not exacerbate heart failure. Respiratory infections, nutritional anemia, atrial fibrillation, stress, pregnancy, and thyroid disorders are among disorders and conditions that will exacerbate heart failure.

The nurse is examining a client in the clinic for follow-up care for heart failure. Which factor, if reported by the client, would not be associated with exacerbating heart failure? A) Peptic ulcer disease B) Nutritional anemia C) Recent upper respiratory infection D) Atrial fibrillation

Answer: A, C, D, and E. A cellphone should not be used near the pacemaker and it's best to keep the phone about half a foot away from the pacemaker. It is not necessary for the client to avoid using a microwave or other electrical devices. However, magnets should be kept away from the device. In order to prevent disruption of the leads after implantation (the most common complication), patients are often taught to limit activity on the affected side for awhile after implantation. Pulses are a good indicator of whether the pacemaker is supplying the body with enough cardiac output.

The nurse is providing discharge teaching to the client who has just received a pacemaker. Which of the following should the nurse include in the plan of care? SATA: A) Use your cell phone on the opposite side of your pacemaker B) You should avoid using a microwave from now on C) For the next week, it would be best to limit activity on the side with your new pacemaker D) You will need to inform airport security about your pacemaker before you fly anywhere E) It would be a good idea to check your pulse daily

B, C, E Rationale: The client undergoing angiography has a large-bore catheter inserted through the femoral artery, so the priority of care is to monitor and prevent bleeding. The client will lie flat for several hours and the groin will be checked regularly. A sandbag may be placed to maintain constant pressure on the arterial puncture site. Elevating the foot of the bed would increase pressure and blood flow to the groin and increase the risk of bleeding, as would applying heat to the leg.

The nurse receives an 82-year-old client with a history of A-fib from the angiography department. What priority actions will the nurse do next? (Select all that apply.) A) Elevate the foot of the bed or place pillows under the legs. B) Check the groin for bleeding or hematoma. C) Place the client supine for several hours. D) Apply heat to the calf of the leg. E) Place a sandbag on the femoral site.

Answer: C. Anemia can contribute to sinus tachycardia.

The nurse sees the following rhythm on the monitor. Which of the following lab values does the nurse identify as being most likely to have caused this dysrhythmia? a) K 3.0 b) Ca 10.5 c) Hgb 9 d) Magnesium 2.1

ANS: B If the pacemaker is set for 70 beats/minute, a slower rate means that the pacemaker is not working properly and the patient's spontaneous rate has taken over. This situation needs to be evaluated. Reference: 836-837

The patient has a pacemaker set for 70 beats/minute. When taking the patient's pulse, you obtain a heart rate of 60 beats/minute. What is the best interpretation of this finding? A. The patient's heart has become more effective. B. The pacemaker is not working properly. C. The patient is tolerating a lower heart rate now. D. The pacemaker is sensing a ventricular rhythm.

Answer: A. Following defibrillation, CPR is immediately initiated if a perfusable rhythm is not initiated. The client may need to be shocked again, but chest compressions must begin first.

The patient who has recently been experiencing runs of ventricular tachycardia suddenly loses consciousness. The patient is defibrillated, and the rate returns as the following. What should the nurse do first? A) Begin compressions B) Shock the client again immediately C) Prepare for intubation D) Administer adenosine

Answer: B. Atrial flutter places the client at high risk for development of clot formation in the atria. Because the client is stable at this time, cardioversion or adenosine would not be performed at this time. Before cardioversion can occur in a patient, anticoagulant therapy should be begun at least 48 hours beforehand if possible.

The patient with a history of hypertension and diabetes has the following rhythm strip. The patient's vitals are as follows: BP 145/89, HR 90, SpO2 95%, RR 19. Which of the following does the nurse expect to do at this time? a) Prepare the client for cardioversion STAT b) Begin administering anticoagulants c) Grab the crash cart for administration of adenosine d) Teach the client about possibility of pacemaker installation

The nurse assess the patient with chronic biventricular heart failure for paroxysmal nocturnal dyspnea (PND) by questioning the patient regarding?

The use of two or more pillows to help breathing during sleep.

Methylxanthines

Theophylline (level: 5-15) decreases inflammation, use with smokers Side effects: increases heart rate (30-40 beats)

Heart failure and circulatory shock are both conditions of circulatory system failure. Which of the following statements regarding these conditions is correct?

They have the same compensatory mechanisms

The nurse is seeing artifact on the telemetry monitor. Which factors could contribute to this artifact? Disabled automaticity Electrodes in the wrong lead Too much hair under the electrodes Stimulation of the vagus nerve fibers

Too much hair under the electrodes Artifact is caused by muscle activity, electrical interference, or insecure leads and electrodes that could be caused by excessive chest wall hair. Disabled automaticity would cause an atrial dysrhythmia. Electrodes in the wrong lead will measure electricity in a different plane of the heart and may have a different wave form than expected. Stimulation of the vagus nerve fibers causes a decrease in heart rate, not artifact.

Anti-cholinergics

Uses: COPD Atrovent Spiriva Side effects: dry mouth*, stomatitis, dizziness -cannot use with peanut allergy

Beta agonists

Uses: COPD, asthma, chronic gas pts Proventil - short-acting Xopenox - kids with cystic fibrosis; breathing treatment Serevent - long-acting, most common Side effects: tachycardia*, shakes, tremors *make sure pt is better after treatment*

A patient is told that she has cardiac valve leaflets, or cusps, that are floppy and fail to shut completely, permitting blood flow even when the valve should be completely closed. The nurse knows that this condition can lead to heart failure and is referred to as which of the following?

Valvular regurgitation

An elderly client asks the nurse why so many older people develop heart failure. The best response would be increased:

Vascular stiffness

Levels of endothelins may be increased in clients with heart failure. Which of the following is the primary action of endothelins?

Vasoconstriction

A client who has progressed to late-stage heart failure is experiencing sympathetic stimulation. The nurse would assess the client for: Select all that apply.

Vasoconstriction Arrhythmias Increased oxygen demand

In hypovolemic shock, the main purpose of treatment is correcting or controlling the underlying cause of the hypovolemia and improving the perfusion of the tissues and organs of the body. Which of the following treatments is not a primary form of therapy for hypovolemic shock?

Vasoconstrictive drugs

Hydralazine , Isosorbide dinitrate

Vasodilators

For which dysrhythmia is defibrillation primarily indicated? Ventricular fibrillation Third-degree AV block Uncontrolled atrial fibrillation Ventricular tachycardia with a pulse

Ventricular fibrillation Defibrillation is always indicated in the treatment of ventricular fibrillation. Drug treatments are normally used in the treatment of uncontrolled atrial fibrillation and for ventricular tachycardia with a pulse (if the patient is stable). Otherwise, synchronized cardioversion is used (as long as the patient has a pulse). Pacemakers are the treatment of choice for third-degree heart block.

Calcium channel blockers

Verapamil, Pricardia, Diltiazem Used for very high HR and BP Pt go into heart block Get a headache

HCTZ (hydrochlorothiazide)

Very mild thiazides Work by itself, works a lot better when combined with Lasix

A) Pulmonary Edema is a manifestation of left sided heart failure

When assessing a patient with chronic heart failure, the healthcare provider would expect to identify which of these clinical manifestations? A) Inspiratory crackles B) Asymmetrical Chest Expansion C) Expiratory Wheezing D) Subcutaneous crepitus

C Coronary artery thrombosis causes an inclusion of the artery, leading to myocardial death. An aneurysm is an outpouching of a vessel and doesn't cause an MI. Renal failure can be associated with MI but isn't a direct cause. Heart failure is usually a result from an MI.

Which of the following conditions is most commonly responsible for myocardial infarction? A Aneurysm B Heart failure C Coronary artery thrombosis D Renal failure

Answer: 3) This is describing ventricular tachycardia (QRS is a giveaway), and the treatment for a stable patient is Amiodarone or cardioversion. If the patient were unstable, we'd go ahead and defibrillate.

Which of the following does the nursing student realize is the treatment for a stable patient presenting with QRS intervals above 0.12 seconds with a regular rhythm and a rate of 100-250 bpm? 1) Atropine 2) Defibrillation 3) Amiodarone 4) Adenosine

Answer: B - prolonged hiccups indicate pacemaker failure. Other signs and symptoms of pacemaker failure are dysrhythmias, dizziness, faintness, chest pain, shortness of breath, increase or decrease in apical rate.

Which of the following signs and symptoms indicate pacemaker failure? a) excessive thirst b) prolonged hiccups c) flushing of the skin d) increased urine output

C Inadequate deactivation of aldosterone by the liver after right-sided heart failure leads to fluid retention, which causes oliguria.

Which of the following symptoms might a client with right-sided heart failure exhibit? A Adequate urine output B Polyuria C Oliguria D Polydipsia

A patient has an implantable cardioverter defibrillator (ICD). In cardioversion shock, the defibrillator is set in the synchronized mode to do what? a. Avoid discharging the shock during the T wave b. Discharge the shock during the R wave c. Discharge the shock during the T wave d. Avoid discharging the shock during the Q wave

a

A patient is diagnosed with torsades de pointes. The nurse prepares to administer which emergency medication? a. Magnesium sulfate b. Epinephrine (Adrenalin) c. Adesnosine (Adenocard) d. Calcium chloride

a

The patient is admitted with pain, edema, and warm skin on her lower left leg. What test should the nurse expect to be ordered first? a. Duplex ultrasound b. Complete blood count (CBC) c. Magnetic resonance imaging (MRI) d. Computed venography (phlebogram)

a

The remote telemetry technician alerts the nurse to the presence of premature ventricular contractions (PVCs) in a newly admitted patient. How does the nurse assess whether the premature complexes perfuse to the extremities? a. Palpate peripheral arteries while observing the monitor for widened complexes. b. Auscultate for the apical heart sounds and listen for irregularities or pauses c. Check the color and temperature of extremities, and capillary refill of fingers and toes d. Assess the ECG strip for regularity and width of QRS complexes

a

What does the P wave in an ECG represent? a. Atrial depolorization b. Atrial repolarization c. Ventricular depolarization d. Ventricular repolarization

a

What are characteristic of arteriospastic disease (Raynaud's phenomenon) (select all that apply)? a. Predominant in young females b. May be associated with autoimmune disorders c. Precipitated by exposure to cold, caffeine, and tobacco d. Involves small cutaneous arteries of the fingers and toes e. Inflammation of small and medium-sized arteries and veins f. Episodes involve white, blue, and red color changes of fingertips

a, b, c, d, f

The surgery area calls the transfer report for a 68-year-old, postmenopausal, female patient who smokes and takes hormone therapy. She is returning to the floor after a lengthy hip replacement surgery. Which factors present in this patient increase her risk for developing venous thromboembolism (VTE) related to Virchow's triad (select all that apply)? a. Smoking b. IV therapy c. Dehydration d. Estrogen therapy e. Orthopedic surgery f. Prolonged immobilization

a, b, d, e ,f

During treatment of a patient with a BP of 222/148 mm Hg and confusion, nausea, and vomiting, the nurse initially titrates the medications to achieve which goal? a. Decrease the mean arterial pressure (MAP) to 129 mm Hg b. Lower the BP to the patient's normal within the second to third hour c. Reduce the systolic BP (SBP) to 158 mm Hg and the diastolic BP (DBP) to 111 mm Hg within the first 2 hours d. Decrease the SBP to 160 mm Hg and the DBP to between 100 and 110 mm Hg as quickly as possible

a. Initially the treatment goal in hypertensive emergencies is to reduce the mean arterial pressure (MAP) by no more than 20% to 25% in the first hour, with further gradual reduction over the next 24 hours. In this case the MAP is 172, so decreasing it by 25% equals 129. MAP = [(2 x diastolic)+systolic] / 3

The normal width of the QRS complex is which of the following? (Select all that apply.) a. 0.06 to 0.10 seconds. b. 0.12 to 0.20 seconds. c. 1.5 to 2.5 small boxes. d. 3.0 to 5.0 small boxes. e. 0.04 seconds or greater.

a. 0.06 to 0.10 seconds. c. 1.5 to 2.5 small boxes.

The diagnosis of acute rheumatic fever is most strongly supported in the patient with a. carditis, polyarthritis, and erythema marginatum b. polyarthritis, chorea, and increased antistreptolysin O titer c. positive C-reactive protein, elevated WBC, subcutaneous nodules d. organic heart murmurs, fever, and elevated erythrocyte sedimentation rate (ESR)

a. carditis, polyarthritis, and erythema marginatum Rationale: Major criteria for the diagnosis of rheumatic fever include evidence of carditis, polyarthritis, chorea (often very late), erythema marginatum, and subcutaneous nodules. Minor criteria include all laboratory findings as well as fever, arthralgia, and a history of previous rheumatic fever.

The nurse recognizes that primary manifestations of systolic failure include: a. decreased EF and increased PAWP b. decreased PAWP and increased EF. c. decreased pulmonary hypertension associated with normal EF d. decreased afterload and decreased left ventricular end-diastolic pressure

a. decreased EF and increased PAWP Rationale: Systolic heart failure results in systolic failure in the left ventricle (LV). The LV loses its ability to generate enough pressure to eject blood forward through the aorta. This results in increased pulmonary artery wedge pressure (PAWP). The hallmark of systolic failure is a decrease in the left ventricular ejection fraction (EF).

The nurse is caring for a patient who is on a cardiac monitor. The nurse realizes that the sinus node is the pacemaker of the heart because it is: a. the fastest pacemaker cell in the heart. b. the only pacemaker cell in the heart. c. the only cell that does not affect the cardiac cycle. d. located in the left side of the heart.

a. the fastest pacemaker cell in the heart.

A patient with chronic heart failure has atrial fibrillation and a left ventricular ejection fraction of 18%. To decrease the risk of complications form these conditions, what drug does the nurse anticipate giving?

anti-coagulants

A 62-yr-old Hispanic male patient with diabetes mellitus has been diagnosed with peripheral artery disease (PAD). The patient is a smoker with a history of gout. To prevent complications, which factor is priority in patient teaching? a. Gender b. Smoking c. Ethnicity d. Comorbidities

b

A 73-yr-old man with dementia has a venous ulcer related to chronic venous insufficiency. The nurse should provide teaching on which type of diet for this patient and his caregiver? a. Low-fat diet b. High-protein diet c. Calorie-restricted diet d. High-carbohydrate diet

b

Which dysrhythmia causes the ventricles to quiver, resulting in absence of cardiac output? a. Ventricular tchycardia b. Ventricular fibrillation c. Asystole d. Third-degree heart block

b

While obtaining subjective assessment data from a patient with hypertension, the nurse recognizes that a modifiable risk factor for the development of hypertension is a. a low-calcium diet. b. excessive alcohol intake. c. a family history of hypertension. d. consumption of a high-protein diet.

b

Which clinical manifestations are seen in patients with either Buerger's disease or Raynaud's phenomenon (select all that apply)? a. Intermittent fevers b. Sensitivity to cold temperatures c. Gangrenous ulcers on fingertips d. Color changes of fingers and toes e. Episodes of superficial vein thrombosis

b, c, d

Electrocardiogram (ECG) paper contains a standardized grid where the horizontal axis measures time and the vertical axis measures voltage or amplitude. The nurse must understand that each horizontal box indicates: a. 200 milliseconds or 0.20 seconds duration. b. 40 milliseconds or 0.04 seconds duration. c. 3 seconds duration. d. millivolts of amplitude.

b. 40 milliseconds or 0.04 seconds duration.

A patient with a diagnosis of heart failure has been started on a nitroglycerin patch by his primary care provider. What should this patient be taught to avoid? a. High-potassium foods b. Drugs to treat erectile dysfunction c. Nonsteroidal antiinflammatory drugs d. Over-the-counter H2 -receptor blockers

b. Drugs to treat erectile dysfunction The use of erectile drugs concurrent with nitrates creates a risk of severe hypotension and possibly death. High-potassium foods, NSAIDs, and H2-receptor blockers do not pose a risk in combination with nitrates.

The patient's heart rate is 70 beats per minute, but the P waves come after the QRS complex. The nurse correctly determines that the patient's heart rhythm is: a. a normal junctional rhythm. b. an accelerated junctional rhythm. c. a junctional tachycardia. d. atrial fibrillation.

b. an accelerated junctional rhythm.

Drugs that the nurse would expect to be prescribed for patients with a mechanical valve replacement include a. oral nitrates b. anticoagulants c. atrial antidysrhythmics d. beta adrenergic blocking agents

b. anticoagulants Rationale: Patients with mechanical valves have an increased risk for thrombus formation. Therefore prophylactic anticoagulation therapy is used to prevent thrombus formation and systemic or pulmonary embolization. Nitrates are contraindicated for the patient with aortic stenosis because an adequate preload is necessary to open the stiffened aortic valve. Antidysrhythmics are used only if dysrhythmias occur, and alpha or beta adrenergic blocking agents may be used to control the HR as needed.

A patient with infective endocarditis of a prosthetic mitral valve develops a left hemiparesis and visual changes. The nurse expects that collaborative management of the patient will include a. an embolectomy b. surgical valve replacement c. administration of anticoagulants d. higher than usual antibiotic dosages

b. surgical valve replacement Rationale: Drug therapy for patients who develop endocarditis of prosthetic valves is often unsuccessful in eliminating the infection and preventing embolization, and early valve replacement followed by prolonged drug therapy is recommended for these patients.

The patient is admitted with a fever and rapid heart rate. The patient's temperature is 103° F (39.4° C).The nurse places the patient on a cardiac monitor and finds the patient's atrial and ventricular rates are above 105 beats per minute. P waves are clearly seen and appear normal in configuration. QRS complexes are normal in appearance and 0.08 seconds wide. The rhythm is regular, and blood pressure is normal. The nurse should focus on providing: a. medications to lower heart rate. b. treatment to lower temperature. c. treatment to lower cardiac output. d. treatment to reduce heart rate.

b. treatment to lower temperature.

An elderly client has been diagnosed with chronic heart failure. He is prescribed an ACE inhibitor to treat the symptoms and improve his quality of life. This drug will alleviate the client's symptoms of heart failure by:

blocking the conversion of angiotensin I to angiotensin II

An elderly client has been diagnosed with chronic heart failure. He is prescribed an ACE inhibitor to treat the symptoms and improve his quality of life. This drug will alleviate the client's symptoms of heart failure by:

blocking the conversion of angiotensin I to angiotensin II.

A 50-year-old woman weighs 95 kg and has a history of tobacco use, high blood pressure, high sodium intake, and sedentary lifestyle. When developing an individualized care plan for her, the nurse determines that the most important risk factors for peripheral artery disease (PAD) that must be modified are a. weight and diet. b. activity level and salt intake. c. tobacco use and high blood pressure. d. sedentary lifestyle and exercise training.

c

A patient in the telemetry unit is having continuous ECG monitoring. The patient is scheduled for a test in the radiology department. Who is responsible for determining when monitoring can be suspended? a. Telemetry technician b. Charge nurses c. Health care provider dd. Primary nurse

c

The provider has completed the placement f lead wires for the invasive temporary pace-maker in a patient who is asystolic. In turning on a pacing unit, which setting does the nurse use? a. Synchronous pacing mode b. Demand pacing mode c. Asynchronous pacing mode d. Temporary pacing mode

c

The patient has chronic hypertension. Today she has gone to the ED, and her blood pressure has risen to 200/140. What is the priority assessment for the nurse to make? a) Is the patient pregnant? b) Does the patient need to urinate? c) Does the patient have a headache or confusion? d) Is the patient taking antiseizure medications as prescribed?

c) Does the patient have a headache or confusion? The nurse's priority assessments include neurologic deficits, retinal damage, heart failure, pulmonary edema, and renal failure. The headache or confusion could be seen with hypertensive encephalopathy from increased cerebral capillary permeability leading to cerebral edema. Pregnancy can lead to secondary hypertension. Needing to urinate and taking antiseizure medication do not support a hypertensive emergency.

A 2400-mg sodium diet is prescribed for a patient with chronic heart failure. The nurse recognizes that additional teaching is necessary when the patient makes which statement? a. "I should limit my milk intake to 2 cups a day." b. "I can eat fresh fruits and vegetables without worrying about sodium content." c. "I can eat most foods as long as I do not add salt when cooking or at the table." d. "I need to read the labels on prepared foods and medicines for their sodium content."

c. Not adding salt to foods will not eliminate enough sodium for the 2400-mg sodium diet. All foods that are high in sodium should be eliminated in a 2400-mg sodium diet, in addition to the elimination of salt during cooking. Examples include obviously salted foods as well as unexpected sodium sources that are identified by reading the label of prepared foods and medicines.

29. Which assessment finding for a patient who has been admitted with a right calf venous thromboembolism (VTE) requires immediate action by the nurse? a. Erythema of right lower leg b. Complaint of right calf pain c. New onset shortness of breath d. Temperature of 100.4 F (38 C)

c. New onset shortness of breath

Digoxin

cardiac Glycoside

The nurse determines that treatment of heart failure has been successful when the patient experiences

clear lung sounds and decreased HR

The pathophysiology of heart failure involves an interaction between decreased pumping ability and the ________ to maintain cardiac output.

compensatory mechanisms

________ __________ is the term for right-sided HF that is the result of chronic pulmonary dz.

cor pulmonale

What is included in the correct technique for BP measurements? a. Always take the BP in both arms. b. Position the patient supine for all readings. c. Place the cuff loosely around the upper arm. d. Take readings at least two times at least 1 minute apart.

d. Correct technique in measuring BP includes taking two or more readings at least 1 minute apart. Initially BP measurements should be taken in both arms to detect any differences. If there is a difference, the arm with the higher reading should be used for all subsequent BP readings. The patient may be supine or sitting. The important point is that the arm being used is at the heart level and the cuff needs to fit snugly.

15. Which topic should the nurse include in patient teaching for a patient with a venous stasis ulcer on the leftlower leg? a. Need to increase carbohydrate intake b. Methods of keeping the wound area dry c. Purpose of prophylactic antibiotic therapy d. Application of elastic compression stockings

d. Application of elastic compression stockings

After having an MI, the nurse notes the patient has jugular venous distention, gained weight, developed peripheral edema, and has a heart rate of 108/minute. What should the nurse suspect is happening? a. ADHF b. Chronic HF c. Left-sided HF d. Right-sided HF

d. Right-sided HF An MI is a primary cause of heart failure. The jugular venous distention, weight gain, peripheral edema, and increased heart rate are manifestations of right-sided heart failure.

1. When discussing risk factor modification for a 63-year-old patient who has a 5-cm abdominal aortic aneurysm, the nurse will focus discharge teaching on which patient risk factor? a. Male gender b. Turner syndrome c. Abdominal trauma history d. Uncontrolled hypertension

d. Uncontrolled hypertension

The nurse is talking with the patient when the monitor alarms and shows a wavy baseline without a PQRST complex. The nurse should: a. defibrillate the patient immediately. b. initiate basic life support. c. initiate advanced life support. d. assess the patient and the electrical leads.

d. assess the patient and the electrical leads.

The nurse prepares a patient for synchronized cardioversion knowing that cardioversion differs from defibrillation in that a. defibrillation requires a lower dose of electrical energy b. cardioversion is indicated to treat atrial bradydysrhythmias c. defibrillation is synchronized to deliver a shock during the QRS complex d. patients should be sedated if cardioversion is done on a non-emergency basis

d. patients should be sedated if cardioversion is done on a non-emergency basis Rationale: Synchronized cardioversion is the therapy of choice for patients with hemodynamically unstable ventricular or supraventricular tachydysrhythmias. A synchronized circuit in the defibrillator delivers a countershock that is programmed to occur on the R wave of the QRS complex of the electrocardiogram. The synchronizer switch must be turned on when cardioversion is planned. The procedure for synchronized cardioversion is the same as for defibrillation with the following exceptions: If synchronized cardioversion is performed on a nonemergency basis, the patient is sedated before the procedure, and the initial energy needed for synchronized cardioversion is less than the energy needed for defibrillation.

A patient with aortic valve endocarditis develops dyspnea, crackles in the lungs, and restlessness. The nurse suspects that the patient is experiencing a. vegetative embolization to the coronary arteries b. pulmonary embolization from valve vegetations c. nonspecific manifestations that accompany infectious diseases d. valvular incompetence with possible infectious invasion of the myocardium

d. valvular incompetence with possible infectious invasion of the myocardium Rationale: The dyspnea, crackles, and restlessness the patient is manifesting are symptoms of heart failure and decreased cardiac output (CO) that occurs in up to 80% of patients with aortic valve endocarditis as a result of aortic valve incompetence. Vegetative embolization from the aortic valve occurs throughout the arterial system and may affect any body organ. Pulmonary emboli occur in right-sided endocarditis.

the nurse recognizes that primary manifestations of systolic failure include

decrease EF and increase PAWP

Digitalis

digoxin (Lanoxin) Improves cardiac contractility Increases force of contraction & slows conduction at AV node Decreasing S/S of systolic HF Caution: renal dysfunction (dig. tox risk) Key: monitor for dig. toxicity & bradycardia

The action of ANP and BNP most resembles the action of which class of meds: Beta 1 adrenergic receptor antagonists aldosterone diuretics ADH

diuretics

The patient with chronic heart failure is being discharged with a diuretic, a renin-angiotensin-aldosterone system inhibitor, and a Beta-adrenergic blocker. When recieved from pharmacy, which medication should not be included for this patient?

dopamine

Systolic heart failure can be caused by anything that: -Impedes expansion of ventricle such as pericardial effusion & constrictive pericarditis -Conditions that decrease wall thickness while increasing chamber size such as dilated hypertrophic cardiomyopathy -Conditions that delay diastolic relaxationIschemic heart dz -Aging -Conditions that unduly increase heart rate T/F

false These are all true about diastolic HF

Wilbur had a mild MI 20 years ago, but has not been able to control his HTN due to lack of compliance with his meds. For the last several months, he has experienced DOE, bouts of unproductive coughing, and is unable to sleep without being propped up on 2 to 3 pillows. It would be reasonable to suspect right-sided heart failure. T/F

false These are signs/symptoms of LEFT side HF

Loop diuretics

furosemide (Lasix) Inhibit sodium & chloride reabsorption in loop of Henle Usually initial treatment for fluid volume overload in HF Increase K excretion Monitor: electrolyte imbalances (hypokalemia, hyponatremia), renal dysfunction, diuretic resistance, hypotension

What is the pathophysiologic mechanism that results in the pulmonary edema of left-sided heart failure

increase hydrostatic pressure

The nurse plans long-term goals for the patient who has had a heart transplant with the knowledge that what is the most common cause of death in heart transplant patients during the first year?

infection

Match the etiology/S & S Anaphylactic shock

itching, hives, wheezing severe allergy to peanuts

Angiotensin-Converting Enzyme (ACE) Inhibitors

lisinopril (Prinivil), enapralapril (Vasotec) Promotes vasodilation & diuresis Diminishes heart workload & improves ventricular emptying Decrease aldosterone secretion (decrease BP & afterload) Excretes sodium & fluid--retains potassium Started low & slow Adverse: hypotension, hyperkalemia (potassium retention--D/C if 5.5 or higher), renal dysfunction, dry cough, ANGIOEDEMA

Furosemide

loop diuretic

Reduce Preload & Intravascular volume

loop diuretic lasix Foley catheter to measure - easier than a bedpan Check K+ and BP vasodilators (Nitroglycerin) NTG IV continuous - vasodilate High dose - will dilate both arterial and venous (low dose does JUST venous) position high fowlers (dangle feet) Don't want blood coming back to the heart/lungs

Neurogenic shock

low heart rate and warm, dry skin spinal cord injury

Which of the following is not a likely complication of shock? acute lung injury gastrointestinal ischemia and necrosis acute renal failure malignant hypertension DIC

malignant hypertension

Thiazide diuretics

metolazone (Zaroxolyn), hydrochlorothiazide (HCTZ) Inhibit Na and Cl reabsorption in distal tubules Also increases K excretion Monitor: electrolyte imbalances, renal dysfunction, diuretic resistance, hypotension

Beta-Adrenergic Blocking Agents (Beta-Blockers)

metoprolol (Lopressor), carvedilol (Coreg) Vasodilation, decrease afterload & cardiac workload (block adverse effects of SNS) Recommended: asymptomatic systolic HF (prevent disease progression) Adverse: hypotension, bradycardia, fatigue, depression Started low & slow Caution: hx of bronchospasm (ex asthma)

a patient with chronic HF and atrial fibrillation is treated witha digitalis glycoside and a loop diuretic. To prevent possible complications of this combination of drugs, what does the nurse need to do

monitor serum potassium levels Teach the patient how to take pulse rate

The nurse monitors receiving treatment for acute decompensated heart failure with the knowledge that marked by hypotension is most likely to occur with the IV administration of which medication?

nitroprusside

Septic shock

overwhelming infection with a gram-negative bacteria

Hypovolemic shock

prolonged vomiting or diarrhea burns excessive polyuria from uncontrolled diabetes mellitus acute blood loss

Obstructive shock

pulmonary embolism cardiac tamponade increased right-sided heart pressures with JVD

Cardiogenic shock

severe left lateral wall MI end-stage heart failure

Noepinephrine, Dopamine , Dobutamine

shock medications

Mucolytics

thins secretions, tastes and smells very bad used with cystic fibrosis kids, near drowning pts

Anaphylactic shock is directly associated with:

type I hypersensitivity response.

Angiotensin-Receptor Blockers (ARBs)

valsartan (Diovan), losartan (Cozaar) Block angiotensin II to block vasoconstriction Alternative to ACE inhibitors for HF Reduces BP and afterload Relieves S/S of HF & prevents disease progression Monitor: hypotension, hyperkalemia, renal dysfunction


Related study sets

TestOut Networking Test Chapter 2-3

View Set

Les déterminants et les substituts du nom

View Set

初一英语 unit 2 (2-family)

View Set

Assignment Energy Workplaces and Tasks

View Set

RUOE PT. 4 - Complete First Unit 9 , pg. 101 (acts. 1 and 2)

View Set

Animal Learning and Cognition Exam 2

View Set

Ciao! Ch.1- La città -vocabulary

View Set